Você está na página 1de 423

Um Curso de Cálculo 1

por Luı́s Gustavo Doninelli Mendes 23

1
Pretendo continuar acrescentando material e corrigindo imperfeições. Por isso
sugiro que o improvável leitor não imprima por enquanto este texto, por ser ainda
um trabalho em desenvolvimento. Sugestões, correções, por favor as envie para
mendes.lg@gmail.com
2
Professor Adjunto do Departamento de Matemática da UFRGS
3
Última atualização: 06/05/2010
Índice

Capı́tulo 1. Introdução 9
1. O Cálculo 9
2. Sobre o Curso 10
3. Livros-texto e Referências 12
Capı́tulo 2. Alguns dos objetivos do Cálculo 13
1. Funções e seus domı́nios 13
2. Função 15
3. Funções definidas a partir de outras funções 15
4. Diferentes domı́nios de funções 17
5. Gráfico descontı́nuo, mas que mesmo assim é gráfico 17
6. Função positiva, negativa e zeros ou raı́zes 18
7. Função crescente ou decrescente 18
8. Máximos e mı́nimos 20
9. Exercı́cios 21
Capı́tulo 3. Propriedade básicas dos números Reais 23
1. Intervalos 23
2. Metamorfoses de cúbicas 26
3. Exercı́cios 33
Capı́tulo 4. Sequências e seus limites 35
1. Sequências 35
2. Limites de sequências 37
3. Definição e Propriedades fundamentais 37
4. Exercı́cios 42
Capı́tulo 5. Limites de funções definidas em intervalos 45
1. Operações elementares com limites de funções 46
2. A definição usual com ǫ e δ 47
3. Limites quando x tende ao infinito 50
4. Quando a parte é do mesmo tamanho do todo 55
5. Exercı́cios 57
Capı́tulo 6. A noção de Continuidade 61
1. Operações com funções contı́nuas 62
2. Polinômios, funções racionais e trigonométricas 64
3. Continuidade da função inversa 68
4. Dois teoremas fundamentais sobre funções contı́nuas 69
3
4 ÍNDICE

5. Primeiras aplicações do T.V.I 70


6. Raı́zes de polinômios cujo grau é ı́mpar 70
7. Raı́zes simples e fatoração de polinômios 71
8. Exercı́cios 74

Capı́tulo 7. Geometria Analı́tica Plana 77


1. Equações de retas, coeficientes angular e linear 77
2. Ortogonalidade 79
3. Teorema de Tales no cı́rculo 80
4. A Reta de Euler 81
5. Função inversa como reflexão de gráfico na diagonal 90
6. O método de Descartes para as tangentes a um gráfico 91
7. Exercı́cios 94

Capı́tulo 8. A Tangente ao gráfico, segundo o Cálculo 97


1. Retas secantes a um gráfico 97
2. A reta tangente a um gráfico 97
3. A reta tangente ao seno em (0, 0) é a diagonal 100
4. Interpretação Fı́sica da reta tangente 103
5. Exercı́cios 104

Capı́tulo 9. A derivada 105


1. Definição, primeiras propriedades e exemplos simples 105
2. Um Árbitro que só avalia as inclinações 107
3. A segunda derivada 109
4. Exercı́cios 110

Capı́tulo 10. Sinal da derivada e crescimento 113


1. Teoremas de Rolle e de Lagrange 113
2. O Teorema 0 das Equações Diferenciais 117
3. Critérios de crescimento e de decrescimento 120
4. Uma confusão frequente sobre o sinal da derivada 120
5. Exercı́cios 122

Capı́tulo 11. Critérios para máximos e mı́nimos e Aplicações 125


1. Primeiro critério 125
2. Critério da segunda derivada 126
3. Um problema tı́pico para os engenheiros 127
4. Mı́nimos de distâncias e Ortogonalidade 128
5. Concavidades 130
6. Mı́nimos quadrados e a média aritmética 133
7. Inflexões 135
8. Critério da derivada de ordem n 136
9. Confecção de gráficos de polinômios e funções racionais 138
10. Exercı́cios 142

Capı́tulo 12. Derivadas de seno e cosseno e as leis de Hooke 147


ÍNDICE 5

1. O cosseno como derivada do seno 147


2. Leis de Hooke com e sem atrito 150
3. Exercı́cios 152
Capı́tulo 13. Indução Matemática e a derivada de xn , ∀n ∈ N. 153
1. Princı́pio de indução matemática 153
2. Derivada do Produto 155
3. Raı́zes múltiplas e fatoração de polinômios 156
4. Derivadas de x−n , ∀n ∈ N 158
5. Exercı́cios 158
Capı́tulo 14. Derivada da composição de funções 159
1. Regra da composta ou da cadeia 159
2. A Derivada do quociente 163
3. Uma função com derivada, mas sem a segunda derivada 165
4. Um problema de máximos/mı́nimos: o problema do freteiro 166
5. Exercı́cios 179
Capı́tulo 15. Derivadas de funções Implı́citas 181
1. Curvas versus gráficos 181
2. Teorema da função implı́cita 183
3. O método das tangentes e pontos racionais de cúbicas 186
4. Derivação implı́cita de segunda ordem 190
5. Exercı́cios 192
Capı́tulo 16. Funções inversas
√ e suas derivadas 193
1. Derivada de y = x 194
1 m −m
2. Derivada da “função”x n , de x n e de x n 195
3. Derivadas do arcoseno e do arcocosseno 196
4. Derivada do arcotangente 199
5. Existe uma função f 6≡ 0 que seja imune à derivação ? 201
6. Quantas funções são imunes à derivação ? 202
7. Exercı́cios 202
Capı́tulo 17. O Princı́pio de Fermat e a refração da luz 205
1. Princı́pio de Fermat 205
2. Refração, distâncias ponderadas e Lei de Snell 207
3. Exercı́cios 211
Capı́tulo 18. As Cônicas e suas propriedades refletivas 213
1. Definição unificada das cônicas 213
2. A Parábola e sua propriedade refletiva 220
3. A Elipse e sua propriedade refletiva 224
4. A Hipérbole e o análogo da propriedade refletiva 228
5. Exercı́cios 233
Capı́tulo 19. Integração e o Primeiro Teorema Fundamental 235
1. Área sob um gráfico positivo 235
6 ÍNDICE

2. Qual função descreve as Áreas sob gráficos? 236


3. Primeira Versão do Primeiro Teorema fundamental do
Cálculo 239
4. A Integral e o Primeiro Teorema fundamental 241
5. Funções com derivada, mas sem segunda derivada 244
6. Exercı́cios 245

Capı́tulo 20. Logaritmo natural e sua inversa, a exponencial 247


m
1. A propriedade ln(x n ) = mn
· ln(x) 247
2. A propriedade ln(x · y) = ln(x) + ln(y) 248
3. A composta ln | x | e sua derivada 249
4. Crescimento muito lento do logaritmo e muito rápido da
exponencial 250
5. Uma função extremamente achatada 253
6. Exercı́cios 256

Capı́tulo 21. Segundo Teorema Fundamental e Áreas 263


1. A descoberta de Gregory e Sarasa sobre área 263
2. Segundo Teorema Fundamental do Cálculo 264
3. Regiões entre dois gráficos 265
4. Exercı́cios 267

Capı́tulo 22. Técnicas de Integração 271


1. Integração por partes 271
2. Integração por substituição 274
3. Substituições trigonométricas, áreas do Cı́rculo e Elipse 278
4. Outras substituições trigonométricas 281
5. Integração
R de algumas funções racionais 282
6. R (ax + bx + c)−1 dx
2
282
7. (Ax3 + Bx2 + Cx + D)−1 dx 285
8. Exercı́cios 288

Capı́tulo 23. A curvatura dos gráficos 291


1. O comprimento de um gráfico 291
2. Velocidade de um gráfico ou de uma curva 293
3. Definição de curvatura e sua fórmula 294
4. Qual a curvatura de uma quina ? 296

Capı́tulo 24. Rudimentos de Equações diferenciais e Aplicações 299


1. A exponencial e as equações diferenciais 299
2. A definição original de Napier para o logaritmo 300
3. Decaimento radioativo e a datação de fósseis, rochas, ossos 302
4. Objetos em queda-livre 304
5. A curva que minimiza o tempo 311
6. Balı́stica e o Super Mário 314
7. Exercı́cios 318
ÍNDICE 7

Capı́tulo 25. Newton e a gravitação 321


1. Atração segundo o inverso do quadrado da distância 321
2. Tempo de colisão e velocidade de escape 322
3. Órbitas planetárias 325
4. Velocidade e aceleração expressas em coordenadas polares 326
5. Cônicas em coordenadas polares 333
6. Oscilador harmônico 335

Capı́tulo 26. Cinética quı́mica e crescimento bacteriano 339


1. Crescimento bacteriano 339
2. Cinética quı́mica 342

Capı́tulo 27. Área de regiões ilimitadas e Séries convergentes 343


1. Séries k-harmônicas, k > 1 344
2. A série geométrica 345
3. Um argumento geométrico para a série geométrica 347

Capı́tulo 28. Aproximação de Números e Funções importantes 349


1. Aproximações de raı́zes quadradas por números racionais 349
2. Raı́zes quadradas que são irracionais 350
3. Como tirar raı́z quadrada só com +, −, ×, / 350
4. Tangentes a parábolas e o formato da sequência de Newton 352
5. O Reais através de sequências de números Racionais 353
6. Aproximações de e = exp(1) por números Racionais 354
7. Arcotangente e cartografia 357
8. A aproximação de π dada por Leibniz 359
9. Aproximações de logaritmos 361
10. Aproximação de logaritmos de números quaisquer 362
11. Aproximação de ln(2) 364
12. Exercı́cios 364

Capı́tulo 29. Séries numéricas e de funções 367


1. Séries de números 367
2. Séries de funções 369

Capı́tulo 30. O discriminante de polinômios de grau 3 373


1. Quase a fórmula de Cardano 373
2. O discriminante como curva 378
3. A curva discriminante entre as cúbicas singulares 380

Capı́tulo 31. Apêndice: O expoente 34 comanda a vida ! 385


1. Metabolismo versus massa corporal 385
2. Escalas log/log para um experimento 386
3. Reta de ajuste - método de mı́nimos quadrados 386
4. A Lei experimental de Kleiber 388
5. Justificação racional da Lei de Kleiber 389
6. O argumento 390
8 ÍNDICE

Capı́tulo 32. Soluções detalhadas de alguns Exercı́cios 399


CAPı́TULO 1

Introdução

1. O Cálculo
O Cálculo Diferencial e Integral ou, simplesmente o Cálculo, é a
matemática que está na base da ciência de hoje.
As ciências mais desenvolvidas como Fı́sica e Quı́mica não podem
expressar seus conceitos sem fazerem uso do Cálculo. Também a Econo-
mia e a Biologia cada vez mais são matematizadas através do Cálculo.
O Cálculo foi fundamental na revolução cientı́fica dos séculos XVII
e XVIII e de lá para cá não cessou de produzir resultados e aplicações.
O Cálculo é uma teoria matemática, ou seja, um modo unificado de
se ver uma série de fatos matemáticos.
Na matemática, quando surge uma nova teoria, ao invés de se elim-
inar os resultados das teorias anteriores, o que a nova teoria faz é:
• reobter os teoremas até então conhecidos,
• dar generalizações deles,
• produzir resultados completamente novos.
Isso só ocorre em matemática: em outras ciências uma nova teoria
pode tornar obsoleta e errada a teoria anterior.
Por exemplo, a determinação exata da Área de certas regiões, que
com métodos elementares exigiu o gênio de Arquimedes, com o Cálculo
vira uma continha de rotina. Mas através do Cálculo aparecem fatos
novos e intrigantes sobre Áreas, como o fato de regiões ilimitadas
poderem ter Área finita.
Além de nos permitir provar tudo que já ouvimos falar de matemática
no colégio, o Cálculo vai nos transformar em verdadeiros McGivers, ou
seja, aquele personagem que com quase nada de recursos faz horrores
de coisas, como aparelhos, armas, etc, e suas missões. Através do
Cálculo , só com as quatro operações +, −, x vamos poder no Capı́tulo
28 aproximar com a precisão que quisermos:
• funções fundamentais como arctan(x), ln(x), etc

• números como p (p primo), π, e = exp(1).
Uma das inspirações fundamentais para o Cálculo foi a Fı́sica, ou
Fı́sica-matemática com a qual Isaac Newton revolucionou a ciência da
época. Vários fenômenos fı́sicos tiveram então uma explicação com-
pleta e unificada, através das técnicas do Cálculo.
Alguns tópicos do Curso:
9
2. SOBRE O CURSO 10
• No Capı́tulo 3 apresentamos cúbicas em forma implı́cita e al-
gumas de suas metamorfoses, usando apenas material do En-
sino Médio. Depois voltaremos a elas no Capı́tulo 15, já com
métodos do Cálculo, onde tocaremos num tema moderno: pon-
tos racionais de cúbicas. O estudo geral das cúbicas foi um dos
trabalhos de Isaac Newton. No Capı́tulo 30 daremos uma visão
unificada dos diferentes gráficos dos polinômios de grau três.
• o Capı́tulo 7 de Geometria Analı́tica traz material que não
tenho visto por aı́: a Reta de Euler dada analiticamente e o
método original de Descartes para determinar retas tangentes.
• Veremos algumas das conexões do Cálculo com a Fı́sica no
Capı́tulo 24 embora a ênfase do curso seja mais geométrica.
Nas Seções 4 e 6 estaremos seguindo a trilha de Galileu. Também
daremos aplicações à Arqueologia e Geologia.
• no Capı́tulo 25 veremos um pouco da revolução feita por New-
ton.
• no Capı́tulo 26 daremos aplicações à Biologia e Quı́mica (esta,
ainda em elaboração).
• O capı́tulo 3 (em elaboração) apresentará a importante noção
geométrica de curvatura dos gráficos.
• como um Apêndice, no Capı́tulo 31 expomos uma justificação
matemática de uma lei Biológica universal, chamada lei de
Kleiber

2. Sobre o Curso
Um alerta: este curso trata de matemática superior. Em várias
universidades, inclusive a nossa, há uma a tentativa de se ensinar o
Cálculo como se fosse uma continuação do Ensino Médio, seu ensino
sendo feito através de tabelas, regrinhas, macetes.
Se refletimos um pouco, vemos que em alguns cursos como Farmácia,
Economia, Biologia, o Cálculo é uma das poucas disciplinas de matemática
que terão na universidade. Desse modo, imitando o Ensino Médio, se
cursaria um Curso Superior sem ter contato com a Matemática Supe-
rior. A formação cientı́fica desses cursos ficaria prejudicada e de fato
não poderiam chamar-se cursos universitários.
Por isso neste Curso sempre que for possı́vel (exceto quando a ex-
plicação for técnica demais) vamos tentar dar justificações matemáticas
corretas, sem apelar para a credulidade do estudante e argumentos de
autoridade, do tipo acreditem em mim.
Os argumentos que damos são concatenações de idéias simples,
mas às vezes exigem um certo fôlego do leitor para acompanhá-lo do
começo ao fim. Esse treino de concentração certamente irá colaborar
na formação técnico-cientı́fica do estudante.
CAPÍTULO 1. INTRODUÇÃO 11

2.1. Sobre os Gráficos. Tentei fazer o máximo possı́vel de gráficos


para ilustrar o conteúdo, usando o programa Maple 9 para fazê-lo nu-
mericamente, ou seja, realisticamente. Este programa é pago, mas o
estudante pode usar o XMaxima ou mesmo o Gnuplot que são progra-
mas livres, do Linux, como auxiliar no estudo.
Sempre que possı́vel usei a mesma escala nos dois eixos, pois isso
determina inclinações das retas e essas inclinações são importantes no
Cálculo1.
Mas nem sempre isso foi possı́vel, por exemplo quando as funções
crescem muito rápido, onde não dá para manter as mesmas escalas nos
eixos x e y.
A teoria tem que ser sempre nossa guia na confecção de gráficos,
pois os computadores erram ao representar funções descontı́nuas ou
funções que estão muito próximas de um certo valor sem alcançar esse
valor.

2.2. Figuras. Também fiz figuras apenas qualitativas e diagramas


usando o programa Winfig, que é pago. Mas há uma versão livre para
o Linux, o Xfig.

2.3. Alerta aos estudantes. Por ser matemática superior, o Curso


exige do aluno um empenho e atenção muito diferente daquele exigido
nos seus contatos anteriores com a matemática.
Principalmente o aluno deve usar de modo preciso os conceitos que
vão sendo apresentados (por ex. limites, continuidade, derivada). Se
não os entender, pergunte ao professor até ter esclarecido o conceito.
Pois embora às vezes pareçam apenas conceitos qualitativos, são de fato
bastante precisos e mais tarde dão resultados quantitativos de absoluta
precisão.
Numa primeira leitura, o estudante pode ler o enunciado dos Teore-
mas e Afirmações, sem ler todas as demonstrações. Mas de fato, só se
entende completamente um fato matemático quando se entende a sua
demonstração.
Por último, é muito importante que o estudante pense nos exercı́cios
propostos em cada Capı́tulo. Mesmo que não responda todos, ao tentar
fazer exercı́cios o conteúdo vai sendo assimilado concretamente. E se
o aluno não consegue fazer quase que nenhum exercı́cio, então precisa
voltar a refletir no conteúdo dado.
Alguns têm solução bastante detalhada, apresentada no Capı́tulo
32. Mas que só devem ser lidas após muito trabalho pessoal do aluno.

1Vejapor exemplo o gráfico do seno que está errado em várias edições do livro
do Anton, pois ele não usou as mesmas escalas
3. LIVROS-TEXTO E REFERÊNCIAS 12

3. Livros-texto e Referências
Livros ruins de Cálculo: vários, de cuyos nombres no quiero acor-
darme.
Bastante razoável o livro do G. Thomas, disponı́vel na biblioteca
em várias edições.
Curto, direto e bom preço: R. Silverman, Essential Calculus with
applications, Dover.
Para mim um dos melhores livros de Cálculo é o de Michael Spi-
vak, Calculus (edições em espanhol e ingles na biblioteca da UFRGS).
Aprende-se muito nesse livro. Claro que é bastante difı́cil como primeiro
livro de Cálculo, mas o esforço de ler qualquer seção dele é sempre rec-
ompensado.
Excepcional e enciclopédico é o livro de R. Courant e F. John,
Introduction to Calculus and analysis, Interscience, 1965 (disponı́vel
em espanhol e inglês na bilioteca da UFRGS).
Usei neste Curso bastante do que aprendi com o Spivak, algumas
coisas que aprendi com o Courant-John, e também com E. Lima Curso
de Análise, Projeto Euclides, SBM.
Usei também informação do belo livro de C.H. Edwards, The his-
torical development of the Calculus, Springer, 1979.
No Capı́tulo sobre equações diferenciais uso material do Courant-
John e do excepcional livro de M. Hirsch e S. Smale Differential equa-
tions, dynamical systems and linear algebra, Academic Press, 1974.

Agradecimentos:

Agradeço a duas estudantes que fizeram Cálculo comigo em 2008:


Pâmela Lukasewicz Ferreira, por ter tomado notas do curso que dei
e que me serviram de roteiro para este texto e Mônica Hoeveler, por
participações em aula e por sugestões de temas.
Agradeço ao Professor Mark Thompson, da UFRGS, por ter me
disponibilizado notas de seus Cursos de Cálculo e também o livro de
G. Gibson, An elementary treatise on the Calculus, with illustrations
from Geometry, Mechanics and Physics, reimpressão de 1956 da edição
de 1901, e que me foram muito úteis.
CAPı́TULO 2

Alguns dos objetivos do Cálculo

A descrição matemática dos fenômenos se faz principalmente a par-


tir da noção de função y = f (x) e de seu gráfico.
Se pudermos entender:

• se f (x) assume somente valores Reais, onde f (x) se anula,


onde é positiva ou negativa,
• se e onde f (x) cresce ou decresce à medida que x cresce,
• se f (x) se aproxima de um certo valor quando x cresce muito,
• se e onde f (x) tem valor máximo ou mı́nimo,
• no caso de y = f (x) ≥ 0, qual a área sob seu gráfico e acima
do eixo dos x,
• se dado y pudermos descobrir qual x gerou y = f (x),

então podemos dizer que entendemos o comportamento da f (x).


Estaremos capacitados a fazer previsões sobre o fenômeno modelado
por essa função.
Esses são alguns dos objetivos do Cálculo.
Nas próximas Seções passamos lembrar / definir essas noções.

1. Funções e seus domı́nios


Os filósofos sempre se espantaram com o fato de que as coisas mu-
dam, e se questionaram tanto sobre o que muda como sobre o que
permanece nessas mudanças.
Os matemáticos também compartilham desse espanto e sempre se
perguntaram, ao ver que há mudanças, como as coisas mudam.
A resposta a essa pergunta pode ser tanto qualitativa como quan-
titativa, as duas são interessantes. Por exemplo é qualitativa quando
um astrônomo afirma que certo cometa voltará a passar algum dia. É
quantitativa no caso de Halley, que previu o ano em que certo cometa
voltaria, usando as ferramentas do Cálculo.
Se um fenômeno (a temperatura de um sistema, por exemplo) de-
pende de um só parâmetro (o tempo, por exemplo) é natural descrever
sua evolução num gráfico da função que associa a cada momento x a
13
1. FUNÇÕES E SEUS DOMÍNIOS 14

temperatura T (x). Esse gráfico formará uma curva no plano.

1
0,8
0,6
0,4
0,2
0
-2 -1 0 1 2
x

Figura: O gráfico de y = T (x) forma uma curva no plano.

Mas é claro que conhecemos fenômenos z = F (x, y) que dependem


de dois fatores e para descrever esse fenômeno precisariamos de gráficos
que formam superfı́cies no espaço, ao invés de curvas no plano. E em
geral os fenômenos dependem de vários parâmetros (em quı́mica, por
exemplo, quantidades de reagentes, pressão, ph, etc).

Figura: O gráfico de z = F (x, y) forma uma superfı́cie no espaço

Os conceitos que aprenderemos neste curso se adaptam facilmente


para superfı́cies, mas vamos nos restringir a gráficos que são curvas.
Ou como se diz, faremos o Cálculo de 1 variável.
A seguir vamos começar a estabelecer conceitos qualitativos sobre
gráficos que são importantes no Curso. O manejo correto desses con-
ceitos é fundamental para a compreensão do resto do curso.
CAPÍTULO 2. ALGUNS DOS OBJETIVOS DO CÁLCULO 15

2. Função
Uma função é uma regra que associa a cada ponto1 de um conjunto
(o domı́nio da função) um ponto de um outro conjunto fixado (o contra-
domı́nio). Dito de outro modo, uma reta vertical traçada passando
por um ponto do domı́nio de uma função y = f (x) corta seu gráfico
exatamente em 1 ponto. Por isso, por exemplo, um cı́rculo não é gráfico
de uma função y = f (x).
O subconjunto do contradomı́nio formado por pontos que são efeti-
vamente valores da função formam a imagem da função. Por exemplo,
f : R → R, f (x) = x2
tem como domı́nio e contradomı́nio os números Reais, mas sua imagem
são apenas os Reais não-negativos2.
Quando dizemos que f : I → J é sobrejetiva isto quer dizer que
não somente a imagem f (I) verifica f (I) ⊂ J, mas que de fato verifica
f (I) = J. Ou seja, que efetivamente todo ponto de J foi atingido
pela f . Por exemplo, f (x) = x2 só é sobrejetiva vista como função
f : R → R≥0 .
É importante notar na definição de função que só há um valor asso-
ciado a cada ponto do domı́nio. Se houver ambiguidade na atribuição
do valor então dizemos que a função não está bem-definida naquele
ponto. Por exemplo, quando perguntamos qual é a raı́z quadrada de 9
há uma ambiguidade: pode ser que tomemos a raı́z positiva 3 ou a raı́z
negativa −3.
Não confunda a definição de função com outra, a de função injetiva:
uma função é injetiva quando não associa o mesmo valor a dois pontos
distintos de seu domı́nio. Por exemplo, f : [0, 3] → R, f (x) = x2 é
injetiva mas f : [−3, 3] → R, f (x) = x2 não é injetiva.

3. Funções definidas a partir de outras funções


3.1. Função inversa. Imagine uma função que desfaz o efeito de
outra função.
Por exemplo, uma dá a a velocidade de um carro em função do
tempo trascorrido v = v(t). Sua inversa diria para cada velocidade v
qual o tempo necessário para atingir essa velocidade t = t(v) (o que dá
uma medida da potência do motor do carro, por ex.)
Ou por exemplo, a temperatura de um objeto vai caindo com o
tempo. Sabendo quanto caiu a temperatura T (t) como determinar o
tempo t transcorrido ?

1Para mim os números Reais formam um reta, portanto uso número ou ponto
indistintamente.
2Várias vezes no curso usaremos isso: o quadrado de um número Real nunca é
negativo
3. FUNÇÕES DEFINIDAS A PARTIR DE OUTRAS FUNÇÕES 16

Para se ter uma função inversa f −1 , a função f necessariamente


tem que ser injetiva !
Se não, vejamos: se y = f (x1 ) = f (x2 ) com x1 6= x2 , o que deve
fazer f −1 com y ? Enviá-lo em x1 = f −1 (y) ou em x2 = f −1 (y) ? Isso
é uma ambiguidade inaceitável para f −1 .
Vamos mais tarde falar do sentido geométrico da função inversa.
3.2. Composição de funções. Dentre os modos mais úteis de
se produzir um função interessante a partir de funções simples está a
composição de funções.
A idéia é simples e fundamental: o resultado de uma função g(x)
vira entrada de uma segunda função f .
A notação usual é: se f : I → J e g : J → K então (f ◦ g) : I → K
faz (f ◦ g)(x) := f ( g(x) ).
É claro que se pode compor um número qualquer de funções.
Pense em quantos exemplos encontramos disso na natureza, nas
reações quı́micas, nas indústrias, em que um processo complicado é
dividido em várias etapas simples concatenadas.
Neste Curso procedermos assim também: vamos primeiro entender
os casos mais simples e depois, via composição de funções, entender os
mais complicados.
3.3. O que é a Área sob um gráfico ? Podemos usar o gráfico
de uma função para definir outra. Por exemplo, tomo a diagonal y = x
como gráfico e me pergunto pela Área do triângulo determinado pela
origem, o eixo horizontal e um segmento vertical de (x, 0) até (x, x). À
medida que x avança no eixo dos x, a Área do triângulo obtido aumenta
e poderı́amos tentar descrever como essa Área depende de x isso num
outro gráfico.
Na definição do Logaritmo Natural, faremos exatamente isso, mas a
área em questão será delimitada sob o gráfico de 1/x e não sob y = x.

x=1 x
Figura: Área sob um o gráfico, de x = 1 até x.

Precisaremos saber primeiro, o que é a Área sob um gráfico curvado


como 1/x. Isso que foge do que sabemos do Ensino Médio, que são áreas
de regiões elementares como triângulos, quadrados, trapézios, setores
CAPÍTULO 2. ALGUNS DOS OBJETIVOS DO CÁLCULO 17

circulares, etc. Só entenderemos isso plenamente na Parte 2 do curso,


com o conceito de Integral.

4. Diferentes domı́nios de funções


A princı́pio o domı́nio de uma função pode ser qualquer conjunto,
mas neste Curso usaremos como domı́nios quase sempre:
• todos os Reais R, ou
• intervalos de números reais, incluindo semi-retas ou
• apenas os Naturais N ⊂ R.
Mas é claro que em certas situações os domı́nios também podem
ser a união de vários intervalos (como se verá por exemplo na Seção ??
do Capı́tulo 6), somente os números Racionais Q ⊂ R, etc.

5. Gráfico descontı́nuo, mas que mesmo assim é gráfico


Há gráficos que sofrem um salto abrupto, mas que mesmo assim são
gráficos.
Por exemplo, o gráfico da função f : R → R, definida condicional-
mente por
f (x) = x − 2, se x < 2 e f (x) = x2 se x ≥ 2.
O ponto 2 de seu domı́nio é um ponto catastrófico: se estamos em
pontos que são um pouquinho menores que 2 a função tem valores
próxima do zero. Mas se mexemos um pouco a coordenada x, chegando
em x = 2 ou acrescentando algo positivo muito pequeno ao 2, o valor
da função já pula para ≥ 22 = 4.

y=4

x=2

Figura: O gráfico de função descontı́nua no ponto x = 2

Outro modo de ver o que acontece é que, enquanto seu domı́nio R


é feito de um só pedaço, sua imagem f (R) = R≤0 ∪ R≥4 é feito de dois
pedaços: a função rasga seu domı́nio em dois pedaços.
Esses gráficos são úteis para modelar matematicamente comporta-
mentos explosivos: uma explosão quı́mica, o comportamento de um
animal à medida que aumenta o stress, etc. Mas em cursos de Cálculo
veremos gráficos que não tem essas variações dramáticas de valores.
7. FUNÇÃO CRESCENTE OU DECRESCENTE 18

6. Função positiva, negativa e zeros ou raı́zes


Uma função f : I → R é positiva (negativa)3 se se sua imagem está
contida nos Reais positivos (negativos).
Muito importante para um técnico ou cientista é determinar os
pontos do domı́nio onde a função se anula (ou, como se diz, onde corta
o eixo dos x, que é dado por y = 0). Ou seja, é importante resolver
uma equação f (x) = 0.
No caso de polinômios esses pontos são as chamadas raı́zes. Acon-
selho o leitor a ler o Teorema 7.1 no Capı́tulo 6, que prova a relação
entre raı́zes e fatores de polinômios.
Mais adiante, no Teorema 3.1 do Capı́tulo 5.1 explicaremos em
termos do Cálculo qual o significado das raı́zes múltiplas.

0
-2 -1 0 1 2
x
-2

-4

-6

Figura: Um gráfico de polinômio com 3 raı́zes

7. Função crescente ou decrescente


Definição 7.1. Uma função f : I → R é estritamente crescente
exatamente quando

∀ x1 , x2 ∈ I, x1 < x2 ⇒ f (x1 ) < f (x2 ).

E dizemos que é apenas crescente exatamente quando

∀ x1 , x2 ∈ I, x1 < x2 ⇒ f (x1 ) ≤ f (x2 ).

Analogamente se define estritamente decrescente, trocando f (x1 ) <


f (x2 ) por f (x1 ) > f (x2 ).

3Para evitar escrever duas frases onde só trocaria uma palavra, ponho em
parênteses a modificação a ser feita na frase
CAPÍTULO 2. ALGUNS DOS OBJETIVOS DO CÁLCULO 19

0,8

0,6

0,4

0,2

0
1 1,5 2 2,5 3
x

Figura: Exemplo de gráfico de y = f (x) crescente.

1
0,8
0,6
0,4
0,2

0 0,5 1 1,5 2 2,5 3


x

Figura: Exemplo de gráfico de y = f (x) decrescente.

Claro que há funções que não são nem crescentes nem decrescentes,
ou sejam, que oscilam.

0,8

0,6

0,4

0,2

0
-0,6 -0,4 -0,2 0 0,2 0,4 0,6
x

Figura: Exemplo de gráfico de y = f (x) que oscila.

Uma observação simples mas útil:


Se uma função f é estritamente crescente (ou estritamente decres-
cente) então f é injetiva.
8. MÁXIMOS E MÍNIMOS 20

De fato, se tomo quaisquer x1 , x2 diferentes de seu domı́nio, posso


sempre me perguntar qual deles é menor, por exemplo, x1 < x2 .
Como a f é estritamente crescente (ou estritamente decrescente), temos
f (x1 ) < f (x2 ) (ou f (x1 ) > f (x2 )), mas de qualquer forma f (x1 ) 6=
f (x2 ). Logo é injetiva.

Um exemplo importante é o que já demos de uma função f que


mede a Área sob um gráfico de uma outra função positiva. É natural
que f seja uma função estritamente crescente, pois à medida que vamos
para a direita no eixo x há mais área sob o gráfico. Logo é natural que
seja injetiva e tenha então uma inversa f −1 . Volto nesse ponto, com f
o Logaritmo Natural e f −1 a Exponencial.

Saber que uma função é crescente pode ser um fato extremamente


relevante do ponto de vista cientı́fico: por exemplo, um dos princı́pios
fı́sicos mais fundamentais é que a função Entropia é uma função cres-
cente, ou seja, que as coisas têm uma tendência a se desorganizar. É
essa Entropia crecente que está na base da nossa distinção entre pas-
sado, presente e futuro.

Por outro lado um exemplo marcante de função decrescente é a


função y = f (x) que dáa quantidade de uma substância radioativa
no tempo x. Uma descoberta cientı́fica fundamental foi a de descr-
ever de modo quantitativamente preciso como é essa função para cada
substância radioativa.

É fundamental neste curso estabelecermos um critério para deter-


minar se uma função é crescente (ou é decrescente).
De preferência um critério que consista em entender uma função que
seja mais simples que a função f ela mesma ! Se não não adiantaria
muito. Isso veremos no Capı́tulo 10, que é muito importante.

8. Máximos e mı́nimos
Uma das grandes utilidades do Cálculo é encontrar pontos onde uma
função atinge seu máximo ou mı́nimo. Ou seja, o Cálculo serve para
minimar ou maximizar: rendimento de um processo, custos, gastos,
etc, desde que o problema seja formulado matematicamente.
Vamos definir um máximo local (analogamente um mı́nimo local).
Definição 8.1. Seja f : I → R e x ∈ I. Dizemos que x é máximo
local se existe algum intervalo
(−ǫ + x, x + ǫ)
centrado em x, tal que
∀x ∈ I ∩ (−ǫ + x, x + ǫ), f (x) ≤ f (x).
CAPÍTULO 2. ALGUNS DOS OBJETIVOS DO CÁLCULO 21

Já x é dito ser um máximo global de f : I → R se


∀x ∈ I, f (x) ≤ f (x).

É a mesma diferença que há entre ser o cara que corre mais rápido
no clube do bairro e ser o cara que corre mais rápido no mundo !

4,2

3,8

3,6

3,4

3,2

-0,6 -0,4 -0,2 0 0,2 0,4 0,6


x

Figura: Função com um mı́nimo global, um máximo local e um mı́nimo local.

Chamo a atenção de que há funções que simplesmente não tem


máximo, como já vimos no caso de f : (0, 5] → R, f (x) = x1 .
E existem as que não tem mı́nimo: por ex. f : R≥1 → R, f (x) = x1 .
De fato, se tomo n ∈ R≥1 , temos f (n) = n1 , que já sabemos fica tão
próximo quanto quisermos de 0, sem nunca atingir zero. Isso diz que f
vai sempre diminuindo um valor, não tendo portanto um ponto de seu
domı́nio onde um valor mı́nimo fosse atingido.
Dá vontade de dizer algo sobre o papel do 0 neste exemplo f :
R≥1 → R, f (x) = x1 . O 0 realmente nunca é atingido pela função mas
de certo modo demarca, delimita o conjunto imagem
f (R≥1 ) = (0, 1].
0 é o que se costuma chamar uma cota inferior do conjunto imagem
f (R≥1 ), isto é,
∀y ∈ f (R≥1 ), 0 ≤ y.
E mais ainda, qualquer número maior que zero não é cota inferior
de f (R≥1 ), pois n1 ∈ f (R≥1 ) se aproxima o que quisermos de zero.
Portanto 0 é a maior cota inferior de f (R≥1 ), que se chama o Ínfimo
desse conjunto.

9. Exercı́cios
Exercı́cio 9.1. Determine em que intervalos as funções a seguir
são negativas ou positivas e onde estão seus zeros:
vi) x2 − x
vii) x2 − 5x + 6
viii) x3 − x2
9. EXERCÍCIOS 22

Exercı́cio 9.2. Dê exemplos de frases do dia a dia que são verdade,
mas cujas recı́procas não são verdade.
Exercı́cio 9.3. Negue as seguintes frases:
i) dado qualquer polı́tico, existe um valor de suborno tal que por
esse valor ele se corrompe.
ii) dada uma distância qualquer, existe um tempo tal que a partir
daquele tempo o asteróide dista da terra menos que a distância dada.
Exercı́cio 9.4. Imagine alguns exemplos, qualitativamente, sem
precisar dar explicitamente a regra f (x), de funções:
i) positivas e crescentes,
ii) negativas e crescentes,
iii) negativas e decrescentes,
iv) negativas e decrescentes,
v) com mı́nimo local, mas sem mı́nimo global
vi) com máximo local e máximo global diferentes.
Exercı́cio 9.5. Faça as composições f ◦ g ◦ h e h ◦ g ◦ f , onde:
i) f = x13 , g = sin(x) h = x + 5
ii) f = x2 , g = x1 , h = sin(x).
iv) Imagine algum exemplo onde aconteça f ◦ g ◦ h = h ◦ g ◦ f (o
que é raro !).
Exercı́cio 9.6. (resolvido)
Determine explicitamente as funções inversas f −1 das funções f (x)
a seguir. Teste sua resposta verificando que x = f −1 (f (x)).
i) f : R → R, f (x) = x3
ii) f : R → R, f (x) = x3 + 1
iii) f : R → R, f (x) = (x − 1)3
iv) f (x) = x1 ,
x
v) f : (0, 1) → R, f (x) = 1−x 2 . Dica: o mais difı́cil neste item é

não se equivocar com os sinais.


CAPı́TULO 3

Propriedade básicas dos números Reais

1. Intervalos
Um intervalo I ⊂ R é definido como o conjunto de todos os números
Reais maiores (ou iguais) a um certo número a e menores (ou iguais)
que um certo b.1
Se impomos que sejam estritamente maiores que a e estritamente
menores que b temos um intervalo aberto
I = {x ∈ R; a < x < b}
denotado I = (a, b). Caso contrário surgem os intervalos semi-abertos,
fechados, etc.
Um tı́pico intervalo que vamos usar no Curso será o intervalo aberto
de raio ǫ > 0 centrado num ponto x:
(−ǫ + x, x + ǫ)
onde x é um ponto da reta dos Reais e ǫ > 0 é um número positivo
fixado por nós.
O modo como vamos usar esses intervalos centrados é o seguinte:
(−ǫ + x, x + ǫ) será uma espécie de gaiola ou cercado em torno de
x, delimitando pontos próximos dele (à medida que ǫ > 0 é tomado
pequeno).
Explico isso em mais detalhe:
Definição 1.1. A distância entre dois pontos x, x da reta dos Reais
é definida pelo módulo2 da diferença entre eles:
|x − x| = |x − x|.

Pela definição de módulo, |x − x| < ǫ significa que


x − x < ǫ, se x − x ≥ 0 ou − (x − x) < ǫ, se x − x < 0.
É importante entender que:

1Podemos considerar a reta R toda ou uma semi-reta também como intervalos:


veremos isso em detalhe na Seção 4. Ao invés de usarmos o sı́mbolo (2, +∞) para
denotar a semi-reta dos números maiores que 2, prefiro usar o sı́mbolo R>2 : o
motivo é evitar o mal uso do sı́mbolo +∞.
2para um número Real △, |△| := △, se △ ≥ 0 ou |△| := −△, se △ < 0

23
1. INTERVALOS 24

Afirmação 1.1. (−ǫ+x, x+ǫ) é exatamente3 o conjunto dos pontos


que distam de x menos que ǫ > 0.

Demonstração.
Vamos mostrar primeiro que

(−ǫ + x, x + ǫ) ⊂ {x ∈ R; |x − x| < ǫ}.

Tome
x ∈ (−ǫ + x, x + ǫ),
com x 6= x (caso x = x não há nada a provar, pois ǫ > 0).
Ou seja x verifica:

−ǫ + x < x < x ou x < x < x + ǫ.

Que equivale (subtraindo x) a:

−ǫ < x − x < 0 ou 0 < x − x < ǫ.

Que equivale4 a:

0 < −(x − x) < ǫ ou 0 < x − x < ǫ,

ou seja, 0 < |x − x| < ǫ, como querı́amos.

Agora vamos mostrar que:

{x ∈ R; |x − x| < ǫ} ⊂ (−ǫ + x, x + ǫ).

.
Tome x ∈ {x ∈ R; |x − x| < ǫ}.
Se 0 ≤ x − x então temos

x−x<ǫ ⇔ x < x + ǫ,

e portanto x ∈ [x , x + ǫ).
Se x − x < 0 então

−(x − x) < ǫ ⇔ −x + x < ǫ ⇔ −ǫ + x < x,

ou seja, x ∈ (−ǫ + x , x).5.




3Dois conjuntos X e Y são iguais se X ⊂ Y e Y ⊂ X


4Atenção:as desigualdade se invertem quando multiplicadas por um número
negativo, por ex., 1 < 2 < 3 mas −3 < −2 < −1
5O quadrado à direita significa que a demonstração terminou
CAPÍTULO 3. PROPRIEDADE BÁSICAS DOS NÚMEROS REAIS
25

1.1. O que é útil num intervalo aberto.


Os intervalos abertos são importante no Cálculo, e o ponto impor-
tante é que um intervalo aberto tem uma certa tolerância com cada
um de seus elementos. Podemos mexer um pouquinho em cada um de
seus elementos sem sair do intervalo aberto. Mais especificamente:
Afirmação 1.2. Dado qualquer x ∈ (a, b) existe um pequeno in-
tervalo aberto centrado em x denotado Ix tal que Ix ⊆ (a, b).
Demonstração.
Considere as distâncias de x ∈ (a, b) até o extremo a e até o extremo
b:
|x − a| := x − a > 0, |x − b| := b − x > 0
(são dois números positivos pois (a, b) é intervalo aberto).
Dentre os dois agora escolho o menor, chamando-o de δ0 > 0:
δ0 := mı́nimo{ x − a, b − x }.
Faça
Ix := (−δ0 + x, x + δ0 ),
e vamos verificar que
(−δ0 + x, x + δ0 ) ⊂ (a, b).
Para isso vamos supor que é o caso que δ0 = x − a, ou seja, que x está
ou no centro do intervalo (a, b) ou um pouco mais próximo de a que de
b (analogamente no outro caso). Então
(−δ0 + x, x + δ0 ) = ( −(x − a) + x, x + (x − a) ) =

= ( a, x + (x − a) ).
Ora supusemos estar na situação em que x − a ≤ b − x, logo:
(a, x + (x − a)) ⊆ (a, x + (b − x)) = (a, b),
portanto:
(−δ0 + x, x + δ0 ) ⊆ (a, b)
como querı́amos.


Observe nessa Prova que à medida que x se aproxima de a ou de b


a tolerância (medida pelo δ0 ) fica menor, mas sempre existe.
Já no intervalo semi-aberto I = (0, 5] não há tolerância nenhuma
com seu elemento 5: ou seja, qualquer número δ > 0 que for somada a
5, já faz que 5 + δ não pertença a (0, 5].
2. METAMORFOSES DE CÚBICAS 26

1.2. O que é útil num intervalo fechado.


Num intervalo aberto acontece de seus elementos estarem se aproxi-
mando cada vez mais de um ponto que ele mesmo não está no intervalo,
por assim dizer de um fantasma. Por exemplo, os pontos 12 , 13 , . . . , n1 de
(0, 5) estão cada vez mais próximos de 0, mas mesmo assim 0 6∈ (0, 5).
Isso não acontece no intervalo fechado [0, 5].
Dito de outro modo, no Curso não estamos apenas interessados em
saber se um certo número z pertence ou não pertence a um conjunto
X ⊂ R, como se fazia no ensino Médio. Também vamos querer saber
se desse ponto z podemos achar elementos x ∈ X tão próximos quanto
quisermos.
• Se I é um intervalo aberto, pode acontecer que z ∈/ I e mesmo
assim hajam elementos de I tão próximos quanto quisermos.
• Se I é intervalo fechado, e há elementos de I tão próximos
quanto quisermos de z, então de fato z ∈ I.
Uma informação extremamente importante para um cientista é
saber se uma função que lhe interessa assume máximo ou mı́nimo em
seu domı́nio e principalmente, saber onde o faz.
Somente os intervalos fechados I = [a, b] garantirão sempre máximos
e mı́nimos globais de funções, senão pode acontecer algo como segue.
Pense em f : (0, 5] → R, f (x) = x1 . À medida que vamos tomando
os pontos 1/n ∈ (0, 5] a função vale
1
f ( ) = n,
n
que fica tão grande quanto quisermos. Note que (0, 5] não é um inter-
valo fechado.

2. Metamorfoses de cúbicas
Nesta Seção resolvi descrever curvas interessantes usando apenas
propriedades básicas do Reais, como regra dos sinais, desigualdades,
módulo, etc.
Coloquei na forma de Exercı́cios deste Capı́tulo a prova de várias
dessas propriedades fundamentais.
Para começar lembro uma propriedade básica, a regra dos sinais::
O produto de dois números Reais não-negativos é um número
Real não-negativo. E o produto de dois números Reais negativos é um
número positivo.
Esse princı́pio acarreta que
∀x ∈ R, x2 := x · x ≥ 0.
Logo não há raı́z quadrada de um número negativo.
Tudo o que vem a seguir nesta Seção é baseado nisso.
Começemos com o conhecido cı́rculo y 2 + x2 = r2 de raio r > 0.
Observe que:
CAPÍTULO 3. PROPRIEDADE BÁSICAS DOS NÚMEROS REAIS
27

• podemos tomar o gráfico de y = √ r2 − x2 para descrever o
semicı́rculo superior (ou tomar y = − r2 − x2 para o inferior).
• se r2 − x2 > 0 há duas escolhas de raı́zes, positiva e negativa,
e quando x = r ou x = −r essas duas escolhas colapsam numa
só, que é y = 0.
• Onde r2 − x2 < 0 deixamos de √ trabalhar sobre os Reais, pois
os valores associados a y = r2 − x2 passam para o terreno
dos números Complexos.6Como só tratamos neste Curso de
funções a valores Reais, não existem pontos do cı́rculo cuja
coordenada x verifique r2 − x2 < 0.
Por último, observe que mudando o valor de r muda o raio do
cı́rculo, portanto podemos pensar em y 2 + x2 = r2 como sendo uma
famı́lia de cı́rculos em que cada elemento fica determinando pelo r.
Veja a Figura:

0,5

y 0
-1 -0,5 0 0,5 1
x

-0,5

-1

Bom, mas tratar de cı́rculos é covardia, pois temos sua imagem


impressa na nossa mente desde a infância.
Que tal tratarmos de alguma curva que não tenha sua imagem
impressa na nossa mente ? E ademaias, que tal tratarmos logo de uma
famı́lia delas ?
Considere a familia de curvas dada por:
y 2 − x3 − r · x = 0, r 6= 0.
Vamos analisar separadamente o que acontece quando r > 0 e
quando r < 0.

Caso r > 0:
Temos
y 2 = x3 + r x ⇔ y 2 = x · (x2 + r).
Como x2 + r ≥ r > 0, o sinal de x · (x2 + r) só depende do de x. Logo
6Há uma versão magnı́fica do Cálculo sobre os números complexos !
2. METAMORFOSES DE CÚBICAS 28

• se x > 0 temos duas opções


p p
y = x · (x2 + r) ou y = − x · (x2 + r).
Ou seja, a curva não é um gráfico, ela tem uma parte no eixo
y > 0 e uma parte no eixo −y. Há uma simetria relativa ao
eixo dos x. √
• ainda se x > 0, |y| = x3 + rx observo que fica tão grande
quanto quisermos. De fato, se dou o valor 7 K >> 1:

3
x ≥ K 2 ⇒ x3 ≥ K 2 ⇒

⇒ x3 + rx ≥ K 2 ⇒ |y| = x3 + rx ≥ K.
p p
• essas duas escolhas y = x · (x2 + r) ou y = − x · (x2 + r)
colapsam numa só se x = 0, pois então y = 0.
• se x < 0 a(s) coordenada(s) y deixa de ser um número Real,
ou seja, para nós deixa de existir.
Uma Figura compatı́vel8 com essa descrição é:

y 0
0 0,4 0,8 1,2 1,6
x
-1

-2

-3

Caso r < 0
Agora
y 2 = x · (x2 + r),
e (x2 + r) pode ser positivo, negativo ou positivo. Por isso o estudo do
sinal de
x · (x2 + r)
é mais delicado.
Note que
√ √
x2 + r > 0 ⇔ x2 > −r > 0 ⇔ x2 > −r.
Só que √
x2 = |x|
7O sinal >> 1 quer dizer bem maior que 1
8Na Figura traçada há mais informação do que a que justificamos. Somente na
Seção 4 do Capı́tulo 15 é que teremos esses dados.
CAPÍTULO 3. PROPRIEDADE BÁSICAS DOS NÚMEROS REAIS
29

e portanto temos

x2 + r > 0 −r. ⇔ |x| >
√ √
Se x >√0, |x| > −r quer √
dizer x > −r mas se x < 0 isso quer dizer
−x > −r, ou seja x < − −r.
Em suma:
√ √
x2 + r > 0 ⇔ x < − −r ou x > −r.
Então
• se x > 0

x · (x2 + r) ≥ 0 ⇔ x≥ −r,
e teremos duas opções de √ raı́zes para determinar y. Que co-
lapsam para y = 0 se x = −r.
• se x ≤ 0, só teremos x · (x2 + r) ≥ 0 se (x2 + r) ≤ 0. Ou seja,

− −r ≤ x ≤ 0.
Nessa faixa de valores de x teremos √
duas opções de y, que
colapsam em y = 0 se x = 0 ou x = − −r.

Uma Figura compatı́vel com essa descrição é (r = −1).

y 0
-1 -0,5 0 0,5 1 1,5 2
x

-1

-2

Por último, note que se |r| vai ficando pequeno, então os pontos
√ √
(− −r, 0), (0, 0) e ( −r, 0)
vão se aproximando. Note que as ovais da parte negativa vão dimin-
uindo de tamanho quando |r| vai diminuindo.
Imagine r vindo de valores positivos, que vão ficando bem próximos
de zero, pulam o valor zero, e passam a assumir então valores negativos.
É como se de um continente fosse expelida uma ilhota, que vai
ficando maior e mais distante do continente: as quatro figuras a seguir
tentam mostrar isso.
2. METAMORFOSES DE CÚBICAS 30

y 0
0 0,4 0,8 1,2 1,6
x
-1

-2

-3

Figura: A curva y 2 − x3 − x = 0.

y 0
0 0,5 1 1,5 2
x
-1

-2

-3

Figura: A curva y 2 − x3 − 0.4 x = 0.

y 0
-0,5 0 0,5 1 1,5 2
x
-1

-2
CAPÍTULO 3. PROPRIEDADE BÁSICAS DOS NÚMEROS REAIS
31

Figura: A curva y 2 − x3 + 0.3 x = 0.

y 0
-1 -0,5 0 0,5 1 1,5 2
x

-1

-2

Figura: A curva y 2 − x3 + x = 0.

2.1. Suavização do caso r = 0.


Há uma pergunta natural: o que acontece na curva y 2 − x3 − 0 x =
y − x3 = 0 ?
2

Já aviso: os programas gráficos ficam bem perdidos para traçar essa
curva, se a coordenada x fica próxima de 0.
Por isso vou proceder como em muitos ramos da ciência, vou tentar
inferir qual o formato dessa curva tomando curvas que entendamos e
que estejam cada vez mais próximas dela.
Num sentido que ficará claro mais tarde, essas curvas próximas são
suaves ou não-singulares (ver Definição 3.1 na Seção 3 do Capı́tulo
30).
Na Figura a seguir traço a curva y 2 − x3 = 0 só que estabeleço
x ≥ 0.4, deixando a região em torno de x = 0 como um mistério.

y 0
0 0,4 0,8 1,2 1,6
x
-1

-2

-3

A curva y 2 − x3 = 0, só que x ≥ 0.4.


2. METAMORFOSES DE CÚBICAS 32

Como quero ter mais luz sobre esse objeto y 2 − x3 = 0 não vou
deformá-lo de novo na famı́lia y 2 −x3 −r x = 0, mas sim noutra famı́lia:

y 2 − x3 + s = 0, s ∈ R>0 .

Observo que a relação

y 2 = x3 − s

permite tirar raı́zes


√ quadradas desde que x3√− s ≥ 0. Portanto há duas
opções de x > s ou apenas y = 0 se x = 3 s.
3

Ou seja:

• a curva y 2 = x3 −√s só tem traço no plano Real se x ≥ 3 s e
• a partir de x > 3 s a curva é simétrica em relação
√ ao eixo
3
x,√já que temos duas opções diferentes: y = x − s e y =
− x3 − s.

Ademais note que se x > 3 s, então
√ √
y = x3 − s < x3

e
√ √
y = − x3 − s > x3 .
ou seja:
2 3
• dado x > 0, o traço da curva
√ y = x + s que tem y > 0 fica
sempre abaixo do de y = x3 .
2 3
• dado x > 0, o traço da curva
√ y = x + s que tem y < 0 fica
sempre acima do de y = − x3 .
A Figura a seguir ilustra isso para y 2 − x3 + 8 = 0:

y 0
0,5 1 1,5 2 2,5
x

-2

-4

A curva y 2 − x3 = 0, só que x ≥ 0.4, e a curva y 2 − x3 − 8 = 0.


As Figuras a seguir ilustram curvas cada vez mais próximas:
CAPÍTULO 3. PROPRIEDADE BÁSICAS DOS NÚMEROS REAIS
33

y 0
0,5 1 1,5 2 2,5
x

-2

-4

A curvas y 2 − x3 = 0, y 2 − x3 + 8 = 0 e y 2 − x3 + 1 = 0.

y 0
0,5 1 1,5 2 2,5
x

-2

-4

A curvas y 2 − x3 = 0, y 2 − x3 + 8 = 0, y 2 − x3 + 1 = 0 e y 2 − x3 + 0.5 = 0.
Será que agora o leitor consegue inferir a forma de y 2 − x3 = 0 ?

3. Exercı́cios
Exercı́cio 3.1. (resolvido)
Passo aulas e aulas repetindo que não se pode dividir por zero.
Mas a final, por quê isso é verdade ? No que podemos nos apoiar
para provar que não existe o número 01 ?
Exercı́cio 3.2. (resolvido)
Um aspecto bonito da matemática é que, após assumir a verdade
de certos fatos simples, podemos deduzir fatos novos, às vezes não tão
simples.
Neste exercı́cio, assuma como verdade os seguintes Princı́pios (Ax-
iomas):
Princı́pio 1: a soma de quaisquer dois números Reais não-negativos
é um número Real não-negativo.
3. EXERCÍCIOS 34

Princı́pio 2: o produto de um número positivo por um número


positivo é positivo.
Princı́pio 3: o elemento neutro multiplicativo 1 é positivo.
Usando esses Princı́pios 1 , 2, 3 prove as seguintes propriedades
fundamentais:
i) x ≥ y e z ≥ w ⇒ x + z ≥ y + w, ∀x, y, z, w ∈ R
ii) Se x > 0 e y ≥ z então x · y ≥ x · z.
iii) Se x < 0 e y ≥ z então x · y ≤ x · z.
iv) se x > 0 então x1 > 0
v) se x > 1 então x1 < 1.
Exercı́cio 3.3. Usando o Exercı́cio anterior, mostre que:
i) 0 < x1 < x2 ⇒ 0 < x12 < x11 .
ii) 0 < x < 1 ⇒ 0 < x2 < x < 1.
iii) 1 < x ⇒ 1 < x < x2
iv) 0 < x1 < x2 < 1 ⇒ 1 < x12 < x11 .
v) 1 < x1 < x2 ⇒ x12 < x11 < 1.
vii): 0 < x < 1 ⇒ 1 < x1 < x12 .
vii): 1 < x ⇒ x12 < x1 < 1.
Dica: O ideal é fazer um item e depois usá-lo nos que seguem. Mas
também é interessante, supôr a verdade de um item e mostrar apenas
como obter o outro.
Exercı́cio 3.4. (resolvido)
Para quais valores de x:
i) −3x + 2 > 0 ?
ii) x2 − x > 0 ?
iii) 3x2 − 2x − 1 > 0 ?
iii) 3x + 2 > 2x − 8 ?
iv) |x − 6| < 2 ?
v) |x + 7| < 1 ?
Exercı́cio 3.5. (resolvido)
Prove que para quaisquer números Reais  e △:
| + △| ≤ || + |△|.
Exercı́cio 3.6. Como são os gráfico das funções (com domı́nio
∀x ∈ R):
i) y = |x|,
ii) y = −| x|,
iii) y = |x − 5|,
iv) y = |x| + |x − 1| + |x − 2| ?
CAPı́TULO 4

Sequências e seus limites

1. Sequências
Neste Curso será importante a situação em que o domı́nio de uma
função será o conjunto dos números Naturais N = {1, 2, 3, ...}. Nesse
caso
f :N→R
é chamada de sequência.
A imagem de uma tal f é uma lista de números Reais. Como cada
ponto de sua imagem é do tipo f (n) é comum denotá-lo por xn e a
sequência toda por (xn )n .

Exemplo 0: f : N → R dada por f (n) = K é a sequência mais


boba de todas, pois sua imagem é somente o conjunto {K} - chama-se
sequência constante.

Exemplo 1: Uma sequência não tão boba é f : N → R dada por


f (n) = 2n, cuja imagem são os números Pares.

Exemplo 2:
Uma sequência fundamental para todo o Curso é
1
f : N → R, f (n) = .
n
No que segue, dizer que N é um conjunto ilimitado em R é dizer
que sempre há um número Natural maior que qualquer número Real
que for dado.
Afirmação 1.1. O fato de que os números naturais N formam um
conjunto ilimitado nos R é equivalente ao fato de que os valores de
f : N → R, f (n) = 1/n ficam tão próximos quanto quisermos de 0,
desde que n seja suficientemente grande.
Demonstração.
Uma equivalência é uma implicação em dois sentidos: ⇔.
Prova do sentido ⇒: Obviamente 1/n nunca é igual a 0: caso
pensássemos o contrário para algum n0 , obterı́amos de n10 = 0 e multi-
plicando por n0 obtemos que 0 = 1: absurdo.
A distância entre f (n) = 1/n e 0 é dada por |1/n − 0| = 1/n.
Suponha que nos foi dado um número positivo muito pequeno ǫ0 > 0.
35
1. SEQUÊNCIAS 36

Queremos confirmar que


1/n < ǫ0
a partir de um certo n, ou seja se n ≥ nǫ (onde uso a notação nǫ para
destacar que esse n depende do ǫ, quanto menor o ǫ maior o nǫ ). Mas
negar o anterior seria dizer:
1
∀n ∈ N, ǫ0 ≤ .
n
n
Mas isso equivale (multiplicando por ǫ0
> 0):
1
∀n ∈ N, n≤
ǫ0
1
Concluirı́amos então que o número ǫ0
é maior que todos os números
naturais, contradizendo a hipótese.
Prova do sentido ⇐:
Se existe um número K ∈ R tal que ∀n ∈ N tenhamos n ≤ K então
∀n ∈ N terı́amos K1 ≤ n1 . Logo a sequência n1 não se aproxima de 0
mais que K1 . Contradição.


Observação: É possı́vel se colocar um Axioma sobre os números


Reais - chamado Axioma de Completamento - que implica a propriedade
de N ser ilimitado em R.
Para nós, neste Curso, o fato dos Naturais serem ilimitados é tomado
como um Axioma.
Podemos também dizer o conteúdo da Afirmação anterior de outro
modo: dada uma cerca (−ǫ+0, 0+ǫ), se tomamos um nǫ suficientemente
grande, então ∀n ≥ nǫ teremos 1/n ∈ (−ǫ+0, 0+ǫ). Ou seja, esperando
o tempo suficiente nǫ , a partir dali a sequência 1/n não sai mais da
gaiola (−ǫ + 0, 0 + ǫ). Simbolicamente escreveremos
1
lim = 0,
n→+∞ n
que lê-se assim: zero é o limite da sequência 1/n ou a sequência tende
a zero
Veremos adiante que há sequências que tendem de diversas maneiras
diferentes a pontos, algumas vão decrescendo em valores como a (xn )n =
1/n, outras vão crescendo como −1/n, outras vão oscilando e assim por
diante, mas o que é importante é que:
• elas entram em qualquer cerca estabelecida em torno de seu
limite, desde que se espere o tempo nǫ suficiente e
• depois de lá entrarem não mais saem.
CAPÍTULO 4. SEQUÊNCIAS E SEUS LIMITES 37

Veremos também que podemos combinar sequências simples (cujo


limite podemos intuir facilmente) para criar sequências complicadas,
das quais não é possı́vel ter uma intuição de seu limite (exceto alguém
com poderes para-normais ...). Mesmo assim poderemos matematica-
mente determinar esses limites.

2. Limites de sequências
O conceito de limite é o conceito fundamental do Cálculo, de onde
surgem outras noções importantes como continuidade, derivada e inte-
gral. Por isso este é um Capı́tulo um pouco mais extenso.
Imagine uma máquina, um sistema ou um processo tal que para
um certo input x dá um certo output f (x). Agora imagine que para
um input parecido x + h (com h pequeno) dá um output parecido:
f (x + h) = f (x) + δ, com δ pequeno.
Apesar de ser uma situação plausı́vel, da qual temos muitos ex-
emplos no dia a dia, também sabemos que há exemplos da situação
oposta, em que, apesar de x + h ∼ x temos f (x + h) muito diferente
de f (x). Essas duas possibilidades são tı́picas de processos contı́nuos
e descontı́nuos, respectivamente.
O objetivo deste capı́tulo é definir essas noções precisamente, pois
nelas se apoiam os dois conceitos centrais do Curso: Derivada e Integral.

3. Definição e Propriedades fundamentais


Vamos começar com a Definição 3.1, que é mais precisa e importante
do que parece.
Nela destaco que há:
• uma enorme exigência: onde dizemos ∀ǫ >, e
• uma imposição: a de que a partir de um certo nǫ a sequência
não mais saia de uma região onde entrou.
Definição 3.1. Um sequência (xn )n tende a um ponto L se ∀ǫ
existe nǫ ∈ N tal que se n ≥ nǫ então xn ∈ (−ǫ + L, L + ǫ).
Há diferentes formas pelas quais uma sequência pode tender a um
limite; em particular, com diferentes velocidades.
Por exemplo, Afirmo que xn = n12 tende a 0 mais rapidamente do
que zn = n1 o faz. Ou seja, Afirmo que o tempo nǫ (zn ) de espera para
ter zn < ǫ é menor que o tempo nǫ (xn ) que tenho de esperar para ter
xn < ǫ. De fato,1:
r
1 1
nǫ (zn ) = ⌈ ⌉, nǫ (xn ) = ⌈ ⌉,
ǫ ǫ
q
e é claro que 1ǫ ≤ 1ǫ para ǫ pequeno.

1onde ⌈△⌉ significa o primeiro número Natural maior ou igual que △ ∈ R.


3. DEFINIÇÃO E PROPRIEDADES FUNDAMENTAIS 38

Nos argumentos discutidos abaixo teremos às vezes que esperar o


tempo n suficiente para que duas ou mais sequências se aproximem de
onde queremos. Como podem ser diferentes, por precaução tomamos
o maior dentre eles, para que as duas ou mais sequências estejam onde
queremos.
Teorema 3.1. (Propriedades fundamentais de sequências)
Sejam (xn )n e (zn )n duas sequências, com
lim xn = L1 e lim zn = L2 .
n→+∞ n→+∞

Então:
1) A sequência soma (xn + zn )n tem
lim (xn + zn ) = L1 + L2 .
n→+∞

2) A sequência diferença (xn − zn )n tem


lim (xn − zn ) = L1 − L2 .
n→+∞

3) Se C ∈ R é uma constante, então a sequência (C · xn ) tem


lim (C · xn ) = C · L1 .
n→+∞

4) Seja (qn )n uma sequência qualquer tal que


∀n, |qn | ≤ K,
para algum K. Se L1 = 0 então limn→+∞ (qn · xn ) = 0
5) A sequência produto (xn · zn )n tem
lim (xn · zn ) = L1 · L2 .
n→+∞

6) Se L2 6= 0, então:
• i) a partir de um certo n, zn 6= 0 e
• ii) limn→+∞ xznn = LL12 .
7) Suponha adicionalmente que a partir de um certo n, xn ≤ L1 e
que, para uma sequência qualquer qn , a partir de um certo n temos
x n ≤ q n ≤ L1 .
Então
lim qn = lim xn = L1 .
n→+∞ n→+∞

Demonstração. (de alguns itens do Teorema 3.1)


Prova de 1) Nesse primeiro item, o ponto a lembrar é que xn e zn
se aproximam cada uma de um número a princı́pio distinto e que cada
uma delas o faz possivelmente com velocidade diferente.
O que queremos provar? Queremos saber se, esperando um tempo
nǫ suficiente, conseguimos que:
xn + zn ∈ (−ǫ + L1 + L2 , L1 + L2 + ǫ),
CAPÍTULO 4. SEQUÊNCIAS E SEUS LIMITES 39

ou seja, como já explicamos, se |xn + yn − (L1 + L2 )| < ǫ. Vamos


traduzir esta última condição de outro modo, que leva em conta as
duas hipóteses sobre xn e zn 2:
|xn + yn − (L1 + L2 )| = |xn − L1 + yn − L2 | ≤
≤ |xn − L1 | + |yn − L2 |.
Agora fazemos o seguinte: esperamos tempo suficiente nǫ para que
tenhamos
ǫ ǫ
∀n ≥ nǫ , |xn − L1 | < e |zn − L2 | < .
2 2
Então obtemos de acima:
ǫ ǫ
|xn + yn − (L1 + L2 )| ≤ |xn − L1 | + |yn − L2 | < + = ǫ,
2 2
exatamente o que querı́amos provar.
Prova de 2): Análoga à do 1), apenas fazendo agora:
|(xn − yn ) − (L1 − L2 )| = |xn − L1 + L2 − zn | ≤ |xn − L1 | + |L2 − zn |.
Prova de 3): agora queremos que a partir de um certo nǫ :
| C · xn − C · L1 | < ǫ.
É claro que posso supor C 6= 0, senão tudo é óbvio.
Ora então o que queremos é provar que:
| C · (xn − L1 ) | < ǫ,
ou seja3 queremos que
|C| · |xn − L1 | < ǫ.
Noto agora que, se espero tempo nǫ suficiente, tenho:
ǫ
|xn − L1 | < , onde C 6= 0
C
pois xn se aproxima tanto quanto quisermos de L1 . Então juntando as
informações:
ǫ
|C · xn − C · L1 | = |C| · |xn − L1 | < C · = ǫ,
C
exatamente o que querı́amos.
Prova de 4): Aqui o que fazemos é esperar o tempo nǫ suficiente
para que |xn | < Kǫ (estou supondo que K 6= 0, pois se K = 0, então
2No último passo uso uma desigualdade (chamada desigualdade triangular, ver
Exercı́cio 3.5) que vale para quaisquer números Reais  e △:
| + △| ≤ || + |△|
, no nosso caso aplicadoa para  = xn − L1 e △ = yn − L2
3Para quaiquer números Reais  e △ sempre vale:

| · △| = || · |△|;
no nosso caso, uso para  = C e △ = xn − L1
3. DEFINIÇÃO E PROPRIEDADES FUNDAMENTAIS 40

a hı́pótese |qn | ≤ 0 diz que qn = 0 ∀n e tudo é óbvio, pois a sequência


0 · xn é a sequência constante, igual a 0). Então para n ≥ nǫ :
ǫ
|qn · xn | = |qn | · |xn | < K · = ǫ,
K
como querı́amos.
Prova de 5): Queremos fazer
| xn · zn − L1 · L2 | < ǫ.
dese que n cresça o suficiente.
Mas posso escrever:
| xn · zn − L1 · L2 | =
= | xn · zn −xn · L2 + xn · L2 −L1 · L2 | =
| {z }
0
= | xn · (zn − L2 ) + L2 · (xn − L1 ) | ≤
≤ | xn · (zn − L2 ) | + | L2 · (xn − L1 ) | =
= | xn | · | (zn − L2 ) | + | L2 | · | (xn − L1 ) |
E agora noto que |xn | ≤ K para alguma K , pois xn tende ao L1 ∈ R.
E tanto | (xn − L1 ) | quanto | (zn − L2 ) | se faz tão pequeno quanto
quisermos, pois zn tende a L2 e xn tende a L1 .
Logo | xn · zn − L1 · L2 | fica tão pequeno quanto quisermos.

Prova de 6): Primeiro afirmo que a partir de um certo n temos


L2
| | < |zn |.
2
Se L2 > 0, a partir de um certo n temos
L2
0< < zn
2
pois L22 < L2 = lim zn . E se L2 < 0, a partir de um certo n
L2
zn < <0
2
pois lim zn = L2 < L22 .
Ou seja, a partir de um certo n:
L2
| | < |zn |
2
e em particular a partir desse n, temos zn 6= 0.
No que segue já suponho que tomei esse n para que a partir dele:
L2
| | < |zn |.
2
Então além de podermos dividir pelos zn , podemos afirmar que
|L2 |2
< |zn | · |L2 |
2
CAPÍTULO 4. SEQUÊNCIAS E SEUS LIMITES 41

e portanto
1 2
< .
|zn · L2 | |L2 |2
Portanto
1 1 L2 − zn
| − |=| |=
zn L2 zn · L2
1
=| | · |L2 − zn | ≤
zn · L2
2
≤ · |L2 − zn |.
|L2 |2
Mas |L2 −zn | se faz tão pequeno quanto quisermos, desde que esperemos
possivelmente um tempo n ainda maior, já que lim zn = L2 .
Por exemplo, podemos esperar um n a partir do qual valha | L22 | <
|zn | e também
ǫ · L22
|L2 − zn | < ,
2
o que dá
1 1 2 ǫ · L22
| − |< · = ǫ.
zn L2 |L2 |2 2
Sobre 7): de fato, após esquecermos um certo número de termos
das sequências, temos
| qn − L1 | ≤ |xn − L1 |
e |xn − L1 | se faz tão pequeno quanto quisermos.


Chamo a atenção para uma propriedade, que provamos como parte


do item 6), e que será bastante útil:

Afirmação 3.1. Se limn→+∞ xn = L e L 6= 0 então a partir de


um certo tempo n, xn 6= 0. Em particular, se L > 0 (ou L < 0) então
a partir de um certo tempo n, xn > 0 (ou xn < 0).

Por último, será útil mais tarde se introduzimos dois sı́mbolos:


Definição 3.2. Dizemos que
lim xn = +∞
n→+∞

se ∀K > 0 existe um tempo nK tal que se n ≥ nK temos xn > K.


Dizemos que
lim xn = −∞
n→+∞
se ∀K < 0 existe um tempo nK tal que se n ≥ nK temos xn < K.
4. EXERCÍCIOS 42

Ou seja, sequências que ficam tão positivas quanto quisermos, ou


sequências que ficam tão negativas quanto quisermos, esperando o
tempo n suficiente. Exemplos: xn = n2 e xn = −n2 , respectivamente.

4. Exercı́cios
Exercı́cio 4.1. Exemplifique com sequências (xn )n bem simples a
diferença entre as seguintes frases:
i) a partir de um certo tempo n a sequência xn dista de L menos
que um ǫ > 0 e
ii) existem tempos n arbitrariamente grandes tais que xn dista de
L menos que um ǫ > 0.
Exercı́cio 4.2. Para as sequências (xn )n abaixo e para a função
y = f (x) = x12 , diga o formato da sequência ( f (xn ) )n :
i) xn = √1n ,
ii) xn = n1 ,
iii) xn = n2 .
Exercı́cio 4.3.
Explique se existem ou não os limites das seguintes sequências:
i) xn := 5 n,
ii) xn := (−1)n 5,
iii) xn := (−1)n (5 + n1 ),
iv) xn := (−1)n n5
v) xn := (−1)n n1 .
vi) xn = n1 + n2 + n3 ,
vii) xn = n1 · n2 · n3 .
Exercı́cio 4.4.
No dia-a-dia sabemos que todo gremista gosta de azul, mas nem
todos que gostam de azul são gremistas.
Tratando-se agora de sequências xn e zn , dê exemplos onde não
existem
lim xn ou lim zn
n→+∞ n→+∞
mas que no entanto existam:
lim (xn + zn ) ou lim (xn · zn ).
n→+∞ n→+∞

Exercı́cio 4.5. (resolvido)


Prove duas propriedades fundamentais de limites:

i) se xn < 0 ∀n e se limxn = L então L ≤ 0. Dê exemplo onde todo


xn < 0 mas onde L = 0.

ii) se limxn = L e se ∀n xn ≤ zn ≤ L, então limzn = L.


CAPÍTULO 4. SEQUÊNCIAS E SEUS LIMITES 43

Exercı́cio 4.6. Usando algumas sequências já estudadas em aula


e propriedades de +, −, ·, / de sequências, calcule:
1 1 300n2 + 35n + 1000
lim 3 · (2 − + 2 ), lim ,
n→+∞ n n n→+∞ n3 + n
300n2 + 35n + 1000 10123456789
lim , lim ,
n→+∞ 150n2 + n + 10000 n→+∞ n
30000000n + 1200000 2n7 + 35n + 1000
lim , lim .
n→+∞ n2 n→+∞ 3n7 + n + 10000
Dica: fatore n à força no numerador e no denominador as potências
mais altas e simplifique, antes de passar ao limite.
Exercı́cio 4.7. As sequências a seguir tendem a zero. Dado ǫ > 0
determine qual n (em função de ǫ) é suficiente para termos |xn | < ǫ
nas seguintes sequências: a): xn = n14 , b): xn = √1n , c): xn = 4√1 n
1
Exercı́cio 4.8. A sequência xn = n
fica dentro do intervalo [0, 1]
e é decrescente, ou seja
xn+1 ≤ xn , ∀n.
Já a sequência xn = 1 − n1 fica também dentro do intervalo [0, 1] mas
é crescente, ou seja xn+1 ≥ xn , ∀n. É verdade o seguinte Teorema:
sequências que ficam dentro de algum intervalo e que são ou bem cres-
centes ou bem decrescentes convergem para algum limite.
Veja em quais sequências a seguir pode-se aplicar esse Teorema:
n 2n
a): xn = 5n1 2 , b): xn = 5n 1
, c): xn = (−2)
n
, d): xn = (−1)
n
, e):
(−1)2n+1
xn = n
.
CAPı́TULO 5

Limites de funções definidas em intervalos

Neste Curso usaremos a noção de continuidade fortemente quando


calcularmos algumas Derivadas e mais adiante na teoria de Integração
do Capı́tulo 19.
Daremos sua definição precisa no próximo Capı́tulo.
Mas para isso, antes precisamos entender a noção de limite de
funções definidas em intervalos. Até agora só vimos limites de um
tipo de função, cujo domı́nio são os Naturais, as chamadas sequências.
Agora vamos definir:
Definição 0.1. Seja uma função f : I → R, y = f (x) definida
num intervalo I. Seja x tal que exista alguma sequência xn ∈ I \ {x}
com limn→+∞ xn = x.
Dizemos que função f tem limite L quando x tende a x, denotado
por
lim f (x) = L, L ∈ R,
x→x

se para toda sequência xn contida em I \ {x}


lim xn = x
n→+∞

temos
lim f (xn ) = L.
n→+∞

Observações importantes sobre a Definição 0.1:


• O ponto importante nesta definição é que, não importa quan-
tas sequências tomemos com limn→+∞ xn = x, sempre as
sequências f (xn ) tendem para o mesmo número L.
• O fato de que não seja relevante como xn se aproxima de x,
mas apenas que xn se aproxima x, fica visı́vel no sı́mbolo que
usamos:
lim f (x).
x→x

• O leitor verá mais tarde que às vezes x não está no domı́nio
das funções, ou seja, que não faz sentido perguntar por quanto
a função vale nele, mas que, como x está arbitrariamente
próximo do domı́nio dessas funções, podemos perguntar quanto
a função vale em pontos do domı́nio cada vez mais próximos
dele.
45
1. OPERAÇÕES ELEMENTARES COM LIMITES DE FUNÇÕES
46

• o valor f (x) pode ser bem diferente de limx→x f (x). Por isso
tomamos sequências xn contidas em I \ {x} (ou seja, que não
valem nunca x).

1. Operações elementares com limites de funções

A noção de limite de funções foi construı́da a partir da de limite


de sequências; assim que é natural que as propriedades de limites de
sequências repercutam nas dos limites de funções definidas em interva-
los.
Teorema 1.1. (Propriedades fundamentais de limites de funções)
Sejam f e g cujos domı́nios são intervalos e seja x tal que existam
sequências nos domı́nios dessas funções que tendam a ele.
Suponha que existam:
lim f (x) = L1 e lim g(x) = L2 .
x→x x→x

Então:
1) A função soma f + g tem
lim (f + g)(x) = L1 + L2 .
x→x

2) A função diferença f − g tem


lim (f − g)(x) = L1 − L2 .
x→x

3) Se C ∈ R é uma constante, então a função (C · f )(x) := C · f (x)


tem
lim (C · f )(x) = C · L1
x→x

4) Suponha uma função q(x) com o mesmo domı́nio da f (x) tal que
|q(x)| ≤ K, ∀x. Suponha adicionalmente que L1 = 0. Então
lim ( f (x) · q(x) ) = 0.
x→x

5) A função produto (f · g)(x) tem


lim (f · g)(x) = L1 · L2 .
x→x

6) Se L2 6= 0, então: i) se x é suficientemente próximo de x então


g(x) 6= 0 e ii) limx→x fg(x)
(x)
= LL12 .
7) Suponha uma outra função q(x) definida no mesmo domı́nio e
que adicionalmente f (x) ≤ q(x) ≤ L1 . Então
lim q(x) = lim f (x) = L1 .
x→x x→x
CAPÍTULO 5. LIMITES DE FUNÇÕES DEFINIDAS EM
INTERVALOS 47
Demonstração.
Prova do Item 1): Queremos saber se
lim ( f (xn ) + g(xn ) ) = L1 + L2 ,
n→+∞

quando tomamos qualquer sequência xn com


lim xn = x.
n→+∞

Mas por hipótese, limn→+∞ f (xn ) = L1 e limn→+∞ g(xn ) = L2 ,


quando tomamos qualquer sequência xn com limn→+∞ xn = x.
Ora, pelo item 1) do Teorema 3.1, aplicado às sequências f (xn ) e
g(xn ), concluimos que limn→+∞ ( f (xn ) + g(xn ) ) = L1 + L2 .

A prova de outros itens fica para o leitor, bastando combinar a


Definição 0.1 com alguns itens do Teorema 3.1, bem como com a Afir-
macao 3.1. 

2. A definição usual com ǫ e δ


Na maioria dos livros texto de Cálculo, o limite de uma função
definida em um intervalo é definido assim:
Definição 2.1. Dizemos que f tende a L quando x tende ao x, ou
em sı́mbolos:
lim f (x) = L
x→x

se ∀ǫ > existe δ > 0 tal que se 0 < |x − x| < δ então |f (x) − L| < ǫ.

Observações:
• pense em ǫ > 0 como um número pequeno, que impõe o desafio
de se encontrar o δ > 0 suficiente para termos |f (x) − L| < ǫ,
desde que 0 < |x − x| < δ.
• o sı́mbolo ∀ǫ > 0 (para todo ǫ > 0) diz que ǫ será feito tão
pequeno quanto quisermos,
• veremos logo abaixo que o δ depende do ǫ, da natureza da f e
também, em geral, de cada ponto x.
• a cláusula 0 < |x − x| existe para que possamos ter funções
com f (x) 6= L = limx→x f (x).
Um pouco mais sobre o último item: suponha que temos uma f
com f (x) bem diferente dos valores f (x), para x próximos de x porém
diferentes de x. Por exemplo suponha que |f (x) − L| ≥ 1 , embora
|f (x) − L| < ǫ é pequeno se x 6= x, mas x próximo de x. Então
|x − x| = 0 < δ, ∀δ > 0 e no entanto |f (x) − L| ≥ 1. Por isso na
Definição 2.1 estamos interessados apenas em controlar os valores f (x)
para x 6= x.
2. A DEFINIÇÃO USUAL COM ǫ E δ 48

Vejamos agora que essa nova Definição 2.1 tem o mesmo conteúdo
da Definição 0.1 do Capı́tulo 4, mesmo que a princı́pio não pareçam o
mesmo.

Afirmação 2.1. A Definição 2.1 é equivalente à Definição 0.1 do


Capı́tulo 4.

Demonstração. (da Afirmação 2.1)


Provar a equivalência de duas definições é mostrar que uma implica
a outra e vice-versa.
Suponha por um momento a Definição 0.1 e por absurdo negue a
Definição 2.1.
Então existe um ǫ0 > 0 especial tal que ∀δ > 0 existe um xδ com

0 < |xδ − x| < δ, mas |f (xδ ) − L| ≥ ǫ0 .

Já que vale para todo δ > tomo-os da forma δ(n) := n1 . Então
concluo que os xδ(n) formam uma sequência de I \ {x} que tende a x,
pois
1
0 < |xδ(n) − x| <
n
e já sabemos que os n1 ficam tão pequenos quanto quisermos. Com essa
sequência (xδ(n) )n no domı́nio da f , formo outra sequência f (xδ(n) ) na
imagem da f , que não tende a L já que

|f (xδ(n) ) − L| ≥ ǫ0 , ∀n,

ou seja, não se aproxima do número L mais que ǫ0 . Isso contradiz a


Definição 0.1.
Agora suponha Definição 2.1 e vamos obter a informação dada pela
Definição 0.1.
Considere qualquer sequência xn de I \{x} que tenda a x: queremos
saber então se é verdade que f (xn ) tende a L. Ou seja, se dado ǫ > 0
existe nǫ ∈ N tal que ∀n ≥ nǫ temos |f (xn ) − L| < ǫ.
O que sei pela Definição 2.1 é que existe um δ > 0 tal que:

0 < |x − x| < δ ⇒ |f (x) − L| < ǫ.

Então tomo esse δ > 0 e, para ele, tomo um nδ ∈ N tal que:

∀n ≥ nδ ⇒ 0 < |xn − x| < δ

(o que funciona pois xn tende a x).


Logo |f (xn ) − L| < ǫ pois os xn entraram na região adequada em
torno de x, que é (−δ + x, x + δ).
A Figura ilustra:
CAPÍTULO 5. LIMITES DE FUNÇÕES DEFINIDAS EM
INTERVALOS 49

L+ ε

f (x_n)
L− ε

x_n

x −δ x x +δ

Lembrando que o δ = δ(ǫ), pois depende de ǫ, obtivemos o que


querı́amos, já que |f (xn ) − L| < ǫ a partir de um certo tempo nδ(ǫ) .


Exemplos:

1)- f (x) = ax+b, polinômio de grau ≤ 1, tem limx→x f (x) = ax+b.


De fato, se a = 0 é claro que a f ≡ b constante tende a b. Caso
ǫ
a 6= 0, quando for dado ǫ > 0 tome por exemplo δ(ǫ) := |a| . Então se
ǫ
|x − x| < |a| temos:

ǫ
|f (x) − L| = |ax + b − (ax + b)| = |a||x − x| < |a| · = ǫ,
|a|

como querı́amos.
2)- No exemplo 1) o δ só dependeu do ǫ. Agora dou um exemplo
em que o δ depende também do x, ficando cada vez menor à medida
que o x vai sendo escolhido mais perto de um extremo do domı́nio da
f.
Seja f : R>0 → R, f (x) = x1 . Veremos na próxima Seção que
limx→x f (x) = x1 . Mas a Figura a seguir ilustra como vai ficando mais
difı́cl encontrar o δ adequado à medida que x > 0 se aproxima do 0.
3. LIMITES QUANDO X TENDE AO INFINITO 50

Figura: Para um mesmo ǫ, preciso cada vez menores valores de δ

3. Limites quando x tende ao infinito


Quando um cientista quer entender um fenômeno, ele pode querer
entender não apenas o comportamento agora, mas sim a longo prazo.
Por exemplo, pode se perguntar se a longo prazo a Lua permanecerá
girando em torno da Terra.
Na linguagem do Cálculo isso se expressa numa pergunta assim: a
que tende o fenômeno quando o tempo x fica arbitrariamente grande ?
O que se põe em sı́mbolos:
lim f (x) = L ∈ R, ou lim f (x) = L ∈ R.
x→+∞ x→−∞

Ambos sı́mbolos admitem dois tipos de definições (equivalentes)


Definição 3.1. Dizemos que
lim f (x) = L ∈ R
x→+∞

se ∀ǫ > 0 existe K > 0 tal que |f (x) − L| < ǫ, se x > K.


Ou
Definição 3.2. Dizemos que
lim f (x) = L ∈ R
x→+∞

se ∀(xn )n contida no domı́nio de f com limn→+∞ xn = +∞ temos


limn→+∞ f (xn ) = L.
(onde limn→+∞ xn = +∞ foi apresentado na Definição 3.2).

Deixo para o leitor verificar a equivalência dessas duas Definições


3.1 e 3.2.
Analogamente se define limx→−∞ f (x) = L ∈ R.
CAPÍTULO 5. LIMITES DE FUNÇÕES DEFINIDAS EM
INTERVALOS 51

Geometricamente, as Definições 3.1 ou 3.2 se ilustram na Figura a


seguir, em que o gráfico se aproxima da altura L cada vez mais:

0,98

0,96

0,94

0,92

50 100 150 200 250 300


x

Figura: Quando x aumenta o gráfico se aproxima de uma altura definida.

As propriedades básicas dessas noções são análogas àquelas do Teo-


rema 1.1:
Teorema 3.1. Sejam f e g funções definidas em um intervalo
ilimitado à direita.1 Suponha2
lim f (x) = L1 ∈ R e lim g(x) = L2 ∈ R.
x→+∞ x→+∞

Então:
1) A função soma f + g tem
lim (f + g)(x) = L1 + L2 .
x→+∞

2) A função diferença f − g tem


lim (f − g)(x) = L1 − L2 .
x→+∞

3) Se C ∈ R é uma constante, então a função (C · f )(x) := C · f (x)


tem
lim (C · f )(x) = C · L1
x→+∞
4 ) Suponha uma função q(x) com o mesmo domı́nio da f (x) tal
que |q(x)| ≤ K, ∀x. Suponha adicionalmente que L1 = 0. Então
lim ( f (x) · q(x) ) = 0.
x→+∞

5) A função produto (f · g)(x) tem


lim (f · g)(x) = L1 · L2 .
x→+∞

6) Se L2 = 6 0, então:
i) se x é suficientemente grande então g(x) 6= 0 e
f (x) L1
ii) limx→+∞ g(x)
= L2
.
1Enuncio apenas para x → +∞, pois é análogo se x → −∞
2
Atenção que L1 , L2 têm que ser números, não podem ser substituı́dos pelos
sı́mbolos +∞ ou −∞
3. LIMITES QUANDO X TENDE AO INFINITO 52

7) Suponha uma outra função q(x) definida no mesmo domı́nio e


que adicionalmente f (x) ≤ q(x) ≤ L1 . Então
lim q(x) = lim f (x) = L1 .
x→+∞ x→+∞

Demonstração.
Prova do item 1): Quero saber se a sequência soma f (xn ) + g(xn )
tende a L1 + L2 , se a sequência xn tem limn→+∞ xn = +∞. Mas
por hipótese f (xn ) tende a L1 e g(xn ) tende a L2 . Logo pelo item
1) do Teorema 3.1 aplicado às sequências f (xn ) e g(xn ) obtemos que
f (xn ) + g(xn ) tende a L1 + L2 .
Os outros itens se demonstram da mesma maneira. 

Exemplos:

1) Obviamente a função constante f ≡ C tem limx→+∞ C = C.


1
2) A função f : R<0 ∪ R>0 → R, f (x) = x
tem
1 1
lim = lim = 0.
x→+∞ x x→−∞ x
De fato, | x1 | < ǫ se |x| > K := 1ǫ , o que está de acordo com a Definição
3.1.

3)
C 1
lim = C · lim =C ·0=0
x→+∞ x x→+∞ x
usando o Teorema 3.1.
4) Também
1 1 1
lim 2
= lim ( · ) = 0 · 0,
x→+∞ x x→+∞ x x

pelo Teorema 3.1.

5)
1 1
lim (C + ) = C + lim =C +0=C
x→+∞ x x→+∞ x
usando o Teorema 3.1.

6)
C1 x C1
lim = ,
x→+∞ C2 x + C3 C2
onde C1 , C2 , C3 são constantes não nulas. De fato, primeiro observe que
se x se faz tão grande quanto quisermos, em particular x > 0. Logo
CAPÍTULO 5. LIMITES DE FUNÇÕES DEFINIDAS EM
INTERVALOS 53

posso escrever:
C1 x x C1 C1
lim = lim C3
= lim
x→+∞ C2 x + C3 x→+∞ x (C2 + x ) x→+∞ (C2 + Cx3 )
e agora uso o Teorema 3.1 e os Exemplos anteriores , concluindo que
C1 C1
lim C3
= .
x→+∞ (C2 + x ) C2

7) O mesmo tipo de argumento do Exemplo 6) dá que:


an xn + an−1 xn−1 + . . . + a0 an
lim n n−1
= ,
x→+∞ bn x + bn−1 x + . . . + b0 bn
onde ai , bi são constantes, an 6= 0, bn 6= 0.
De fato, como posso supor x > 0:
an xn + an−1 xn−1 + . . . + a0
lim =
x→+∞ bn xn + bn−1 xn−1 + . . . + b0
an−1 a0
xn · (an + x
+ ... + xn
)
= lim bn−1 b0
=
x→+∞ xn · (bn + x
+ ... + xn
)
an−1 a0
(an + x
+ ... + xn
) an
= lim bn−1 b0
= ,
x→+∞ (bn + x
+ ... + xn
) bn
usando novamente o Teorema 3.1 e Exemplos prévios.
Ilustro o Exemplo 7) nas Figura que segue, onde an = a2 = 2 e
bn = b2 = 1:

1,8

1,6

1,4

1,2

0,8

0,6

0 50 100 150 200


x

2x2 +x+4
Figura: Gráfico de x2 +3x+7
com x ∈ [0, 200].

8)
Se m < n, am 6= 0, bn 6= 0:
am xm + am−1 xm−1 + . . . + a0
lim = 0.
x→+∞ bn xn + bn−1 xn−1 + . . . + b0
3. LIMITES QUANDO X TENDE AO INFINITO 54

De fato,
am−1
xm · (am + x
+ . . . + xam0 )
lim =
x→+∞ xm · xn−m · (bn + bn−1
x
+ . . . + xb0n )
am−1
1 (am + x
+ . . . + xam0 ) am
= lim bn−1
=0· = 0,
x→+∞ xn−m (bn + x
+ . . . + xb0n ) bn
usando o Teorema 3.1.
Ilustro este Exemplo 8) na Figura a seguir, com am = a2 = 20 e
bn = b3 = 0.01. Escolhi o coeficiente b3 = 0.01 bem pequeno em relação
ao a2 = 20 de propósito, para indicar que não adianta, pois a longo
prazo o grau 3 do denominador é mais importante.

8000

6000

4000

2000

0
5 10 15 20 25 30
x

20x2 +30x+40
Figura: Gráfico de (0.01)x3
, para x ∈ [1, 30]

Estes dois Exemplos 7) e 8) ilustram o seguinte princı́pio: a longo


prazo o que importa são os graus mais altos dos polinômios envolvidos
num quociente de polinômios.

9) Lembrando apenas que a função seno tem | sin(x)| ≤ 1, então


sin(x)
=0lim
x→+∞ x
1
pois limx→+∞ x
= 0 (use o Teorema 3.1).

0,4

0,3

0,2

0,1

0
20 40 60 80 100 120
x
-0,1

-0,2

sin(x)
Figura: O gráfico de x
para x ∈ [2, 130]
CAPÍTULO 5. LIMITES DE FUNÇÕES DEFINIDAS EM
INTERVALOS 55

4. Quando a parte é do mesmo tamanho do todo

Nesta Seção proponho explicar o seguinte Teorema, que parece um


total absurdo:
Afirmação 4.1. A reta inteira de números Reais tem tantos pontos
quanto o intervalo aberto (−1, 1).

Em primeiro lugar preciso lembrar o que significa dois conjuntos


terem o mesmo número de elementos. O exemplo que mais gosto,
para explicar essa noção, li num um livro de Tarski.
Imagine num garçom colocando, para cada cliente, um garfo e uma
faca ao lado do prato. Ao final da tarefa, ele têm a seguinte conversa
com o cozinheiro:
• cozinheiro: para preparar a refeição, gostaria de saber quantos
clientes temos hoje.
• garçom: não contei, não sei.
• cozinheiro: mas você não estava pondo os garfos e facas para
cada um deles ?
• garçom: sim, mas só o que tenho certeza é que há tantos garfos
quanto facas à mesa.
• cozinheiro: mas como você pode ter certeza disso, sem saber
quantos garfos e facas você pôs, já que não contou ?
• garçom: ora, é fácil, sei que há tantos garfos quanto facas
porque para cada faca colocada, coloquei um garfo, e não mais
de um garfo.
A moral dessa história é a seguinte: dois conjuntos têm o mesmo
número de elementos quando há uma função f sobrejetora (nenhuma
faca sem garfo) e injetora (não mais de um garfo) entre eles. Apesar
de que não saibamos exatamente quantos elementos os conjuntos têm.

Um exemplo conhecido já por Galileu é que há tantos números


Naturais N quanto números Pares 2N: de fato, existe a bijeção
f : N → 2N, f (n) = 2n,
cuja inversa dá f −1 (2n) = n. Apesar disso 2N ⊂ N, por isso se diz que,
nesse caso, a parte é do tamanho do todo !

Para provar a Afirmação 4.1, considero a seguinte função:


x
f : R → R, f (x) := .
|x| + 1
Primeiro noto que está bem definida em todos os Reais, pois seu de-
nominador nunca se anula. Agora afirmo que f (R) ⊂ (−1, 1), ou seja,
4. QUANDO A PARTE É DO MESMO TAMANHO DO TODO 56

que
x
∀x ∈ R, −1 < < 1.
|x| + 1
De fato, primeiro f (0) = 0 e se x > 0 então |x| = x e portanto:
x
0< < 1,
x+1
pois 0 < x < x + 1. E se x < 0, então |x| = −x e portanto:
x
−1 < < 0,
−x + 1
pois −1 · (−x + 1) = x − 1 < x.
O que não está ainda nada claro é se f é sobrejetora, ou seja, se
(−1, 1) ⊂ f (R), ou seja f (R) = (−1, 1).
Estou assumindo neste momento, sem demonstrar, que a imagem
de f é algum intervalo f (R) = (a, b) ⊂ (−1, 1).
O que quero mostrar agora é que não acontece que −1 < a nem
que b < 1. Para isso meu argumento é o seguinte: vou mostrar que
x x
lim =1 e lim = −1,
x→+∞ | x | + 1 x→−∞ | x | + 1

ou seja, pela Definição de limite, que f atinge valores tão próximos de


1 e de −1 quanto quisermos. Isso impedirá que −1 < a e que b < 1.
Mas se x → +∞ então em particular x > 0 e
x x x·1
lim = lim = lim = 1,
x→+∞ | x | + 1 x→+∞ x + 1 x→+∞ x · (1 + 1 )
x
pelo Teorema 3.1 e Exemplos que o seguem.
E se x → −∞ então em particular x < 0 e
x x x·1
lim = lim = lim = −1,
x→−∞ | x | + 1 x→−∞ −x + 1 x→−∞ x · (−1 + 1 )
x
pelo Teorema 3.1 e Exemplos que o seguem.
Agora só falta ver que f é injetiva: mas note que se x > 0, de
x
y = x+1 obtenho y = x − xy e daı́:
y
x= ,
1−y
x
que é bem definido pois y < 1. E se x < 0 então de y = −x+1 obtenho
y = x + xy e daı́:
y
x= ,
1+y
que é bem definido pois −1 < y.
Isso mostra que y = f (x) é injetiva, já que tenho explicitamente
sua função inversa x = f −1 (y).

As Figuras a seguir mostram parte dos gráficos de f e de f −1 ,


respectivamente:
CAPÍTULO 5. LIMITES DE FUNÇÕES DEFINIDAS EM
INTERVALOS 57

0,8
0,4
0
-4 -2 0 2 4
-0,4
-0,8x

0
-0,8
-0,40 0,4
0,8
x

-2

-4

Para terminar, chamo a atenção do leitor que f −1 : (−1, 1) → R


faz uma espantosa expansão do intervalo (−1, 1). A expansão feita
por f −1 (y) depende sensivelmente de y e aumenta cada vez mais à
medida que y vai para os extremos do intervalo. Na Parte 2 do Curso
poderemos justificar e explicar melhor a seguinte Afirmação sobre f −1 :

Afirmação 4.2. Se y ∈ [0, 1) então a taxa de expansão de f −1 é


1 −1 1
de (1−y) 2 e a taxa de expansão de f (y) para y ∈ (−1, 0] é de (1+y) 2.

Uma comparação é natural: um dos fenômenos mais bizarros do


Universo é que não apenas ele se expande, e que quanto mais longe
mais ele se expande, mas também, como se descobriu faz pouco tempo,
que essa expansão está aumentando...

5. Exercı́cios
Exercı́cio 5.1. A seguir dado ǫ > 0 determine δ > 0 (em função
de ǫ) tal que |x − x0 | < δ implique |f (x) − L| < ǫ:
5. EXERCÍCIOS 58

a): x0 = 1, f (x) = 555x, L = 555,

b): x0 = 0, f (x) = x2 , L = 0,

c): x0 = 0, f (x) = 555x2 , L = 0.


Exercı́cio 5.2.
1

0,5

x
0 10 20 30 40 50
0

-0,5

-1

A figura mostra o gráfico da função f : R>0 → (−1, 1) dada por


x−1
f (x) = .
x+1
Prove aquilo que é sugerido pelo gráfico, ou seja, que
lim f (x) = −1 e lim f (x) = 1.
xց0 x→+∞

Exercı́cio 5.3. Determine:


2
a): limx→2 x +5x+6
x+2
,
1
b): limx→2 (x−2)2
,
−1
c): limx→−6 (x+6)2
,
−1
d): limxր−6 x+6
,
−1
e): limxց−6 x+6
.
Exercı́cio 5.4. Considere os seguintes limites
x3 − 3x + 2 x3 − 3x + 2
lim e lim .
x→1 x−1 x→1 (x − 1)2
i) Antes de fazer contas, diga qual a diferença qualitativa que há
entre os dois casos.
ii) Calcule os limites.
iii) será que existe o
x3 − 3x + 2
lim ?
x→1 (x − 1)3
CAPÍTULO 5. LIMITES DE FUNÇÕES DEFINIDAS EM
INTERVALOS 59

Exercı́cio 5.5. Calcule


x3 − 2x2 − 4x + 8 x3 − 2x2 − 4x + 8
lim e lim .
x→1 x−2 x→1 (x − 2)2
Exercı́cio 5.6. i) Considere a função f : R → R definida por
partes:
f (x) = −x, se x < −1,
f (x) = x2 + x + 1, se − 1 ≤ x ≤ 1,
f (x) = 2 · x, se 1 < x.
Existem os limites lim f (x) ou lim f (x)?
x→−1 x→1

ii) Ajuste os parâmetros b, c para que g : R → R definida por partes:


g(x) = −x, se x < −1,
2
g(x) = x + b · x + c, se − 1 ≤ x ≤ 1,
g(x) = 2 · x, se 1 < x.
tenha ambos os limites lim g(x) e lim g(x)
x→−1 x→1
CAPı́TULO 6

A noção de Continuidade

Na Definição a seguir pediremos um pouco mais que o que foi


exigido na Definição 0.1, pois vamos pedir que:
• x ∈ I (domı́nio da função) e que
• limx→x f (x) = f (x)
ou seja que o limite L da função coincida com f (x):
Definição 0.1. Uma função f : I → R é contı́nua em x ∈ I se
toda sequência xn de pontos de seu domı́nio com
lim xn = x
n→+∞

tenha também
lim f (xn ) = f (x).
n→+∞
Quando dissermos apenas que f é contı́nua estamos querendo dizer f
que é contı́nua em cada ponto de seu Domı́nio.

Observações:
• Quer dizer então que, se uma função é contı́nua em x, é porque
ela manda todas sequências contidas no Domı́nio I de f que
se aproximam de x em sequências no Contra-Domı́nio que se
aproximam de f (x).
• Concluı́mos que, para não termos a continuidade de f em
x ∈ I, tem que haver pelo menos uma sequência xn de pon-
tos de seu domı́nio com limn→+∞ xn = x, mas para as qual
limn→+∞ f (xn ) 6= f (x) .
Isso pode acontece ou porque simplesmente não existe esse
limite ou, mesmo existindo, pode ser que seja diferente de valor
esperado f (x).
• Só faz sentido dizer que f é descontı́nua (não-contı́nua) em
pontos x de seu Domı́nio1

Exemplos de descontinuidades:
1- f : R → R definida condicionalmente por: f (x) = x se x ≤ 0
e por x + 4 se x > 0. Nesse exemplo, sequências xn < 0 que tendem
1Ao contrário do que faz o Anton em seu livro de Cálculo, para quem f :
R \ {0} → R é descontı́nua em x = 0 !!!
61
1. OPERAÇÕES COM FUNÇÕES CONTÍNUAS 62

a zero tem f (xn ) tendendo a 0; mas sequências xn > 0 que tendem a


zero tem f (xn ) tendendo a 4.
2- f : [0, 5] → R, definida condicionalmente por f (0) = 3 e f (x) =
1/x, se x ∈ (0, 5]. Aqui, sequências de números positivos xn que tendam
a 0 tem f (xn ) ficando tão grande quanto quisermos, ou seja se afastando
de f (0) := 3.
3- f : [0, π1 ] → R, f (0) = 0 e f (x) = sen(1/x), se x ∈ (0, π1 ] (aqui
apelo apenas para o conhecimento de base, de que seno é uma função
periódica, que tem valores em [−1, 1] e que se anula em π). Aqui se
tomamos xn > 0 conveniente tendendo a 0, podemos conseguir f (xn )
tendendo para qualquer Lxn ∈ [−1, 1].

0,5

x
0,05 0,1 0,15 0,2 0,25 0,3
0

-0,5

-1

Figura: O gráfico de f (0) = 0 e f (x) = sin( x1 ) se x ∈ (0, π1 ].

1. Operações com funções contı́nuas


O próximo Teorema simplesmente re-escreve alguns itens do Teo-
rema 1.1, no caso em em x está no domı́nio de ambas as funções e em
que L1 = f (x) e L2 = g(x).
Teorema 1.1. (Propriedades das funções contı́nuas) Suponha que
f e g ambas são contı́nuas em x, ou seja:
lim f (x) = f (x) e lim g(x) = g(x).
x→x x→x

Então:
1) A função soma f + g é também contı́nua em X ou seja
lim (f + g)(x) = (f + g)(x).
x→x

2) A função diferença f − g é também contı́nua em X ou seja


lim (f − g)(x) = (f − g)(x).
x→x

3) Se C ∈ R é uma constante, então a função (C · f )(c) := C · f (x)


é contı́nua, ou seja:
lim (C · f )(x) = C · f (x)
x→x
CAPÍTULO 6. A NOÇÃO DE CONTINUIDADE 63

4) A função produto (f · g)(x) tem


lim (f · g)(x) = (f · g)(x).
x→x

5) Se g(x) 6= 0:
• i) se x é suficientemente próximo de x, então g(x) 6= 0 e
• ii) lim fg(x)
(x)
= fg(x)
(x)
.

A Afirmação 3.1 e a definição de função contı́nua implicam:


Afirmação 1.1. (Princı́pio de Inércia das funções contı́nuas) Seja
f : I → R contı́nua em x, definida num intervalo aberto I.
• se f (x) > 0 então f (x) > 0 num intervalo aberto centrado em
x.
• se f (x) > 0 então f (x) > 0 num intervalo aberto centrado em
x.
Deixo a prova como um exercı́cio para o leitor, se bem que a figura
a seguir diz quase tudo:

L+ ε

L>0

L−ε

x
x −δ x +δ

Figura: f é contı́nua e positiva m x.

O Teorema a seguir é enunciado para a composição de 2 funções,


mas pode ser adaptado facilmente para qualquer número (finito) de
composições de funções.
Teorema 1.2. Seja g : I → J e f : J → K funções de intervalos
em intervalos. Suponha que g é contı́nua em x e que f é contı́nua em
g(x). Então (f ◦ g)(x) := f (g(x)) é contı́nua em x.
Demonstração.
Queremos saber se para qualquer sequência (xn )n com xn ∈ I que
tende a x temos que a sequencia f (g(xn )) ∈ K tende para f (g(x)).
O que sabemos pelas hipóteses sobre f e sobre g é, primeiro, que
se xn ∈ I tende a x então g(xn ) ∈ J tende a g(x).
Mas agora, se olhamos z := g(x), vemos zn := g(xn ) é um sequência
que tende a ele. Pela hipótese de continuidade sobre f , f manda qual-
quer sequência que tenda a g(x) em uma sequência f (zn ) = f ( g(xn ) )
que tende a f (z) = f (g(x)), exatamente o que querı́amos.
2. POLINÔMIOS, FUNÇÕES RACIONAIS E
TRIGONOMÉTRICAS 64

2. Polinômios, funções racionais e trigonométricas


2.1. Polinômios.
Não imagino um exemplo mais simples de função contı́nua que a
função constante : f (x) ≡ C, C ∈ R. É claro que limx→x f (x) = C,
pois f (x) = C simplesmente não depende de x ou de x particulares.
Outro exemplo que é contı́nua é a função identidade f (x) = x, pois
obviamente
lim f (x) = lim x = x.
x→x x→x

Uma consequência do Teorema 1.1 é que os polinômios:


f (x) := an · xn + an−1 · xn−1 + . . . + a1 · x + a0 , onde ai ∈ R
são funções contı́nuas. De fato, para um polinômio usamos um número
finito de vezes os itens 1), 2) , 3) e 4).

2.2. Funções racionais.


O item 5) do Teorema 1.1 diz então que a função F : R \ {0} :→ R,
F (x) = x1 é contı́nua, pois numerador e denominador são contı́nuos.
Isso é um pouco chocante, pelo aspecto do gráfico dessa, formado de
duas partes. Se lê em alguns livros que uma função contı́nua não tem
rasgos no seu gráfico, mas o correto é dizer que uma função contı́nua
não introduz rasgos. Se o próprio domı́nio dela já é formado como neste
exemplo de dois pedaços como o de x1 ,
R \ {0} = R>0 ∪ R<0
então o gráfico pode ter dois pedaços, só não poder ter mais de dois
pedaços.
O que sempre ficaria descontı́nua é qualquer tentativa de estender
f (x) = x1 ao ponto x = 0, pois se aproximando x pela direita 1/x > 0
fica tão positivo quisermos e aproximando x pela esquerda 1/x < 0 fica
tão negativo quanto quisermos.

Generalizando o exemplo x1 , defino uma função racional como o


quociente PP21 (x)
(x)
de dois polinômios. Resta saber, se adotamos esta
definição, onde a função racional está bem definida como função.
Vale o seguinte: se P1 (x) e P2 (x) não têm raı́zes comuns, então PP21 (x)
(x)

tem como Domı́nio exatamente o conjunto


{ x ; P2 (x) 6= 0 }.
P1 (x)
E P2 (x)
é uma função contı́nua.
CAPÍTULO 6. A NOÇÃO DE CONTINUIDADE 65

Porém, suponha que P1 (x) e P2 (x) têm alguma raı́z comum x, que
é de ordem m1 ≥ 1 para P1 (x) e de ordem m2 ≥ 1 para P2 (x). Então
P1 (x)
P2 (x)
estará definida em x se e somente se
m1 ≥ m2 .
Relembro essas noção de ordem ou multiplicidade de uma raı́z:
Definição 2.1. Seja f (x) polinômio a coeficientes Reais.
Dizemos que x é raı́z de ordem exatamente m, se
f (x) = (x − x)m · g(x), m ∈ N,
para um g(x) polinômio a coeficientes Reais que não se anula em x.
2.3. Trigonométricas.
Considere agora um cı́rculo de raio 1.
Podemos usar o comprimento do arco do cı́rculo (medido no sentido
antihorário desde o eixo x > 0) como uma medida do ângulo central.
Assim um ângulo de 360 graus (antihorário, desde o eixo x > 0))
mede +2π (onde π é tomado no sentido elementar de quociente entre
o perı́metro e diâmetro de um cı́rculo). Um ângulo de 90 graus an-
tihorário mede +π/2, o de 180 antihorário mede +π. É claro que há
sempre uma ambiguidade de k · 2π nesse modo como medimos o ângulo
central.
A medida da projeção no eixo y (orientada como o eixo y) do arco
de comprimento θ é o seno do ângulo θ. Assim como a medida da
projeção no eixo x (orientada como o eixo x) do arco de comprimento
θ é o cosseno do ângulo θ.

tan θ
senθ
θ

1 cos θ

Figura: Definição elementar de seno e cosseno

Seno e cosseno naturalmente são periódicos de perı́odo 2π, devido


à ambiguidade na medida do ângulo.
Agora vamos usar a intuição que temos de que, se variamos um
pouquinho o arco θ para θ + h, então as duas projeções vertical e
horizontal mudam pouco (as projeções são funções contı́nuas).
Ou seja, Afirmamos que seno e cosseno são funções contı́nuas por
serem definidas a partir de projeções.
2. POLINÔMIOS, FUNÇÕES RACIONAIS E
TRIGONOMÉTRICAS 66

Lembro que seno retrito a [ −π , π ] é uma função estritamente cres-


2 2
cente; sua função inversa chamada de arcoseno (pois diz de que arco o
número dado é um seno) também é estritamente crescente.
Isso vale em geral:

Se uma função y = f (x) é estritamente crescente, sua inversa x =


−1
f (y) também é.
De fato, se por absurdo ocorresse que y 1 < y 2 mas f −1 (y 1 ) ≥
f −1 (y 2 ) então terı́amos x1 = f −1 (f (x1 )) ≥ f −1 (f (x2 )) = x2 con-
tradizendo que y = f (x) é estritamente crescente.
sin(x)
Pelo item 5) do Teorema 1.1, a função cos(x) é contı́nua nos pontos
onde cos(x) 6= 0, ou seja para x 6= π/2 + k · π, k ∈ Z. Essa função é
por definição a função tangente

sin(x)
tan(x) := .
cos(x)

Será importante mais adiante, quando falarmos dos coeficientes angu-


lares de retas.
A periodicidade do seno do cosseno repercute na função tangente,
que é periódica de perı́odo π. Seu domı́nio é uma união de infinitos
intervalos de comprimento π:

−π π −π π −π π
... ∪ ( − π, − π) ∪ ( , )∪( + π, + π) ∪ . . .
2 2 2 2 2 2

e não é difı́cil de ver que quando restrita a cada intervalo ela é uma
função:

• i) estritamente crescente e
• ii) que fica em módulo tão grande quanto quisermos se nos
aproximamos suficentemente dos extremos

sin(θ)
pois o denominador cos(θ) de cos(θ) se aproxima de zero enquanto o
numerador sin(θ) se aproxima de 1 ou de −1.
CAPÍTULO 6. A NOÇÃO DE CONTINUIDADE 67

0
-1-0,5
0 0,51
x

-2

-4

Figura: Gráfico feito no computador de tan restrita a ( −π


2
+ 0.2, π2 − 0.2)

Nessa Figura, feita numericamente no computador, não pude pedir


para o computador trabalhar no intervalo ( −π , π ), pois os valores de
2 2
tan explodem em módulo.
A restrição
−π π
tan : ( , )→R
2 2
tem uma inversa arctan : R → ( −π , π ). Também é uma função es-
2 2
tritamente crescente, como já explicamos acima, mas seus valores não
sobrepassam em módulo a π2 .

1
0,5
0
-4 -2 -0,5 0 2 4
-1x

Figura: Gráfico de arctan(x)


3. CONTINUIDADE DA FUNÇÃO INVERSA 68

Podemos expressar o comportamento de arctan(x) usando a notação


da Seção 3:

π
lim arctan(x) =
x→+∞ 2
para dizer que arctan(x) fica tão próximo quanto quisermos
de π2 se deixarmos x crescer o suficiente;

π
lim arctan(x) = −
x→−∞ 2
para dizer que arctan(x) fica tão próximo quanto quisermos
de − π2 se deixarmos x decrescer o suficiente;
E podemos introduzir novos sı́mbolos para comparar com o com-
portamento de tan(x):

lim tan(θ) = −∞
θց− π2

significa que tan(θ) fica tão negativo quanto quisermos desde


que θ > − π2 decresça e se aproxime o suficiente de − π2 .

lim tan(θ) = ∞
θր π2

significa que tan(θ) fica tão positivo quanto quisermos desde


que θ < π2 cresça e se aproxime o suficiente de π2 .

3. Continuidade da função inversa


É possı́vel provar (mas a prova é um pouco técnica demais) que:
Afirmação 3.1. Se f : I → R, y = f (x) definida num intervalo I
é contı́nua e tem inversa, então f −1 : f (I) → I também está definida
num intervalo f (I) e f −1 também é contı́nua.
Chamo a atenção que essa Afirmação pode ser falsa se o domı́nio
da f não é um intervalo2
Para ver um exemplo disso, considere uma f definida numa união
de intervalos: [0, a] ∪ (a + 1, b], que seja contı́nua e que tenha inversa.
Note que a continuidade em x = a só se refere ao comportamento a f
em relação a sequências xn ∈ [0, a] que tendam a x = a. As sequências
xn ∈ (a + 1, b] do domı́nio da f não tendem ao ponto a, pois distam
dele pelo menos 1, então não interessam na análise da continuidade da
f em a. O gráfico que segue é um exemplo de uma tal f :

2Como esqueceu o Anton, na pag. 156, Teorema 2.6.2, da Oitava Edição do seu
livro de Cálculo.
CAPÍTULO 6. A NOÇÃO DE CONTINUIDADE 69

y = f(x)

0 a a+1 b

Figura: f : [0, a] ∪ (a + 1, b] → R contı́nua,


com x = f −1 (y) descontı́nua em f (a)

Agora Afirmo que a função inversa x = f −1 (y) é descontı́nua em


y = f (a). De fato, se yn < f (a) é uma sequência de pontos da imagem
da f que tende a f (a) vemos na Figura que limn→+∞ f −1 (yn ) = a.
Mas se tomamos yn > f (a) uma sequência de pontos da imagem da f
que tende a f (a), vemos que limn→+∞ f −1 (yn ) = a + 1.
A Figura a seguir ilustra:

y = f^{−1} (x)

y = f(x)

0 a a+1 b

Figura: Aqui y = f (x) e y = f −1 (x) estão no mesmo sistema cartesiano

4. Dois teoremas fundamentais sobre funções contı́nuas


A demonstração dos dois Teorema a seguir foge do conteúdo
usual do Cálculo, é visto em disciplinas mais avançadas de Análise
Matemática.
É importante que o estudante medite sobre seus enunciados.

Teorema 4.1. (Teorema do Valor Intermediário - abrev.: T.V.I.)


Seja f : [a, b] → R função contı́nua com A = f (a) e B = f (b), com
A 6= B, por exemplo A < B.
Seja C qualquer número C ∈ (A, B). Então existe algum x ∈ (a, b)
tal que f (x) = C (pode haver mais de um x desse tipo)
Teorema 4.2. (Teorema de Bolzano-Weierstrass)
6. RAÍZES DE POLINÔMIOS CUJO GRAU É ÍMPAR 70

Seja f [a, b] → R contı́nua, onde [a, b] é intervalo fechado e limitado.


Então f tem mı́nimo e máximo globais assumidos em pontos de [a, b]

5. Primeiras aplicações do T.V.I

Vamos dar agora algumas aplicações iniciais do T.V.I. Mais tarde


ele será importante na prova do Teorema Fundamental do Cálculo, na
Parte 2 do Curso.
Primeiro um tı́pico teorema bem geral, mas que não diz nada sobre
a solução em cada caso especı́fico:
Proposição 5.1. Dado qualquer f : [0, 1] → [0, 1] contı́nua, existe
x ∈ [0, 1] tal que f (x) = x.
Demonstração.
Observe que geometricamente o que queremos é saber se o gráfico
de y = f (x) corta o gráfico da diagonal y = x.
Se f (0) = 0 ou se f (1) = 1 então corta e acabou, não há nada mais
a provar. Portanto vamos supor que f (0) ∈ (0, 1] e que f (1) ∈ [0, 1),
para termos algo a provar.
É razoável olhar a função diferença entre elas: f (x) − x. Por ser
uma diferença de duas funções contı́nuas, f (x) − x também é função
contı́nua. Ademais, f (0) ∈ (0, 1] e f (1) ∈ [0, 1) dizem que:
f (0) − 0 > 0 e f (1) − 1 < 0.
Pelo T.V.I. existe algum x ∈ (0, 1) tal que:
f (x) − x = 0,
como querı́amos. 

6. Raı́zes de polinômios cujo grau é ı́mpar


A segunda aplicação do T.V.I.:
Proposição 6.1. Todo polinômio de coeficientes Reais e de grau
ı́mpar tem algum zero Real: f (x) = 0.
Observe que há polinômios de grau par sem zeros Reais, como
f (x) = x2 + 1.
Demonstração. Seja f o polinômio de grau 2n − 1:
f (x) := a2n−1 · x2n−1 + a2n−2 · x2n−2 + . . . + a1 · x + a0 , ai ∈ R, n∈N
Caso a2n+1 > 0:
Escrevo para x > 0:
a2n−2 a0
a2n−1 ·x2n−1 +a2n−2 ·x2n−2 +. . .+a1 ·x+a0 = a2n−1 x2n−1 ·(1+ +. . . 2n−1 ).
x x
CAPÍTULO 6. A NOÇÃO DE CONTINUIDADE 71

Pelo Teorema 3.1 e pelos Exemplos que o seguem, temos que


a2n−2 a0
lim ( + . . . 2n−1 ) = 0.
x→+∞ x x
Portanto para x > 0 suficientemente grande temos que
a2n−2 a0
1+ + . . . 2n−1 > 0.
x x
Logo, para x > 0 suficientemente grande, o sinal de
a2n−2 a0
a2n−1 x2n−1 · (1 + + . . . 2n−1 )
x x
2n−1 2n−1
é o mesmo sinal de a2n−1 x , que é a2n−1 x > 0.
Argumentando do mesmo jeito para x → −∞, concluimos que o
sinal de
a2n−2 a0
a2n−1 x2n−1 · (1 + + . . . 2n−1 )
x x
para x < 0 suficientemente grande é o mesmo sinal de a2n−1 x2n−1 , que
nesses pontos é a2n−1 x2n−1 < 0.
Então
f (x) = a2n−1 · x2n−1 + a2n−2 · x2n−2 + . . . + a1 · x + a0
assumiu valores negativos e positivos.
Pelo T.V.I. e pela continuidade do polinômio f (x), tem que haver
um ponto onde f (x) = 0.
Caso a2n+1 < 0: completamente análogo.


Esse teorema (e sua prova) não dão nenhuma pista de como achar
concretamente algum ponto x onde f (x) = 0.
Em dois trabalhos, de 1690 e 1691, Michel Rolle tentou estabelecer
um método para determinar concretamente esses zeros.
Ele o fez de um modo bem confuso, pois não tinha uma boa definição
de Derivada, mas seu nome ficou associado ao teorema que estabele-
ceremos mais adiante no Capı́tulo 10 e que nos permitirá criar métodos
para encontrar raı́zes de polinômios (e de funções mais gerais).

7. Raı́zes simples e fatoração de polinômios


Acho que pode ser útil na formção dos estudantes, ter uma prova
do seguinte fato fundamental:
Teorema 7.1. Seja f (x) = an xn +an−1 xn−1 +. . .+a0 um polinômio
de grau n, com coeficientes ai ∈ R.
São equivalentes:
• i) f (x) = 0 para alguma raı́z x ∈ R e
7. RAÍZES SIMPLES E FATORAÇÃO DE POLINÔMIOS 72

• ii) f (x) = (x − x) · g(x) onde g(x) é um polinômio de grau


n − 1 com coeficientes Reais.

Demonstração.
ii) obviamente implica i), pois:
f (x) = (x − x) · g(x) = 0.
A prova de que i) implica ii) será dividida em duas etapas.
A parte interessante é construir o g(x) que queremos em:
f (x) = (x − x) · g(x) + r,
onde r é uma constante.
Se tivermos feito isso, avaliaremos tudo em x:
0 = f (x) = (x − x) · g(x) + r = r,
para concluir que r = 0.
Para chegarmos na desejada expressão f (x) = (x − x) · g(x) + r,
temos um algoritmo a executar.
Para f (x) = an xn + an−1 xn−1 + . . . + a0 , faço
g1 (x) := an · xn−1
e subtraio
r1 (x) := f (x) − (x − x) · g1 (x).
O g1 (x) foi escolhido para que r1 (x) não tenha termo de grau n. Ou
seja que esse novo polinômio r1 (x) tem grau ≤ n − 1. Se por acaso
r1 (x) ≡ 0 então
f (x) = (x − x) · g1 (x)
e já temos o que queremos, com r = 0 e g(x) := g1 (x).
Caso contrário r1 (x) = bk xk + bk−1 xk−1 + . . ., onde k ≤ n − 1; defino
xk−1
g2 (x) := ,
bk
e subtraio
r2 (x) := r1 (x) − (x − x) · g2 (x).
Pela definição do g2 (x) esse novo polinômio r2 (x) tem grau ≤ n − 2.
Se dermos sorte e r2 (x) ≡ 0 então
f (x) = (x − x) · [g1 (x) + g2 (x)],
e já temos o que queremos com r = 0 e g(x) = g1 (x) + g2 (x).
Caso contrário continuamos, considerando agora r2 (x) = cj xj +
cj−1 xj−1 + . . ., onde j ≤ n − 2 e definindo g3 (x) e r3 (x) como fizemos
antes.
O que importa é que o grau desse novo r3 (x) será ≤ n − 3. Ou seja,
como vão caindo os graus dos rk (x) a cada etapa, após no máximo n
etapas chegaremos a um rk (x) (k ≤ n) que ou bem é ≡ 0 ou bem tem
CAPÍTULO 6. A NOÇÃO DE CONTINUIDADE 73

grau zero, uma constante. Esse será o r. E g(x) := g1 (x) + . . . + gk (x),


k ≤ n. 

Digressão sobre o Teorema 7.1:


Se observarmos a prova desse Teorema vemos que, na fatoração
f (x) = (x − x) · g(x)
os coeficientes do polinômio g(x) são soma, subtrações, produtos, quo-
cientes da raı́z x e dos coeficientes ai de f (x).
Por isso, se a raı́z x fossse um número Complexo e a1 são Reais ou
Complexos, deveria haver uma fatoração de f onde o polinômio g(x)
tivesse coeficientes Complexos.
Por exemplo, temos
x3 − 1 = (x − 1) · (x2 + x + 1)
e isso é tudo que podemos fazer se estamos limitados a trabalhar com
coeficientes Reais.
Mas x2 + x + 1 tem raı́zes Complexas:
√ √ √ √
−1 − −1 3 −1 + −1 3
x1 := e x2 := ,
2 2
ous seja, as raı́zes Reais ou Complexas de x3 − 1 = 0 são 1, x1 , x2 .
Portanto deveria haver uma fatoração:
x3 − 1 = (x − x1 ) · g(x),
com os coeficientes desse novo g(x) nos Complexos.
Seguindo os passos do algoritmo dado na prova do Teorema 7.1
(com a mesma notação), faço:
g1 (x) := x2
r1 := x3 − 1 − x2 · (x − x1 ) =
= x1 x2 − 1.
Agora
g2 (x) := x1 x,
r2 := r1 − x1 x · (x − x1 ) =
= x21 x − 1.
E também
g3 (x) := x21 ,
r3 := r2 − x21 · (x − x1 ) =
= −1 + x31 = 0.
Portanto
g(x) := g1 (x) + g2 (x) + g3 (x) =
= x2 + x1 x + x21 ,
8. EXERCÍCIOS 74

e a fatoração é
√ √
3 2 −1 − −1 3
x − 1 = (x − x1 ) · ( x + x1 x + x21 ), onde x1 := .
2
Note que:
(x − 1) · (x − x2 ) = x2 − (x2 + 1) x + x2 =
= x2 + x1 x + x21 ,
pois claramente
x2 + 1 = −x1 ,
e
x21 = x2 .

8. Exercı́cios
Exercı́cio 8.1. Considere a função definida assim: f (x) = 0 se x
é um número racional e f (x) = 1 se x é um número irracional.

i): Como é seu gráfico ?


ii): em que pontos ela é contı́nua ou é descontı́nua?
Exercı́cio 8.2. A soma, o produto e a composição de funções
contı́nuas produz funções contı́nuas. Usando isso calcule:
i) lim (3x − 4x) · (x5 − 2x)4 ,
x→1

ii) lim 4x − 3x · (x5 − 2x)4 .
x→1

Exercı́cio 8.3. Dê um exemplo de f (x) descontı́nua em algum


ponto mas tal que f 2 (x) é contı́nua em todos os pontos.
Exercı́cio 8.4. (resolvido)
Prove que a função definida por f (x) = x · sin( x1 ), se x > 0 e
f (0) = 0 é contı́nua.
Exercı́cio 8.5. Prove a Afirmação 1.1, que chamei de princı́pio de
inércia das funções contı́nuas.
Exercı́cio 8.6. Um aluno me disse que, para descobrir em quais
intervalos um polinômio y = f (x) de grau n é positivo ou negativo, ele
faz o seguinte.
Ele primeiro descobre todas as raı́zes Reais x1 , x2 , . . . , xk , onde k ≤
n.
Depois considera os intervalos (−∞, x1 ), (x1 , x2 ), etc , (xk−1 , xk ),
(xk , +∞). Então para saber o sinal de f em cada intervalo desses, ele
examina o sinal de f (x) em um único x de cada intervalo.
O método dele está correto ? Se está, justifique-o com conceitos/
teoremas do Cálculo.
CAPÍTULO 6. A NOÇÃO DE CONTINUIDADE 75

Exercı́cio 8.7. Dê um exemplo de uma função f positiva em um


ponto x, mas tal que f (xn ) = 0 em pontos xn que formam um sequência
com limn→+∞ xn = x.

Exercı́cio 8.8. Encontre o domı́nio da função racional f (x) =


1
x2 −1
.Descreva o que acontece com o módulo e o sinal de f quando x
se aproxima pela esquerda e pela direita dos pontos onde ela não está
definida.

Exercı́cio 8.9. (resolvido)


Prove que


5 · x2 + x √
lim = 5
x→+∞ x+2

2,2

1,8

1,6

1,4

1,2

0,8
20 40 60 80 100
x


5·x2 +x

Figura: Gráfico de y = x+2
, x ∈ [1, 100], 5 ≈ 2.23.

Exercı́cio 8.10. Um exemplo que não parece estar ligado a quo-


cientes, mas que se calcula introduzindo quocientes:

√ 1
lim ( x2 + x − x ) = .
x→+∞ 2
8. EXERCÍCIOS 76

0,5

0,48

0,46

0,44

0,42

20 40 60 80 100
x

Figura: Gráfico de y = x2 + x − x, x ∈ [1, 100].

Exercı́cio 8.11. É um fato que o polinômio


y = x5 − 2x4 + x3 + x2 + 1
só tem uma raı́z Real. Não é muito fácil achá-la explicitamente...
Mas com o Teorema do Valor Intermediário você pode concluir que
a raı́z Real é um ponto do intervalo [−1, 1]. Por quê ?
CAPı́TULO 7

Geometria Analı́tica Plana

1. Equações de retas, coeficientes angular e linear


A equação de uma reta vertical por dois pontos (x, y1 ) e (x, y 2 ) é
x − x = 0.
Mas a equação de uma reta não-vertical por (x1 , y 1 ) e (x2 , y 2 ) é do
tipo:
y = a1 · x + a0 , a1 , a0 ∈ R.
Ou seja, sua equação é um tipo bem simples de polinômio, cujo grau
em x é ≤ 1.
Vamos usar uma notação mais habitual:
y = a · x + b, a, b ∈ R.
Afirmação 1.1. Os coeficientes a, b da equação y = ax + b da reta
passando pelos dois pontos (x1 , y 1 ) e (x2 , y 2 ) com x1 6= x2 são dados
por:
y − y1
a= 2 ,
x2 − x1
e
b = y 1 − a · x1 = y 2 − a · x2 .

Demonstração. De
y 1 = a · x1 + b e y 2 = a · x2 + b,
subtraindo-as, obtemos:
y 2 − y 1 = a · (x2 − x1 ),
de onde
y2 − y1
a= ,
x2 − x1
(onde é crucial que x2 6= x1 ). E daı́ sai que:
y − y1
b = y1 − ( 2 ) · x1 ,
x2 − x1
ou o que dá no mesmo:
y2 − y1
b = y2 − ( ) · x2 .
x2 − x1
77
1. EQUAÇÕES DE RETAS, COEFICIENTES ANGULAR E
LINEAR 78

Note que esse número b é a altura em que a reta y = ax+b intersecta


o eixo dos y, que é dado por x = 0: de fato,
y = a · 0 + b = b.
Definição 1.1. Dados dois pontos distintos do plano (x1 , y 1 ) e
(x2 , y 2 ) com coordenadas x1 6= x2 , definimos o coeficiente angular da
reta ligando esses dois pontos por:
y2 − y1 y − y2
= 1 .
x2 − x1 x1 − x2
Afirmação 1.2. O coeficiente angular é uma informação da reta,
não dependendo dos pontos particulares que usamos para calculá-lo.
Demonstração.
De fato, se tomo qualquer ponto (x3 , y 3 ) da reta y = a · x + b
determinada por (x1 , y 1 ) e (x2 , y 2 ), como y 3 = ax3 + b, então:
y3 − y1 (a · x3 + b) − (ax1 + b)
= = a,
x3 − x 1 x3 − x1
e já vimos na Afirmação 1.1 que
y − y1
a= 2 ,
x2 − x1
ou seja,
y3 − y1 y − y1
= 2 .
x3 − x1 x2 − x1


Como consequência temos a seguinte observação útil para o Curso:


Afirmação 1.3. Dado um ponto (x1 , y 1 ) e um coeficiente angular
pré-estabelecido valendo a, então a única reta que passa por (x1 , y 1 ) e
tem esse coeficiente angular é dada por
y = a · x + (y 1 − a · x1 ).
Demonstração. De fato, tomando um ponto (x, y) genérico dessa
reta, então pela Afirmação 1.2
y − y1
= a,
x − x1
o que dá, isolando-se y:
y = a · x + (y 1 − a · x1 ).

CAPÍTULO 7. GEOMETRIA ANALÍTICA PLANA 79

Exemplos:
1)- a diagonal y = x tem coeficente angular 1 e a anti-diagonal
y = −x tem coeficiente angular −1.
2)- A reta horizontal y = b tem coeficiente angular 0, pois y = b =
0 · x + b.

Observações:
• Se x1 = x2 então a reta que liga (x1 , y 1 ) e (x2 , y 2 ) é vertical e
não tem um coeficiente angular definido.
Temos a tentação de dizer que o coeficiente angular da reta
vertical é +∞. Mas se começamos com a anti-diagonal e a va-
mos levantando, os coeficientes angulares ficam cada vez mais
negativos e ao atingir a posição vertical ficariam −∞: essa
ambiguidade entre +∞ e −∞ para o candidato a coeficiente
angular da reta vertical é que faz que seja melhor desistirmos
de atribuir um coeficiente angular à reta vertical.
• Geometricamente o coeficiente angular a representa o quo-
ciente entre o cateto oposto y 2 −y 1 e o cateto adjacente x2 −x1
do triângulo retângulo formado pelos pontos (x1 , y 1 ), (x2 , y 1 )
e (x2 , y 2 ): logo a = tan(α) ( tangente do ângulo (anti-horário)
α formado pela reta e o eixo horizontal). Vimos na Seção ??
que se um ângulo que tende a +π 2
sua tangente tende a +∞,
enquanto que, se o angulo tende a −π 2
, sua tangente tende a
−∞.
• Se fixamos a e variamos b em y = a · x + b estamos descrevendo
uma famı́lia de retas paralelas com a mesma inclinação.

2. Ortogonalidade

Deve estar claro pelo que já explicamos que duas retas y = ax + b1
e y = ax + b2 , com b2 6= b1 , são de fato paralelas.
Agora gostaria de explicar que uma par de retas y = ax + b1 e
y = − a1 x + b2 , com a 6= 0, são ortogonais.
Posso me restringir a considerar retas pela origem: y = ax e
y = − a1 x, pois estas são translações verticais das retas anteriores, e
portanto têm entre elas o mesmo ângulo que as anteriores. Posso su-
por também que a > 0 (caso a < 0 então − a1 > 0 e poderia trabalhar
com este coeficiente angular).
Se escrevo a = B A
, com A, B > 0, então − a1 = − BA
.
Agora considero 3 triângulos (ilustrados na Figura a seguir):
• ∆1 dados pelos pontos (0, 0), (A, 0) e (A, B) e
• ∆2 dado pelos pontos (0, 0), (−B, 0) e (−B, A).
• ∆3 dado pelos pontos (0, 0), (A, B) e (−B, A).
3. TEOREMA DE TALES NO CÍRCULO 80

( A,B )

(−B , A )
∆3
∆1
∆2

(−B , 0) (0, 0) ( A, 0 ) x

Observe que ∆1 e ∆2 são triângulos retângulos e que a reta que


contém a hipotenusa de ∆1 é y = ax , enquanto que a reta que contém a
hipotenusa de ∆2 é a reta y = − a1 x. Então por Pitágoras as hipotenusas

de ∆1 e de ∆2 valem o mesmo: A2 + B 2 .
Por outro lado o comprimento do segmento de reta ligando (−B, A)
a (A, B) vale, por definição:
p √
(B − A)2 + (A − (−B))2 = 2A2 + 2B 2 .

Portanto o triângulo
√ ∆3 é isósceles, pois tem dois lados de mesmo
tamanho λ := A + B 2 . Esses lados formam um ângulo em (0, 0) que
2

denoto por α. E o terceiro lado de ∆3 , oposto a α, mede


√ √
2A2 + 2B 2 = λ2 + λ2 .

Lembro agora que é válida a recı́proca do Teorema de Pitágoras (coisa


pouco lembrada no Ensino Médio), ou seja, se um lado maior de um
triângulo é soma de quadrados de outros dois lados menores, então o
triângulo é retângulo no ângulo oposto ao maior lado. Logo o triângulo
∆3 tem que ter ângulo reto em α, por ter um lado cuja medida é λ2 +λ2 .
Logo y = ax e y = −1 a
x são de fato ortogonais, pois α é reto.

Apenas com as noções de coeficiente angular e de ortogonalidade é


possı́vel provar fatos bonitos e fundamentais da Geometria Euclidiana.
É o que faremos nas duas Seções seguintes.

3. Teorema de Tales no cı́rculo


Um dos mais bonitos teoremas da geometria Euclidiana é o Teorema
de Tales no Cı́rculo, que diz:
Afirmação 3.1. (Teorema de Tales)
Todos os ângulos inscritos no cı́rculo determinados pelo diâmetro
são ângulos retos (= π2 radianos).
CAPÍTULO 7. GEOMETRIA ANALÍTICA PLANA 81

Figura: O Teorema de Tales no Cı́rculo

Demonstração.
Vamos provar para pontos do Cı́rculo com coordenada y > 0 (para
os outros é análogo).
Tome
√ um ponto no do Cı́rculo de raio r > 0, de coordenadas
(x, + r2 − x2 ), onde x ∈ [−r, r]. √
(x, + r2 − x2 )
Queremos ver se os coeficiente angular a da reta ligando √
a (r, 0) e o coeficiente angular a′ da reta ligando (x, + r2 − x2 ) a
(−r, 0) satisfazem a condição que expressa a ortognalidade:
a′ · a = −1.
Mas √ √
′ r 2 − x2 − 0 r 2 − x2
a = = ,
x − (−r) x+r

r 2 −x2
enquanto que a = e portanto:
x−r
√ √
′ r 2 − x2 r 2 − x2 r 2 − x2
a ·a= · = 2 = −1.
(x + r) (x − r) x − r2


4. A Reta de Euler
Um Teorema muito geral, que escapou de Euclides, mas não de
Euler, é o seguinte:
Afirmação 4.1. (Reta de Euler)
Considere qualquer triângulo.
Se o triângulo não é equilátero, o Baricentro B, o Circuncentro C
e o Ortocentro H são pontos distintos mas são colineares. Ademais as
distâncias entre eles verificam:
HB = 2 · BC.
Se o triângulo é equilátero, os três pontos coincidem num mesmo
ponto.
4. A RETA DE EULER 82

Essa reta que contém esse três pontos é a reta de Euler.

1,5

0,5

0
0 0,2 0,4 0,6 0,8 1

Figura: A reta de Euler representada por segmento intersectando


uma mediana, uma altura e uma mediatriz, para P = ( 23 , 2)

1,5

0,5

0
0 0,2 0,4 0,6 0,8 1

Figura: A reta de Euler representada por segmento intersectando


uma mediana, uma altura e uma mediatriz, para P = ( 15 , 2)

À medida que formos demonstrando esse fato iremos relembrando os


conceitos envolvidos. A demosntração dará as coordenadas explı́citas
dos pontos e a equação explı́cita da reta de Euler.

Demonstração.
Não perdemos muita generalidade se supusermos que o triângulo
tem vértices:
(0, 0), (1, 0) e (A, B), B 6= 0,
pois isso se obtém escolhendo um sistema de coordenadas cartesiano
adequado.
CAPÍTULO 7. GEOMETRIA ANALÍTICA PLANA 83

Os lados do triângulo fazem parte de três retas, das quais obvia-


mente a primeira é
l1 : y = 0.
A reta l2 é a que contém (0, 0) e (A, B), cuja equação é:
B
l2 : y = · x, se A 6= 0,
A
ou a reta vertical:
l2 : x = 0, se A = 0.
E a terceira é a que contem (1, 0) e (A, B), cuja equação é:
B B
l3 : y = ·x− , se A 6= 1
A−1 A−1
ou a reta vertical
l3 : x = 1, se A = 1.
Os pontos médios de cada lado do triângulo são:
1 A+1 B A B
( , 0), ( , ) e ( , ).
2 2 2 2 2
Considero agora as três medianas : retas ligando vértices a pontos
médios dos lados opostos.
A reta que liga (0, 0) a ( A+1
2
, B2 ) é
B
2 B
m1 : y= A+1
·x= · x, se A 6= −1,
2
A+1
ou a reta vertical
m1 : x = 0, se A = −1.
A reta que liga (1, 0) a ( A2 , B2 ) é
B B
m2 : y= ·x− , se A 6= 2,
A−2 A−2
ou a reta vertical
m2 : x = 1, se A = 2.
A reta que liga (A, B) a ( 12 , 0) é:
2B B 1
m3 : y= x− , se A 6=
2A − 1 2A − 1 2
ou a reta vertical:
1 1
m3 :x = , se A = .
2 2
Supondo por um instante que estamos no caso geral, em que A 6= −1, 2,
a intersecção m1 ∩ m2 se obtem facilmente, resolvendo:
B B B
x= ·x−
A+1 A−2 A−2
4. A RETA DE EULER 84

que dá (usando B 6= 0):


A+1
x=
3
e portanto é
A+1 B
B := (
, ).
3 3
Agora tratemos dos casos particulares que faltaram.
Se A = −1, então m1 ∩ m2 consiste na intersecção de x = 0 e
y = − B3 x + B3 . Ou seja é o ponto
B
(0, ),
3
que coincide com o B.
B
Se A = 2, então m1 ∩ m2 é dada por y = 3
x intersectada com
x = 1, que dá o ponto:
B
(1, ),
3
que coincide também com o B.
Agora Afirmo que
B ∈ m3 .
1
Se A 6= 2 então o fato ques eja verdade
2B A+1 B B
( )·( )− =
2A − 1 3 2A − 1 3
diz que B ∈ m3 .
Se A = 12 , então m3 é dada por x = 21 , que obviamente passa por
1
+1 B 1 B
B = (2
, ) = ( , ).
3 3 2 3
Esse ponto B, que em todos os casos possı́veis é
B = m1 ∩ m2 ∩ m3
é chamado Baricentro.
Considero agora as três mediatrizes: retas saindo de cada ponto
médio em ângulo reto com o lado.
A mediatriz pelo ponto médio ( 21 , 0) é fácil, é a reta:
1
md1 : x = .
2
A B
O lado que contém o ponto médio ( 2 , 2 ) está na reta l2 e essa reta
ou é y = B
A
x, se A 6= 0, ou a reta vertical x = 0 se A = 0.
Portanto mediatriz md2 pelo ponto médio ( A2 , B2 ) ou é horizontal
B
md2 : y= , se A = 0,
2
ou a reta:
A B A2
md2 : y=− ·x+( + ), se A 6= 0,
B 2 2B
CAPÍTULO 7. GEOMETRIA ANALÍTICA PLANA 85

(lembre que nunca B = 0).


Então md1 ∩ md2 é o ponto:
1 B
C : ( , ), se A = 0
2 2
ou
1 A · (A − 1) B
C: ( , + ), se A 6= 0.
2 2B 2
Afirmo agora que em qualquer caso:
C ∈ md3
onde md3 é a mediatriz do lado contendo om ponto médio ( A+1 2
, B2 ).
De fato, o lado está contido em l3 , cujas equações são:
B B
l3 : y = ·x− , se A 6= 1
A−1 A−1
ou a reta vertical
l3 : x = 1, se A = 1.
Portanto ou md3 é y = B2 no caso A = 1 e claramente passa por
1 B
C: ( , ),
2 2
ou
A−1 B A2 − 1
md3 : y=− ·x+ + , se A 6= 1,
B 2 2B
que passa também por
1 A · (A − 1) B
C=( , + ),
2 2B 2
como se vê em seguida.
Esse ponto C que verifica:
C = md1 ∩ md2 ∩ md3
é chamado Circuncentro (o Exercı́cio 7.7 ajudará a justificar essa nomen-
clatura).
Já podemos nos perguntar o que acontece se
B = C.
Isso ocorre quando:
A+1 1 B A · (A − 1) B
= e = + .
3 2 3 2B 2
1
A primneira dá A = 2 , que posta na segunda dá:
3
B2 = ,
4
√ √
ou seja B = 23 ou B = − 23 . √ √
Esse triângulo com (A, B) = ( 12 , 23 ) ou (A, B) = ( 21 , − 23 ) e com os
outros vértices em (0, 0) e (1, 0) é equilátero.
4. A RETA DE EULER 86

Agora consideremos as três alturas: retas que saem de vértices e


são ortogonais ao lado oposto.
Como veremos no Exercı́cio 7.6, se
P = (x, y) 6∈ r,
a reta P Q intersecta ortogonalmente r : y = ax + b em Q ∈ r com
coordenadas
Q = (x, b) se a = 0
ou coordenadas
x − a(b − y) x − a(b − y)
Q=( , a · ( ) + b ), se a 6= 0.
a2 + 1 a2 + 1
A altura que sai de (A, B) e vai ortogonal até o lado l1 : y = 0 é
portanto:
h1 : x = A.
A altura que sai de (0, 0) é:
h3 : y = 0, se A = 1,
pois nesse caso l3 : x = 1. Ou
A−1
h3 = − · x, se A 6= 1,
B
pois no caso geral
B B
l3 : y = ·x− .
A−1 A−1
A intersecção h1 ∩ h3 é portanto:
(1, 0), se A = 1
ou
A · (A − 1)
(A, − ), se A 6= 1.
B
Em qualquer caso,
A · (A − 1)
H = ( A, − ) = h1 ∩ h2 .
B
Afirmo que

H ∈ h2 ,
onde h2 é a altura que sai de (1, 0) e chega ortogonal a l2 .
Se l2 : x = 0 (quando A = 0) então
h2 : y=0
B
obviamente passa por H. E se l2 : y = A
· x (no caso A 6= 0) então:
A A
h2 : y = − ·x+ .
B B
Nesse caso também H ∈ h2 .
Esse ponto de encontro das três alturas é o Ortocentro.
CAPÍTULO 7. GEOMETRIA ANALÍTICA PLANA 87

Quando H = B ?
Quando
A+1 B A(A − 1)
A= e =− .
3 3 B
Que é exatamente quando:
1 3
A= e B2 = ,
2 4
que diz que se trata de triângulo equilátero, como já vimos.
Falta vermos também quando o Ortocentro coincide com o circun-
centro. Isso se dá quando
1 A(A − 1) A · (A − 1) B
A= e − = + ,
2 B 2B 2
que também dão
1 3
A= e B2 = ,
2 4
formando triângulos equiláteros.
Agora, supondo que nosso triângulo não seja equilátero, só nos resta
encontrar a equação da reta ligando B a C e conferir que ela passa pelo
H.
A reta por B e C é ou bem a reta vertical
1 1
x = , se A = ,
2 2
quando o triângulo é isósceles, ou bem se A 6= 21 :
B 2 + 3A2 − 3A A(B 2 + A2 − 1)
y=− ·x+ .
B(2A − 1) B(2A − 1)
Esta é a reta de Euler !
Só falta agora verificarmos as distâncias.
Os quadrados das distâncias são:
2 2 1 A(A − 1) 1 2
HB := ( A − )2 + ( + B) =
3 3 B 3
10A B − 10AB + B + 9A − 18A + 9A2 + B 4
2 2 2 2 4 3
= .
9B 2
Enquanto que

2 1 1 A(A − 1) 1 2
BC := ( A − )2 + ( + B) =
3 6 2B 6
10A B − 10AB + B + 9A − 18A + 9A2 + B 4
2 2 2 2 4 3
= .
36B 2
ou seja
2 2
HB = 4 · BC ,
como querı́amos.

4. A RETA DE EULER 88

Observação 1:
Observe que temos a equação explı́cita e portanto podemos deter-
minar casos onde a reta de Euler é horizontal. Que ocorrem para pontos
da forma

p
P = ( A, ± 3A(1 − A) ).

0,8

0,6

0,4

0,2

0
0 0,2 0,4 0,6 0,8 1


6
Figura: A reta de Euler é horizontal para pontos da forma P = ( 32 , 3
).

Observação 2:
É natural termos curiosidade por qual seria o gráfico da função
z = z(A, B), B 6= 0 dada por

z = 10A2 B 2 − 10AB 2 + B 2 + 9A4 − 18A3 + 9A2 + B 4 ,

pois vimos z = 0 está associado a um ponto muito especial no plano


formado pelos parâmetros (A, B): o ponto


1 3
( , ) ∼ (0.5, 0.8).
2 2
CAPÍTULO 7. GEOMETRIA ANALÍTICA PLANA 89

A Figura a seguir mostra uma parte dessa superfı́cie, com A ∈ [0, 1]


e B ∈ [0.1, 1.3] (na figura o eixo x é o dos A e o eixo y é o dos B).

0 1
1,2 0,8
1 0,6
0,8
y 0,6 0,4 x
0,4 0,2
0,2 0

Mas não se vê muita coisa. Já as próximas duas Figuras são per-
fis da superfı́cie, e elas sim ilustram bem que um ponto próximo de
(0.5, 0.8) é o mı́nimo dessa função z = z(A, B) (na figura o eixo x é o
dos A e o eixo y é o dos B).

0
1 0,8 0,6 0,4 0,2 1 ,2
0,8
00,2
0,4
0,6
x
y
5. FUNÇÃO INVERSA COMO REFLEXÃO DE GRÁFICO NA
DIAGONAL 90

0 1
0
0,8 x
0,6
0,4
0,2
1,2 1 0,8 0,6 0,4 0,2
y

5. Função inversa como reflexão de gráfico na diagonal

Imagine uma função f : I → J, y = f (x) que admita uma função


inversa f −1 : J → I, x = f −1 (y).
Vamos supor agora que temos ambos os gráficos, de f e de f −1 ,
no mesmo sistema de coordenadas (x, y), ou seja, por um momento
pensemos em g = f −1 tomada com as mesmas abcissas e oordenadas
que a f , ou seja, vamos ver ao mesmo tempo y = f (x) e y = g(x).
Agora ligamos com uma reta r o ponto (A, B) := (x, f (x)) do
gráfico de y = f (x) com o ponto (B, A) do gráfico de y = g(x). Então
o coeficiente angular dessa reta é:
A−B
a := = −1.
B−A
Ou seja que a reta r que os liga tem a mesma inclinação da anti-
diagonal, a = −1, ou seja, r é ortogonal à diagonal y = x. A equação
dessa r é pelo que vimos na Afirmação 1.3:
r: y = −x + (A + B).
E r corta a diagonal y = x no ponto cuja abcissa satisfaz:
x = −x + (A + B),
ou seja x = A+B 2
, ou seja, no ponto com coordenadas ( A+B 2
, A+B
2
). E
A+B A+B
(A, B) e (B, A) são equidistantes de ( 2 , 2 ).
Concluı́mos que a diagonal y = x funciona como um espelho para
os gráficos de y = f (x) e y = g(x):
O gráfico da f −1 referido ao mesmo sistema (x, y) é um reflexão na
diagonal do gráfico da y = f (x)
CAPÍTULO 7. GEOMETRIA ANALÍTICA PLANA 91

y=x
(B,A)
r
y= f^{−1}(x)

(A,B)

y= f(x)

Figura: Os gráficos de f e f −1 no mesmo sistema cartesiano

6. O método de Descartes para as tangentes a um gráfico


Como a Geometria analı́tica foi um criação de René Descartes, nada
mais justo que indicarmos um bonito método criado por ele1
Pelo menos no meu caso, durante meu tempo de ensino Médio,
só me lembro da palavra reta tangente ser usada para referir a reta
tangente de um cı́rculo.
Nesse caso, para um cı́rculo C de raio r e centro O, pode ser definida
como a reta t pelo ponto P que é ortogonal ao raio do Cı́rculo.
Em geral uma reta por um ponto P de C o intersecta noutro ponto,
mas a reta tangente t a P não pode intersectar C noutro ponto P ′ : se
por absurdo t ∩ C = {P, P ′ } então no triângulo ∆ OP P ′ a hipotenusa
OP ′ mediria o mesmo que o cateto OP , absurdo.
Descartes se perguntou pelo significado da reta ortogonal a um
gráfico qualquer, pois isso está ligado a questões de Óptica, de reflexão
da luz em lentes, que lhe interessavam.
Responder a essa questão dá a chave também para o significado da
reta tangente a um gráfico qualquer (pois uma é ortogonal à outra).
De fato não vamos lidar coma questão assim tão geral: suponhamos
gráficos de polinômios y = f (x).
Ele pensou em usar o que sabia de cı́rculos para atacar o caso geral
de gráficos. Para isso, considerou um ponto P = (x, f (x)) do gráfico
e considerou Cı́rculo com centro (c, 0) no eixo dos x, de raios r que
passem por P = (x, f (x)).
Ou seja, escolhidos c, r teremos que x é raı́z de:

(f (x) − 0)2 + (x − c)2 − r2 = 0.

1Me baseei mais no livro de Edwards, mas o leitor pode comparar com o que
está nas páginas 95-113 de The geometry of René Descartes, Dover.
6. O MÉTODO DE DESCARTES PARA AS TANGENTES A UM
GRÁFICO 92

Em geral, se c é escolhido de qualquer jeito, pode haver outra raı́z x′


dessa equação, pois o cı́rculo
y 2 + (x − c)2 − r2 = 0
pode cortar o gráfico de y = f (x) em mais de um ponto.

problema: Como escolher c para que x seja raı́z dupla de:


(f (x) − 0)2 + (x − c)2 − r2 = 0,
ou seja, para que uma segunda raı́z x′ colida com x ?

Se conseguı́ssemos resolver esse Problema estarı́amos colocando o


Cı́rculo de modo a tocar, tangenciar o gráfico em P .
Ora, como sabemos qual a tangente ao Cı́rculo usarı́amos essa reta
como tangente ao gráfico !
Melhor do que explicar o método em abstrato será fazermos dois
Exemplos.

Exemplo 6.1. Consider y = Cx2 uma parábola e tome P =


(x, Cx2 ), com x > 0. Comos os Cı́rculos com centro (c, 0) tem equação:
y 2 + (x − c)2 = r2 ,
queremos encontrar uma raı́z dupla x de:
(Cx2 )2 + (x − c)2 − r2 = 0,
ou seja queremos encontrar uma fatoração:
(Cx2 )2 + (x − c)2 − r2 = (x − x)2 q(x)
onde q(x) é um polinômio de grau 2.
Ou seja queremos encontrar uma fatoração do tipo:
(Cx2 )2 + (x − c)2 − r2 = (x − x)2 · (a2 x2 + a1 x + a0 ).
Expandindo ambos os lados, formam-se dois polinômios de grau 4 em
x, à esquerda e à direita. Igualando os coeficientes do monômios x4 à
esquerda e à direita faz aparecer
C 2 − a2 = 0 ⇔ a2 = C 2 .
Igualando os coeficientes de x3 à esquerda e à direita faz aparecer:
−a1 + 2xa2 = 0
ou seja
−a1 + 2x(C 2 ) = 0 ⇔ a1 = 2xC 2 .
Igualando os coeficientes de x2 à esquerda e à direita faz aparecer:
1 + 2xa1 − a0 − x2 a2 = 0,
ou seja
1 + 2x(2xC 2 ) − a0 − x2 C 2 = 0 ⇔ a0 = 1 + 3x2 C 2 .
CAPÍTULO 7. GEOMETRIA ANALÍTICA PLANA 93

Por último, igualando os coeficientes de x à esquerda e à direita faz


aparecer:
−2c + 2xa0 − x2 a1 = 0
ou seja,
−2c + 2x(1 + 3x2 C 2 ) − x2 (2xC 2 ) = 0 ⇔ c = x + 2x3 C 2 .
Logo o Cı́rculo cujo centro é o ponto
O = (c, 0) = (x + 2x3 C 2 , 0)
e que passa por P = (x, Cx2 ) tangencia o gráfico de y = Cx2 nesse
ponto P .

y 1

0
0 1 2 3 4 5
x
-1

-2

Figura: O gráfico de y = x2 e o cı́rculo tangente em P = (1, 1), de centro (3, 0).

O coeficiente angular da reta ligando O a P é:


f (x) Cx2 1
− =− 3 2
=− .
c−x x + 2x C − x 2xC
Ora, para passarmos ro raio do cı́rculo para a tangente basta tomar
1
a reta ortogonal. E o coeficiente angular ortogonal ao anterior − 2xC é:
2Cx.
Logo a reta tangente ao gráfico em P vem dada por:
y − Cx2
= 2Cx ⇔ y = (2Cx) x + (Cx2 − 2Cx2 ).
x−x
Exemplo 6.2. Considere y = Cx3 e tome P = (x, Cx2 ), com x > 0.
Queremos uma raı́z dupla de:
(Cx3 )2 + (x − c)2 − r2 = 0,
ou seja queremos encontrar uma fatoração:
(Cx3 )2 + (x − c)2 − r2 = (x − x)2 q(x)
7. EXERCÍCIOS 94

onde q(x) agora é um polinômio de grau 4.


Ou seja queremos encontrar uma fatoração do tipo:
(Cx3 )2 + (x − c)2 − r2 = (x − x)2 · (a4 x4 + a3 x3 + a2 x2 + a1 x + a0 ).
Expandindo ambos os lados, formam-se dois polinômios de grau 6, à
esquerda e à direita. Comparando como fizemos antes os coeficientes
de cada monômio, fazemos surgir equações, que vão sendo resolvidas
uma a uma, produzindo nesta ordem:
a4 = C 2 , a3 = 2xC 2 , a2 = 3x2 C 2 ,
a1 = 4x3 C 2 , a0 = 1 + 5x4 C 2 , c = x + 3x5 C 2 .
Logo o Cı́rculo cujo centro é o ponto
O = (c, 0) = (x + 3x5 C 2 , 0)
e que passa por P = (x, Cx3 ) tangencia o gráfico de y = Cx3 nesse
ponto P .

1
y
0
0 1 2 3 4 5 6 7
x
-1

-2

-3

Figura: O gráfico de y = x3 e o cı́rculo tangente em P = (1, 1), de centro (4, 0).

O coeficiente angular da reta ligando O a P é:


f (x) Cx3 1
− =− 5 2
=− 2 ,
c−x x + 3x C − x 3x C
O coeficiente angular da reta ortogonal a esta é
3x2 C
e daı́ se obtém em seguida a equação toda da reta tangente ao gráfico.

7. Exercı́cios
Exercı́cio 7.1. Qual é o coeficiente angular da reta y = y(x)
determinada pela equação 3y + 4x − 27 = 0 ?
CAPÍTULO 7. GEOMETRIA ANALÍTICA PLANA 95

Exercı́cio 7.2. i) determine a reta, na forma y = a · x + b, que


passa por (1, 2) e (4, 13).

ii) determine a reta, na forma y = a · x + b, que passa por (1, 2) com


coeficiente angular 5.
Exercı́cio 7.3. (resolvido)
Tentei resolver o sistema de equações:
y − 5x − 2 = 0 e 2y − 10x − 1 = 0,
e fiz o seguinte: da primeira equação obtive y = 5x + 2 e substitui esse
y na segunda, obtendo:
2(5x + 2) − 10x − 1 = 3 = 0,
o que é um absurdo, pois 3 6= 0.
Você poderia explicar, com os conceitos deste Capı́tulo por quê
chego nesse absurdo?
Exercı́cio 7.4. Agora tentei resolver os sistemas de duas equações:
y − ax + 1 = 0 e y − x + 2 = 0
(sim são vários sistemas de duas equações pois a ∈ R pode ser mudado).
Da primeira obtive: y = ax − 1 e substituindo na segunda obtive:
(ax − 1) − x + 2 = x(a − 1) + 1 = 0.
i) Supondo a − 1 6= 0 continue a resolução dos sistemas.
ii) explique geometricamente qual o significado da condição a − 1 6=
0.
Exercı́cio 7.5. Um outro modo se pensar a questão de como de-
terminar a reta y = a · x + b passando por dois pontos P1 = (x1 , y1 ) e
P2 = (x2 , y2 ) é resolver o sistema:
y1 = a · x1 + b e y2 = a · x2 + b,
cujas incógnitas são a, b.
i) qual a condição sobre P1 = (x1 , y1 ) e P2 = (x2 , y2 ) para que o
sistema tenha solução única ? O que diz a chamada Regra de Cramer
neste caso ?
Agora considere o problema de determinar qual a curva da forma
y 2 = x3 + b · x + a
passa pelos pontos P1 = (−3, 0) e P2 = (4, 0).
ii) qual o sistema de equações a ser resolvido ? É muito diferente
do anterior ?
iii) qual a solução (a, b) ?
7. EXERCÍCIOS 96

Exercı́cio 7.6. (resolvido)


Seja y = ax + b a equação de uma reta r e seja P = (A, B) 6∈ r.
i) Encontre o ponto Q na reta r tal que o segmento P Q é ortogonal
a r em Q.
ii) pode acontecer que a coordenada x de Q seja A ? Exatamente
em que situações ?
Exercı́cio 7.7. Prove que o circuncentro
1 A(A − 1) B
C=( , + ),
2 2B 2
equidista dos três vértices (0, 0), (1, 0) e (A, B) do triângulo (B 6= 0).
Conclua que há um cı́rculo centrado em C que passa pelos vértices
do triângulo.
Dica: expanda os quadrados e simplifique.
CAPı́TULO 8

A Tangente ao gráfico, segundo o Cálculo

No final do Capı́tulo anterior vimos que Descartes desenvolveu um


engenhoso método algébrico para definir e calcular retas tangentes a
gráficos de polinômios.
Mas precisamos de um método mais geral. Para isso, estudare-
mos primeiro as secantes a gráficos e depois, via o conceito de limite,
definiremos as tangentes a gráficos.

1. Retas secantes a um gráfico


Será interessante para nós pegarmos dois pontos de um mesmo
gráfico e calcularmos a equação da reta que os liga, chamada secante
ao gráficos pelos dois pontos.
Estaremos interessados pricipalmente em seu coeficiente angular.
Por exemplo, (x1 , f (x1 ) e (x2 , f (x2 ) definem uma reta y = ax + b
com coeficiente angular
f (x2 ) − f (x1 )
a= ,
x2 − x1
e coeficiente linear
f (x2 ) − f (x1 )
b = f (x1 ) − ( ) · x1 .
x2 − x1

Exemplos:
1)- Tome um x1 > 0 e fixe no gráfico da função f (x) = |x| o
ponto (x1 , x1 ). Note que os x2 próximos de x1 também são positivos e
portanto as secantes determinadas por (x1 , x1 ) e (x2 , x2 ) são sempre as
mesmas, de fato, são todas iguais à diagonal y = x. Analogamente, se
x1 < 0 as secantes que envolvem o ponto (x1 , −x1 ) e outro do gráfico
bem próximo coincidem com a antidiagonal y = −x.
2) - Certamente nenhuma secante ao gráfico de y = x2 coincide com
o gráfico; vemos que aqui as secantes mudam de inclinação.

2. A reta tangente a um gráfico


Olhe agora somente o coeficiente angular da secante ao gráfico de
y = f (x) por dois de seus pontos :
f (x2 ) − f (x1 )
.
x2 − x1
97
2. A RETA TANGENTE A UM GRÁFICO 98

Imagine que (x1 , f (x1 )) fica parado mas que (x2 , f (x2 )) está se movendo,
no gráfico de f , indo cada vez mais próximo de (x1 , f (x1 )). Se f é
contı́nua, basta supor que a coordenada x2 fica próxima de x1 para
necessariamente f (x2 ) ficar mais próxima de f (x1 ).
Como x2 fica próximo de x1 sua diferença
h := x2 − x1
tem módulo pequeno. Para deixarmos o ponto (x1 , f (x1 )) em destaque,
vamos escrever o coeficiente angular acima como:
f (x1 + h) − f (x1 )
ax1 ,h := , onde x1 + h = x2 .
h

0
0 0,5 1 1,5 2
x
-1

-2

Figura: Duas secantes pelo ponto (1, 1) do gráfico de y = x2

A grande questão é:


Será que esses coeficientes angulares ax1 ,h tendem a um valor es-
pecı́fico bem determinado ax1 1, quando h → 0 (independentemente do
modo como h se faz pequeno) ?

É nesse ponto que se vê importância de podermos falar de algo como


o h tender a zero, sem precisar nunca ser zero: pois simplesmente não
podemos dividir por h = 0 e precisamos calcular limh→0 ax1 ,h .
Atenção ! pois em geral pode não existir esse limite, como algo
bem definido.
O exemplo mais simples é (que é uma função contı́nua !):
y = f (x) = |x| e x = 0.

1Claro que em geral ax1 depende do x1 escolhido


CAPÍTULO 8. A TANGENTE AO GRÁFICO, SEGUNDO O
CÁLCULO 99

De fato, se h > 0 e tende a zero, obtenho:


|0 + h| − |0| h
lim = lim =
h→0
h>0
h h→0
h>0
h
= lim 1 = 1,
h→0
h>0

e no entanto:
|0 + h| − |0| −h
lim = lim =
h→0
h<0
h h→0
h<0
h
= lim −1 = −1,
h→0
h<0

0,8

0,6

0,4

0,2

0
-1 -0,5 0 0,5 1
x

Figura: Gráfico de y = | x |, para x ∈ [−1, 1].

Definição 2.1. Quando há uma posição limite de secantes, ou


seja, quando existe
f (x1 + h) − f (x1 )
a := lim ax1 ,h , onde ax1 ,h := ,
h→0 h
dizemos que existe a Reta Tangente ao gráfico de f em (x1 , f (x1 )). É
a reta dada por:
y = a · x + b, pondo a := lim ax1 ,h
h→0
e onde b fica determinado pela imposição de que essa reta passe por
(x1 , f (x1 ).

De f (x1 ) = a · x1 + b, obtenho o coeficiente linear:


b = f (x1 ) − (lim ax1 ,h ) · x1 .
h→0

É interessante que, embora as secantes não tenham muito a ver com


o gráfico:
a tangente ao gráfico em um de seus ponto dá informação relevante
sobre ele, ela dá informação do formato do gráfico naquele ponto.
Dentre todas a retas passando por aquele ponto, a tangente ao
gráfico é a mais informativa do formato do gráfico.
3. A RETA TANGENTE AO SENO EM (0, 0) É A DIAGONAL 100

3. A reta tangente ao seno em (0, 0) é a diagonal

Vamos dar uma justificação bem geométrica para o fato de que no


gráfico do seno existe uma reta tangente bem definida no ponto (0, 0):
de fato sua equação é a mesma da diagonal y = x.
Para isso começamos observando que:
Afirmação 3.1. Valem:
sin(θ) < θ e θ < tan(θ), para 0 < θ < π/4,
e
tan(θ) < θ e θ < sin(θ), para − π/4 < θ < 0.
Demonstração.
Seja 0 < θ < π/4.
Considere três Áreas envolvidas:
• do triângulo △ com vértices em (0, 0), (1, 0) e em (cos(θ), sin(θ)).
Note que a base dele mede 1 e que sua altura é o sin(θ). Logo
A△ (θ) = sin(θ)
2
.
• do Setor circular (fatia do disco) de abertura θ do disco de
raio 1, s(θ). Sua área é denotada As (θ). Temos As (2π) = π e
em geral vale As (θ) = 2θ .
• do triângulo ∆ com vértices em (0, 0), (1, 0) e no ponto (1, tan(θ)),
que é um triângulo retângulo em (1, 0) Denote sua área por
A∆ (θ). A base dele mede 1 e que sua altura é tan(θ). Logo
A∆ (θ) = tan(θ)
2
.

(1, tan θ )

( cos θ, sen θ)

θ
(1,0)
(0,0)

Figura: Observe que △ ⊂ s(θ) ⊂ ∆

Das inclusões:
△ ⊂ s(θ) ⊂ ∆
obtemos:
A△ (θ) < As (θ) < A∆ (θ)
CAPÍTULO 8. A TANGENTE AO GRÁFICO, SEGUNDO O
CÁLCULO 101

ou seja para 0 < θ < π/4:


sin(θ) θ tan(θ)
< < ,
2 2 2
que é o que queremos (se eliminamos o 1/2).
Por outro lado, se −π/4 < θ < 0 (isto é, θ é ângulo no sentido
horário),
A△ (θ) < As (θ) < A∆ (θ)
agora significa (já que para cálculo de áreas tomo os módulos de números
negativos):
− sin(θ) −θ − tan(θ)
< < ,
2 2 2
ou seja (multiplicando por −1):
tan(θ) θ sin(θ)
< <
2 2 2
o que queremos (eliminando o 1/2).


Afirmação 3.2. (Um Limite fundamental)


sin(θ)
lim =1
θ→0 θ
Demonstração.
Para 0 < θ < π/4, da Afirmação 3.1 temos
sin(θ)
θ< ,
cos(θ)
cos(θ)
e obtenho (multiplicando por θ
> 0):
sin(θ)
cos(θ) < .
θ
Ainda da Afirmação 3.1, para 0 < θ < π/4,:
sin(θ) < θ
e obtenho:
sin(θ)
< 1.
θ
Ou seja,
sin(θ)
cos(θ) < < 1, se 0 < θ < π/4.
θ
Uso agora o item 6) do Teorema 1.1, combinado com continuidade do
cosseno, obtendo:
sin(θ)
lim = lim cos(θ) = cos(0) = 1.
θց0 θ θ→0
3. A RETA TANGENTE AO SENO EM (0, 0) É A DIAGONAL 102

Por outro lado, quando −π/4 < θ < 0 ainda temos cos(θ) > 0 e pela
Afirmação 3.1 tı́nhamos:
sin(θ)
< θ,
cos(θ)
cos(θ)
de onde obtenho (multiplicando por θ
< 0):
sin(θ)
> cos(θ).
θ
De novo da Afirmação 3.1 para −π
2
< θ < 0:
θ < sin(θ)
e obtenho (já que θ < 0):
sin(θ)
< 1.
θ
Então como antes obtenho:
sin(θ)
lim = lim cos(θ) = cos(0) = 1,
θր0 θ θ→0

o que é suficiente para sabermos que


sin(θ)
lim = 1.
θ→0 θ


1
0,8
0,6
0,4
0,2
0
-3 -2 -1 0 1 2 3
x

sin(θ)
Figura: Gráfico de y = f (x) = θ
para 0 6= θ ∈ [−π, π] e f (0) = 0.

Como consequência da Afirmação 3.2 e da definição de Reta Tan-


gente ao gráfico do seno em (0, 0), a tangente ao gráfico do seno em
(0, 0) é exatamente a diagonal, pois os coeficientes angulares de secantes
por (0, 0) são:
sin(θ) − sin(0)
θ−0
e
sin(θ) − sin(0) sin(θ)
lim = lim = 1.
θ→0 θ−0 θ→0 θ
CAPÍTULO 8. A TANGENTE AO GRÁFICO, SEGUNDO O
CÁLCULO 103

1,5

0,5

0
-1,5 -1 -0,5 0 0,5 1 1,5
x
-0,5

-1

-1,5

Figura: A diagonal é tangente ao seno em (0, 0)

4. Interpretação Fı́sica da reta tangente


Uma das fontes do Cálculo é a Fı́sica. Os conceitos de secantes e
tangente a um gráfico têm uma interpretação fı́sica natural.
Se x é pensado como sendo o tempo, podemos pensar em f (x) como
a posição de um objeto, determinada em relação a um ponto de origem,
do qual nos afastamos para a direita (valores positivos de f ) ou para a
esquerda (valores negativos de f ).
Então
f (x2 ) − f (x1 )
é a distância percorrida no tempo transcorrido x2 − x1 e
f (x2 ) − f (x1 )
x2 − x1
é o que se costuma chamar a velocidade média.
É o que no dia-a-dia nos perguntam: você vai de casa até a faculdade
em quanto tempo ? E daı́ se deduz a velocidade média do seu trajeto.
Mas também poderia haver interesse de alguém nas velocidades
marcadas no velocimetro do seu carro a cada instante, para saber onde
pegou engarrafamento, se teve excesso de velocidade em alguns trechos,
etc. O que é essa velocidade instantânea no instante x1 ? Ora, é o
limite:
f (x1 + h) − f (x1 )
lim .
h→0 h
Ou seja, o coeficiente angular da tangente ao gráfico da função
posição f no instante x1 dá a velocidades instantânea no momento x1 .
Isso é o que marca o velocı́metro do carro.
Essa interpretação que estamos dando dos conceitos que vimos ao
caso do movimento de um objeto, nos motiva a falar da aceleração, um
conceito que usamos muito no dia a dia. Falaremos disso na Seção 3
do Capı́tulo 9.
5. EXERCÍCIOS 104

5. Exercı́cios
Exercı́cio 5.1. i) Determine os intervalos em que coeficientes an-
gulares das secantes da função f (−∞, 0) ∪ (0, +∞) → R, f (x) = 1/x
são positivos ou negativos.
ii) Diga (ainda de modo bem intuitivo) o que acontece com esses
coeficientes angulares de secantes quando o ponto fixado x fica próximo
de zero (separadamente se x < 0 ou se x > 0) ou com módulo de x
muito grande (x > 0 ou x < 0).
Exercı́cio 5.2. (resolvido)
Sabendo que o gráfico de uma f e de sua f −1 são reflexões um do
outro na diagonal, que consequência tiramos disso para o gráfico de
1/x ?
Exercı́cio 5.3. Calcule as equações y = ax + b das retas tangentes
no ponto (1, 1) dos gráficos de:
i): y = x2
ii): y = x3
iii): y = x4
sin(x)
Exercı́cio 5.4. Pedi para o programa Maple plotar y = x
e
2
y = sinx(x) para x ∈ [−3, 3] e ele repondeu:

0,8

0,4

0
-3 -2 -1 0 1 2 3
x

-0,4

Mas essas funções a princı́pio não estão sequer definidas em x = 0 !


Explique com os conceitos de limite e continuidade o que o programa
fez.
CAPı́TULO 9

A derivada

1. Definição, primeiras propriedades e exemplos simples


A grandeza
f (x + h) − f (x)
, h 6= 0
h
é conhecida como quociente incremental. Ela compara, através do quo-
ciente, o incremento (aumento, variação) dos valores da função com o
incremento (aumento, variação) na entrada da função.
E é assim que pensamos no dia-a-dia: não é muito informativo se
dissermos quanto aumentou o salário de alguém, de f (x) para f (x + h),
se não dissermos quanto tempo h foi necessário para o reajuste.
Também se dissermos que um carro passa de f (x) km/h para f (x +
h) km/h e não dissermos em quanto tempo h o faz, não teremos uma
idéia da potência do motor. E assim por diante, há inúmeros exem-
plos de processos só são descritos corretamente se usarmos quocientes
incrementais.
Definição 1.1. A Derivada da função y = f (x) num ponto x de
seu domı́nio é o limite:
f (x + h) − f (x)
lim .
h→0 h
Denotamos1 esse limite por f ′ (x).

Observações:
• Não estamos dizendo que sempre exista f ′ (x), ao contrário, é
uma bela propriedade para uma f ter derivada f ′ (x). Quando
dissermos apenas que f tem Derivada (ou também, é De-
rivável ), estamos dizendo que ela tem Derivada em cada ponto
de seu domı́nio.
• após a definição de derivada, podemos redefinir a reta tangente
ao gráfico de y = f (x) no ponto (x, f (x)) como a reta que passa
por esse ponto e tem coeficiente angular f ′ (x). Essa reta se
determina assim: pondo
y − f (x)
= f ′ (x)
x−x
1Essa notação lembra a de I. Newton, mas o outro criador do Cálculo, G.
Leibniz usava a notação dd fx (x), muito usada nos livros de Cálculo.
105
1. DEFINIÇÃO, PRIMEIRAS PROPRIEDADES E EXEMPLOS
SIMPLES 106

obtenho:
y = f ′ (x) · x + (f (x) − f ′ (x)x).

Note o milagre que há numa derivada: o denominador da fração


tende a zero e mesmo assim a fração tende a um número definido.
Isso certamente está ligado ao fato de que o numerador tende a zero
também, como vemos agora:
Teorema 1.1. Se existe o limite
f (x + h) − f (x)
lim ,
h→0 h
então:
• limh→0 ( f (x + h) − f (x) ) = 0
• limh→0 f (x + h) = f (x).
• f é contı́nua em x.
Demonstração.
Prova de i):
Fixe um ponto x qualquer do domı́nio da f . Parto de que existe
f (x + h) − f (x)
lim .
h→0 h
Então adaptando a nossa notação2 àquela do item 4) do Teorema 1.1,
obtenho:
f (x + h) − f (x)
lim ( h · ) = 0.
h→0 h
Ou seja,
lim ( (f (x + h) − f (x)) = 0.
h→0
Prova de ii):
Dizer que limh→0 ( (f (x + h) − f (x)) = 0 é exatamente o mesmo
que dizer limh→0 f (x + h) = f (x).
Prova de iii): O iem ii) é a definição de continuidade da f em x.


A recı́proca desse Teorema é falsa, como o mostra f (x) = |x| que,


apesar de contı́nua em todo seu domı́nio, não tem derivada no x = 0.
De fato, já vimos que:
|0 + h| − |0| |0 + h| − |0|
lim = −1, mas lim = 1.
hր0 h hց0 h
Existem funções contı́nuas bastante bizarras, sem derivada em nen-
hum ponto. Tente imaginar (sem conseguir, é claro !) uma espécie de
serrote com uma infinidade de dentes, que entre dois dentes tem mais
2Na f (x+h)−f (x)
notação do Teorema 1.1, x = 0, x = h, uma das funções de h é h
e a outra é a identidade g(h) = h
CAPÍTULO 9. A DERIVADA 107

outro e assim por diante. Um exemplo é construı́do no livro Calculus,


de M. Spivak.

2. Um Árbitro que só avalia as inclinações


Comparando com a Seção 2 do Capı́tulo 8, concluı́mos que a Derivada
f ′ (x) na Definição 1.1 é o coeficiente angular da Tangente ao gráfico
de y = f (x) em (x, f (x)). Se o valor da Derivada f ′ (x) muda quando
mede x isso significa que as inclinações das tangentes variam ao longo
do gráfico.
Vamos dar 4 Exemplos dos mais simples.
Imagine uma competição de surf em que 4 participantes realizam
manobras descritas por quatro gráficos diferentes: y = f1 (x) ≡ 1 (con-
stante), y = f2 (x) = x, y = f3 (x) = x2 e y = f4 (x) = x3 . Imagine
também que um certo Árbitro da competição tem a tarefa exclusiva
de só medir e avaliar as inclinações das pranchas em cada instante
x, sem se interessar em medir as alturas atingidas pelos participantes.
Quem controla as alturas quem controla é outro Árbitro (e por sinal,
nesses exemplos tão simples é fácil saber onde cada função tem valores
positivos, zero ou negativos).
Ou seja, que o Árbitro que só mede as inclinações calcula as Derivadas
e apresenta o gráfico de cada Derivada. A seguir, o resultado para cada
um dos 4 concorrentes:

1): f1 (x) = 1:

1−1
f1′ (x) = lim = lim 0 = 0.
h→0 h h→0

0,8

0,6

0,4

0,2

0
-1 -0,5 0 0,5 1
x

Figura: y = f1 (x) ≡ 1 em vermelho e f1′ (x) ≡ 0 em verde.


2): f2 (x) = x:

(x + h) − x
f2′ (x) = lim = lim 1 = 1.
h→0 h h→0
2. UM ÁRBITRO QUE SÓ AVALIA AS INCLINAÇÕES 108

0,5

0
-1 -0,5 0 0,5 1
x

-0,5

-1

Figura: y = f2 (x) = x em vermelho e f2′ (x) ≡ 1 em verde.


3): Para f3 (x) = x2 , f3′ (x) = 2x: já fizemos essa conta na Seção 2
do Capı́tulo 8, onde vimos a equação da tangente a esse gráfico.
2

0
-1 -0,5 0 0,5 1
x

-1

-2

Figura: y = f3 (x) = x2 em vermelho e f3′ (x) = 2x em verde.


4): f4 (x) = x3 :
(x + h)3 − x3 x3 + 3x2 h + 3x h2 + h3 − x3
f4′ (x) = lim = lim =
h→0 h h→0 h
h · (3x2 + 3x h + h2 )
= lim == lim (3x2 + 3x h + h2 ) = 3x2 ,
h→0 h h→0
pois o polinômio em h de grau ≤ 2 dado por 3x2 + 3xh + h2 é uma
função contı́nua !
3

0
-1 -0,5 0 0,5 1
x

-1

Figura: y = f4 (x) = x3 em vermelho e f4′ (x) = 3x2 em verde.

Para confeccionarmos um gráfico interessante mais adiante, será


útil se calculamos à mão a derivada de:
5) f5 (x) = x4 :
(x + h)4 − x3 x4 + 4x3 h + 6x2 h2 + 4x h3 + h4 − x4
f4′ (x) = lim = lim =
h→0 h h→0 h
CAPÍTULO 9. A DERIVADA 109

h · (4x3 + 6x2 h + 4x h2 + h3 )
= lim
h→0 h
= lim (4x3 + 6x2 h + 4x h2 + h3 ) = 4x3 ,
h→0
pois o polinômio em h de grau ≤ 3 dado por 4x3 + 6x2 h + 4x h2 + h3
é uma função contı́nua !

0
-1-0,50 0,5 1
x

-2

-4

Figura: y = f5 (x) = x4 em vermelho e f5′ (x) = 4x3 em verde.

3. A segunda derivada
Pisando no acelerador do carro vemos o ponteiro do velocimêtro mu-
dar de posição, pois aumentamos a velocidade instantânea. Enquanto
que, pisando no freio do carro, desaceleramos o carro, diminuimos sua
velocidade instantânea.
Vamos usar o sı́mbolo da derivada
f ′ (x)
para denotar a velocidade instantânea em cada tempo x.
Note que antes tı́nhamos uma função f (x) que dava a posição em
cada instante. Agora estamos interessados em variar não a posição
f (x) em cada instante, mas sim a velocidade f ′ (x) em cada instante.
Então podemos perguntar agora quanto f ′ (x) variou num tempo
determinado, ou seja podemos falar da aceleração média:
f ′ (x2 ) − f ′ (x1 )
.
x2 − x1
Exemplo dessa grandeza no dia-a-dia: nas revistas especializadas em
carros sempre falam do carro que passa de zero a 100 km/h em tantos
segundos.
Agora passando ao limite:
f ′ (x1 + h) − f ′ (x1 )
lim .
h→0 h
obtemos a aceleração instantânea no instante x1 . Um sı́mbolo para
ela é:
f ′′ (x1 ) := (f ′ )′ (x1 )
4. EXERCÍCIOS 110

e em geral, em cada instante x:


f ′′ (x) := (f ′ )′ (x)
Infelizmente nos carros de passeio normais não temos uma aparelho
que meça isso, um acelerômetro, para nos dizer qual a aceleração in-
stantânea. Porém num escândalo recente na Fórmula 1 se soube que se
registra também os valores de aceleração em cada instante dos carros
de corrida. Na Seção 2 do Capı́tulo 10 daremos um Exemplo em que a
aceleração/velocidade/posição de um carro contradiz o senso comum.
Na Fı́sica de Newton a aceleração instantânea f ′′ (x) := (f ′ )′ (x)
joga um papel primordial, pois ela (multiplicada pela massa) é a resul-
tante de todas as forças que agem sobre um corpo.
O que ele descobriu foi como, matematicamente, passar da acel-
eração instantânea (f ′ )′ (x) para a velocidade instantânea f ′ (x) e dai
finalmente para a posição f (x) do objeto em cada instante de tempo.
Começou postulando um formato para a aceleração resultante da
força de atração gravitacional do sol sobre os planetas, e chegou, matem-
aticamente, no formato exato das órbitas dos planetas (elipses,cônicas)
(ou seja na f (x) ) e em suas velocidades f ′ (x) (a lei de Kepler). Com
isso transformou a astronomia em ciência.
No Capı́tulo 24 sobre Equações Diferenciais entenderemos um pouco
do método que ele usou.

4. Exercı́cios
Exercı́cio 4.1. Qual o gráfico de f (x) = |x + 1|?
Onde é contı́nua e onde não tem derivada ?
Exercı́cio 4.2. Consider as funções definidas por:
f (x) = x2 + x + 2, se x < 1,

f (x) = −x2 + b · x + c, se x ≥ 1.
Ajuste os parâmetros b, c para que f seja contı́nua e derivável em
x = 1.
Dica: impondo a continuidade se produz uma relação entre c = c(b).
E o valor de b sai de impôr-se a derivabilidade.
Exercı́cio 4.3. Usando apenas a definição, derive (onde C é uma
constante ):
i) y ≡ C
ii) y = C · x,
iii) y = C · x2
iv) y = C · x3 ,
v) y = ( x − C )2
vi) y = ( x − C )3
CAPÍTULO 9. A DERIVADA 111

Exercı́cio 4.4. A Figura a seguir mostra uma parte do gráfico de


y = f (x) = | x x|+1 (vermelho) (estudada na Seção 4 do Capı́tulo 5) e
parte do gráfico de y = x (verde).
1

0,5

0
-1 -0,5 0 0,5 1
x

-0,5

-1

Ela sugere que f ′ (0) = 1. Prove isso mostrando separadamente que:


h
( h+1 )
lim =1
hց0 h
e
h
( −h+1 )
lim =1
hր0 h

Exercı́cio 4.5. Para fazer este Exercı́cio, lembre que x = y é
inversa de f : R>0 → R>0 , y = f (x) = x2 e que, pela Afirmação 3.1,

x = y é uma função contı́nua. √
i) Sem calcular a derivada de f : R>0 → R>0 , √ f (x) = x, o que
podemos prever que aconteça com a derivada de x quando x > 0
tende a zero?
ii) Usando apenas a definição√de derivada, calcule a derivada da
função f : R>0 → R>0 , f (x) = x (Dica: quando ficar complicado
lidar com a raı́z quadrada, lembre que (a − b)(a + b) = a2 − b2 .)
iii) compare a fórmula obtida em ii) com o que previu em i).
Exercı́cio 4.6. (resolvido)
Seja f : R<0 ∪ R>0 → R, f (x) = x1 .
i) Sem calcular a derivada de f o que se pode pre-dizer do sinal
dessa derivada ? Em que intervalos é positiva ou negativa ? Pode se
anular ?
ii) para calcular a derivada de f via a definição, só é preciso sabe
somar e subtrair duas frações e saber que as funções racionais são
contı́nuas. Calcule-a via definição.
Exercı́cio 4.7. Defino uma função f : R → R condicionalmente
por:
f (x) = 3x2 + 2, se x < 1, e f (x) = 3x + b, se x ≥ 1.
i) Escolha o coeficiente linear b para que f : R → R seja uma função
contı́nua em todos os pontos.
4. EXERCÍCIOS 112

ii) Dá para escolher b de modo que f : R → R além de contı́nua


também fique derivável em todos os pontos ? Ou há algum ponto onde
não haverá derivada ? Por quê ?
iii) com b escolhidos para f ser contı́nua, qual o gráfico de f ′ (x) ?
CAPı́TULO 10

Sinal da derivada e crescimento

1. Teoremas de Rolle e de Lagrange


Tudo que precisamos sobre zeros, crescimento e decrescimento de
funções sai de dois Teoremas: de Rolle e de Lagrange (que de fato são
equivalentes entre si).

Teorema 1.1. (Teorema de Rolle) Seja f : [a, b] → R contı́nua em


[a, b] e derivável em (a, b). Se f (a) = f (b) então existe algum ponto
x ∈ (a, b) tal que f ′ (x) = 0.

Demonstração.
Considere o mı́nimo global mf e o máximo global Mf de f em [a, b].
Se mf = Mf isso quer dizer que f é constante: então para qualquer
ponto de (a, b) temos f ′ (x) = 0 e acabou.
Supomos então que mf < Mf .
Vamos nos convencer agora que não é possı́vel que ambos os valores
mf e Mf sejam valores de f nos pontos extremo a, b de [a, b]. De fato,
se por exemplo f (a) = mf , como por hipótese f (a) = f (b), então
f (b) = mf ; como Mf > mf então Mf será atingido por x ∈ (a, b). Vice
versa se supomos que f (a) = Mf , concluimos que mf é atingido em
x ∈ (a, b).
Agora vamos mostrar que num x ∈ (a, b) onde f (x) = mf ou onde
f (x) = Mf temos que ter f ′ (x) = 0.
Por exemplo, suponha x ∈ (a, b) onde f (x) = mf e por absurdo,
suponha que f ′ (x) 6= 0:
Há dois Casos a considerar:
Caso 1): f ′ (x) < 0.
Já que x vive num intervalo aberto (a, b) existe pela Afirmação 1.2
um intervalo centrado em x,

(−δ0 + x, x + δ0 ) ⊂ (a, b)

e por isso podemos tomar 0 < h < δ0 suficientemente pequeno para


que x + h ∈ (a, b).
Então pela definição de derivada, temos:

f (x + h) − f (x)
lim <0
h→0 h
113
1. TEOREMAS DE ROLLE E DE LAGRANGE 114

e nesse limite h pode ser tomado positivo ou negativo: tomando h


positivo e pequeno temos:
f (x + h) − f (x)
lim < 0,
hց0 h
o que implica que os quocientes incrementais f (x+h)−f
h
(x)
são negativos
para h positivo suficientemente pequeno.
Mas o denominador é h > 0: logo os numeradores são negativos:
f (x + h) − f (x) < 0,
para 0 < h suficientemente pequeno. Portanto, f (x + h) < f (x) para
0 < h suficientemente pequeno. Ora, isso contradiz a hipótese de que
f (x) = mf é mı́nimo global. Essa contradição veio de supor f ′ (x) < 0
nesse x.
A Figura a seguir apenas serve para ilustrar a situação absurda
obtida, onde a reta em vermelho simboliza a tangente ao gráfico em
(x, f (x)) = (x, mf ) (em vermelho).

m_f

x x+h ( h >0 )

Figura: Chegamos num absurdo deste tipo supondo f ′ (x) < 0 em x.

Caso 2): f ′ (x) > 0:


Novamente, já que existe um intervalo centrado em x,
(−δ0 + x, x + δ0 ) ⊂ (a, b),
podemos tomar h < 0 de módulo suficientemente pequeno (|h| < δ0 )
para que x + h ∈ (a, b). Então pela definição de derivada, temos:
f (x + h) − f (x)
lim >0
h→0 h
e tomando h < 0 temos
f (x + h) − f (x)
lim > 0,
hր0 h
o que implica que os quocientes incrementais f (x+h)−f
h
(x)
são positivos
para h < 0 de módulo suficientemente pequeno.
Mas o denominador é h < 0: logo os numeradores são negativos,
ou seja,
f (x + h) < f (x)
CAPÍTULO 10. SINAL DA DERIVADA E CRESCIMENTO 115

para h < 0 de módulo suficientemente pequeno. Contradizendo a


hipótese de que f (x) = mf é mı́nimo global. Essa contradição veio
de supor f ′ (x) > 0 nesse x. Como antes, ilustramos a situação na
Figura que segue1:

m_f

x+h x ( h<0 )

Figura: Chegamos nesse tipo de absurdo supondo f ′ (x) > 0 em x.

Logo concluimos que f ′ (x) = 0.


A prova análoga se f (x) = Mf .


O uso que Rolle fazia desse fato era para localizar zeros (raı́zes) de
polinômios apenas.
Ele pensava assim, sempre que houver duas raı́zes a e b sucessivas
de um polinômio p(x) de grau n tem que haver uma raı́z do polinômio
p′ (x) situada no intervalo [a, b] (veremos na Parte 2 que sempre a função
Derivada de um polinômio é também um polinômio). Mais ainda, como
vimos já em alguns exemplos simples, o grau de p′ (x) é n − 1. Logo
pode ser mais fácil achar as raı́zes de p′ (x) que as do polinômio original
p(x). E aı́ teremos alguma informação sobre a possı́vel localização das
raı́zes a e b de p(x).
(obs.: Na Figura a seguir os eixos horizontal e vertical não estão na
mesma escala)

1A f não precisa ser crescente nessa região, como parece sugerir a Figura; f
precisa apenas valer menos que f (x). Voltaremos nisso na Seção 4 deste Capı́tulo
1. TEOREMAS DE ROLLE E DE LAGRANGE 116

10

0
-2 -1 0 1 2
x

-5

-10

Figura: Polinômio p(x) com 5 raı́zes Reais e p′ (x) com 4 raı́zes Reais.

O Teorema de Rolle pode ser generalizado:

Teorema 1.2. (Teorema do Valor Médio de Lagrange)2


Seja f : [a, b] → R contı́nua e derivável em (a, b). Então existe
algum x ∈ (a, b) tal que
f (b) − f (a)
f ′ (x) =
b−a

0,5

0
-1 -0,5 0 0,5 1
x
-0,5

-1

Figura: O gráfico em vermelho ilustra o Teo. de Lagrange em dois pontos.

Demonstração.
2Atenção: muitos estudantes confundem o que diz o Teorema de Lagrange com
o que diz a definição da Derivada.
CAPÍTULO 10. SINAL DA DERIVADA E CRESCIMENTO 117

Seja p(x) a equação da reta passando por (a, f (a)) e (b, f (b)). Con-
sidere uma nova função, a função diferença f −p dada por (f −p)(x) :=
f (x) − p(x).
Então f − p é contı́nua, pelo item 1) do Teorema 1.1. E afirmo que
ela é derivável em (a, b), com (f − p)′ (x) = f ′ (x) − p′ (x).
De fato:
(f − p)(x + h) − (f − p)(x)
(f − p)′ (x) := lim =
h→0 h
f (x + h) − f (x) p(x + h) − p(x)
= lim ( − )
h→0 h h
e pelo item 1) do Teorema 1.1 esse último limite vale:
f (x + h) − f (x) p(x + h) − p(x)
lim − lim =: f ′ (x) − p′ (x),
h→0 h h→0 h
ou seja, provamos que (f − p)′ (x) = f ′ (x) − p′ (x). Agora noto que
(f − p)(a) = f (a) − p(a) = 0, e (f − p)(b) = f (b) − p(b) = 0,
e portanto estamos em condições de aplicar em (f − p) o Teorema de
Rolle: portanto existe algum x ∈ (a, b) onde
(f − p)′ (x) = 0,
ou seja onde
f ′ (x) = p′ (x).
Por outro lado p(x) = a1 · x + a0 já que é um polinômio de grau ≤ 1 e
sua derivada é o coeficiente angular da reta: p′ (x) ≡ a1 e sabemos que
f (b) − f (a)
a1 = .
b−a
f (b)−f (a)
Portanto f ′ (x) = b−a
como querı́amos.


2. O Teorema 0 das Equações Diferenciais


Para motivar o importante Teorema 2.1, começo descrevendo um
exemplo.
Imagine um motorista que está dirigindo seu carro do Sul para o
Norte numa rodovia e que vê uma placa indicando que dali a alguns
kilômetros há um posto da polı́cia rodoviária. Como é usual, ele começa
a freiar o carro mas o faz assim: começa pisando no freio assim que vê a
placa e vai gradualmente tirando o pé do freio de modo bem cuidadoso,
para que bem em frente do posto da polı́cia esteja acabando de tirar o
pé do freio e passe então para o acelerador, começando a acelerar bem
suavemente e depois aumentando a aceleração.
Freiar e acelerar são tipos de acelerações. Aceleração negativa ao
freiar e positiva quando pisamos no acelerador. Como explicamos na
2. O TEOREMA 0 DAS EQUAÇÕES DIFERENCIAIS 118

Seção 4 do Capı́tulo 8, podemos representar matematicamente o que


o motorista fez com as acelerações através da função segunda derivada
f ′′ (x) (Seção 3 do Capı́tulo 9), onde f ′ (x) é a função que dá a velocidade
a cada instante e f (x) a posição do carro a cada instante. A função
posição será f (x) < 0 ao Sul do posto policial e f (x) > 0 ao Norte do
posto e seu aumento significa ir mais para o Norte.
Quando ele estava pisando no freio, f ′′ (x) < 0, quando pisa no
acelerador, f ′′ (x) > 0. Onde f ′′ (x) < 0, a velocidade f ′ (x) estava
decrescendo, e quando f ′′ (x) > 0 a função velocidade f ′ (x) deve voltar
a crescer.
Um exemplo disso seria:
f (x) = x3 , f ′ (x) = 3x2 , f ′′ (x) = 6x.

10

0
-2 -1 0 1 2
x

-5

-10

Figura: f vermelho, f ′ verde, f ′′ amarelo, escalas diferentes nos eixos.

O que é interessante neste exemplo é que em frente ao posto da


polı́cia, quando x = 0, a velocidade que aparece no velocı́metro é
f ′ (0) = 0 e mesmo assim, em nenhum instante o carro parou, já que
f (x) = x3 é estritamente crecente.
Mas isso contradiz o nosso senso-comum, já que algo que se move
a 0 km/h deveria estar parado, pelo menos por algum tempo !
Para fazermos as pazes com o senso-comum, temos o seguinte Teo-
rema, onde a condição f ′ (x) = 0 se supõe que vale para x em todo um
intervalo, mesmo que pequeno:
Teorema 2.1. Seja f : I → R definida em um intervalo I não-
degenerado.3
Suponha f ′ (x) ≡ 0. Então f (x) ≡ C (ou seja, f é constante).

3Não-degenerado significa não se reduzindo a um ponto. Claro que I pode ser


todo R. Mas atenção que pode a conclusão pode ser falsa, se a f tem o domı́nio
composto de mais de um intervalo (disjuntos).
CAPÍTULO 10. SINAL DA DERIVADA E CRESCIMENTO 119

Demonstração.
Não temos a capacidade de predizer qual a constante que iremos
encontrar. O que podemos apenas é raciocinar por absurdo: suponha
que f não é constante.
Então existem x1 , x2 ∈ I tais que f (x1 ) 6= f (x2 ). Restrinja f ao
domı́nio [x1 , x2 ]. Então pelo Teorema do Valor Médio de Lagrange
aplicado à restrição f : [x1 , x2 ] → R tem que haver um x ∈ (x1 , x2 ) tal
que:
f (x1 ) − f (x2 )
f ′ (x) = .
x 1 − x2
f (x1 )−f (x2 )
Mas x1 −x2
6= 0 e isso contradiz a hipótese de que f ′ (x) ≡ 0.


E dele decorre o Teorema a seguir (que chamo de 0 por um dos mais


básicos):

Teorema 2.2. (O Teorema 0 das Equações Diferenciais) Sejam


f : I → R e g : I → R deriváveis, com f ′ (x) = g ′ (x), ∀x ∈ I, onde I é
um intervalo. Então f (x) ≡ g(x) + C.

Ilustro esse Teorema através da seguinte Figura:

12

0
-1 -0,5 0 0,5 1
x

Figura: Translações verticais de um gráfico e o gráfico da função derivada.

Demonstração.
Como já observamos, ∀x ∈ I, (f − g)′ = f ′ (x) − g ′ (x). A hipótese
dá então que (f − g)′ (x) ≡ 0. Logo pelo Teorema 2.1, (f − g)(x) ≡ C
(é constante) ; logo f (x) ≡ g(x) + C.

4. UMA CONFUSÃO FREQUENTE SOBRE O SINAL DA
DERIVADA 120

3. Critérios de crescimento e de decrescimento


Decorrem facilmente de Rolle e Lagrange os desejados critérios:
Teorema 3.1. (Critérios de crescimento e de decrescimento)
Seja f : I = (a, b) → R derivável.
• i) se ∀x ∈ I, f ′ (x) ≥ 0 então f é crescente em I;
• ii) se ∀x ∈ I, f ′ (x) > 0 então4 f é estritamente crescente em
I.
• iii) se ∀x ∈ I, f ′ (x) ≤ 0 então f é decrescente em I;
• iv) se ∀x ∈ I, f ′ (x) < 0 então f é estritamente decrescente
em I.
Demonstração.
De i): por absurdo suponha que f não é crescente. Significa que
existem x1 , x2 ∈ I com x1 < x2 para os quais:
f (x1 ) > f (x2 ).
Mas então o Teorema do Valor Médio de Lagrange aplicado à restrição
f : [x1 , x2 ] → R dá que existe algum x ∈ (x1 , x2 ) com:
f (x2 ) − f (x1 )
f ′ (x) = < 0,
x 2 − x1
contradizendo a hipótese de que f ′ (x) ≥ 0 ∀x ∈ I.
De ii): Se supomos por absurdo que f não é estritamente crescente,
significa que existem x1 , x2 ∈ I com x1 < x2 para os quais:
f (x1 ) ≥ f (x2 ).
Novamente o Teorema do Valor Médio de Lagrange aplicado a f :
[x1 , x2 ] → R dá que existe algum x ∈ (x1 , x2 ) com:
f (x2 ) − f (x1 )
f ′ (x) = ≤ 0,
x2 − x1
contradizendo a hipótese de que f ′ (x) > 0 ∀x ∈ I.
De iii) e iv): são completamente análogas, mutatis mutandis 5


4. Uma confusão frequente sobre o sinal da derivada

Peço atenção agora, para que se evite uma confusão que aparece
em algumas exposições.
4A recı́proca é falsa, como mostra f (x) = x3
5Essa expressão latina quer dizer, desde que adaptando, mudando, o que for
conveniente; no nosso caso, sinais, desigualdades.
CAPÍTULO 10. SINAL DA DERIVADA E CRESCIMENTO 121

As hipóteses dos itens ii) e iv) do Teorema 3.1 pedem que o sinal
da função derivada seja positivo (ou negativo) em todo um intervalo
aberto I.
Seria falso um enunciado assim:

(falso !) Seja f : (a, b) → R derivável com algum x ∈ (a, b) onde


f (x) > 0 (f ′ (x) < 0). Então existe um intervalo centrado em x onde

a restrição da f é crescente (decrescente).

Claro que isso pode até funcionar em alguns exemplos, mas um


teorema tem que funcionar sempre !
A Figura a seguir ilustra uma função f que é derivável, com f ′ (0) >
0, e que no entanto não é nem crescente nem decrescente em nenhum
intervalo centrado em x (a Figura não mostra bem, mas as oscilações
continuam a existir até a origem). Deduzimos então, após o Teorema
3.1, que a derivada f ′ (x) muda de sinal tão perto de x = 0 quanto
quisermos.

0,08

0,04

0
-0,2 -0,1 0 0,1 0,2
x

-0,04

-0,08

Figura: A função f oscila à esquerda e à direita de x = 0, embora f ′ (0) > 0.

A única propriedade que a f da Figura tem é que:

f vale mais que f (0) em pontos x um pouco maiores que x = 0 e f


vale menos que f (0) em pontos x um pouco menores que x = 0

(é isso nós aprendemos na prova do Teorema de Rolle 1.1). Vamos


destacar isso como uma afirmação:

Afirmação 4.1. Seja uma f derivável e x um ponto do intervalo


aberto I onde f está definida.
Se f ′ (x) > 0 então existe um intervalo J centrado em x, onde
f (x) < f (x) se x < x, x ∈ J e f (x) < f (x) se x < x, x ∈ J.
5. EXERCÍCIOS 122

Se f ′ (x) < 0 então existe um intervalo J centrado em x, onde


f (x) > f (x) se x < x, x ∈ J e f (x) > f (x) se x < x, x ∈ J.

Demonstração.
Contida na demonstração do Teorema de Rolle.


5. Exercı́cios
Exercı́cio 5.1. A figura que exemplifica o T.V.M de Lagrange no
texto é o gráfico de y = x3 . Quando x ∈ [−1, 1] em quais pontos do
gráfico a inclinação da reta tangente é 1 ?

Exercı́cio 5.2. 2) Explique (com os conceitos do Cálculo) o que se


modifica e o que não se modifica nos gráficos a seguir quando variamos
o parâmetro b 6= 0 em:
i): y = fb (x) = bx2
ii) y = fb (x) = x2 + b
iii) y = fb (x) = x2 + bx − 1.
(Obs.: nos itens i) e iii) há certos pontos em que se vê bem as
diferenças entre os gráficos).

Exercı́cio 5.3. Encontre o ponto (ou os pontos) do gráfico de


y = (x − 1)3 em que sua(s) reta(s) tangente(s) é (são) paralela(s) à reta
y = 3x.
Encontre o ponto (ou os pontos) do gráfico de y = x3 em que sua(s)
reta(s) tangente(s) é (são) ortogonal (s) à reta y = − 61 x.
Obs. Não precisa desenhar nada.

Exercı́cio 5.4. (resolvido)


Considere a famı́lia de gráficos

y = fb (x) := (−b + 4/3) · x2 + b · x + (2b − 7/3), b ∈ R,

dos quais plotei apenas 7 representantes (b = 1, 1.2, 1.3, 4/3, 1.6, 1.8, 2):
CAPÍTULO 10. SINAL DA DERIVADA E CRESCIMENTO 123

x
-3 -2 -1 0 1 2 3 4
0

-5

-10

Como se vê são gráficos bem diferentes, à medida que mudamos


o parâmetro b. Mas quando fiz um zoom na região x ∈ [0.3, 0.7] do
domı́nio, obtive pedaços dos 7 gráficos de y = fb (x) que se parecem
muito:
2,5

1,5

0,5

0
0,4
0,5
0,6
0,7
x

Explique o que aconteceu quando fizemos o zoom, após confirmar que


que os pontos (−1, −1) e (2, 3) pertencem a esses gráficos todos, ∀b ∈
R).
Dica: Teorema Valor Médio de Lagrange.
Exercı́cio 5.5. Na Seção 4 apresento um exemplo (sem dar ainda
a equação explı́cita) de uma função derivável, com f ′ (x) > 0 num
ponto x mas que não é crescente nem decrescente em nenhum intervalo
centrado em x.
Pondo isso ao lado dos itens i) ou ii) do Teorema 3.1, qual a con-
clusão que se deve tirar sobre a função derivada f ′ (x)? Será que f ′ (x)
pode ser contı́nua em x ?
CAPı́TULO 11

Critérios para máximos e mı́nimos e Aplicações

1. Primeiro critério
Se olharmos bem a demonstração que demos do Teorema de Rolle,
veremos que de fato já provamos o seguinte:

Afirmação 1.1. Seja f : (a, b) → R derivável. Se1 x ∈ (a, b) é


ponto de Mı́nimo Local ou de Máximo Local, então f ′ (x) = 0.

A recı́proca dessa Afirmação é em geral falsa: f (x) = x3 tem f ′ (0) =


0 e x = 0 não é nem Mı́nimo nem Máximo local.
No entanto temos o seguinte:

Afirmação 1.2. Seja f : (a, b) → R derivável, com x ∈ (a, b) onde


f ′ (x) = 0.
• i) Suponha que existe um intervalo J centrado em x onde a
função derivada vale f ′ ≤ 0, se x < x, e f ′ ≥ 0, se x < x.
Então x é Mı́nimo Local da f .
• ii) Suponha que que existe um intervalo centrado em x onde a
função derivada vale f ′ ≥ 0, se x < x, e f ′ ≤ 0, se x < x. .
Então x é Máximo Local da f .

Demonstração.
De i): Temos que f ′ (x) ≤ 0 se x ∈ (−δ + x, x) e f ′ (x) ≥ 0 se
x ∈ (x, x + δ).
Mas então pelo item iii) do Teorema 3.1, a função original f (x) é
decrescente em (−δ + x, x). E pelo item i) do Teorema 3.1 a função
original f (x) é crescente em (x, x + δ).
A conclusão é que x é ponto de Mı́nimo da f restrita a (−δ+x, x+δ),
um Mı́nimo local portanto.
De ii): completamente análoga, mutatis mutandis.


1Émuito importante que (a, b) seja aberto, pois f : [0, 1] → R, f (x) = x tem
pontos de máximo e mı́nimo e no entanto f ′ (0) = f ′ (1) = 1, onde essas derivadas
′ ′
devem ser entendidas como derivadas à direita f+ (0) e à esquerda f− (1).
125
2. CRITÉRIO DA SEGUNDA DERIVADA 126

2. Critério da segunda derivada


Primeiro vamos relembrar e reforçar o tema da segunda derivada
ou aceleração instantânea em termos fı́sicos.
Para definir uma aceleração instantânea usamos um limite do tipo:
f ′ (x + h) − f ′ (x)
lim ,
h→0 h

onde f (x) é a função velocidade instantânea (e onde a f (x) de partida
era a função posição em cada instante).
Segundo a definição de derivada, o que fizemos lá foi derivar a
função f ′ (x), ela mesma já uma derivada da função f (x). Fizemos
então uma segunda derivada:
f ′′ (x) := ( f ′ (x) )′ .
Sua definição então é essencialmente a mesma que demos para a derivada
(que passamos agora a chamar de primeira derivada), só que a matéria-
prima para compôr os quocientes incrementais não é uma função f (x)
mas sim uma função f ′ (x).
Desse modo, posso enunciar:
Afirmação 2.1. Seja f : (a, b) → R derivável, tal que f ′ (x)
também seja derivável.
• i): se f ′ (x) = 0 e f ′′ (x) > 0 então2 x é Mı́nimo local da f
original.
• ii): se f ′ (x) = 0 e f ′′ (x) < 0 então x é Máximo local da f
original.
Este teorema será generalizado na Afirmação 8.1, um critério da
derivada n-ésima.
Demonstração. (da Afirmação 2.1)
De i): Pela Afirmação 4.1 do Capı́tulo 10, aplicada agora à função
derivada f ′ (x), temos que para x ∈ J centrado em x, f ′ (x) < 0 = f ′ (0)
se x < x e 0 = f ′ (x) < f ′ (x) se x < x.
Então recaı́mos exatamente no item i) da Afirmação 1.2. A con-
clusão portanto é que x é Mı́nimo local.

De ii): completamente análoga, mutatis mutandis.




Com o material deste Capı́tulo 11 e do Capı́tulo anterior 10 esta-


mos em condições de confeccionar gráficos qualitativamente corretos de
polinômios simples, de grau baixo, e é o que faremos como Exercı́cio.
2Recı́proca
falsa: f (x) = x4 tem Mı́nimo local em x = 0 e se pode provar que
′ ′′
f (0) = f (0) = 0
CAPÍTULO 11. CRITÉRIOS PARA MÁXIMOS E MÍNIMOS E
APLICAÇÕES 127

3. Um problema tı́pico para os engenheiros

Suponha que você tem o seguinte problema prático:

Construir um objeto retangular, onde a construção de cada x met-


ros da largura custa a metade da construção de cada z metros de com-
primento. Gastando 10 reais na fabricação de cada unidade, quais as
medidas de x e z que maximizam a área do objeto?
Traduzimos o problema assim: queremos maximizar a área
A(x, z) := z · x
com uma função custo 3 c(x, z) := x + 2z fixada em c(x, z) = 10:
x + 2z = 10.
Note que a princı́pio a função área depende tanto de x como de z. Mas
a condição c(x, z) = 10 me permite escrever z = 10−x 2
e a função área
como dependendo só de uma variável:
10 − x x2
A(x) = ( ) · x = 5x − .
2 2
O domı́nio natural de A(x) é I = (0, 10), pois a largura x tem que ser
positiva, e ao mesmo tempo a condição c(x, z) = 10 diz que, quando z
se aproxima de zero, x se aproxima de 10.
Mas considerar A(x) definida num domı́nio um pouco maior, o inter-
valo [0, 10], que tem a vantagem de ser um intervalo limitado e fechado,
onde podemos usar o Teorema 4.2 de Bolzano-Weiersstras, já que A(x)
claramente é contı́nua.
Esse Teorema garante que existe um ponto de Máximo global de
A : [0, 10] → R. Mas onde ? Não adianta só sabermos que há uma
solução, queremos achá-la !
Certamente não será em x = 0 ou em x = 10, pois nesses pontos a
Área fica zero, já que não largura ou comprimento. Então esse ponto
x buscado está em (0, 10), o que é promissor, pois poderemos tentar
usar a Afirmação 1.2.
Para isso precisamos examinar alguns candidatos.
Conforme a Afirmação 1.1, eles terão que ser pontos onde
A′ (x) = 0.
x2
Ora, isso significa para A(x) = 5x − 2
que:
5 − x = 0,
pelo que já sabemos das derivadas, ou seja, o ponto é x = 5.
3Também poderia dizer que a função custo é 2x + 4z, já que há dois lados
que são largura e dois que são comprimento. Mas a solução seria completamente
análoga.
4. MÍNIMOS DE DISTÂNCIAS E ORTOGONALIDADE 128

Mas claramente A′ (x) = 5 − x > 0 se x < 5 e A′ (x) = 5 − x < 0


se 5 < x. Logo o item ii) da Afirmação 1.2 diz que realmente x é
um Máximo local e portanto o Máximo global, já que não há outro
candidato. A área máxima desses objetos então será
25
A(5) = .
2

12

10

0
0 2 4 6 8 10
x

x2
Figura: O gráfico de A : [0, 10] → R, A(x) = 5x − 2
.

Em geral, nos problemas desse tipo, aparecem diferentes candidados


a Máximos global, que foram aprovados no teste para Máximos locais
dado pelo item ii) da Afirmação 1.2, e então se faz necessário comparar
os valores da função em questão em cada um deles.

4. Mı́nimos de distâncias e Ortogonalidade

Suponha que P = (2, 1) e queremos descobrir qual o menor seg-


mento de reta de P até uma reta de equação y = ax + 1 (com algum
a 6= 0 fixado) que não passe por P .
Vamos fazê-o de dois modos distintos, que esperamos que dêem os
mesmos resultados.
Primeiro vamos usar nossa intuição, que diz que deve se tratar do
segmento saindo de P que é ortogonal à reta y = ax + 1. Ou seja,
pelo que aprendemos na Seção 2 do Capı́tulo 8, deve ser um ponto
(x, ax + 1) tal que:
(ax + 1) − 1 −1
= ,
x−2 a
pois o lado esquerdo é o ceoeficiente angular da reta contendo o seg-
mento que sai de (2, 1). Então disso obtemos:
2
x= 2
a +1
e daı́ facilmente descobrimos o tamanho do segmento.
Por outro lado podemos, via as técnicas de Cálculo, tentar descobrir
o mı́nimo da função que mede a distância de P aos pontos da reta dada.
Para não cairmos numa derivada mais complicada, vamos modificar
um pouco o problema, tentando minimizar a função que é o quadrado
CAPÍTULO 11. CRITÉRIOS PARA MÁXIMOS E MÍNIMOS E
APLICAÇÕES 129

da distância de P à reta, que na prática dará também o ponto que


minimiza a distância.
Essa função quadrado da distância é dada por:
(x − 2)2 + (y − 1)2 = (x − 2)2 + (ax + 1 − 1)2 =
= (a2 + 1)x2 − 4x + 5.
Então essa f (x) = (a2 +1)x2 −4x+5 tem derivada f ′ (x) = 2(a2 +1)x−4
e f ′ (x) = 0 exatamente em x = a22+1 , o mesmo ponto encontrado acima.
É claro que f ′ (x) < 0 para x < x = a22+1 e f ′ (x) > 0 para x > x =
2
a2 +1
. Portanto pelo item i) da Afirmação 1.2 f tem mı́nimo local, que
de fato é o global nesse ponto x.
Agora vejamos um Exemplo mais interessante. Quero minimizar a
2
distância entre P = (0, 7) e os pontos da parábola y = x2 .
Usando a intuição geométrica vou buscar esse ponto Q de mı́nima
distância entre aqueles em que o segmento desde P é ortogonal à tan-
gente da parábola em Q.
Então, já que conheço as inclinações das tangentes à parabola em
(x, ax2 ) como sendo 2( x2 ) = x, a ortogonalidade que busco é dada por:
x2
2
−7 −1
= ,
x−0 x
ou seja,
x2
x·( − 6) = 0.
2
A solução x = 0, onde claramente há ortogonalidade, é nitidamente
um ponto de máximo local√ da distância
√ entre P = (0, 7) e a parábola.
Mas as soluções x = 12 e x = − 12 corresponderão, como vere-
mos a seguir, a dois pontos de mı́nimos. A Figura a seguir mostra esses
pontos de ortogonalidade.

5
x
-4 -2 0 2 4
0

-5

-10

-15

-20
5. CONCAVIDADES 130

Figura: No gráfico aparecem dois pontos onde há ortogonalidade.

Visto de outro modo, via a técnica do Cálculo, considero o quadrado


da função distância entre P = (0, 7) e a parábola:
x2
(x − 0)2 + (y − 7)2 = x2 + ( − 7)2 =
2
x4
= − 6x2 + 49.
4
x4
A derivada de f (x) = 4
− 6x2 + 49 é
f ′ (x) = x3 − 12x = x(x2 − 12).
O zero da derivada em x = 0 corresponde √ a um máximo
√ local.
Verificamos agora que os pontos x = 12 e x = − 12 são mı́nimos
locais (e globais). √
Observe
√ que se 0 < x < 12 temos x(x2 − 12) < 0, enquanto que
se x > √ 12 temos x(x2 − 12) > 0. Logo o item i) da Afirmação 1.2 diz
que x = 12 é mı́nimo√ de f . √
Agora se x < − 12 temos x(x2 −12) > 0, enquanto que se − 12 <
x < 0√ temos x(x2 − 12) > 0. Logo o item i) da Afirmação 1.2 diz que
x = − 12 é mı́nimo de f .

5. Concavidades
Na Definição 5.1 a seguir só me interesso no comportamento da
função próxima a cada um dos pontos de seu gráfico.
Definição 5.1. Diremos que uma função é localmente côncava
para cima num ponto (x, f (x)) de seu gráfico se existe um intervalo Ix
centrado em x em que
f (x) > ax + b, ∀x ∈ Ix \ {x},
onde y = ax + b é a reta tangente ao gráfico em (x, f (x)).
Para definir localmente côncava para baixo num ponto (x, f (x))
basta trocar > por <.

2
x
-2 -1 0 1 2
0

-2

-4

-6
CAPÍTULO 11. CRITÉRIOS PARA MÁXIMOS E MÍNIMOS E
APLICAÇÕES 131

Figura: Um função localmente côncava para cima em cada ponto do domı́nio


Afirmação 5.1. Suponha uma função f : I → R duas vezes de-
rivável.
• i) Se ∀x ∈ I, f ′′ (x) > 0 então, f é localmente côncava para
cima em cada um dos pontos de seu gráfico.
• ii) Se ∀x ∈ I, f ′′ (x) < 0 então f tem localmente côncava para
baixo em cada um dos pontos de seu gráfico.
Demonstração.
De i):
Tome um ponto (x, f (x)) do gráfico. Seja y = ax + b a equação da
reta tangente ao gráfico nesse ponto.
Note que a função
φ(x) := f (x) − (ax + b)
tem
φ(x) = 0 e φ′ (x) = f ′ (x) − a = 0.
Ademais
φ′′ (x) = f ′′ (x) > 0.,
já que supomos que sempre f ′′ (x) > 0.
Então o Critério da Segunda Derivada (Afirmação 2.1, Capı́tulo 11)
quando aplicado a φ diz que φ tem um mı́nimo local em x (local pois
φ tem que ser restrita a um intervalo Ix centrado em x para ter aı́ um
ponto de mı́nimo).
Ou seja,
φ(x) > φ(x), ∀x ∈ Ix \ {x},
que significa
f (x) > ax + b, ∀x ∈ Ix \ {x},
como querı́amos provar.

De ii): Análogo, bastando usar o Critério da Segunda Derivada


para ter um máximo local.


Na Definição 5.2 a seguir impomos um comportamento global sobre


a função: ela terá que ficar por cima (ou por baixo) de todas as retas
tangentes a seu gráfico.
Definição 5.2. Direi que uma função f : I → R é côncava para
cima se para todo ponto x ∈ I,
f (x) > ax + b, ∀x ∈ I \ {x}
onde y = ax + b é a reta tangente ao gráfico em (x, f (x)).
5. CONCAVIDADES 132

25

20

15

10

0
-3 -2 -1 0 1
x
-5

Figura: Um função que não é côncava para cima, mas que


é localmente localmente côncava para cima se x < 0.
Afirmação 5.2. Suponha uma função f : I → R duas vezes de-
rivável.
• i) Se ∀x ∈ I f ′′ (x) > 0 então f é côncava para cima.
• ii) Se ∀x ∈ I f ′′ (x) < 0 então f é côncava para baixo.
Demonstração.
De i):
Vamos fazer a prova por absurdo.
Pela Afirmação 5.1 sabemos f é localmente concava para cima em
cada ponto de seu domı́nio. Ou seja, dado qualquer x ∈ I existe um
intervalo Ix centrado nele onde
f (x) > ax + b, ∀x ∈ Ix \ {x},
para y = ax + b reta tangente em (x, f (x)).
Portanto, se pensamos esta demonstração por absurdo, tem que
existir4 algum ponto (x, f (x)) para o qual existe um x0 ∈
/ Ix tal que
f (x0 ) ≤ ax0 + b,
para y = ax + b reta tangente em (x, f (x)).
Sem perda de generalidade suponhamos x0 > x.
Faço agora uma alteração na f , para que a reta tangente a (x, f (x))
seja horizontal. Defino
φ(x) := f (x) − (ax + b).
Note que φ(x) = φ′ (x) = 0, mas φ′′ (x) = f ′′ (x) > 0, ∀x ∈ I. Agora
temos
φ(x0 ) ≤ 0.
Caso φ(x0 ) = 0:
Nesse caso, aplico o Teorema de Rolle a
φ : [x, x0 ] → R
4Confira um exemplo disso na Figura anterior, com x ∼ −0.5 e x0 ∼ 1
CAPÍTULO 11. CRITÉRIOS PARA MÁXIMOS E MÍNIMOS E
APLICAÇÕES 133

e obtenho um ponto ξ ∈ (x, x0 ) onde φ′ (ξ) = 0.


Mas ξ > x e isso contradiz o fato que φ′ (x) é uma função estrita-
mente crescente (já que φ′′ (x) > 0), que partiu do valor φ′ (x) = 0.

Caso φ(x0 ) < 0:


Pelo que vimos na Afirmação 5.1, perto de x temos φ(x) > 0.
Como φ(x) é contı́nua e φ(x0 ) < 0 então o T.V.I. diz que há um
ponto x̂0 ∈ [x, x0 ] onde φ(x̂0 ) = 0. Portanto com esse novo x̂0 recaio
na situação do Caso φ(x̂0 ) = 0 já tratado.

De ii): completamente análoga. 

6. Mı́nimos quadrados e a média aritmética


Dados x1 , . . . , xk pontos na Reta dos Reais, que ponto x minimiza
a soma dos quadrados das distâncias a todos eles ?
O interesse prático desta questão é que os valores x1 , . . . , xk podem
ter sido obtidos após k aferições de um certo dado relevante (o com-
primento de um objeto, uma temperatura, um peso, etc) e o ponto x
servirá para corrigir os prováveis erros nas aferições.
Afirmação 6.1. Sejam dados x1 , . . . , xk ∈ R pontos. Então
• i) o ponto de mı́nimo global da função
f (x) := (x − x1 )2 + . . . + (x − xk )2
é o ponto
x1 + . . . + xk
x= ,
k
chamado de média arimética dos valores x1 , . . . xk .
• ii) sempre vale a desigualdade
k · (x21 + . . . + x2k ) > (x1 + . . . + xk )2
exceto se x1 = . . . = xk , quando vale então:
k · (x21 + . . . + x2k ) = (x1 + . . . + xk )2 .
Demonstração.
Item i)
Trata-se então de minimizar a função:
y = f (x) := (x − x1 )2 + . . . + (x − xk )2 .
que é uma parábola com concavidade para cima, já que:
f (x) = k · x2 − 2 · (x1 + . . . xk ) · x + (x21 + . . . + x2k ).
Portanto seu mı́nimo está onde f ′ (x) = 0, ou seja, na raı́z de:
2k · x − 2 · (x1 + . . . xk ) = 0,
6. MÍNIMOS QUADRADOS E A MÉDIA ARITMÉTICA 134

ou seja, em
x1 + . . . + xk
x=
k
que é chamada de média arimética dos valores x1 , . . . xk .

Item ii)
Note que, por ser uma soma de quadrados,
y = f (x) = (x − x1 )2 + . . . + (x − xk )2 ≥ 0
e se para algum x0 ∈ R temos f (x0 ) = 0 então
(x0 − x1 )2 + . . . + (x0 − xk )2 = 0 ⇔ x0 = x1 = . . . = xk .
Portanto, se algum xi é diferente de algum outro xj , na lista que demos
de x1 , . . . , xk , a equação quadrática em x:
y = f (x) = k · x2 − 2 · (x1 + . . . xk ) · x + (x21 + . . . + x2k ) = 0
não tem solução Real. Ou seja, se seu discriminante é negativo. Mas
esse discriminante é:
(2 · (x1 + . . . xk ))2 − 4 · k · (x21 + . . . + x2k ) < 0,
ou seja,
(x1 + . . . xk )2 < k · (x21 + . . . + x2k ),
como querı́amos.


6.1. Retas de ajuste.


Agora trato de um problema parecido, mas diferente. Que só será
considerado no caso geral na Seção 3 do Capı́tulo 31.
Considere o quadrado da distância vertical de um ponto (x1 , y1 ) a
uma reta y = ax + b, ou seja:
(ax1 + b − y1 )2 ≥ 0
e = 0 exatamente quando (x1 , y1 ) está na reta.
Suponhamos que queremos encontrar a reta pela origem y = ax
(não vertical) que minimiza a soma dos quadrados das distâncias ver-
ticais até k pontos (x1 , y1 ), . . . (xk , yk ) (não todos os xi iguais a zero).
Denote as retas pela origem por y = ξx para deixar claro que a
incógnita agora é o coeficiente angular ξ.
E faça a função que dá a soma de quadrados de distâncias verticais:
f (ξ) := (ξx1 − y1 )2 + . . . + (ξxk − yk )2 .
Note que
f (ξ) = (x21 + . . . + x2k ) · ξ 2 − 2(x1 y1 + . . . + xk yk )ξ + y12 + . . . + yk2 .
Então f (ξ) é uma parábola com concavidade para cima, já que
x21 + . . . + x2k > 0
CAPÍTULO 11. CRITÉRIOS PARA MÁXIMOS E MÍNIMOS E
APLICAÇÕES 135

(se esse número fosse zero todos os pontos tem coordenada x igual a
zero).
Portanto se procuramos por um mı́nimo de f basta procurarmos
onde f ′ (ξ) = 0. Mas:
f ′ (ξ) = 2(x21 + . . . + x2k ) · ξ − 2(x1 y1 + . . . + xk yk ),
e portanto f ′ (ξ) = 0 se dá em:
x1 y1 + · · · + xk yk
ξ= .
x21 + . . . + x2k
Ou seja a reta a ser escolhida é:
x1 y1 + · · · + xk yk
y=( ) · x.
x21 + . . . + x2k
O problema interessante em geral é quando a reta buscada forma
y = ξx + τ não precisa passsar pela origem.
Essa reta aproximará simultâneamente vários pontos, que podem
ser resultado de aferições de dados relevantes.
O Capı́tulo 31 tratará de uma reta que minimiza soma de quadrados
de distâncias verticais de pontos xi , yi de interesse na Biologia, e cujo
coeficiente angular ξ é universal.

7. Inflexões
Definição 7.1. Seja f com segunda derivada f ′′ (x) e tal que f ′′ (x)
seja ao menos contı́nua.
Chamamos de ponto de inflexão um ponto x onde f ′′ (x) = 0 e em
torno do qual muda o sinal da f ′′ (x).
Ou seja, um ponto de inflexão marca a mudança de concavidade
de uma função (se era para cima, vira para baixo e vice-versa).

Exemplos:
• y = f (x) = x3 , que tem f ′′ (x) = 6x e ponto de inflexão em
x = 0.
• em geral, y = f (x) = x2n+1 , ∀n ∈ N, têm inflexão em x = 0,
já que
f ′′ (x) = 2n · (2n + 1) · x2n−1 .
• o gráfico de y = f (x) (em vermelho) na Figura a seguir repre-
senta a população de bactérias colocada num meio favorável,
no tempo x.
A taxa de crescimento f ′ (x) (em verde) vai aumentando
até atingir um valor máximo (no ponto de inflexão x ≈ 1.1.),
a partir do qual fatores como escassez de nutrientes, aumento
de detritos, começam a diminuir essa taxa de crescimento.
No ponto de inflexão a aceleração f ′′ (x) do processo (em
amarelo) é nula.
8. CRITÉRIO DA DERIVADA DE ORDEM N 136

2
x
0 0,5 1 1,5 2 2,5 3
0

-2

-4

-6

A função f (x) será dada explicitamente na Seção 1 do


Capı́tulo 26.

8. Critério da derivada de ordem n


Uma função como y = f (x) = sin4 (x) claramente tem um ponto de
mı́nimo local em x = 0, já que se anula em zero e é positiva por perto.
No entanto
f ′′ (x) = 4 sin(x)2 · (4 cos(x)2 − 1) e f ′′ (0) = 0,
por isso não está ao alcance do critério da segunda derivada (Afirmação
2.1). Também
f ′′′ (x) = 8 sin(x) cos(x) · (8 cos(x)2 − 5)
se anula em x = 0, porém:
f (iv) (x) = 256 cos(x)4 − 272 cos(x)2 + 40
tem valor f (iv) (0) = 24.

A Afirmação 2.1 se generaliza assim:


Afirmação 8.1. Suponha f : (a, b) → R com derivadas de todas
as ordens5. Seja n ∈ N.
i) se f ′ (x) = f ′′ (x) = . . . = f (2n−1) (x) = 0 mas f (2n) (x) > 0 então
x é ponto de mı́nimo local.
ii) se f ′ (x) = f ′′ (x) = . . . = f (2n−1) (x) = 0 mas f (2n) (x) < 0 então
x é ponto de máximo local.
5Não confunda a derivada de ordem n, f (n) , com a potência n-ésima f n .
CAPÍTULO 11. CRITÉRIOS PARA MÁXIMOS E MÍNIMOS E
APLICAÇÕES 137

ii) se f ′ (x) = . . . = f (2n) (x) = 0 mas f (2n+1) (x) 6= 0 então x é ponto


de inflexão.
Demonstração.
Item i):
A prova completa seria ∀n ∈ N e aı́ então a indução matemática
seria exigida. Por isso, para simplificar mas mesmo assim dar uma
ı́déia da prova, me atenho ao primeiro caso relevante, ou seja quando
n = 2.
Temos por hipótese:
f ′ (x) = f ′′ (x) = f ′′′ (x) = 0 mas f (iv) (x) > 0.
Como há derivadas de todas as ordens, a função f (iv) (x) é contı́nua
em x, pois é até mesmo derivável. Logo pelo princı́pio de inércia das
funções contı́nuas, existe um intervalo Ix = (−δ + x, x + +δ) centrado
em x tal que
f (iv) (x) > 0, ∀x ∈ Ix .
Então no intervalo Ix a função f ′′′ (x) é uma função estritamente cres-
cente. Como por hipótese f ′′′ (x) = 0, concluimos que:
f ′′′ (x) < 0 em (−δ + x, x) e f ′′′ (x) > 0 em (x, x + δ).
Ou seja que a função f ′′ (x) é estritamente decrescente em (−δ + x, x)
e f ′′ (x) é estritamente crescente em (x, x + δ). Como f ′′ (x) = 0 isso
diz que:
f ′′ (x) > 0 em (−δ + x, x) ∪ (x, x + δ).
Agora então f ′ (x) é estritamente crescente em (−δ + x, x) ∪ (x, x + δ).
Como f ′ (x) = 0 temos que
f ′ (x) < 0 em (−δ + x, x) e f ′ (x) > 0 em (x, x + δ).
Por último isso diz que f é estritamente decrescente em (−δ + x, x) e
f é estritamente crescente em ((x, x + δ). Logo x é ponto de mı́nimo.

Iem ii): Análogo, mutatis mutandis.

Item iii):
Temos por hipótese:
f ′ (x) = f ′′ (x) = f ′′′ (x) = f (iv) (x) = 0
mas f (v) (x) 6= 0. Por exemplo suponhamos
f (v) (x) > 0.
o caso negativo é análogo.
Como há derivadas de todas as ordens, a função f (v) (x) é contı́nua
em x, pois é até mesmo derivável. Logo pelo princı́pio de inércia das
9. CONFECÇÃO DE GRÁFICOS DE POLINÔMIOS E FUNÇÕES
RACIONAIS 138

funções contı́nuas, existe um intervalo Ix = (−δ + x, x + +δ) centrado


em x tal que
f (v) (x) > 0, ∀x ∈ Ix .
Então no intervalo Ix a função f (iv) (x) é uma função estritamente cres-
cente. Como por hipótese f (iv) (x) = 0, concluimos que:
f (iv) (x) < 0 em (−δ + x, x) e f (iv) (x) > 0 em (x, x + δ).
Ou seja que a função f ′′′ (x) é estritamente decrescente em (−δ + x, x)
e f ′′′ (x) é estritamente crescente em (x, x + δ). Como f ′′′ (x) = 0 isso
diz que:
f ′′′ (x) > 0 em (−δ + x, x) ∪ (x, x + δ).
Agora então f ′′ (x) é estritamente crescente em (−δ + x, x) ∪ (x, x + δ).
Como f ′′ (x) = 0 temos que
f ′′ (x) < 0 em (−δ + x, x) e f ′′ (x) > 0 em (x, x + δ).
Por definição, x é um ponto de inflexão.


9. Confecção de gráficos de polinômios e funções racionais


Considere a função polinomial y = f (x) = x3 − x.
O objetivo é fazer seu gráfico, de modo qualitativamente correto,
sem qualquer calculadora.
Primeiro noto onde f = 0, onde f > 0 ou f < 0 (pois essas in-
formações não serão fornecidas pela f ′ (x)).
Ora f (x) = x · (x2 − 1) e daı́ sai que
• f (x) = 0 exatamente para x = 0, −1, 1;
• f (x) > 0 para −1 < x < 0 ou x > 1;
• f (x) < 0 para x < −1 ou 0 < x < 1.
A derivada é f ′ (x) = 3x2 − 1 e portanto
q q
• f ′ (x) = 0 em x = 13 , − 13 .
q q
• f (x) > 0 se x > 3 ou x < − 13 .
′ 1
q q
• f ′ (x) < 0 se − 13 < x < 13 .
• f ′ (0) = −1
q
1
Essas informações sobre f ′ (x) já dizem que x = 3
é ponto de
q
mı́nimo local de f (x) e que x = − 13 é ponto de máximo local de
q q
f (x). E também que f é crescente se x > 3 ou x < − 13 e que f (x)
1
q q
é decrescente se − 13 < x < 13 . Por último, f ′ (0) = −1 diz que o
gráfico perto da origem se parece com y = −x.
CAPÍTULO 11. CRITÉRIOS PARA MÁXIMOS E MÍNIMOS E
APLICAÇÕES 139

Agora f ′′ (x) = 6x, ou seja f ′′ (0) = 0, e em x = 0 há mudança


de sinal da f ′′ (x). Logo x = 0 é ponto de inflexão. Para x < 0 a
concavidade de f é para baixo e para x > 0 a concavidade de f é para
cima.
A Figura a seguir recolhe essas informações, mas como as escalas
são diferentes nos dois eixos a informação f ′ (0) = −1 não é respeitada:

0
-1,5 -1 -0,5 0 0,5 1 1,5
x

-4

-8

Figura: y = f (x) = x3 − x (verm.), f ′ (x) (verde), f ′′ (x) (amar.)

Os Exercı́cios 10.5 10.6 no final do Capitulo desafiarão o leitor a


fazer gráficos qualitativamente corretos de polinômios, sem usar nen-
huma calculadora.
Agora vamos fazer o gráfico da função racional
x3 + 8x
f : R \ {−1, 1} → R, f (x) = 2 .
x −1
Novamente queremos estar corretos apenas qualitativamente.
Como o numerador de f (x) é x · (x2 + 8), temos que f (x) = 0
exatamente se x = 0. O numerador de f é negativo se x < 0 e positivo
se x > 0. Já o denominador de f (x) é negativo se −1 < x < 1 e
positivo no resto do domı́nio.
Ou seja,
• f (x) = 0 exatamente se x = 0;
• f (x) > 0 se −1 < x < 0 ou x > 1.
• f (x) < 0 se x < −1 ou se 0 < x < 1.
Não é difı́cil ver que:
lim f (x) = −∞ lim f (x) = +∞,
xր−1 xց−1

lim f (x) = −∞ lim f (x) = +∞.


xր1 xց1
Agora examino (derivando pela regra do quociente):
x4 − 11x2 − 8
f ′ (x) = .
(x2 − 1)2
9. CONFECÇÃO DE GRÁFICOS DE POLINÔMIOS E FUNÇÕES
RACIONAIS 140

O numerador é do tipo z 2 − 11z − 8, com z = x2 .


Então f ′ (z) = 0 exatamente se
p √ √
11 ± (11)2 + 4 · 8 11 ± 153 11 ± 3 · 17
z= = = .
2 2 2

Mas 11−3·2 17 < 0, portanto, se queremos determinar x ∈ R onde
f ′ (x) = 0, devemos tomar:
s √
11 + 3 · 17
x=± .
2

√ q √ √
Podemos aproximar grosseiramente 17 ≈ 4 e 11+3·2 17 ≈ 15 ≈ 3.
Ou seja que a derivada f ′ (x) se anula num ponto x1 ≈ 3 e noutro
x2 ≈ −3.
Antes de examinar f ′′ (x), note que não é difı́cil se convencer de que:

lim f (x) = +∞,


x→+∞

Como limxց1 f (x) = +∞ isso indica que x1 ≈ 3 é ponto de mı́nimo


local da f (sem usar qualquer teste).
Por outro lado como

lim f (x) = −∞
x→−∞

e limxր−1 f (x) = −∞, isso indica que x2 ≈ −3 é máximo local da f


(sem usar qualquer teste).
Agora, com a regra da derivada do quociente, da composta e após
simplificações, obtemos:

18x(x2 + 3)
f ′′ (x) = .
(x2 − 1)3

Claramente f ′′ (x) se anula apenas em x = 0 e nesse ponto muda de


sinal. Logo x = 0 é um ponto de inflexão.
Para −1 < x < 0 ou para x > 1 temos f ′′ (x) > 0 e concavidade
para cima.
Mas para x < −1 ou 0 < x < 1 temos concavidade para baixo.
Em particular, f ′′ (x1 ) > 0 e f ′′ (x2 ) < 0 o que comprova que são
mı́nimo e máximo locais respectivamente.
As três Figuras a seguir resumem essas observações: a primeira
pega parte da região x < −1, a segunda, parte da região −1 < x < 1
e a terceira, parte da região x > 1.
CAPÍTULO 11. CRITÉRIOS PARA MÁXIMOS E MÍNIMOS E
APLICAÇÕES 141

x
-5 -4,5 -4 -3,5 -3 -2,5 -2 -1,5

-7

-8

-9

-10

-11

-12

x3 +8x
Figura: O gráfico de y = x2 −1
, x ∈ [−5, −1.5].

15

10

0
-0,8 -0,4 0 0,4 0,8

-5x

-10

-15

x3 +8x
Figura: O gráfico de y = x2 −1
, x ∈ [−0.8, 0.8].

12

11

10

2 3 4 5 6 7
x

x3 +8x
Figura: O gráfico de y = x2 −1
, x ∈ [1.5, 5].
10. EXERCÍCIOS 142

10. Exercı́cios
2
Exercı́cio 10.1. 3) Encontre o ponto do gráfico de y = x2 que
minimiza a distância até P = (2, 1) pelos metodos i): de buscar pontos
de ortogonalidade com o gráfico e ii): via mı́nimo da função quadrado
da distância.
Exercı́cio 10.2. 4) As Figuras i) e ii) abaixo dão dois exemplos
de funções derivadas f ′ (x), apenas dadas qualitativamente. Encontre
f (x) (qualitativamente) que sejam compatı́veis com cada f ′ dada.

0
-3 -2 -1 0 1 2 3
x
-2

-4

-6

Figura i): Gráfico de uma função derivada f ′ .

15

10

5
x
-2 -1 0 1 2 3 4
0

-5

-10

-15

-20

Figura ii): Gráfico de uma função derivada f ′ .


Exercı́cio 10.3. A Figura mostra o gráfico de uma função e o
de sua derivada. Qual é qual e por quê ? (Justifique analisando a
relação entre zero/sinal da f ′ e a f ter máximo/mı́nimo ou ser cres-
cente/decrescente).

80

40

0
-2 -1 0 1 2 3 4
x

-40

-80
CAPÍTULO 11. CRITÉRIOS PARA MÁXIMOS E MÍNIMOS E
APLICAÇÕES 143

Exercı́cio 10.4. Veja o gráfico a seguir como o gráfico de uma


função derivada y = f ′ (x).
i) Sobreponha a ele o gráfico de uma y = f (x) qualitativamente
compatı́vel (Atenção à relação entre zero/sinal de f ′ (x) e máximo,
mı́nimo, crecimento, decrescimento da f ).
ii) faça com detalhe a região da f que corresponde ao máximo da

f (x).

1
x
-2 -1 0 1 2 3
0

-1

-2

-3

-4

Exercı́cio 10.5. (resolvido)


O objetivo deste Exercı́cio é confeccionar gráficos apenas qualita-
tivamente corretos, sem qualquer tipo de calculadora, de polinômios
relativamente simples como:
i) y = f1 (x) = x3 − x2
ii) y = f2 (x) = x2 − x3 .
iii) y = f3 (x) = −2x2 + x3
iv): y = f4 (x) = x4 − 2x2 .
v): y = f5 (x) = 3x4 − 4x3 .
Faça-o seguindo o seguinte roteiro:
a) determine os zeros de f , e em quais intervalos a função f é
positiva ou negativa.
b) calcule a derivada f ′ .
c) determine os zeros da função derivada f ′ , e em quais intervalos
a função derivada é positiva ou negativa.
d) calcule a segunda derivada e determine onde ela é zero, positiva
e negativa.
e) com as informações de a), b), c) e d) esboce o gráfico de f ′′ (x);
com base nesse, o de f ′ (x) e com base nesse o de f (x).
Dica: em cada item fatore a maior potência possı́vel de x e então,
para examinar onde cada função é positiva e negativa basta usar a regra
de sinais + · + = +, + · − = − e − · − = +.
Depois de pensar bastante, pois cada item pode exigir tempo, con-
fira seus resultados com as Soluções no Capı́tulo 32.
Exercı́cio 10.6. (resolvido)
Suponhamos que, seguindo o roteiro do Exercı́cio anterior, você
entendeu o gráfico de y = x3 − C · x2 , onde C ≥ 1 é uma constante.
E que chegou em algo do seguinte tipo:
10. EXERCÍCIOS 144

x
-4 -2 0 2 4
0

-20

-40

-60

-80

-100

Sem fazer nenhuma conta mais, apenas raciocinando geometrica-


mente, como deve ser o gráfico de y = x3 + C · x2 ? (para C ≥ 1).
Exercı́cio 10.7. Dê um exemplo bem simples de uma f : [a, b] →
R contı́nua tal que f ′ (x) 6= 0 ∀x ∈ (a, b). Localize em seu exemplo
onde estão o(s) máximo(s) e mı́nimo(s).
Exercı́cio 10.8. Encontre dois números x, y pertencentes ao in-
tervalo [0, 1] cuja soma é x + y = 1 e tais que
i) x2 + y 2 é máximo (justifique)
ii) x2 + y 2 é mı́nimo (justifique).
iii): para responder ao i) e ii) você estudou máximo e mı́nimo de
uma função f (x). Esboce seu gráfico, indicando onde sua derivada
f ′ (x) é negativa, zero ou positiva.
Exercı́cio 10.9. Uma fábrica de azulejos fabrica pequenos reves-
timentos cerâmicos (pastilhas) retangulares, que têm x cm de largura
e y cm de comprimento.
O perı́metro de cada pastilha será fixado em 2 · (x + y) = 2.
i) descreva a função que dá a Área de cada pastilha como uma
função A(x) só de x.
ii) em qual domı́nio A(x) não é negativa ? Onde A(x) se anula ?
Onde A(x) é positiva ?
iii) Esboce o gráfico de A(x) (apenas qualitativamente). Como
determinar x para que o valor de A(x) seja máximo ?
iv) qual o formato e medidas da pastilha de maior Área ?
Exercı́cio 10.10. O custo de fabricação um objeto Retangular
3
é dado por C(x, y) = x6 + y, pois o material usado na fabricação da
lateral x é muitı́ssimo mais caro que o da frente y. Supondo que sempre
1 ≤ x e que a Área tem que ser igual a 8, quais as medidas x, y que
minimizam o custo de fabricação ?
CAPÍTULO 11. CRITÉRIOS PARA MÁXIMOS E MÍNIMOS E
APLICAÇÕES 145

Exercı́cio 10.11. O custo de fabricação um objeto Retangular é


dado por C(x, y) = x2 + y, pois o material usado na fabricação da
lateral x é muito mais caro que o da frente y. Supondo que sempre
1 ≤ x e que a Área tem que ser igual a 16, quais as medidas x, y que
minimizam o custo de fabricação ?
Um aluno pensou assim sobre esse problema: já que o custo em
função de x é muito maior que em função de y, por que não usar o
mı́nimo de x, ou seja, x = 1 e y = 16, obtendo área de 16 e custo de
12 + 16 = 17 ?
Será que ele está certo ? Esse é mesmo o mı́nimo de custo ?
Exercı́cio 10.12. A área de um objeto retangular é A(x, y) = xy.
O custo da construção depende das dimensões x e y segundo a fórmula
C(x, y) = 5x2 + y.
Maxime a área supondo fixado o custo em C(x, y) = 30.
Exercı́cio 10.13. Explique com os conceitos do Cálculo que relação
pode haver entre os dois gráficos apresentados em cada uma das três
Figuras que seguem.
ii) Que muda de uma Figura para a outra ? O que não muda ?
iii) destaque propriedades geométricas relevantes de cada Figura
(mı́nimos/máximos, inflexões, raı́zes, etc).

10

0
-2 -1 0 1 2
x
-5

-10

10

0
-2 -1 0 1 2
x

-5

10

0
-2 -1 0 1 2
x
-2

-4

Exercı́cio 10.14. Entendendo zeros e sinais de , de sua derivada


f ′ e da segunda derivada f ′′ , confeccione o gráfico de f ′′ , o de f ′ e o de
f , qualitativamente.
Apresente um gráfico acima do outro, identificando pontos impor-
tantes.
Exercı́cio 10.15. Entendendo zeros e sinais de f (x) = x2 − x3 , de
sua derivada f ′ e da segunda derivada f ′′ , confeccione o gráfico de f ′′ ,
o de f ′ e o de f , qualitativamente.
Apresente um gráfico acima do outro, identificando pontos impor-
tantes.
10. EXERCÍCIOS 146

Exercı́cio 10.16. Considere que a Figura a seguir dá em vermelho


o gráfico de y = x3 , restrito a x ∈ (−2, 1); e em verde dá o gráfico de
x3 − 3x2 + 3x − 2, x ∈ (−2, 1).

Prove que existe um ponto C no gráfico verde tal que, apoiando-


se nele, é possı́vel passar uma única reta (infinita) pela região entre
esses dois gráficos, sem que essa reta intersecte o gráfico vermelho em
nenhum ponto.
CAPı́TULO 12

Derivadas de seno e cosseno e as leis de Hooke

Hooke é sempre associado aos temas expostos na próxima Seção.


Mas sua importância cientı́fica vai muito além disso, como mostra o
trecho da carta de Hooke a Newton, de 1689, citado por James Gleick
em Isaac Newton, uma biografia, Companhia das Letras, p.132:

Resta agora conhecer as propriedades de uma linha curva [...] feita


por uma força atrativa central [...] em uma uma proporção duplicada
em relação às distâncias tomadas reciprocamente. Não duvido que por
seu excelente método o senhor descobrirá [...]

1. O cosseno como derivada do seno


No final de Star Wars descobrimos queo mocinho é filho do grande
vilão. Pois nesta Seção vamos descobrir que o cosseno é a derivada do
seno !
A derivada do seno em θ = 0 foi vista: sin′ (0) = 1 (Seção 5 do
Capı́tulo 5 da Parte 1).
Ou seja, sin′ (0) = cos(0). Será que isso é uma coincidência apenas?
Ou será que sin′ (θ) = cos(θ), ∀θ ∈ R ?
Vamos pôr um gráfico abaixo do outro e ver se são os gráficos são
coerentes com o que aprendemos no Capı́tulo 7 da Parte 1, sobre como
a derivada determina o comportamento de uma função.

1
0,5
0
0 1 2 3 4 5 6
-0,5 x
-1

Figura: O gráfico de y = sin(θ) (vermelho) e y = cos(θ)


(verde), para θ ∈ [0, 2π].

Observe que:
147
1. O COSSENO COMO DERIVADA DO SENO 148

• em θ = π2 ≈ 1.6 o seno tem seu máximo e nesse ponto θ = π2 o


cosseno se anula, passando de positivo para negativo.
• em θ = π ≈ 3.1 o cosseno tem seu mı́nimo −1 e nesse ponto
θ = π a inclinação do gráfico do seno parece ser −1. Ademais,
as inclinações do gráfico do seno vinham ficando mais negativas
desde π2 e a partir de θ = π vão ficando menos negativas.
• em θ = 3π 2
≈ 4.7 o cosseno se anula, passando de negativo a
positivo e em θ = 3π 2
o seno tem seu mı́nimo.
• por último, onde o cosseno é positivo (negativo) o seno é cres-
cente (decrescente).
Todas essas observações são coerentes com o que aprendemos no
final da Parte 1 e de fato:
Afirmação 1.1.
sin′ (θ) = cos(θ), ∀θ ∈ R.
Demonstração.
Começo com a definição de derivada em algum θ0 fixado e uso depois
a formula de seno de uma soma:

sin(θ0 + θ) − sin(θ0 )
sin′ (θ0 ) = lim =
θ→0 θ
sin(θ0 ) cos(θ) + cos(θ0 ) sin(θ) − sin(θ0 )
= lim .
θ→0 θ
Para poder continuar, agora vou usar o limite
sin(θ)
lim =1
θ→0 θ
provado na Seção 5 do Capı́tulo 5 da Parte 1 e um limite tão funda-
mental quanto este:
cos(θ) − 1
lim = 0,
θ→0 θ
cuja prova omito por brevidade.
Então as propriedades de limites de somas e produtos (Seção 4 do
Capı́tulo 3 da Parte 1) permitem que re-escreva o de acima como:
(cos(θ) − 1) sin(θ)
sin′ (θ0 ) = lim [sin(θ0 ) · + cos(θ0 ) · ]=
θ→0 θ θ
(cos(θ) − 1) sin(θ)
= sin(θ0 ) · lim + cos(θ0 ) · lim =
θ→0 θ θ→0 θ
= sin(θ0 ) · 0 + cos(θ0 ) · 1 = cos(θ0 ),
como querı́amos. 

Um complemento:
CAPÍTULO 12. DERIVADAS DE SENO E COSSENO E AS LEIS
DE HOOKE 149

A Figura a seguir exibe os gráficos de


sin(θ)
f1 (θ) = , para θ 6= 0 e f1 (0) := 1
θ
e de
cos(θ) − 1
f2 (θ) = , para θ 6= 0 e f2 (0) := 0
θ
(note que defino separadamente os valores para θ = 0, para que as
funções resultantes sejam contı́nuas).

0,8
0,4
0
-3 -2 -1 0 1 2 3
-0,4
x

Figura: O gráficos de y = f1 (θ) (vermelho) e y = f2 (θ)


(verde) para θ ∈ [−π, π].

A vingança do cosseno ! Seu filho (sua derivada) é o oposto do


malvado avô, o seno:

Afirmação 1.2.
cos′ (θ) = − sin(θ), ∀θ ∈ R.
Demonstração. Seguindo as mesmas etapas da prova anterior,
obtemos:
cos(θ0 + θ) − cos(θ0 )
cos′ (θ0 ) = lim =
θ→0 θ
cos(θ0 ) cos(θ) − sin(θ0 ) sin(θ) − cos(θ0 )
= lim =
θ→0 θ
(cos(θ) − 1) sin(θ)
= cos(θ0 ) · lim − sin(θ0 ) · lim =
θ→0 θ θ→0 θ
= cos(θ0 ) · 0 − sin(θ0 ) · 1 = − sin(θ0 ).
como querı́amos. 
2. LEIS DE HOOKE COM E SEM ATRITO 150

2. Leis de Hooke com e sem atrito


A lei de Hooke diz que a força que um objeto sofre quando se estica
uma mola presa a ele é do tipo
F = −kf (x)
onde k > 0 é uma constante e f (x) é a posição do objeto (veja a Figura
a seguir). O sinal negativo significa que a força é no sentido oposto do
deslocamento. Se ignora o atrito entre o objeto e a superfı́cie nessa
formulação da lei.

Se tomamos a força F como sendo o produto de massa m pela


aceleração f ′′ (x) então a lei de Hooke é da forma
mf ′′ (x) = −k · f (x).
A seguir, na Afirmação 2.1, para simplificar e dispensar a derivada
da composta (que não vimos ainda), ponho k = 1.
Afirmação 2.1.
i): As funções f (x) = a · cos(x) + b sin(x) são periódicas de perı́odo
2π, têm f (0) = a e f ′ (0) = b e satifazem
f ′′ (x) = −f (x), ∀x ∈ R.
ii): Ademais a · cos(x) + b sin(x) ≡ A · cos(x − q), onde
√ a
A = a2 + b2 e cos(q) = √ .
a2 + b 2

A Afirmação 2.1 será reforçada na Seção 6 do Capı́tulo 25, onde se


mostrará, entre outras coisas, que as funções f (x) = a · cos(k · x) +
b sin(k · x) são as únicas a satisfazer: f ′′ (x) = −k · f (x), k ∈ R.

Demonstração. (da Afirmação 2.1)


De i):
Como o seno e o cosseno têm perı́odo 2π essas funções também têm
esse perı́odo. Pela derivada da soma e de seno e cosseno, obtemos
f ′′ (x) = (f ′ (x))′ = (a(− sin(x)) + b cos(x))′ =
= −a cos(x) − b sin(x) = −f (x).
Ademais, f (0) = acos(0) = a e f ′ (0) = b cos(0) = b.
De ii):
CAPÍTULO 12. DERIVADAS DE SENO E COSSENO E AS LEIS
DE HOOKE 151

Note para o que segue que, se cos(q) = √ a , então


a2 +b2

b
sin(q) = √ .
a2 + b2
Temos então

A · cos(x − q) = A · [cos(x) · cos(−q) − sin(x) · sin(−q) =

= A · [cos(x) · cos(q) + sin(x) · sin(q)] =

√ a √ b
= a2 + b 2 · √ · cos(x) + a2 + b2 · √ · sin(x) =
2
a +b 2 a + b2
2

= a · cos(x) + b · sin(x),

Na figura a seguir note que não só a posição f (0) é relevante, mas
que também a inclinação f ′ (0) determina o tipo de oscilação que haverá.

0
0 1 2 3 4 5 6
x
-1

-2

Figura: Gráficos de y = a sin(θ) + b cos(θ) para alguns a, b e θ ∈ [0, 2π].

Claro que na realidade fı́sica sempre há algum atrito entre o objeto
e a superfı́cie e sabemos que com o tempo o objeto pára. Uma lei de
Hooke mais realista levaria em conta o atrito que surge com o desloca-
mento do objeto, ou seja, dependente da velocidade f ′ (x) do objeto e
seria do tipo
f ′′ (x) = −f (x) − kf ′ (x).

Na Figura a seguir ponho uma função satisfazendo f ′′ (x) = −f (x) ao


lado de uma função satisfazendo f ′′ (x) = −f (x) − 0.1 · f ′ (x). Uma
3. EXERCÍCIOS 152

função deste último tipo envolve senos e cossenos e a função exponen-


cial, que veremos mais adiante.

0,5

0
0 5 10 15 20 25 30 35
x
-0,5

-1

Figura: Funções satisfazendo a lei de Hooke


sem atrito (vermelho) e com atrito (verde).

E se o atrito for maior, por exemplo, em f ′′ (x) = −f (x)−0.3·f ′ (x),


então nesse caso o objeto vai parar bem mais rápido, como na Figura
a seguir:

0,5

0
0 5 10 15 20 25 30 35
x
-0,5

-1

Figura: Funções satisfazendo a lei de Hooke


sem atrito (vermelho) e com muito atrito (verde).

3. Exercı́cios
Exercı́cio 3.1. (resolvido)
Use que limx→0 sin(x)
x
= 1 para provar que
sin(k · x)
lim = k, ∀k ∈ R.
x→0 x
Exercı́cio 3.2. Determine se o ponto (0, 0) é máximo/mı́nimo ou
inflexão de f, sabendo que f ′ (x) = sen5 (x) cos(x).
CAPı́TULO 13

Indução Matemática e a derivada de xn , ∀n ∈ N.

Já vimos na Primeira parte do Curso que a derivada de f (x) = 1 =


x é f ′ (x) = 0, que a de f (x) = x = x1 é f ′ (x) = 1 = 1x0 , que a de
0

f (x) = x2 é f ′ (x) = 2x1 e até mesmo que a de f (x) = x4 é f ′ (x) = 4x3 .


Ou seja, nos sentimos motivados a conjecturar que ∀n ∈ N, f (x) =
x tem f ′ (x) = nxn−1 .
n

Como podemos provar isso, se não podemos percorrer todos os


Naturais ? Isso se faz através do princı́pio de indução matemática.

1. Princı́pio de indução matemática


Em geral a palavra indução é usada nas ciências experimentais
para referir ao processo pelo qual alguém tenta concluir após um certo
número de evidências que certo fenômeno valerá sempre (ou qual a
probabilidade disso ocorrer).
Já em matemática o significado é o seguinte: quando queremos
provar uma certa propriedade para todo n ∈ N, o que fazemos é:
• prová-la para n = 1,
• supô-la válida até n − 1 e
• prová-la para o próximo natural, ou seja, para n.
(A etapa em que supomos a propriedade válida até n − 1 é chamada
de hipótese de indução).
Se conseguimos fazer essa últı́ma etapa, a propriedade vale para
todo n ∈ N. A validade deste princı́pio está ligada à própria natureza
(axiomas) dos números Naturais.
Vejamos três exemplos, que além de bonitos em si mesmos, serão
úteis mais adiante no Capı́tulo 19:
Afirmação 1.1. ∀n ∈ N:
i) 1 + 2 + . . . + (n − 1) + n = (n+1)·n
2
.
ii) (1 + 2 + . . . + (n − 1) + n)2 = 13 + 23 + . . . + (n − 1)3 + n3 .
iii) 12 + 22 + . . . + n2 = n(n+1)(2n+1)
6

Demonstração.
2·1
Prova de i): Para n = 1 a fórmula diz simplesmente 1 = 2
o que
é óbvio.
A hipótese de indução é
((n − 1) + 1) · (n − 1) n(n − 1)
1 + 2 + . . . + (n − 1) = = .
2 2
153
1. PRINCÍPIO DE INDUÇÃO MATEMÁTICA 154

De agora em diante temos que fazer algo para mostrar quanto vale
1 + 2 + . . . + (n − 1) + n. Ora
1 + 2 + . . . + (n − 1) + n = (1 + 2 + . . . + (n − 1)) + n =
n(n − 1) n(n − 1) + 2n
= +n= =
2 2
(n + 1) · n
= ,
2
como querı́amos.
Prova de ii): Para n = 1 a fórmula diz simplesmente que 12 = 13 o
que é óbvio. Faço a hipótese de indução:
(1 + 2 + . . . + (n − 2) + (n − 1))2 = 13 + 23 + . . . + (n − 2)3 + (n − 1)3 ,
e quero saber se vale também:
(1 + 2 + . . . + (n − 1) + n)2 = 13 + 23 + . . . + (n − 1)3 + n3 .
Agora vamos ter que fazer algo, trabalhar um pouco. Escrevo pelo
binômio:
(1+2+. . .+(n−1)+n)2 = (1+2+. . .+(n−1))2 +2·(1+2+. . .+(n−1))·n+n2
e para continuar uso a hipótese de indução:
(1+2+. . .+(n−1)+n)2 = 13 +23 +. . .+(n−1)3 +2·(1+2+. . .+(n−1))·n+n2 .
Para terminar onde gostaria, preciso ver que
2 · (1 + 2 + . . . + (n − 1)) · n + n2 = n3 .
Mas posso usar a parte i) já provada para qualquer n, mesmo que da
forma n − 1, obtendo:
n · (n − 1)
(1 + 2 + . . . + (n − 1)) = ,
2
e portanto:
2 · (1 + 2 + . . . + (n − 1)) · n + n2 = (n · (n − 1)) · n + n2 =
= n3 ,
como precisávamos.
Prova de iii): para n = 1 a fórmula está correta 1 = 1(1+1)(2+1)
6
.
suponha válida até n − 1 e faço:
(n − 1)(n − 1 + 1)(2n − 2 + 1)
12 + 22 + . . . (n − 1)2 + n2 = + n2 =
6
2n3 − 3n2 + n
= + n2 =
6
2n3 − 3n2 + n + 6n2
= =
6
2n3 + 3n2 + n n(n + 1)(2n + 1)
= ,
6 6
como querı́amos.
CAPÍTULO 13. INDUÇÃO MATEMÁTICA E A DERIVADA DE
X N , ∀N ∈ N. 155

2. Derivada do Produto
Voltemos ao problema original: como derivar f (x) = xn ? Para
n = 1 já sabemos que a fórmula x′ = 1x0 está ok.
Gostariamos de supor a fórmula até n − 1 e prová-la então para n,
de acordo com o princı́pio de indução.
Mas quando escrevo xn e tento relacioná-lo com xn−1 só consigo
imaginar a seguinte relação:
xn = x · xn−1 .
Quando for derivar o lado esquerdo dessa expressão terei que derivar,
no lado direito, um produto de funções.
Como fazê-lo ? Certamente a derivada do produto não é o produto
das derivadas, pois (x2 )′ 6= x′ · x′ = 1 · 1.
Por isso precisamos de:
Teorema 2.1. Sejam f (x) e g(x) duas funções deriváveis com
mesmo domı́nio de definição. Então a função produto (f · g)(x) :=
f (x) · g(x) também é derivável e
(f · g)′ (x) := f ′ (x) · g(x) + f (x) · g ′ (x).
Demonstração.
Seja x e considere a definição de derivada:
f (x + h)g(x + h) − f (x)g(x)
(f · g)′ (x) = lim .
h→0 h
Agora vou fazer um truque, para fazer aparecer f ′ (x) e g ′ (x) nessa
estória. Escrevo
f (x + h)g(x + h) − f (x)g(x) =
= f (x + h)g(x + h) −f (x)g(x + h) + f (x)g(x + h) −f (x)g(x) =
| {z }
0
= (f (x + h) − f (x)) · g(x + h) + f (x) · (g(x + h) − g(x)).
Portanto através deste truque obtemos que
(f (x + h) − f (x)) (g(x + h) − g(x))
(f · g)′ (x) = lim [ · g(x + h) + f (x) ].
h→0 h h
Mas limh→0 g(x + h) = g(x) pela continuidade de g e
f (x + h) − f (x) g(x + h) − g(x)
lim = f ′ (x) e lim = g ′ (x),
h→0 h h→0 h
portanto juntando isso (e lembrando que o produto de limites é o limite
do produto):
(f · g)′ (x) = f ′ (x)g(x) + f (x)g ′ (x)
3. RAÍZES MÚLTIPLAS E FATORAÇÃO DE POLINÔMIOS 156

Agora estamos em condições de terminar a prova de que


(xn )′ = nxn−1 .
Pra n = 1 vale, suponho válida até n − 1.
Escrevo xn = x · xn−1 e aplico o teorema da derivada do produto:

(x · xn−1 )′ = 1 · xn−1 + x · (xn−1 ) =
= xn−1 + x · (n − 1) · xn−1−1 =
= xn−1 + (n − 1) · xn−1 =
= n · xn−1 .

3. Raı́zes múltiplas e fatoração de polinômios


Agora que sabemos derivar xn , para qualquer n ∈ N, também saber-
emos derivar qualquer polinômio de grau n:
f (x) = an xn + an−1 xn−1 + . . . + a0 , an 6= 0,
bastando para isso usar (n vezes) a regra da derivada da soma/subtração:
f ′ (x) = ( an xn + an−1 xn−1 + . . . + a0 )′ =
= (an xn )′ + (an−1 xn−1 )′ + . . . + a′0 =
= nan xn−1 + (n − 1)an−1 xn−2 + . . . + a1 .
Será conveniente chamar de derivada de ordem zero de uma f (x)
a própria função, em sı́mbolos: f (0) (x) := f (x).
Também chamar de derivada de ordem 1 a derivada usual: f (1) (x) :=
f ′ (x), bem como f (2) (x) := f ′′ (x) e assim por diante.
É fundamental o fato seguinte:
Teorema 3.1. Seja f (x) um polinômio de grau n a coeficientes
Reais.
São equivalentes as seguintes afirmações:
• i) f (x) = (x − x)k+1 · g(x), onde g(x) é um polinômio de grau
n − (k + 1) a coeficientes Reais.

• ii) f (0) (x) = f (1) (x) = . . . = f (k) (x) = 0 , onde 0 ≤ k ≤ n − 1.


Demonstração.
i) implica ii) :
Suponho f (x) = (x − x)k+1 · g(x), onde g(x) é um polinômio de
grau n − (k + 1).
Note que f ′ (x) = (k + 1)(x − x)k g(x) + (x − x)k+1 g ′ (x) é uma soma
e cada parcela dessa soma tem um fator (x − x)k ou (x − x)k+1 . Asssim
CAPÍTULO 13. INDUÇÃO MATEMÁTICA E A DERIVADA DE
X N , ∀N ∈ N. 157

também ocorre com qualquer das derivadas f (i) (x), com 0 ≤ i ≤ k ≤


n − 1: são somas onde cada parcela da soma tem algum fator dentre:
(x − x)k+1 , (x − x)k , . . . , (x − x)2 , (x − x).
Logo f (i) (x) = 0, se 0 ≤ i ≤ k.

ii) implica i) :
Procederemos por indução em k.
Se k = 0, ou seja, k + 1 = 1, já vimos no Teorema 7.1 do Capı́tulo
6 que
f (0) (x) := f (x) = 0 ⇒ f (x) = (x − x) · g(x),
onde o grau de g é n − 1.
Tentemos provar para k = m ≤ n − 1, supondo válido o resultado
para todo k ≤ m − 1.
Nossa hipótese será que
f (0) (x) = f (1) (x) = . . . = f (m) (x) = 0.
Em particular:
f (0) (x) = f (1) (x) = . . . = f (m−1) (x) = 0
e a hipótese de indução dá:
f (x) = (x − x)m · g(x)
para um polinômio g(x) de grau n − m. Precisamos ver que
g(x) = (x − x) · g(x)
para termos o resultado desejado:
f (x) = (x − x)m · [(x − x) · g(x)] = (x − x)m+1 · g(x).
Pensemos por absurdo, que
g(x) 6= (x − x) · g(x)
para todo g(x) de grau n − m − 1.
Pelo Teorema 7.1 do Capı́tulo 6 aplicado ao g(x):
g(x) 6= 0.
Mas como
f (x) = (x − x)m · g(x) = (x − x)k · g(x)
então a derivada f (m) (x) = f (k) (x) é uma soma onde cada parcela tem
algum fator dentre
(x − x)k , . . . , (x − x)2 , (x − x)
exceto uma última parcela que é do tipo C · g(x), C ∈ R \ {0}.
As parcelas todas que formam f (m) (x) = f (k) (x) se anulam x, exceto
a parcela que contém o fator C · g(x). Logo f (m) (x) 6= 0: contradição.
Portanto, como querı́amos:
g(x) = (x − x) · g(x).
5. EXERCÍCIOS 158

4. Derivadas de x−n , ∀n ∈ N
Se define x−n := x1n , ∀n ∈ N, onde claramente x 6= 0.
Com essa definição se obtem:
1
x−n · xn = · n = 1
n
−n n n−n
e portanto x · x = x .
Queremos derivar essas funções x−n , e novamente o faremos via a
indução matemática.
Vimos a derivada de f (x) = x−1 = x1 , x 6= 0 diretamente pela
definição, na Parte 1 deste Curso. Como um Exercı́cio, vejamos agora
como re-obter a derivada de x−1 = x1 usando a regra da derivada do
produto.
Escrevo a identidade para x 6= 0:
1 = x−1 · x
e derivo. Á esquerda na identidade obtenho 0 e à direita a regra do
produto dá:
0 = (x−1 )′ · x + x−1 · 1,
ou seja (x−1 )′ = − x12 = −x−2 .
Ou seja, que vale (x−1 )′ = −1 · x−1−1 .
Suponha provada a fórmula até n − 1 > 1: ou seja, que a derivada
de x−(n−1) é
−(n − 1) · x−(n−1)−1 = −(n − 1) · x−n .
Então escrevo x−n = x−(n−1) · x−1 e pela derivada do produto:
(x−n )′ = (x−(n−1) )′ · x−1 + x−(n−1) · (−x−2 ) =
= −(n − 1) · x−n · x−1 − x−(n−1)−2 =
= −(n − 1) · x−n−1 − x−n−1 = −n · x−n−1 ,
como querı́amos.
5. Exercı́cios
Exercı́cio 5.1. (resolvido)
Prove por indução: n! ≥ 2n−1 , ∀ n ≥ 2.
Exercı́cio 5.2. Derive o produto de três funções (deriváveis):
( f (x) · g(x) · h(x) )′
CAPı́TULO 14

Derivada da composição de funções

A composição de funções simples produzindo funções complicadas


é o análogo matemático da composição de processos simples que pro-
duzem efeitos complicados na natureza, nas reações quı́micas, nos pro-
cessos biológicos, etc.
Daı́ a importância de sabermos derivar composições.

1. Regra da composta ou da cadeia


A palavra que costuma se usar regra cadeia poderia ser substituı́da
pelo sinônimo regra da corrente, pois uma corrente é algo feito de elos
simples.
A regra de derivação da função composta combina as derivadas de
cada constituinte da corrente de um modo bem determinado, como
veremos.
Antes de enunciá-la em geral, considero algumas composições es-
pecı́ficas, que nos ajudarão a entender a regra geral.
Considere as funções fn (x) := n·x, com n ∈ N fixado, g(x) = sin(x)
e as compostas (g◦fn )(x) = sin( n·x ). Suponha que fazemos a restrição
g : [0, 2π] → R. Então quando x percorre [0, 2π] o parâmetro z := n · x
percorre n vezes esse intervalo. Ou seja que o gráfico da a função
sin( n · x ) é formado por n cópias do gráfico do seno, claro que mais
comprimidas. Abaixo pot o seno e sin(3x):

1
0,5
0
0 1 2 3 4 5 6
-0,5 x
-1

Figura: Gráfico de y = sin(x) (vermelho) e de y = sin(3x)


(verde) para x ∈ [0, 2pi].
Como vimos no Capı́tulo 12, o cosseno é a derivada do seno: onde
o cosseno é positivo (negativo) o seno é crescente (decrescente), onde
159
1. REGRA DA COMPOSTA OU DA CADEIA 160

o cosseno se anula o seno tem seus máximos ou mı́nimos, etc. Ora,


a função cos(nx) satisfaz qualitativamente todas essas exigências, ou
seja, se comporta qualitativamente como se fosse a derivada de sin(nx).
Ou seja, como fizemos na Parte 1 deste curso, onde os gráficos de f ′ e
f eram corretos apenas qualitativamente.
Veja isso na próxima Figura, com n = 3:

0,5

0
0 0,5 1 1,5 2
x

-0,5

-1

Figura: Gráfico de y = sin(3x) (vermelho) e de y = cos(3x)


(verde) para x ∈ [0, 2π].

Mas o que esta Figura não tem de quantitativamente correto é o


fato de que para que sin(3x) faça 3 vezes o que o seno usual faz quando
x percorre [0, 2π], sin(3x) tem que ser mais rápido que o seno usual.
Ou seja, em cada ponto as inclinações das tangentes de sin(3x) são
maiores que as do seno usual. Quanto maiores? Exatamente 3 vezes
maiores.
Por isso a derivada de sin(3x) quantitativamente correta não é
cos(3x) mas sim:
sin(3x)′ = 3 cos(3x)
e mais em geral:
sin(nx)′ = n cos(nx)
Mostro isso na Figura a seguir:

0
0 0,5 1 1,5 2
x
-1

-2

-3

Figura: Gráfico de y = sin(3x) (vermelho) e de sua


derivada (verde) para x ∈ [0, 2π].

Agora consider uma outra composição: f (x) = x2 e g(x) = sin(x),


ou seja (g ◦ f )(x) = sin(x2 ). A diferença para o exemplo anterior,
CAPÍTULO 14. DERIVADA DA COMPOSIÇÃO DE FUNÇÕES161

sin(3x) é que à medida que x se aproxima de 2π x2 cresce cada vez


mais rápido e a função sin(x2 ) faz aquilo que o seno faz em cada vez
menores intervalos, como mostra a figura a seguir:

1
0,5
0
0 1 2 3 4 5 6
-0,5 x
-1

Figura: Gráfico de y = sin(x) (vermelho) e


de y = sin(x2 ) (verde) para x ∈ [0, 2π].

Qualitativamente falando, cos(x2 ) se comporta como esperamos da


derivada de sin(x2 ):

1
0,5
0
0 1 2 3 4 5 6
-0,5 x
-1

Figura: Gráfico de y = sin(x2 ) (vermelho) e


de y = cos(x2 ) (verde) para x ∈ [0, 2π].

De novo, o que está quantitativamente errado: as inclinações do


gráfico de y = sin(x2 ) estão ficando cada vez maiores quando x se
aproxima de 2π. De quanto precisamos multiplicar a função qualitati-
vamente correta da derivada para termos uma função quntitativamente
exata da derivada ? A resposta como vermos é: precisamos multiplicar
pela função 2x ! Ou seja, para cada x > 0 a correção muda neste
exemplo:
A Figura a seguir superpõe os gráficos y = sin(x2 ) e de sua derivada,
que veremos é cos(x2 ) · 2x, e, ademais dá os gráficos de y = 2x e
y = −2x. Essas retas passam pelos pontos de máximo e mı́nimo locais
da derivada.
1. REGRA DA COMPOSTA OU DA CADEIA 162

10

0
0123 456
x

-5

-10

Figura: y = sin(x2 ) (vermelho), sua derivada (verde), y = 2x e


y = −2x, para x ∈ [0, 2π].

Por último, volto num limite calculado como Exercı́cio 3.1 do Capı́tulo
12:
sin(k · x)
lim = k.
x→0 x
Podemos olhá-lo do seguinte modo:
sin(k · x) − sin(k · 0)
lim =k
x→0 x
e reconhecemos então a definição da derivada da composta sin(k · x)
em x = 0.
O Teorema a seguir generaliza essas observações:
Teorema 1.1. Sejam f : I → J e g : K → L funções definidas em
intervalos, com a imagem J de f contida no domı́nio K de g, J ⊂ K.
Se f e g são seriváveis então a função composta (g ◦ f ) : I → L,
definida por (g ◦ f )(x) := g(f (x)) também é derivável e ademais:
(g ◦ f )′ (x) = g ′ (f (x)) · f ′ (x).

A notação de Leibniz:
dy
A notação de G. Leibniz para a derivada de y = f (x) é dx . O valor
de sua notação fica claro quando escrevemos a regra da derivada da
composta. Para y = f (x), u = g(y) e u = g(f (x)):
du du dy
= · .
dx dy dx
A prova da Afirmação 1.1 é técnica, prefiro tirar consequências.

A primeira consequência é que se pode derivar um número qualquer


de composições. Por exemplo, para tres funções podemos afirmar:
CAPÍTULO 14. DERIVADA DA COMPOSIÇÃO DE FUNÇÕES163

Afirmação 1.1. Sejam f : I → J, g : K → L e h : M → N , com


J ⊂ K e L ⊂ M . Se f, g, h são deriváveis, então a função composta
(h ◦ g ◦ f ) : I → L, definida por (h ◦ g ◦ f )(x) := h(g(f (x))) é derivável
e ademais:
(h ◦ g ◦ f )′ (x) = h′ (g(f (x))) · g ′ (f (x)) · f ′ (x).
Demonstração. De fato, associo h ◦ g ◦ f = h ◦ (g ◦ f ) e uso o
Teorema 1.1 duas vezes:
(h ◦ (g ◦ f ))′ (x) = h′ (g(f (x))) · (g ◦ f )′ (x) =
= h′ (g(f (x))) · g ′ (f (x)) · f ′ (x).


No Capı́tulo 16 sobre funções inversas vamos dar aplicações impor-


tantes da derivada da composta.
Vejamos agora alguns exemplos simples:
• f = sin(x), g = x2 , então (g ◦ f )′ = 2 · (sin(x)) · cos(x)
• f = cos(x), g = x2 , (g ◦ f )′ = 2 · (cos(x)) · (− sin(x)) =
−2 · cos(x) · sin(x).
• como consequência desse dois itens e da derivada da soma:
(sin(x)2 + cos(x)2 )′ = 2 · sin(x) · cos(x) − 2 · cos(x) · sin(x) ≡ 0,
o que é natural já que sin(x)2 + cos(x)2 ≡ 1.
• f (x) = x2 e g(x) = sin(x), então (g ◦ f )′ (x) = cos(x2 ) · 2 · x.

2. A Derivada do quociente
Agora uma aplicação da regra da composta aos quocientes de funções:
Afirmação 2.1. Sejam f e g funções deriváveis com g nunca nula.
Então
f (x) ′ f ′ (x) · g(x) − f (x) · g ′ (x)
( ) (x) = .
g(x) g 2 (x)
Em particular:
1 g ′ (x)
( )′ (x) = − 2 .
g g (x)
Demonstração.
Vou escrever primeiro
f (x) 1
= f (x) ·
g(x) g(x)
e derivar esse produto:
f (x) ′ 1 1 ′
( ) (x) = f ′ (x) · + f (x) · ( ) (x),
g(x) g(x) g(x)
2. A DERIVADA DO QUOCIENTE 164
1
Agora olho g(x) como a composição de duas funções f1 (x) = g(x) e
1 −1
f2 (x) = x = x :
1
= (f2 ◦ f1 )(x).
g(x)
Já sabemos derivar f2 (x) = x1 = x−1 , de fato: f2′ (x) = − x12 = −x−2 .
Então a regra da composta dá:
1 ′
( ) (x) = (f2 ◦ f1 )′ (x) =
g(x)
= f2′ (f1 (x)) · f1′ (x) =
1
=− 2 · g ′ (x).
g (x)
Junto tudo:
f (x) ′ 1 1 ′
( ) (x) = f ′ (x) · + f (x) · ( ) (x) =
g(x) g(x) g(x)
1 1
= f ′ (x) · + f (x) · (− 2 · g ′ (x)) =
g(x) g (x)
f ′ (x) · g(x) − f (x) · g ′ (x)
= ,
g 2 (x)
como querı́amos. 

Exemplos:
• Funções racionais são quocientes de polinômios fg . Onde g não
se anula, a fórmula da Afirmação 2.1 nos diz como derivá-las.
• A tangente é um quociente de funções deriváveis tan(x) =
sin(x)
cos(x)
. Onde o cosseno não se anula podemos derivá-la obtendo:
cos(x) · cos(x) − sin(x) · (− sin(x))
tan′ (x) = =
cos2 (x)
1
=
cos2 (x)
1
e com a nomenclatura conhecida sec(x) := cos(x) o que temos

tan′ (x) = sec2 (x).
Então claramente tan′ (0) = cos12 (0) = 1 e
lim tan′ (x) = lim tan′ (x) = +∞.
xր π2 −π
xւ 2

A seguir plotei os gráficos da tangente e de sua derivada


restritas ao intervalo (−1, 1). Não pude usar um intervalo mais
parecido com o domı́nio (− π2 , π2 ) porque os valores da tangente
ficam muito grande em módulo.
CAPÍTULO 14. DERIVADA DA COMPOSIÇÃO DE FUNÇÕES165

0
-1 -0,5 0 0,5 1
x

-1

Figura: A função tangente (vermelho) e sua derivada (verde) restritas a (−1, 1).

3. Uma função com derivada, mas sem a segunda derivada


Agora que já sabemos derivar quocientes, podemos considerar no-
vamente a função
x
f : R → ( −1, 1 ), f (x) = ,
|x| + 1
estudada na Seção 4 do Capı́tulo 5.
x
Afirmação 3.1. Seja f : R → ( −1, 1 ) dada por f (x) = |x|+1
.
1 1
• f ′ (x) = (x+1) ′ ′
2 se x > 0; f (x) = (−x+1)2 se x < 0 e f (0) = 1.
−2 −2
• f ′′ (x) = (x+1) ′′
3 se x > 0; f (x) = (−x+1)3 se x < 0; mas não

existe f ′′ (0).
Demonstração.
No Exercı́cio 4.4 do Capı́tulo 9 já vimos que f ′ (0) = 1.
Se x > 0 podemos usar a regra da derivada do quociente:
x ′ x · (x + 1)′ − x′ · (x + 1) 1
f (x)′ = [ ] = =
x+1 (x + 1)2 (x + 1)2
e analogamente, se x < 0:
x 1
f (x)′ = [ ]′ = .
−x + 1 (−x + 1)2
Agora sobre f ′′ (x). Se existisse
f ′ (h) − f ′ (0)
f ′′ (0) := lim .
h→0 h
teriam que exister ambos lmites laterais
f ′ (h) − f ′ (0) f ′ (h) − f ′ (0)
lim e lim
hց0 h hր0 h
e ademais serem iguais !
4. UM PROBLEMA DE MÁXIMOS/MÍNIMOS: O PROBLEMA
DO FRETEIRO 166

Porém, já que f ′ (0) = 1:


1
f ′ (h) − f ′ (0) (h+1)2
−1
lim = lim =
hց0 h hց0 h

= lim (−h − 2) = −2,


hց0

enquanto que
1
f ′ (h) − f ′ (0) (−h+1)2
−1
lim = lim =
hր0 h hր0 h

= lim (2 − h) = 2.
hր0

Os gráficos de f ′ e de f ′′ são mostrados a seguir:

x
-3 -2 -1 0 1 2 3
0

-1

-2

Figura: Note que f ′ (x) (vermelho) tem um bico em (0, 1).


Em verde está f ′′ (x). Note que f ′′ (0) não está definido.

4. Um problema de máximos/mı́nimos: o problema do


freteiro
Agora que já sabemos derivar um conjunto grande de funções, pode-
mos nos colocar problemas de máximos e mı́nimos mais interessantes.
Imagine que você está transportando, numa mudança, um objeto
retangular de largura L dada. Durante o transporte ele não poderá ser
deformado, nem vergado.
Você vem com ele por um corredor que mede l1 de largura e que
dobra em ângulo reto, chegando numa sala de largura l2 = k · l1 ≥ l1 ,
como mostra a Figura a seguir:
CAPÍTULO 14. DERIVADA DA COMPOSIÇÃO DE FUNÇÕES167

Pensando o problema como um problema no plano, não espacial,


trata-se de determinar o comprimento máximo do objeto retangular
para que você consiga passá-lo para a sala.

4.1. Caso E ≈ 0. Vamos primeiro considerar o caso em que a


largura L do objeto retangular é muito pequena (por exemplo, uma vara
de alumı́nio de diâmetro muito pequeno mas bem comprida). Vamos
pensar então que E = 0 e o objeto é uni-dimensional.
Primeiro noto que, se consigo passar uma vara de um certo tamanho
para a sala sem ter tocado o ponto C da Figura, então certamente
passaria uma vara um pouco maior, apoiando-me e pivotando em C.
Por isso, de agora em diante, posso pensar que me apoiarei em C,
pivotando nesse ponto.
A chave da resolução do problema é a seguinte: é notar que a
restrição, o impedimento, para se passar a vara está no mı́nimo da
distância do segmento P1 P2 , à medida que muda θ ∈ [0, π2 ]. Veja a
Figura que segue:

P 2

l 2
d 2

θ C

d 1

P 1
l 1
4. UM PROBLEMA DE MÁXIMOS/MÍNIMOS: O PROBLEMA
DO FRETEIRO 168

Portanto trata-se de descobrir qual o mı́nimo de P1 P2 . Para isso,


penso em
P1 P2 = P1 C + CP2
e ademais noto (identificando ângulos opostos pelo vértice) que:
l1 l2
cos(θ) = e sin(θ) = .
P1 C CP2
Ou seja:
P1 P2 (θ) = P1 C(θ) + CP2 (θ) =
l1 l2
= + .
cos(θ) sin(θ)
Repare que é natural que quando θ ≈ π2 (antes de começar a esquina)
tenhamos CP2 (θ) ≈ l2 mas P1 C(θ) fique arbitrariamente grande, ou
seja não há retrições sobre ele. Porém se θ ≈ 0 (após vencer a esquina)
aı́ P1 C(θ) ≈ l1 enquanto CP2 (θ) fica arbitrariamente grande.
Agora:
′ l1 · sin(θ) −l2 · cos(θ)
P1 P2 (θ) = + =
cos2 (θ) sin2 (θ)

l1 · sin3 (θ) − l2 · cos3 (θ)


= ,
sin2 (θ) cos2 (θ)
e portanto
′ l2 1 1
P1 P2 (θ) = 0 ⇔ tan(θ) = ( ) 3 = k 3 .
l1
1
Ou seja, a derivada se anula em um único ponto: θ0 = arctan(k 3 ).
Para concluir que θ0 é o ponto de mı́nimo, basta conferir que
l1 l2
lim + = +∞
θց0 cos(θ) sin(θ)
e
l1 l2
limπ + = +∞.
θր 2 cos(θ) sin(θ)
Assim o valor máximo do comprimento da vara que poderemos
passar é
l1 l2
P1 P2 (θ0 ) = + .
cos(θ0 ) sin(θ0 )
Vejamos Exemplos:

A Figura a seguir mostra a função P1 P2 (θ), para l1 = 1.2 e l2 =
1
2.4, quando θ0 = arctan(2 3 ) ≈ 0.8999083481 e o valor máximo de
comprimento é 4.99432582244 (plotado como reta horizontal em verde)
CAPÍTULO 14. DERIVADA DA COMPOSIÇÃO DE FUNÇÕES169

5,06

5,04

5,02

0,8 0,84 0,88 0,92 0,96


x


Já a próxima figura dá a função P1 P2 (θ) no caso l1 = l2 = 1.2, em
que θ0 = arctan(1) = π4 ≈ e o valor máximo da vara é 3.394112550
(horizontal em verde).

3,56

3,52

3,48

3,44

3,4

0,65 0,7 0,75 0,8 0,85 0,9


x

4.2. Para um objeto retangular. Agora vamos para o caso em


que a largura não pode ser considerada zero, ou seja L > 0, quando o
objeto é bi-dimensional.
A Figura a seguir dá a geometria da situação (note que paralelismo/ortogonalidade
de retas transportam o ângulo θ para dois triângulos retângulos):
4. UM PROBLEMA DE MÁXIMOS/MÍNIMOS: O PROBLEMA
DO FRETEIRO 170

P 2
θ
D2 − d2

d 2 l 2

d 1 C
P 1

D1− d1
θ
l 1

Note que
l1 l2
cos(θ) = e sin(θ) = ,
D1 D2
de onde:
l1 l2
D1 = (D1 − d1 ) + d1 = e D2 = (D2 − d2 ) + d2 = ,
cos(θ) sin(θ)
e portanto:
l1 L l2
L · tan(θ) + d1 = e + d2 = ,
cos(θ) tan(θ) sin(θ)
o que dá:
l1 l2 1
(d1 + d2 )(θ) = + − L · (tan(θ) + )=
cos(θ) sin(θ) tan(θ)
l1 l2 L
= + − .
cos(θ) sin(θ) sin(θ) · cos(θ)
Essa é a função que quero minimizar, pois seu mı́nimo é o impedimento,
a obstrução para que continue se movendo a face externa (relativa a
C) do objeto retangular.
A sua derivada é:
l1 · sin3 (θ) − l2 · cos3 (θ) − L · (2 · cos2 (θ) − 1)
(d1 + d2 ) ′ (θ) = .
sin2 (θ) cos2 (θ)
Queremos saber onde (d1 + d2 ) ′ (θ) = 0, e no caso L > 0 devemos
usar métodos numéricos (aproximações). Os programas como Maple/
Xmaxima , etc a resolvem numericamente.
Aparecem algumas soluções complexas e uma solução Real positiva.
Para concluir que θ0 é o ponto de mı́nimo, basta conferir que
lim (d1 + d2 )(θ) = +∞
θց0
e
lim (d1 + d2 )(θ) = +∞.
θր π2
CAPÍTULO 14. DERIVADA DA COMPOSIÇÃO DE FUNÇÕES171

Como
l1
lim = l1
θ→0 cos(θ)
basta analisar
l2 L
lim − =
θ→0 sin(θ) sin(θ) · cos(θ)
1 L
= lim · (l2 − ).
θ→0 sin(θ) cos(θ)
Mas
L
lim =L
θ→0 cos(θ)
e como l2 ≥ l1 > L, então
1 L 1
lim · (l2 − ) = lim = +∞.
θ→0 sin(θ) cos(θ) θ→0 sin(θ)

Quando θ se aproxima de π2 pela direita então é o sin(θ) que se aproxima


de 1 e o cos(θ) se aproxima de 0. Analogamente com o caso anterior,
se obtém:
1
limπ (d1 + d2 )(θ) = limπ = +∞.
θր 2 θր 2 cos(θ)

Também se pode avaliar (d1 + d2 )′′ (θ0 ) e o valor dá positivo.

Uma questão aparece naturalmente:

Questão 1: haverá outro modo de resolver o problema com L > 0


em que a solução (θ0 ) seja dada por um expressão exata ?

Um Exemplo: a figura a seguir dá a função P1 P2 (θ), para um objeto


de largura L = 1, quando l1 = 1.2, l2 = 2.4. Nesse caso o ponto θ0 onde

P1 P2 (θ0 ) = 0 é θ0 ≈ 1.065134018 e o valor máximo de comprimento
do objeto é 2.860890636 (plotado como reta horizontal em verde).

2,94

2,92

2,9

2,88

2,86
0,9 0,95 1 1,05 1,1 1,15 1,2
x

Outra questão é natural:


4. UM PROBLEMA DE MÁXIMOS/MÍNIMOS: O PROBLEMA
DO FRETEIRO 172

Questão 2: Qual a modelagem matemática do problema em di-


mensão 3 ? Ou seja, quando damos largura e espessura fixadas, mas
podemos girar o objeto no espaço ? Dito de outro modo, o que fazer
quando queremos passar um objeto como uma escada bem comprida
numa esquina ?

4.3. Área máxima do retângulo que dobra a esquina? Qual


a área máxima de uma figura retangular que consiga dobrar a esquina,
no caso l1 = l2 = 1 ?
Se a figura é um quadrado de lado l é fácil de ver que l = 1 é o
máximo, como na Figura a seguir.

Portanto a área máxima de um quadrado que dobra essa esquina é


1. Mas, e se fosse um retângulo não-quadrado ?
Como antes vou imaginar os retângulos se apoiando em C.
Pela simetria (l1 = l2 = 1 e o ângulo reto na esquina), posso pensar
que a figura retangular que se apoia em C é formada de duas partes de
mesma área e formato, uma para a direita de C e outra para a esquerda
de C.
Ademais, para um mesmo perı́metro, o quadrado é o retângulo de
maior área (ver Exercı́cio 10.9). Por isso, imagino à esquerda de C
um quadrado de lado l e à esquerda de C, outro, também de lado l,
formando então um retangulo de comprimento 2l e largura l. Veja a
Figura:
CAPÍTULO 14. DERIVADA DA COMPOSIÇÃO DE FUNÇÕES173

P 2

l
l
l
P 1
l
C

Agora continuo o lado da figura, de modo a obter triângulos como


na figura que segue:
P 2
θ
l r
l
1
l
P 1
l θ
C

Dos triângulos formados obtemos:


1 l
= sin(θ) e = tan(θ).
l+r r
Logo
l 1
r= e l+r = ,
tan(θ) sin(θ)
ou seja:
1 1
l · (1 + )=
tan(θ) sin(θ)
de onde:
tan(θ)
l(θ) = ,
sin(θ) · (1 + tan(θ))
Se encontramos um mı́nimo dessa função l(θ), para 0 < θ < π2 , esse
será o impedimento a passar a figura retangular pela esquina, ou seja,
dará o máximo da medida l do retângulo (e com esse valor saberemos
a área máxima da figura retangular).
4. UM PROBLEMA DE MÁXIMOS/MÍNIMOS: O PROBLEMA
DO FRETEIRO 174

Mas
sin(θ) − cos(θ)
l′ (θ) = .
1 + 2 · sin(θ) cos(θ)
Claramente, para 0 < θ < π2 :
π
l′ (θ) = 0 ⇔ sin(θ) = cos(θ) ⇔ θ= .
4
1
Como limθ→0 1+tan(θ)
= 1, então
tan(θ) 1
lim l(θ) = lim = lim = 1,
θց0 θց0 sin(θ) θց0 cos(θ)

1
e como limθ→ π2 sin(θ)
= 1, então
tan θ
limπ l(θ) = limπ = 1.
θր 2 θր 2 1 + tan(θ)
Então
π 1
l( ) = √
4 2
é o mı́nimo global de l(θ). Veja a Figura:

0,9

0,85

0,8

0,75

0,2 0,4 0,6 0,8 1 1,2 1,4


theta

Figura: Gráfico de y = l(θ), θ ∈ (0.1, π2 − 0.1), onde π


4
≈ 0.78

Portanto a área máxima da figura retangular que dobra a esquina


é:
1
2 · ( √ ) = 1,
2
a mesma que encontramos para o quadrado de área máxima que dobra
essa esquina.
Está ainda um problema em aberto determinar a área máxima da
figura capaz de dobrar a esquina, mesmo no caso l1 = l2 = 1, se
deixamos livre o formato da figura. Ou seja, valem figuras feitas de
pedaços distintos, alguns curvados , etc.
CAPÍTULO 14. DERIVADA DA COMPOSIÇÃO DE FUNÇÕES175

Há cotas máximas para a área, mas não se obteve ainda explici-
tamente uma figura da qual se possa dizer: é esta ! É conhecido na
literatura como o problema do sofá.

4.4. O caso L ≈ 0, mas com uma parede suave. Retomo o


caso em que L ≈ 0 e ainda na situação bem simples em que l1 = l2 = 1.
Coloque a Figura de um corredor que dobra em ângulo reto num
sistema de coordenadas cartesianas (x, y) de modo que:
• o ponto C seja C = (1, 1),
• a parede vertical externa faça parte da reta x = 0,
• a vertical interna, de x = 1,
• a parede horizontal externa faça parte de y = 2 e
• a vertical interna, de y = 1.
Imagine agora que as paredes internas (vertical e horizontal) da
Figura sejam derrubadas e substituı́das por uma parede suave, curvada,
que faça parte do gráfico de:
ǫ
y = fǫ (x) := 1 − , x > 1,
1−x
onde sempre ǫ > 0.
A figura a seguir mostra o que acontece para três escolhas de ǫ:

ǫ
Gráficos de y = 1 − 1−x com ǫ = 1 (vermelho)
ǫ = 0.5 (verde), ǫ = 0.2 (amarelo), y = 1 em azul
ǫ
Diminuindo ǫ o gráfico de y = 1 − 1−x vai se apertando sobre a
parede horizontal interna (em azul y = 1): de fato, cada x > 1 fixado,
fǫ (x) > fǫ′ (x), se ǫ < ǫ′ .
E também é claro que, fixado qualquer ǫ > 0,
lim fǫ (x) = 1
x→+∞
4. UM PROBLEMA DE MÁXIMOS/MÍNIMOS: O PROBLEMA
DO FRETEIRO 176

Note que se ǫ 6= 0, ainda que pequeno, a função é derivável e


ǫ
fǫ′ (x) = .
(x − 1)2
Então
lim fǫ′ (x) = +∞,
xց1
o que mostra que os gráficos de fǫ vão ficando cada vez mais verticais
próximos de x = 1.
Você também pode escrever a partir de fǫ (x):
(y − 1) · (x − 1) = −ǫ,
o que mostra que quando ǫ → 0 obtemos1:
(y − 1) · (x − 1) = 0
que é a união de retas x = 1 e y = 1.
Ou seja que as paredes internas foram substituı́das por um curvada
como na Figura a seguir (fixado um ǫ) e que a medida que o ǫ fica pe-
queno mais vai ficando próxima da parede interna original em formato
de letra L.

O Problema agora para o freteiro:


Problema: passar a maior vara possı́vel, sem entortá-la, possivel-
mente apoiando a vara em algum ponto da parede interna suavizada.

A solução que proponho é a seguinte:


Estratégia: usar a resposta do caso original, com parede em forma
de letra L, para solucionar o caso em que a parede é suave

Comecemos com l1 = l2 = 1 (depois passo ao geral, l1 , l2 quaisquer).


1A curvatura κǫ desses gráficos e seu limite quando ǫ → 0 serão estudados na
Seção 4 do Capı́tulo 23
CAPÍTULO 14. DERIVADA DA COMPOSIÇÃO DE FUNÇÕES177

Quero encontrar o ponto Cǫ = (x, fǫ (x)) e a inclinação da vara V


em Cǫ tais que seja minimizada a distância P1 P2 onde
P1 := V ∩ (x = 0) e P2 := V ∩ (y = 2).
Meu candidato a ponto Cǫ será o ponto (xǫ , fǫ (xǫ )) do gráfico de
y = fǫ (x) que tem
l2 1
fǫ′ (xǫ ) = ( ) 3 = 1
l1
já que a solução do caso original era em
l2 1 π
θ0 = arctan(( ) 3 ) = arctan(1) = .
l1 4
E as retas que se apoiam na parede curvada serão as suas retas tan-
gentes.
As soluções de fǫ′ (x) = 1 são

1 + ǫ1/2 e 1 − ǫ.
Fico apenas com √
xǫ := 1 + ǫ,
pois a outra solução está à esquerda da reta x = 1.
As retas tangentes de y = fǫ (x) num ponto geral (x, fǫ (x)) são:
ǫ x2 − 2(1 + ǫ) · x + 1 + ǫ
y= · x + .
(x − 1)2 (x − 1)2
e em particular em (xǫ , fǫ (xǫ )) a reta tangente é:
y = x − 2ǫ1/2 .

A intersecção de y = x − 2 ǫ com y = 2 é o ponto:

P2 := (2 + 2 ǫ, 2)
enquanto que a intersecção dela com x = 0 é:

P1 := (0, −2 ǫ).
A distância P1 P2 é (para l1 = l2 = 1):
q √ 2 √ 2 √ q √
mǫ := (2 + 2 ǫ) + (2 + 2 ǫ) = 2 · (2 + 2 ǫ)2 ,
e note que √
lim dmǫ = 2 2 ≈ 2.828427124,
ǫ→0
o comprimento da diagonal do quadrado de lado 2, solução do caso
original na figura em forma de L.
Queremos ver se mǫ é o mı́nimo das distâncias P1 P2 onde P2 é a
intersecção de uma reta tangente genérica de y = fǫ (x) com y = 1+l2 =
2 e P1 a intersecção da reta tangente genérica com x = 0.
Ora,
2ǫx − ǫ − x2 + 2x − 1
P1 = (0, − ),
(x − 1)2
4. UM PROBLEMA DE MÁXIMOS/MÍNIMOS: O PROBLEMA
DO FRETEIRO 178

2ǫx − ǫ + x2 − 2x + 1
P2 = ( , 2),
ǫ
e
s
(2ǫx − ǫ + x2 − 2x + 1)2 2ǫx − ǫ − x2 + 2x − 1 2
P1 P2 (x) = + (2 + ).
ǫ2 (x − 1)2

O numerador da fração2 que é P1 P2 (x) é dado pelo polinômio de grau
8 em x:

(ǫx5 −5ǫx4 +10ǫx3 −10ǫx2 +5ǫx−ǫ+x6 −6x5 +15x4 −20x3 +15x2 −6x+1−ǫ3 x)·

·2 · (2ǫx − ǫ + x2 − 2x + 1),

e verifica-se que em x0 = 1 + ǫ:
′ √
P1 P2 (1 + ǫ) = 0

pois x0 = 1 + ǫ é raiz do fator de grau 5 em x:

ǫx5 −5ǫx4 +10ǫx3 −10ǫx2 +5ǫx−ǫ+x6 −6x5 +15x4 −20x3 +15x2 −6x+1−ǫ3 x.
′′ √
Já a enorme fração que é P1 P2 (x) avaliada em x0 = 1 + ǫ vale:
√ √
2 2(2ǫ2 + 3 + 15ǫ + 11 ǫ + 9ǫ3/2 )
√ > 0.
ǫ(1 + ǫ)3

Logo x0 = 1 + ǫ é minimo local de P1 P2 (x).
Mas é bem claro que, para cada ǫ fixado:

lim P1 P2 (x) =
xց1

s
(2ǫx − ǫ + x2 − 2x + 1)2 2ǫx − ǫ − x2 + 2x − 1 2
= lim + (2 + ) = +∞
xց1 ǫ2 (x − 1)2

assim como
lim P1 P2 (x) =
x→+∞

s
(2ǫx − ǫ + x2 − 2x + 1)2 2ǫx − ǫ − x2 + 2x − 1 2
= lim + (2 + ) = +∞.
x→+∞ ǫ2 (x − 1)2

2Conferi as contas que seguem no Maple, pois ficam grandes.


CAPÍTULO 14. DERIVADA DA COMPOSIÇÃO DE FUNÇÕES179

400

300

200

100

0
1,5 2 2,5 3 3,5 4
x

As funções P1 P2 (x) para ǫ = 1 (vermelho) e ǫ = 0.1 (verde)


x0 = 2 e 1.316227766 resp., m1 = 5.656854249 e m0.1 = 3.722854312.

5. Exercı́cios
Exercı́cio 5.1. Usando a regra do quociente e definições/relações
trigonométricas, prove que
cot′ (x) = − csc2 (x),
1 1
onde cot(x) = tan(x) e csc(x) := sin(x)
.
Também mostre que:
sec′ (x) = tan(x) sec(x),
1
onde sec(x) := cos(x)
.

Exercı́cio 5.2. (resolvido)


Imagine que voce se lembra de cor da fórmula do seno da soma:
sin(x + y) = sin(x) · cos(y) + cos(x) · sin(y),
mas que se esqueceu completamente da fórmula do cosseno da soma.
i) Como o Cálculo pode obter a formula para o cosseno? Ou seja,
como saber derivar pode ajudar ?
ii) E se sei a do cosseno da soma, como obter a do seno da soma
via Cálculo ?
Exercı́cio 5.3. Um ponto P move-se sobre a curva de equação
y 3 − x2 = 0.
Determine a taxa de variação da coordenada y no instante em que
P = (8, 4), se a taxa de variação da coordenada x no mesmo instante
é 1cm/s.
5. EXERCÍCIOS 180

Em outras palavras, a coordenada y ao longo dessa curva aumenta


ou diminui, no ponto P , quando aumentamos a coordenada x.
Obs. você não precisa esboçar a curva.
CAPı́TULO 15

Derivadas de funções Implı́citas

1. Curvas versus gráficos


Comecemos com a equação do cı́rculo de raio r:
x2 + y 2 = r 2 .
É importante nos darmos conta de que o cı́rculo como um todo não é
gráfico de nenhuma função f : R → R1.
Mas, dado um ponto P (x, y) do cı́rculo, uma porção do cı́rculo perto
de P pode ser descrita:
• como gráfico de y = y(x), para x num intervalo centrado em
x, ou
• como gráfico de x = x(y), para y num intervalo centrado em
y.

De fato, há dois casos a considerar:


Caso 1: se P = (x, y) no cı́rculo tem coordenada
x 6= −r, r,

então
√ perto de P o cı́rculo é gráfico de y = 1 − x2 ou de y =
− 1 − x2 .

Caso 2: se P p é (−r, 0) ou P = (r, p


0), então perto de P o cı́rculo é
gráfico de x = 1 − y ou de x = − 1 − y 2 .
2

No Caso 1 podemos calcular a derivada da função y = y(x), para x


num intervalo, do seguinte modo: derivo a expressão x2 + y(x)2 = r2
pela regra da composta:
(x2 + y(x)2 )′ = (r2 )′ ⇔ 2x + 2y(x)y ′ (x) = 0 ⇔
−2x
⇔ y ′ (x) = .
2y(x)
√ √
E agora substituindo y(x) por 1 − x2 , se y > 0, ou por y = − 1 − x2
se y < 0, temos:
−2x −x
y ′ (x) = =√ , se y > 0,
2y(x) 1 − x2
1Não confunda essa afirmação com o fato do cı́rculo ser uma curva de nı́vel r2
da função F : R2 → R, F (x, y) = x2 + y 2 .
181
1. CURVAS VERSUS GRÁFICOS 182

ou
−2x x
y ′ (x) = =√ , se y < 0.
2y(x) 1 − x2
No Caso 2 podemos obter a derivada da função x = x(y), para y num
intervalo , do seguinte modo: derivo a expressão (x(y))2 + y 2 = r2 em
y, pela regra da composta:
( (x(y))2 + y 2 )′ = (r2 )′ ⇔ 2x(y)x′ (y) + 2y = 0 ⇔
−2y
⇔ x′ (y) = .
2x(y)
p p
E agora substituindo x(y) por 1 − y 2 , se x > 0, ou por x = − 1 − y 2
se x < 0:
−2y −y
x′ (y) = =p , se x > 0,
2x(y) 1 − y2
ou
−2y y
x′ (y) = =p , se x < 0.
2x(y) 1 − y2
Isso que fizemos se chama derivação implı́cita. É útil mesmo quando
não sabemos a expressão explı́cita de y = y(x) ou de x = x(y).

Por exemplo, se nos damos uma curva no plano através de uma


equação do tipo:
x2 y 2 − 3y 2 + y 4 − 8y + 2y 3 − 4 = 0
verificamos facilmente que (0, 2) é um ponto dessa curva.
Será que, num pequeno trecho perto de (0, 2) temos a curva dada
como um gráfico y = y(x) ? Ou seja, ∀x num intervalo aberto centrado
em x = 0, será que
x2 y(x)2 − 3y(x)2 + y(x)4 − 8y(x) + 2y(x)3 − 4 = 0 ?.
Veremos que neste Exemplo esse é o caso (graças ao Teorema 2.1 a
seguir).
Então supondo por um momento que sabemos que há um gráfico
y = y(x) perto de (0, 2) qual o valor de y ′ (x) em (x, y) = (0, 2) ?
Fazemos a derivada em x:
(x2 y(x)2 − 3y(x)2 + y(x)4 − 8y(x) + 2y(x)3 − 4)′ = 0 ⇔
2xy(x)2 +x2 2y(x)y ′ (x)−6y(x)y ′ (x)+4y(x)3 y ′ (x)−8y ′ (x)+6y(x)2 y ′ (x) = 0
⇔ 2xy(x)2 + y ′ (x)[x2 2y(x) − 6y(x) + 4y(x)3 − 8 + 6y(x)2 ] = 0
−2xy(x)2
⇔ y ′ (x) = 2
x 2y(x) − 6y(x) + 4y(x)3 − 8 + 6y(x)2
que dá em (x, y) = (0, 2)
0
y ′ (0) = = 0,
48
CAPÍTULO 15. DERIVADAS DE FUNÇÕES IMPLÍCITAS 183

ou seja que o gráfico y = y(x) em torno de (x, y) = (0, 2) tem reta


tangente horizontal nesse ponto.

2. Teorema da função implı́cita


Como saberemos se lidamos com y = y(x) ou x = x(y) em torno
de um ponto P = (x, y) de uma curva F (x, y) = 0 ?
O Teorema 2.1 a seguir dá uma resposta (sua prova se vê em Análise
Matemática):
Para poder enunciá-lo vamos introduzir um sı́mbolo novo: dada
uma expressão F (x, y) em duas variáveis, defino ∂F∂x (x,y)
como sendo a
derivada dessa expressão em x (se houver), onde se considera y fixado.
Por exemplo: se F (x, y) = yx2 +y 2 então ∂F∂x(x,y)
= 2yx. Se F (x, y) = y 2
então ∂F∂x
(x,y)
≡ 0. Se F (x, y) = exp(x)y 2 , então ∂F∂x
(x,y)
= exp(x)y 2 .
∂F (x,y)
E analogamente, ∂y se define como a derivada dessa expressão
em y (se houver), onde se considera x fixado.
Teorema 2.1. (Teorema da função Implı́cita).
Seja F (x, y) um polinômio em duas variáveis.2
Suponha que exista (x, y) com F (x, y) = 03
Se ∂F∂y
(x,y)
6= 0 quando avaliada em (x, y), então para x, y em (pos-
sivelmente pequenos) intervalos abertos centrados em x, y:
• a curva F (x, y) = 0 é um gráfico do tipo y = y(x) e
∂F (x,y)
• y ′ (x) = − ∂F∂x
(x,y) .
∂y
∂F (x,y)
Se 6= 0 quando avaliada em (x, y), então para x, y em (pos-
∂x
sivelmente pequenos) intervalos abertos centrados em x, y::
• a curva F (x, y) = 0 é um gráfico do tipo x = x(y) e
∂F (x,y)

• x′ (y) = − ∂F∂y
(x,y) .
∂x

Esse Teorema tem vários detalhes, que se vêem melhor nos Exem-
plos.

∂F (x,y)
Exemplo 2.1. No cı́rculo F (x, y) = x2 +y 2 −r2 = 0 temos ∂y
=
2y 6= 0 se y 6= 0. Nesse caso:
∂F (x,y)
2x
y ′ (x) = − ∂F∂x
(x,y)
=− ,
2y(x)
∂y

como vimos antes.


2há versões mais gerais desse enunciado, onde F é muito geral, sujeito apenas
a certas exigências de derivabilidade
3Não queremos ter conjuntos vazios como F (x, y) = x2 + y 2 + 3 = 0.
2. TEOREMA DA FUNÇÃO IMPLÍCITA 184

Mas se P no cı́rculo tem y = 0 então P = (−r, 0) ou P = (r, 0)


e nesse caso ∂F∂x
(x,y)
= 2x 6= 0. Então é preciso usar funções x = x(y)
para descrever o cı́rculo como gráfico.

O Teorema 2.1 tem sutilezas que ficam evidentes no Exemplo a


seguir:

Exemplo 2.2. Voltando ao exemplo que analisamos acima,

F (x, y) = x2 y 2 − 3y 2 + y 4 − 8y + 2y 3 − 4 = 0

temos
∂F (x, y)
= 2xy 2 ,
∂x
que se anula em P = (0, 2), mas temos

∂F (x, y)
= x2 2 y − 6 y + 4 y 3 − 8 + 6 y 2
∂y

que não se anula em P = (0, 2). Logo há um gráfico y = y(x) em torno
de (0, 2) e já calculamos y ′ (0) = 0 acima.
Até agora não comentei o fato de que P = (0, −1) também satisfaz:

x2 y 2 − 3y 2 + y 4 − 8y + 2y 3 − 4 = 0.

Isso é interessante pois diz que para o mesmo valor x = 0 há dois
valores y que satisfazem F (x, y) = 0 !
Ou seja que é só num pequeno entorno de (0, 2) que pode ser descrito
como gráfico de y = y(x) , mas não todo o conjunto F (x, y) = 0.
Por outro lado, em (0, −1) tanto ∂F∂x(x,y)
= 2xy 2 quanto

∂F (x, y)
= x2 2 y − 6 y + 4 y 3 − 8 + 6 y 2
∂y

se anulam !
Nessa caso o Teorema 2.1 não tem nada a dizer ! Ele não pode
garantir nenhum tipo de gráfico local y = y(x) ou x = x(y).
Ainda bem que o Teorema se calou nessa caso, pois em (0, −1) a
curva F (x, y) = 0 tem uma espécie de laço, que não se deixa descrever
nem como gráfico de y = y(x) nem como gráfico de x = x(y).

A Figura a seguir dá uma idéia da curva, que não por acaso se
chama conchóide:
CAPÍTULO 15. DERIVADAS DE FUNÇÕES IMPLÍCITAS 185

y 0
-4 -2 0 2 4
-1x

-2

Figura: Em (0, 2) vemos um pequeno gráfico horizontal y = y(x). Mas


em (0, −1) forma-se um laço.

Exemplo 2.3. O caso de


3x2
x3 + xy 2 − − y2 = 0
2
expõe outra sutileza do Teorema 2.1.
Note que essa curva tem sobre o eixo dos x exatamente dois pontos:
(0, 0) e (0, 23 ).
Em (0, 23 ) temos (como o leitor pode verificar)
∂F (x, y) ∂F (x, y) 9
= 0, =
∂y ∂x 4
e o Teorema 2.1 diz que a curva F (x, y) = 0 se representa localmente
como gráfico x = x(y). Ademais calcula x′ ( 32 ) como
3 0
x′ ( ) = − 9 = 0,
2 (4)
ou seja que o gráfico é vertical.
Mas em (0, 0) temos
∂F (x, y) ∂F (x, y)
= = 0.
∂y ∂x
De fato esse ponto é completamente isolado do resto da curva ! Ou
seja, não pode ser visto como gráfico de uma função cujo domı́nio é um
intervalo aberto em torno de x = 0.
Na Figura a seguir o Maple não enxerga o (0, 0) na curva !
3. O MÉTODO DAS TANGENTES E PONTOS RACIONAIS DE
CÚBICAS 186

y 0
1,1 1,2 1,3 1,4 1,5
x
-1

-2

-3

3. O método das tangentes e pontos racionais de cúbicas


Consideremos uma cúbica em forma implı́cita, ou seja, uma curva
dada por:
y 2 − x3 − b x − a = 0, a, b ∈ R,
ou equivalentemente:
y 2 = x3 + b x + a a, b ∈ R.
Quando se trabalha com computadores, o melhor dos mundos é lidar
com números Racionais. E duas questões muito importantes e atuais,
que estão relacionadas com a aplicação da matemática à criptografia,
são:

Questão 1: Seja a curva dada por


y 2 = x3 + b x + a a, b ∈ Q.
Quem são ou quantos são os pontos P = (x, y) da curva que têm ambas
coordenadas Racionais ?

Questão 2: Dado um ponto P dessa curva com coordenadas Racionais,


como produzir outros pontos dela que também tenham coordenadas
Racionais ?

Usaremos a notação P = (x, y) ∈ Q × Q para dizer que ambas as


coordenadas são Racionais.
A seguinte Afirmação é um método para atacar a segunda questão:
Afirmação 3.1. (Método das tangentes)
Seja P = (x, y) ∈ Q × Q pertencente à curva
F (x, y) = y 2 − x3 − b x − a a, b ∈ Q.
∂F
Suponha que ∂y
= 2y não se anula em P .
CAPÍTULO 15. DERIVADAS DE FUNÇÕES IMPLÍCITAS 187

Tome a reta tangente a F (x, y) em P , denotada rP .


Então rP ∩ F (x, y) = {P, Q} e o ponto Q também tem coordenadas
Racionais.
Demonstração.
Pelo Teorema 2.1, F (x, y) localmente em torno de P é um gráfico
de y = y(x), com
∂F
−3x2 − b
y ′ (x) = − ∂F
∂x
=− .
∂y
2y
Como b, x, y ∈ Q então y ′ (x) avaliada em P = (x, y) é um número
Racional, que denoto aqui de A.
A equação da reta tangente é do tipo:
rP : y = Ax + B
onde o valor do coeficiente linear B se obtêm de:
y = Ax + B ⇔ B = y − A x,
e portanto B também é um número Racional.
As coordenadas x dos pontos na intersecção F (x, y) ∩ rP são as
soluções de:
F (x, y) = 0 e y = A x + B,
ou seja, soluções de
(A x + B)2 − x3 − b x − a = 0,
ou, equivalentemente,
−x3 + A2 x2 + (2AB − b) x + B 2 − a = 0.
Agora é o momento de lembrar que a coordenada x de P = (x, y) é
uma raı́z dupla ou tripla desse polinômio, já que rP é tangente à curva
F (x, y) nesse ponto (tripla seria o caso de um ponto de inflexão).
No caso em que x é raı́z dupla exatamente, pelo Teorema 3.1 do
Capı́tulo 13:
−x3 + A2 x2 + (2AB − b) x + B 2 − a = (x − x)2 · q(x).
onde o grau de q(x) é 3 − 2 = 1.
Mas, como se viu na prova do Teorema 7.1 do Capı́tulo 6 e na
Digressão que se seguiu, os coeficientes de q(x) são Racionais.
Ou seja, q(x) = q1 x + q0 , com q0 , q1 ∈ Q e a raı́z de q(x) é
−q0
.
q1
O ponto Q 6= P buscado é portanto:
−q0 −q0
Q=( , A( ) + B ),
q1 q1
que nitidamente tem coordenadas Racionais.
3. O MÉTODO DAS TANGENTES E PONTOS RACIONAIS DE
CÚBICAS 188

Se P é ponto de inflexão, então Q = P , ou seja,


rP ∩ F (x, y) = {P, Q} = {P }.


Exemplo 3.1. Considere a curva analisada por Billing, em 1937:


y 2 − x3 + 82 x = 0.
Fora o óbvio (0, 0) há três pontos com coordenadas Racionais relativa-
mente simples
49 231
P1 = (−1, 9), P2 = (−8, 12), P3 = ( , ).
4 8
A Figura a seguir mostra como o Maple plota para essa curva:

100

50

y 0
-5 0 5 10 15 20
x

-50

-100

Vou implementar neste Exemplo o que a prova da Afirmação 3.1


nos ensinou (as contas tediosas foram feita com o Maple).
A reta tangente ao gráfico local y = y(x) de F (x, y) = 0 em P1 =
(−1, 9) é:
79 83
r P1 : − x + .
18 18
A intersecção rP1 ∩ F (x, y) = {P1 , Q1 } tem
6889 517339
Q1 = ( ,− ) ∼ (21, −88).
324 5832
Ver a Figura:
CAPÍTULO 15. DERIVADAS DE FUNÇÕES IMPLÍCITAS 189

100

50

y 0
-10 -5 0 5 10 15 20
x

-50

-100

Agora podemos continuar o processo.


Tomo Q1 , a tangente rQ1 e determino rQ1 ∩ F (x, y) = {q1 , Q2 } onde
Q2 terá coordenadas Racionais.
Faço as contas e obtenho:
44588977 4653507299
rQ1 : − x+
6208068 72701712
3143435938720609 6994054838592555031151
Q2 = ( ,− ) ∼ (9, −1).
346860974633616 6460009551215289641664
A Figura a seguir mostra isso:

100

50

y 0
-10 -5 0 5 10 15 20
x

-50

-100
4. DERIVAÇÃO IMPLÍCITA DE SEGUNDA ORDEM 190

Um Teorema de Billing diz que se continuamos o processo, agora


em Q2 e assim sucessivamente, produzimos uma infinidade de pontos
da curva com coordenadas Racionais.
O mesmo ocorreria se tivéssemos começado com P2 ou P3 .
Note a supreendente forma dos números Racionais envolvidos: essa
complexidade é que interessa a quem quer criar um código secreto.

4. Derivação implı́cita de segunda ordem


Na Seção 2 do Capı́tulo 3 associamos a Figura:

y 0
-1 -0,5 0 0,5 1 1,5 2
x

-1

-2

à curva y 2 − x3 − 1 = 0. Mas tem algo que não ficou plenamente


justificado. Parece na Figura que há 2 pontos de inflexão, em torno de
x ∼ 0.8.
Vamos considerar ao invés daquela curva, outra bem parecida (mas
mais adequada para nossas contas):
F (x, y) = y 2 − x3 − 4x = 0.
A inflexão deve aparecer onde a segunda derivada y ′′ (x) muda de
sinal, ou seja onde y ′′ (x) = 0.
Só que já sabemos que aqui não se trata de um gráfico, mas apenas
de uma curva. Por isso precisamos da derivação implı́cita, só que agora
para calcular a segunda derivada.
Já sabemos que se y 6= 0:
∂F
′ ∂x 3x2 + 4
y (x) = − ∂F = .
∂y
2y
Então calculo
3x2 + 4 ′
y ′′ (x) = ( )
2y
pela regra do quociente, obtendo:
12x · y − (3x2 + 4) · 2y ′ (x)
y ′′ (x) = =
4y 2
CAPÍTULO 15. DERIVADAS DE FUNÇÕES IMPLÍCITAS 191
2
12x · y − (3x2 + 4) · 2( 3x2y+4 )
= =
4y 2

12xy 2 − 9x4 − 24x2 − 16)/4y 3


=
.
Preciso ver as raı́zes de y ′′ (x), ou seja, as raı́zes de

12x(x3 + 4x) − 9x4 − 24x2 − 16

já que y 2 = x3 + 4x.


Ora,

12x(x3 + 4x) − 9x4 − 24x2 − 16 = 3x4 + 24x2 − 16,

que sabemos resolver (pense em z = x2 e resolva 15z 2 + 72z − 16 = 0).


Assim obtenho as raı́zes:
q q q q
2 √ 2 √ 2 √ 2 √
− −9 + 6 3, −9 + 6 3, − −9 − 6 3, −9 − 6 3,
3 3 3 3

das quais a única Real e positiva é


q
2 √
x := −9 + 6 3 ∼ 0.78.
3

Para este valor de x há dois valores de y na curva y 2 = x3 + 4x:


r q
2 √ √
3/2
6(−9 + 6 3) + 54 −9 + 6 3 ∼ 1.9
9
e
r q
2 √ √
− 3/2
6(−9 + 6 3) + 54 −9 + 6 3 − 1.9
9
Agora, já que já temos y ′ (x), é um trabalho tedioso achar a equação
da reta tangente em por exemplo:

q r q
2 √ 2 √ √
( −9 + 6 3 , 3/2
6(−9 + 6 3) + 54 −9 + 6 3 ).
3 9

Com essa equação posso plotar a cúbica e sua tangente, que mostra
bem que há uma inflexão nesse ponto:
5. EXERCÍCIOS 192

y 0
-2 -1 0 1 2 3 4 5
x

-4

-8

5. Exercı́cios
Exercı́cio 5.1. (resolvido)
Considere F (x, y) = y 2 − x3 = 0. Considere o ponto (1, 1) dessa
curva.
i) usando o Teorema 2.1 verifique que perto de (1, 1) essa curva é o
gráfico de uma função y = y(x).
ii) calcule a derivada da função do item i) em (1, 1).
iii) note que (1, −1) também está na curva F (x, y) = y 2 − x3 = 0 e
portanto ela não é globalmente um gráfico de y = y(x).
Exercı́cio 5.2. Considere a cúbica F (x, y) = y 2 − x3 − 4x = 0.
Um fato muito bonito é que esta curva só tem 3 pontos com coor-
denadas Racionais:
(0, 0), (2, 4) e (2, −4).
Suponha esse fato.
Por outro lado ∂F∂y (x,y)
= 2y não se anula em (2, 4) nem em (2, −4), o
que nos dá a oportunidade de usar o método das tangentes (Afirmação
3.1) para obter pontos racionais a partir deles.
i) conclua sem fazer nenhuma conta que as retas tangentes a F (x, y)
em (2, 4) e em (2, −4) passam pela origem (0, 0).
ii) faça as contas e obtenha as equações dessas duas retas tangentes.
CAPı́TULO 16

Funções inversas e suas derivadas

Vimos na Seção 1.2 do Capı́tulo 5 da Parte 1, que quando referidos


ao mesmo sistema cartesiano os gráficos de y = f (x) e de sua inversa
y = f −1 (x) , então elas se relacionam por uma reflexão na diagonal
y = x.
Logo uma reta tangente ao gráfico y = f (x) de coeficiente angular
a = B/A 6= 0 se transforma numa reta tangente ao gráfico refletido,
mas agora de coeficiente angular a1 = A/B (já que os acréscimos na
coordenada x e y que definem A e B ficam invertidos quando refletimos
na diagonal). Ilustro isso nas Figura a seguir:

0,8

0,6

0,4

0,2

0
0 0,2 0,4 0,6 0,8
-0,2 x

-0,4

Figura: Reflexão na diagonal de um gráfico e de sua reta tangente

Quero motivar com isso o seguinte fato:


Teorema 0.1. Seja y = f (x) derivável com f ′ (x) 6= 0 e com uma
função inversa f −1 (x) também derivável. Então:
′ 1
f −1 (x) = ′ −1 .
f (f (x))
Demonstração. Considero a composição entre f e g = f −1 , que
resulta em uma anular o efeito da outra:
(f ◦ f −1 )(x) ≡ x.
Então o Teorema 1.1 dá:

(f ◦ f −1 )′ (x) = f ′ (f −1 (x)) · (f −1 ) (x).
193

1. DERIVADA DE Y = X 194

Mas por outro lado:


1 ≡ (f ◦ f −1 )′ (x)
pois (f ◦ f −1 )(x) ≡ x. Asim que:

1 ≡ f ′ (f −1 (x)) · (f −1 ) (x),
de onde
′ 1
(f 1 ) (x) = .
f ′ (f −1 (x))



1. Derivada de y = x

Vejamos o que é a derivada de y = √x de dois modos distintos,
um pela definição e outro lembrando que :R>0 → R>0 é a inversa de
y = x2 : R>0 → R>0 .
Pela definição temos:
√ √
√ ′ x+h− x
x (x) := lim
h→0 h
e para x > 0 e h com |h| suficientemente pequeno para que x + h > 0,
escrevo:
√ √ √ √ √ √
x+h− x x+h− x x+h+ x
lim = lim ·√ √ .
h→0 h h→0 h x+h+ x
Agora uso que ( + △) · ( − △) = 2 − △2 , para obter que:
√ ′ x+h−x
x (x) = lim √ √ =
h→0 h · ( x + h + x)
1
= lim √ √ .
h→0 x+h+ x

E agora uso a continuidade de y = x (por ser inversa de função
contı́nua definida num intervalo) para fazer:
√ ′ 1 1
x (x) = lim √ √ = √ .
h→0 x+h+ x 2· x
Observe que
1
lim √ = +∞
xց0 2 · x

o que diz que o gráfico de y = x fica vertical na origem.
Agora quero comparar esse resultado com o que obtemos pelo Teo-
rema 0.1 sobre a derivada da inversa. √
Seja f : R>0 → R>0 dada por f (x) = x2 e sua inversa f −1 (x) = x.
Como f ′ (x) = 2x, então
√ √
f ′ ( x) = 2 · x
CAPÍTULO 16. FUNÇÕES INVERSAS E SUAS DERIVADAS 195

e portanto pelo Teo 0.1:


√ 1
x ′ (x) = √ ,
2· x
como querı́amos.
1 m −m
2. Derivada da “função”x n , de x n e de x n

Seja a função f (x) = xn . Se n é par, precisamos restringir f a um


semi-eixo para termos uma função inversa f −1 (uma raı́z n-ésima).
Com essa ressalva, considere g = f −1 a inversa de f (x) = xn . Ou
seja g(f (x)) = x.
1
A notação usual para g(x) é g(x) = x n , feita de propósito a que
valha
1 n
g(f (x)) = (xn ) n = x = x n .
1
Afirmação 2.1. Considere a função x n , para n ∈ N, (com a
ressalva acima). Então para x 6= 0 vale que
1 ′ 1 1
(x n ) (x) = x n −1 .
n
Demonstração.
O Teorema 0.1 diz que para x 6= 0, combinado com a derivada de
n
x , dá:
1 ′ 1
(x n ) = 1 n−1
.
n · (x n )
1
De aı́ em diante basta fazer algumas manipulações (usando (x n )k =
k
x n ):
1′ 1 1 1 n−1
x n = · n−1 = · x− n = .
n x n n
1 1−n 1 1
= · x n = · x n −1 .
n n


m
Podemos agora derivar funções do tipo x n com m, n ∈ N usando
as regras da composta e da inversa, pois
m 1
x n = (x n )m .
Então pelo Teorema 1.1 (a regra da composta) e o que já sabemos
1
para x n :
1 m′ 1 m−1 1 1
(x n ) = m · (x n ) · ( · x n −1 ) =
n
m m−1 1
−1 m m −1
= · x n · xn = ·xn
n n
3. DERIVADAS DO ARCOSENO E DO ARCOCOSSENO 196
m
Para podermos derivar funções do tipo x− n com m, n ∈ N podemos
m m
escrever x− n = 1mn e usar o que sabemos de quocientes e de x n :
x

m
1 ′ − m x n −1 m m 2m
( m ) = n 2m = − · x n −1− n =
xn xn n

m −m −1
− ·x n .
n

Qual o sentido de dizermos que em geral se f (x) = xα então f ′ (x) =


αxα−1 ? √
E se α 6∈ Q? Por exemplo α = 2 ou α = π? Após darmos um
sentido a essa expressão (e precisaremos da função exponencial para
isso), será que essa função é derivável ? Será que sua derivada também
é α · xα−1 ? Voltaremos...

3. Derivadas do arcoseno e do arcocosseno


É claro que o seno visto como função periódica sin : R → R ou
mesmo visto em sin : [0, 2π] → R não tem uma função inversa.
Mas sua restrição sin : (− π2 , π2 ) → (−1, 1) mostrada na Figura a
seguir sim tem função inversa ! De fato, nessa região (− π2 , π2 ) o seno
é uma função injetora, pois sua derivada sin′ (x) = cos(x) é sempre
positiva em (− π2 , π2 ), logo sin(x) é estritamente crescente e portanto
uma função injetora.

0,5

0
-1,5 -1 -0,5 0 0,5 1 1,5
x
-0,5

-1

Figura: Restrição do seno ao intervalo ((− π2 , π2 ).

A inversa de sin : (− π2 , π2 ) → R é chamada de valor principal do arco


seno ou apenas arcoseno, no sentido de que dado sin(θ) em (−1, 1) ela
diz de que arco θ ele proveio, π2 < θ < π2 .
É denotada arcsin. Guardaremos o sı́mbolo sin(x)−1 para denotar
1
sin(x)
.
CAPÍTULO 16. FUNÇÕES INVERSAS E SUAS DERIVADAS 197

1,5

0,5

0
-1 -0,5 0 0,5 1
x
-0,5

-1

-1,5

Figura: Gráfico de arcoseno, domı́nio (−1, 1) e imagem (− π2 , π2 ).

Como explicado no Teorema que trata da inversa de funções contı́nuas,


o arcoseno e o arcocosseno são funções contı́nuas. Mas vamos assumir
que seja derivável, para calcularmos sua derivada.
Agora considere na Figura a seguir a restrição do cosseno ao inter-
valo [0.π].

0,5

0
0 0,5 1 1,5 2 2,5 3
x
-0,5

-1

É uma função estritamente decrescente, cuja inversa (também es-


tritamente decrescente) é denotada arccos : (−1, 1) → [π, 0].
Note para o que segue que vale:
π
arccos(x) ≡ − arcsin(x).
2
Isso fica bem ilustrado na Figura a seguir que mostra em vermelho o
arccos : (−1, 1) → [π, 0] e em verde o − arcsin : (−1, 1) → ( π2 , − π2 ).

0
-1 -0,5 0 0,5 1
x

-1
3. DERIVADAS DO ARCOSENO E DO ARCOCOSSENO 198

Afirmação 3.1. A derivada de arcsin : (−1, 1) → (− π2 , π2 ) é


1
arcsin′ (x) = √ .
1 − x2
A derivada de arccos : (−1, 1) → [π, 0] é
1
arccos′ (x) = − √ .
1 − x2
Demonstração.
Pelo Teorema 0.1
1
arcsin′ (x) = ′ .
sin (arcsin(x))
Mas já sabemos que a derivada do seno é o cosseno, logo:
1
arcsin′ (x) = .
cos(arcsin(x))
Agora uso a relação trigonométrica
cos2 (arcsin(x)) + sin2 (arcsin(x)) ≡ 1
e
sin2 (arcsin(x)) = ( sin(arcsin(x) )2 = x2
para obter:
cos2 (arcsin(x)) = 1 − x2 ,
e como cos(arcsin(x)) > 0 quando arcsin(x) ∈ (− π2 , π2 ) então obtenho:

cos(arcsin(x)) = + 1 − x2
e portanto
1
arcsin′ (x) = √ ,
1 − x2
como querı́amos.
Agora uso
π
arccos(x) ≡ − arcsin(x)
2
para obter
arccos′ (x) = − arcsin′ (x).


1
Note que a função √1−x 2 para x ∈ (−1, 1) é sempre positiva, vale 1

na origem e tem
1 1
lim √ = +∞, e lim √ = +∞.
xր1 1 − x2 xց1 1 − x2
Tudo isso se vê na figura abaixo, onde plotei o arcoseno e sua
derivada, para x ∈ [−0.95, 0.95] (não posso me aproximar demais de
−1 ou de 1 se não o gráfico fica muito alto !)
CAPÍTULO 16. FUNÇÕES INVERSAS E SUAS DERIVADAS 199

0
-0,8-0,4 0 0,4 0,8
x

-1

Figura: Gráfico de y = arcsin(x) (vermelho) e de sua derivada y = √ 1 (verde).


1−x2

Essa figura é tão parecida (qualitativamente) com a que já vimos


no Capı́tulo anterior da função y = tan(x) e sua derivada que resolvi
plotá-las juntas, para que o leitor possa fazer comparações:

0
-0,8-0,4 0 0,4 0,8
x

-1

Figura: y = tan(x) (vermelho), sua derivada (verde), y = arcsin(x) (amarelo) e


sua derivada (azul) restritas a (−0.9, 0.9).

4. Derivada do arcotangente
Como tan′ (x) = cos12 (x) > 0 se x ∈ (− π2 , π2 ) então a função tangente é
estritamente crfecente, logo injetora, logo tem inversa denotada arctan :
R → (− π2 , π2 ).
Afirmação 4.1.
1
arctan′ (x) = , ∀x ∈ R.
1 + x2
Demonstração.
Pelo Teorema 0.1 e pela derivada da função tan(x):
1
arctan′ (x) = ′
=
tan (arctan(x))
4. DERIVADA DO ARCOTANGENTE 200

1
= 1 =
cos2 (arctan(x))

= cos2 (arctan(x)).
Agora arctan(x) é um arco/ângulo e portanto vale:

sin2 (arctan(x)) + cos2 (arctan(x)) = 1

e daı́ dividindo por cos2 (arctan(x)) temos:

sin2 (arctan(x)) 1
2
+1= 2
cos (arctan(x)) cos (arctan(x))
ou seja
1
tan2 (arctan(x)) + 1 = ,
cos2 (arctan(x))
e como
tan2 (arctan(x)) = (tan(arctan(x)))2 = x2 ,

1
x2 + 1 =
cos2 (arctan(x))
quer dizer:
1
cos2 (arctan(x)) =
1 + x2
Logo
1
arctan′ (x) = .
1 + x2


1
0,5
0
-3 -2 -1 0 1 2 3
-0,5
x
-1

Figura: A função arcotangente (vermelho) e sua derivada (verde) restritas a (−4, 4)


CAPÍTULO 16. FUNÇÕES INVERSAS E SUAS DERIVADAS 201

5. Existe uma função f 6≡ 0 que seja imune à derivação ?


Exceto pela função f ≡ 0, todas as funções que vimos até agora
mudam ao serem derivadas (os polinômios perdem grau, etc). Como
poderı́amos criar uma função f (x) imune à derivada ? Ou seja, com
f ′ (x) = f (x) ?
Imagine que tivéssemos uma função f : R> 0 → R com
1
f ′ (x) = .
x

Então f (x) > 0 ∀x ∈ R> 0 e daı́ f (x) é estritamente crescente. Logo
f −1 : R → R>0 existiria e se fosse derivável, pelo Teorema 0.1 da
derivada da inversa, terı́amos:
′ 1
(f −1 ) (x) = ′ −1 =
f (f (x))
1
= =
( f −11(x) )
= f −1 (x).

Ou seja (f −1 ) = f −1 : voilà a função imunizada.
Ou seja a sonhada função imune será a inversa daquela f (x) que
tem f ′ (x) = x1 .
Mas será que já não temos uma função com f ′ (x) = x1 em nossa
lista de funções já conhecidas ?
Se quiséssemos ao invés de f ′ (x) = x−1 algo do tipo f ′ (x) = x−k ,
k 6= 1, bastaria tomar
1
f (x) = · x−k+1
−k + 1
e pelo que já aprendemos f ′ (x) = x−k . Mas, justamente, não podemos
1
escrever −k+1 se k = 1.
Para criarmos essa desejada função com f ′ (x) = x1 precisaremos se
uma fábrica de funções, chamada de Integral Indefinida, que veremos
no próximo Capı́tulo.

Assim como vimos que há leis fı́sicas importantes modeladas a partir
da propriedade f ′′ (x) = −f (x) do seno e do cosseno, há processos muito
importantes modelados matematicamente pela relação:
f ′ (x) = f (x).
Essa relação entre a derivada e a função diz por exemplo que quanto
mais f (x) fica positivo mais aumenta sua velocidade. É a modelagem
de algum processo que tem um crescimento extraordinário.
Por exemplo, f (x) pode ser uma população em um certo tempo, e
que quanto mais elementos tem mais cruzamentos efetua, aumentando
a população, e assim por diante. Ou por exemplo uma dı́vida, sobre
7. EXERCÍCIOS 202

a qual incidem juros que aumentam a dı́vida e sobre ela mais juros
incidem, assim por diante.

6. Quantas funções são imunes à derivação ?

Acima propusemos um método para criar uma função imune à


derivação (como inversa de uma outa função também misteriosa) Chamemos
nossa função imune f1 (x) (com f1′ (x) = f1 (x) ∀x portanto).
Ssuponhamos por um momento que f1 (x) nunca se anula (será ver-
dade!).
Será que há alguma outra função f2 (x) com f2′ (x) = f2 (x) ∀x,
bem diferente da nossa f1 (x) e que quem sabe será criada por um
outro método completamente diferente desse nosso? A resposta é que
essencialmente não !
E o argumento é o seguinte. Suponha outra f2 (x) com f2′ (x) = f2 (x)
∀x e defina:
f2 (x)
.
f1 (x)
Então a derivada do quociente dá:
f2 (x) ′ f ′ (x) · f1 (x) − f2 (x) · f1′ (x)
( ) (x) = 2 =
f1 (x) f12 (x)
f2 (x) · f1 (x) − f2 (x) · f1 (x)
=
f12 (x)
0
= 2 ≡ 0.
f1 (x)
Mas então pela Parte 1 do Curso concluı́mos que
f2 (x)
≡C
f1 (x)
onde C é uma constante. Dito de outro modo f2 (x) = C · f1 (x) ou seja
que f2 é apenas f1 multiplicada por uma constante.
Note que se C = 0 então f2 (x) ≡ 0 é imune à derivação.

7. Exercı́cios
Exercı́cio 7.1. (resolvidos: iii, iv, v, xv.)

Derive usando regras de derivação de +, −, x, /, e a derivada da
composta:
p
i) sin(x3 ), se sin(x3 ) > 0 ii) cos5 (x) + sin(x5 ),
x4 + x2 + 1
iii) sin3 (x3 ), iv) sin(x) cos(x), v) ,
3x4 + 4x2 + 1

vi) 1 − x2 , se |x| < 1, vii) sin(x3 ), viii) cos3 (x) + sin3 (x),
CAPÍTULO 16. FUNÇÕES INVERSAS E SUAS DERIVADAS 203

x7 − x2 − 1 x3 − x + 1
ix) 4 , x) 4 ,
x + 4x2 + 8 x − x3 + x2 − 1
2
xi) sin3 (x) − sin(x3 ), xii) 3 , 0 < x,
x
xiii) (sin(x) · cos (x)) , xiv) (x + 3) , xv) (3x + 4)100 .
2 2 100

Exercı́cio 7.2. Suponha que você só conhece a reta tangente ao


Cı́rculo como o fizemos aqui neste curso de Cálculo, ou seja, como reta
cujo coeficiente angular é dado por uma derivada, etc.
Prove que essa reta tangente é ortogonal ao raio do Cı́rculo, ou seja,
que coincide com a definição do Ensino Médio (dica: basta considerar
pontos do cı́rculo x2 + y 2 = 1 com coordenada y > 0).
Exercı́cio 7.3. Agora que sabemos derivar a raı́z quadrada e as
composições, volto a considerar o problema de minimizar a distância
2
entre o ponto (0, 7) e a parábola y = x2 (Seção 4 do Capı́tulo 11).
i) defina a função d(x) que dá a distância usual entre um ponto da
2
parábola y = x2 . e (0, 7).
ii) obtenha d′ (x)
iii) determine onde d′ (x) = 0.
iv) determine qual dos pontos que satisfazem iii) é mı́nimo local
(global) ou máximo local (apenas calculando d(x) nesses pontos e com-
parando os valores obtidos).
O gráfico de d(x) está plotado a seguir:

7,5

6,5

5,5

4,5

-4 -2 0 2 4
x

v) mudando um pouco a escolha do ponto (0, 7) para (0, a) o gráfico


da função d(x) associada a esse ponto muda um pouco em relação
ao dado na Figura acima, mas qualitativamente é o mesmo. Porém,
quando escolho (0, a) = (0, 21 ) a função d(x) associada vira d(x) =
1 2
2
+ x2 , ou seja, d(x) vira uma translação vertical do gráfico da parábola.
Por quê isso acontece com o ponto (0, 21 ) ? Se não souber explicar, leia
o começo do Capı́tulo 18.
Exercı́cio 7.4. Considere a função f : R>0 → [−1, 1] dada por
f (x) = sin( x1 ).
7. EXERCÍCIOS 204

i) derive-a pela regra da composta, ii) comprove que |f ′ (x)| fica


arbitrariamente grande quando x tende a zero, iii) interprete geometri-
camente o resultado, sobre o que acontece com o gráfico de f próximo
à origem, iv) agora considere a função dada por f (x) = x2 · sin( x1 )
(para x > 0). v) derive-a , vi) veja se o módulo da derivada f ′ (x) fica
arbitrariamente grande próximo à origem, ou não.
Exercı́cio 7.5. Considere a Figura a seguir, que dá o gráficos de
f (x) = arctan(x) (função inversa da tangente), de sua derivada f ′ (x) =
1
1+x2
(assuma que sua derivada é essa) e de sua segunda derivada f ′′ (x),
restritas ao eixo positivo x > 0.

0,5

0
0 0,5 1 1,5 2 2,5 3 3,5
x
-0,5

1
Vemos que o gráfico de f ′ (x) = 1+x 2 tem um ponto de inflexão, ou

seja, onde as inclinações de suas tangentes tem um mı́nimo e depois


vão aumentando, ficando cada vez mais próximas de zero quando x >>
1. Dito de outro modo, um ponto onde a segunda derivada f ′′ (x) =
(f ′ (x)′ ) têm um mı́nimo.
Para encontrar onde é esse mı́nimo de f ′′ (x), calcule pela regra do
quociente a terceira derivada f ′′′ (x) e procure por seus zeros ! (Vão
ser duas soluções, uma positiva e outra negativa, pois o gráfico de
1
f ′ (x) = 1+x 2 é simétrico em relação ao eixo dos y).

Exercı́cio 7.6. Considere a função g : (−1, 1) → R dada por


y
g(y) = , se y ∈ [0, 1),
1−y
y
g(y) = , se y ∈ (−1, 0].
1+y
(Chamo a variável de y pois foi assim que a vimos na Parte 1 do Curso).
Já vimos que g é uma tremenda expansão, pois a imagem do intervalo
pela g é toda a reta R ! Prove que a derivada da g em y ∈ [0, 1) é
1 1
(1−y)2
e que a derivada da g em y ∈ (−1, 0] é de (1+y) 2 . Chamamos

essas derivadas de taxas de expansão.


Exercı́cio 7.7. Calcule a segunda derivada da função
sin(x)
tan(x) := .
cos(x)
CAPı́TULO 17

O Princı́pio de Fermat e a refração da luz

1. Princı́pio de Fermat
Suponhamos dois pontos P1 = (x1 , y 1 ) e P2 = (x2 , y 2 ) com coorde-
nadas y > 0.
O problema é: Encontrar o ponto P = (x, 0) no eixo dos x que
minimiza a soma das distâncias P P1 + P P2 .
Não é uma perda de generalidade muito grande supôr que P1 =
(0, 1) (basta escolher sistema de coordenadas adequado).
Chamemos o ângulo 1) formado em P pelo eixo dos x e a reta P P1
de ângulo de incidência; e de ângulo refletido o ângulo formado pelo
eixo dos x e a reta P P2 .
Afirmação 1.1. (Princı́pio de Fermat)
• i) o ponto no eixo dos x que minimiza a soma de distâncias a
P1 := (0, 1) e a P2 := (x2 , y 2 ), com y 2 > 0, é
x
P = (x, 0) = ( 2 , 0).
1 + y2
• ii) os ângulos de incidência e refletido formados nesse P são
iguais.

2,5

1,5

0,5

0
0 0,5 1 1,5 2 2,5 3
x

Figura: Três exemplos do princı́pio de Fermat, com P1 = (0, 1)


P2 : (3, 1), (3, 2), (3, 3) e P : ( 32 , 0), (1, 0), ( 34 , 0) respectivamente.
Demonstração.
Do Item i):
1convexo, ou seja, 0 ≤ θ ≤ π, e não-orientado, ou seja, não distingo entre
ângulos horários e anti-horários.
205
1. PRINCÍPIO DE FERMAT 206

Queremos encontrar o ponto P = (x, 0) no eixo dos x que minimiza


a função:
p q
d(x) := (x − 0) + (0 − 1) + (x − x2 )2 + (0 − y 2 )2 =
2 2

√ q
= x + 1 + (x − x2 )2 + y 22 .
2

Queremos usar o critério da segunda derivada (Afirmação 2.1 do


Capı́tulo 10) para determinar o mı́nimo de d(x).
Para isso precisamos calcular d ′ (x), o que ainda não sabemos fazer.
Então, adiantando o que aprenderemos sobre derivadas de funções
compostas e da raı́z quadrada, Afirmo que:
x x − x2
d ′ (x) = √ +q =
x2 + 1 2
(x − x2 ) + y 2
2
q √
x · (x − x2 )2 + y 22 + (x − x2 ) · x2 + 1
= √ q ,
x2 + 1 · (x − x2 )2 + y 22
e claramente:
q √
d ′ (x) = 0 ⇔ x· (x − x2 )2 + y 22 + (x − x2 ) · x2 + 1 = 0.
Ao invés de resolver diretamente:
q √
x · (x − x2 )2 + y 22 = (x2 − x) · x2 + 1,
elevo ambos os lados ao quadrado, obtendo:
x2 · [(x − x2 )2 + y 22 ] = (x2 − x)2 · (x2 + 1),
o que equivale, após simplificações, a resolver:
(y 22 − 1) x2 + 2x2 x − x22 = 0.
Aqui há dois casos a considerar (dos quais daremos o significado geométrico
a seguir):
Caso y 22 − 1 = 0, ou seja, y 2 = ±1, então a solução buscada é
x
P = (x, 0) = ( 2 , 0).
2
2
Caso y 2 − 1 6= 0, então temos uma equação quadrática em x, cujas
soluções são:
x2 x2
e .
1 + y2 1 − y2
x
Note que o ponto Q := ( 1−y2 , 0) é colinear com (0, 1) e (x2 , y 2 )
2
(basta calcular os coeficientes angulares das retas por dois deles). Então
essa solução não nos interessa.
Porém a solução
x
P = (x, 0) = ( 2 , 0)
1 + y2
CAPÍTULO 17. O PRINCÍPIO DE FERMAT E A REFRAÇÃO
DA LUZ 207
x
é interessante. Note que se y2 = 1 esse ponto se reduz a P = ( 22 , 0),
ou seja, coincide com a solução obtida no caso y 22 − 1 = 0.
x x
Temos d ′ ( 1+y2 ) = 0 e agora precisarı́amos ver que d ′′ ( 1+y2 ) > 0,
2 2
para termos um mı́nimo de d(x).
A segunda derivada d ′′ (x) existe, como veremos nos Capı́tulos seguintes
sobre regras de derivação.
O cálculo de d ′′ (x) é tedioso e ainda mais tedioso2 é obter:
x2 (1 + y 2 )4
d ′′ ( )= q ,
1 + y2 y 2 (x22 + 1 + 2y 2 + y 22 )3
x
e vemos que d ′′ ( 1+y2 ) é positivo se y 2 > 0.
2
Está provado que o ponto minimiza a soma de distâncias.

Do Item ii):
Calculo o coeficiente angular da reta P P1 :

1−0 (1 + y 2 )
a := x2 = − .
0 − 1+y x2
2

Agora calculo o coeficiente angular da reta P P2 :


y2 − 0 1 + y2
a′ := x2 = ,
x2 − 1+y 2
x2
logo a′ = −a, ou seja, formam o mesmo ângulo (não-orientado) com a
reta vertical.
Portanto também há igualdade de ângulos formados em P com a
horizontal.


2. Refração, distâncias ponderadas e Lei de Snell


Na Seção anterior buscamos minimizar a soma das distâncias
P P1 + P P2 ,
onde P1 , P2 estão no semi-plano superior e P no eixo dos x
Agora imaginemos um problema um pouco mais geral.

Suponha que no semiplano superior nos movimentamos com uma


velocidade constante v1 enquanto no semiplano inferir nos movimenta-
mos com uma velocidade constante v2 . E que queremos sair de P1 no
semiplano superior, atingir P no eixo dos x e daı́, no semiplano-inferior,
ir até P2 , fazendo isso no menor tempo possı́vel. Como escolher P ?
2É útil para essas contas tediosas usar algum programa como o Maple.
2. REFRAÇÃO, DISTÂNCIAS PONDERADAS E LEI DE SNELL
208

Esse problema está ainda relacionado com o princı́pio de Fermat,


que em geral não é simplesmente de minimar distância entre dois pon-
tos, mas de minimizar o tempo gasto para ir de um a outro ponto.
Na prática é o problema do salva-vidas, que estando em P1 tem
correr pela areia (com v1 ) e escolher o ponto P na praia de onde sair
nadando (com v2 < v1 ) até chegar em algum banhista P2 .
Claro que se vv21 = 1, a solução é seguir a reta que liga P1 a P2 . E se
v2
v1
<< 1, o ponto P ficará cada vez mais próximo da projeção vertical
de P2 no eixo dos x. Porém a resposta não é tão clara se vv12 ∼ 1.
Como distância é o mesmo que velocidade multiplicada pelo tempo,
podemos pensar que no semiplano superior e inferior as medidas de
distância são diferentes. Como se tivéssemos diferentes réguas para
medir distância: um certo trecho que mede d no semiplano superior
(onde sou mais rápido) dever ser considerado como medindo k · d > d
no semiplano-inferior, onde sou mais lento.
Podemos então reformular o problema do seguinte modo:
Como minimizar a soma das distâncias ponderadas
d1,k (x) := P P1 + k · P P2 ?
(onde P1 , P2 estão em semi-planos diferentes e P no eixo dos x)
Isso é o que acontece quando a luz passa de um meio para outro.
Por exemplo, a razão entre velocidade da luz no ar (v1 ) e na água (v2 )
é da ordem de
v2 1
= ,
v1 1.33
ou seja, devemos usar a soma de distâncias ponderadas 3:
d1,1.33 (x) := P P1 + 1.33 · P P2 ,
(onde P1 está no ar e P2 na água).
Suponha que P1 = (0, 1) e que por exemplo
P2 = (x2 , −1), x2 > 0.
Imitando o que fizemos na Seção anterior, vamos querer derivar d1,k (x)
e saber onde d1,k ′ (x) = 0.
Agora, derivando obtemos:
x (x − x2 )
d1,k ′ (x) = √ +k p =
+1 x2 (x − x2 )2 + 1
p √
x · (x − x2 )2 + 1 + k x2 + 1 · (x − x2 )
= √ p .
x2 + 1 · (x − x2 )2 + 1
3O chamado optical path length- OPL é definido como o produto da distância
usual pelo ı́ndice de refração - suposto constante - do meio onde a luz se propaga.
Então no nosso caso d1,1.33 (x) = OPL( ar ) + OPL( água )
CAPÍTULO 17. O PRINCÍPIO DE FERMAT E A REFRAÇÃO
DA LUZ 209

Como
x (x − x2 )
d1,k ′′ (x) = ( √ )′ + (k p )′ =
2
x +1 2
(x − x2 ) + 1

1 k
+ 2 > 0,
(x2 + 1)3/2 (x2 − 2x2 x + x2 + 1)3/2
a solução de d1,k ′ (x) = 0 será um ponto de mı́nimo de d1,k .
Mas
p √
d1,k ′ (x) = 0 ⇔ x · (x − x2 )2 + 1 = k x2 + 1 · (x2 − x)

e elevando ao quadrado ambos os lados, obtenho:

x2 ( (x − x2 )2 + 1 ) = k 2 (x2 + 1) (x2 − x)2 ,

ou seja, temos que resolver uma equação de grau 4:

(1−k 2 ) x4 +(−2x2 +2k 2 x2 ) x3 +(x22 +1−k 2 x22 −k 2 ) x2 +2k 2 x2 x−k 2 x22 = 0.

Claro que se k = 1 (ou seja, d1,1 (x) é a soma de distâncias usuais),


a equação acima vira uma equação quadrática:
x2
2x2 x − x2 = 0 ⇔ x= .
2
x
Logo P = ( 22 , 0) está na reta ligando P1 e P2 .
Mas se k 6= 1 temos uma verdadeira equação de grau 4.
Resovi fazer três exemplos, com o k = 1.33 (ı́ndice de refração da
água) onde sempre P1 = (0, 1), mas P2 assume três valores

(2, −1), (3, −1), (4, −1).

Nesses três casos o Maple resolve as equações de grau 4 acima4,


dando em cada caso um par de soluções complexas, uma solução real
negativa e uma real positiva.
Listo as soluções reais positivas de cada um dos três casos:

se P2 = (2, −1), P = (1.268409214, 0),

se P2 = (3, −1), P = (2.078744326, 0),

se P2 = (4, −1), P = (2.983414222, 0).


A Figura a seguir representa as linhas quebradas ligando P1 a P e
daı́ passando por P2 , em cada um dos três casos, com k = 1.33:

4Pois existe a fórmula de Tartaglia para equações de grau 4.


2. REFRAÇÃO, DISTÂNCIAS PONDERADAS E LEI DE SNELL
210

x
0 1 2 3 4
0

-1

-2

-3

A figura a seguir dá os gráficos das d1,1.33 para


P2 = (2, −1), (3, −1), (4, −1).

6,5

5,5

4,5

3,5

0 1 2 3 4
x

Gráficos de y = d1,1.33 (x) para três escolhas de P2

Voltando ao que obtivemos como derivada:


p √
d1,k ′ (x) = 0 ⇔ x · (x − x2 )2 + 1 = k x2 + 1 · (x2 − x),
note que essa última expressão equivale a:
x (x − x)
√ =kp 2 .
x2 + 1 (x − x2 )2 + 1
Agora note que
x
sin(α) = √
x2+1
CAPÍTULO 17. O PRINCÍPIO DE FERMAT E A REFRAÇÃO
DA LUZ 211

onde α é o ângulo em P = (x, 0) do triângulo


∆ P P1 (x, 1).
E veja que
(x − x)
sin(β) = p 2
(x − x2 )2 + 1
onde β é o ângulo em P = (x, 0) do triângulo
∆ P P2 (x, −1).
Essa é a lei de refração de Snell :
sin(α) = k · sin(β).
Para uso posterior, podemos reescrever a lei de Snell assim:
v1
sin(α) = ,
v2
ou seja
sin(α) sin(β)
= .
v1 v2
Para terminar, é natural nos perguntarmos que acontece com a
trajetória da luz ao viajar por um meio com ı́ndice de refração variável.
Qual o formato da trajetória da luz, qual a sua equação ?
A resposta a esse tipo de pergunta depende de mais teoria matemática,
por exemplo do Cálculo de Variações.
3. Exercı́cios
Exercı́cio 3.1. (resolvido)
O Problema do salva-vidas: estando em (8, 0), na areia da praia, o
salva-vidas tem que sair correndo para salvar alguém que se afoga no
ponto B = (0, 5), denro do mar.
Suponha que a velocidade do salva-vidas na praia é v1 m/s e na
água é v2 < v1 , com razão:
v1
k := < 1.
v2
Para que ele chegue o mais rápido possı́vel, até que ponto (x, 0) ele
deve correr pela praia, para daı́ ir em linha reta nadando até B.
i) se k ≤ 0.5 então g(0) < g(8).
ii) Expresse x como função de k, x(k).
iii) mostre que limk→0 x(k) = 0.
iv) se 0 ≤ k ≤ 0.5 então x(k) < g(0).
CAPı́TULO 18

As Cônicas e suas propriedades refletivas

1. Definição unificada das cônicas


No colégio se insiste em apresentar as cônicas separadamente, sem
que se dê uma definição unificada.
A Definição 1.1 a seguir englobará todas as cônicas, menos uma, o
Cı́rculo. Mas veremos em seguida que a Definição 1.1 compreende a
Definição 1.3, a qual se estende naturalmente ao Cı́rculo.
Lembre que a distância de um ponto P a uma reta r, denotada P r
a seguir, é a distância do ponto P ao pé da perpendicular a r por P .
Definição 1.1. Fixe uma reta r e um ponto F ∈ / r. Uma cônica
é o lugar geométrico no plano dos pontos P cuja distância P F está
numa razão constante para a distância P r. Ou seja:
PF
= e, e > 0.
Pr
A grandeza e será chamada de excentricidade da cônica, F , de foco e
r, de diretriz.

Afirmação 1.1. Considere uma cônica de foco F , diretriz r e ex-


centricidade e. Então existe um sistema cartesiano de coordenadas em
que
• a origem (0, 0) pertence à conica,
• a diretriz vira a reta vertical x = −ρ, com ρ > 0,
• o foco é F = (eρ, 0)
• os pontos P = (x, y) da cônica satisfazem a equação:
(1 − e2 ) · x2 − 2e(1 + e)ρ · x + y 2 = 0.
Ademais, se e = 1 a equação vira:
1
x= · y2

assim como o foco vira F = (ρ, 0) e a diretriz, x = −ρ.
Se e < 1 , a equação geral vira
x2 2 y2
− · x + = 0,
a2 a b2
213
1. DEFINIÇÃO UNIFICADA DAS CÔNICAS 214

onde
eρ p
a := > 0 e b := a2 · (1 − e2 ) > 0.
1−e
Se e > 1, a equação geral vira:
x2 2 y2
+ · x − = 0,
a2 a b2
onde
eρ p
a := >0 e b := a2 (e2 − 1) > 0.
e−1

Definição 1.2. A cônica


1
x= · y2,

do caso e = 1 da Afirmação 1.1, é chamada parábola.
• Ela tem óbvia simetria no eixo dos y e o eixo x é chamado de
eixo da parábola.
• Um reta vertical pelo foco F = (ρ, 0) intersecta a parábola em
dois pontos (ρ, ±2ρ). A distância de F a cada um deles, que
é 2ρ, é chamada semi-latus rectum 1 da parábola.
• Num novo sistema cartesiano (x, y) em que o vértice P0 está
em (x, y) = (h, k) e o foco está na reta y = k a parábola
y 2 = 4ρx
se escreve como:
(y − k)2 = 4ρ(x − h)
que expandido dá:
y 2 − 2ky − 4ρx + k 2 + 4h = a1 y 2 + a2 y + a3 x + a4 = 0.
Em Exercı́cios pode se pedir para, a partir de uma equação do
tipo:
a1 y 2 + a2 y + a3 x + a4 = 0
determinar a parábola, com o vértice, o foco e a diretriz.
Também o papel de x e y pode estar trocado.
• A pista para chegar na parábola está em que só há grau 2 em
uma das coordenas.

Para entendermos melhor as cônicas nos casos e 6= 1:

1semi largura ortogonal


CAPÍTULO 18. AS CÔNICAS E SUAS PROPRIEDADES
REFLETIVAS 215

Afirmação 1.2. No caso 0 < e < 1 da Afirmação 1.1, existe um


novo sistema de coordenadas (x, y) dado por
x=x−a e y=y
em que a equação vira:
x y
2
+ 2 =1
a b
e no qual as coordenadas do foco são

F = (− a2 − b2 , 0),
para
eρ p
a := > 0 e b := a2 · (1 − e2 ) > 0.
1−e
Ademais2: √
a2 − b 2
e= .
a
No caso 1 < e da Afirmação 1.1, existe um novo sistema de coor-
denadas (x, y) dado por
x=x−a e y=y
em que a equação vira:
x y
2
− 2 =1
a b
e no qual as coordenadas do foco são

F = ( a2 + b2 , 0),
onde
eρ p
a := >0 e b := a2 (e2 − 1) > 0.
e−1
Ademais3: √
a2 + b 2
e= .
a

Definição 1.3. A cônica do caso 0 < e < 1 da Afirmação 1.2 é


chamada elipse. √
Um reta vertical
√ por F 1 = (− a2 − b2 , 0) intersecta a elipse em
b 2
dois pontos (− a2 − b2 , ± a ). A distância de F1 a cada um deles, que
2
é ba , é o semi-latus rectum da elipse.
Note que:

2Na

apostila c := √a2 − b2 para elipses
3Na apostila, c := a2 + b2 para hipérboles
1. DEFINIÇÃO UNIFICADA DAS CÔNICAS 216

• A elipse tem simetria tanto no eixo dos x como no eixo dos y.


Daı́ se obtem que ela poderia √ ser definida também com base
num segundo foco√F2 := ( a2 − b2 , 0) como o foi com base
em F1 := F = (− a2 − b2 , 0). Haverá uma segunda diretriz,
cuja distância ao foco F2 é a mesma da primeira diretriz a F1 .

r1 r2
b
F1 F2

ρ a a ρ
b

• Se na equação
x2 y 2
+ 2 =1
a2 b
fazemos a = b então os dois focos coincidem em (0, 0) e temos
o Cı́rculo de raio a.
2
• O raio a = aa do cı́rculo é um caso particular de semi-latus
rectum.
• Num novo sistema cartesiano (x, y) em que o vértice P0 está
em (x, y) = (h, k) e os focos estão na reta y = k, a elipse
x2 y 2
+ 2 =1
a2 b
se escreve como:
(x − h)2 (y − k)2
+ =1
a2 b2
que expandido dá uma expressão do tipo:
a1 x2 + a2 x + a3 y + a4 y 2 + a5 = 0.
Em Exercı́cios pode se pedir para, a partir de uma equação de
elipse do tipo
a1 x 2 + a2 x + a3 y + a4 y 2 + a5 = 0
determinar focos, eixos e a excentricidade.
Também o papel de x e y pode estar trocado.
2 2
• A pista para chegar na elipse na forma (x−h)
a2
+ (y−k)
b2
= 1 está
em completar os quadrados, ou seja, agrupar os termos em x
CAPÍTULO 18. AS CÔNICAS E SUAS PROPRIEDADES
REFLETIVAS 217

separadamente dos em y e forçar a parecer binômios (x − h)2


e (y − k)2
Definição 1.4. A cônica do caso 1 < e da Afirmação 1.2 é chamada
hipérbole e tem simetria4 no eixo√ x e no eixo y.
Um reta
√ vertical por F1 = ( a2 + b2 , 0) intersecta a elipse em dois
b2 2
pontos ( a + b , ± a ). A distância de F1 a cada um deles, que é ba ,
2 2

é o semi-latus rectum da hipérbole.

Demonstração. (da Afirmação 1.1)


Seja então R ∈ r o pé da perpendicular a r desde F . Considere o
segmento de reta RF .
Afirmo que existe apenas um ponto5 P0 no segmento RF tal que
P0 F = e · P0 r.
De fato, se identificamos a reta RF com os Reais, e se usamos a coor-
denada 0 para R e f > 0 para F , queremos resolver a equação:
f − x = e · (x − 0) = e · x,
o que dá:
(e + 1) · x = f,
f
cuja única solução é x0 = e+1
.
Noto que 0 < x0 < f , pois e > 0.
Escolho como sistema cartesiano de coordenadas (x, y) aquele que
tem origem em P0 , eixo horizontal P0 F (orientado de R para F ) e eixo
vertical a perpendicular a P0 F por P0 .
Nesse sistema, P0 = (0, 0) e se ρ := P0 r > 0 a diretriz é
x = −ρ e F = (eρ, 0).
Ademais, pela sua Definição, qualquer ponto P = (x, y) da cônica
verifica: p p
(x − eρ)2 + y 2 = e · (x + ρ)2 ,
p p
pois P F = (x − eρ)2 + y 2 e P r = (x + ρ)2 . Portanto os pontos da
cônica satisfazem:
(x − eρ)2 + y 2 = e2 · (x + ρ)2 ,
ou seja, após simplificar:
(1 − e2 ) · x2 − 2e(1 + e)ρ · x + y 2 = 0.
Caso e = 1:
Nesse caso a equação acima vira:
4ρ · x = y 2 ,
com F = (ρ, 0) e a diretriz vira x = −ρ.
4Daı́ se obtem que poderia ser definida também com √
base num segundo foco

F2 := (− a2 + b2 , 0) como o foi com base em F1 := F = ( a2 + b2 , 0).
5Será chamado de vértice
1. DEFINIÇÃO UNIFICADA DAS CÔNICAS 218

Caso 0 < e < 1:


Nesse caso podemos dividir a equação
(1 − e2 ) · x2 − 2e(1 + e)ρ · x + y 2 = 0
por 1 − e2 obtendo:
2eρ y2
x2 − ·x+ = 0.
1−e 1 − e2
Introduzo uma constante a e depois uma b pela regra:
eρ p
a := > 0 e b := a2 · (1 − e2 ) > 0.
1−e
Então a última equação vira:
a2 2
x2 − 2ax + ·y =0
b2
que dividida por a2 dá:
x2 2 y2
− · x + = 0.
a2 a b2
Caso 1 < e: Nesse caso, analogamente ao que fizemos no Caso
anterior, mas com
eρ p
a := > 0 e b := a2 (e2 − 1) > 0
e−1
obtemos a equação:
x2 2 y2
+ · x − = 0.
a2 a b2


Demonstração. (da Afirmação 1.2)


No caso 0 < e < 1 já temos a equação
x2 2 y2
− · x + =0
a2 a b2
para a cônica, onde

a := > 0.
1−e
Portanto vemos que essa cônica intersecta a reta y = 0 em P0 = (0, 0)
e em
P1 := (2a, 0).
Considere o ponto médio do segmento P0 P1 :
C := (a, 0).
Vamos transladar a origem do sistema de coordenadas para C. Para
isso estabeleçamos um novo sistema de coordenadas (x, y) onde:
x = x − a e y = y.
CAPÍTULO 18. AS CÔNICAS E SUAS PROPRIEDADES
REFLETIVAS 219

Então a equação da cônica vira:


(x + a)2 2 y2
− · (x + a) + 2 = 0,
a2 a b
ou seja:
x y
2
+ 2 = 1.
a b
O foco F tinha coordenada x dada por eρ e agora, no novo sistema,
terá coordenada x dada por:
eρ e2 ρ
eρ − a = eρ − =− =
1−e 1−e
p p
e4 ρ2 e2 ρ2 − e2 ρ2 (1 − e2 )
=− =− =
1−e 1−e
s
e2 ρ2 e2 ρ2 (1 − e2 )
=− − =
(1 − e)2 (1 − e)2

= − a2 − b 2 .
Da definição de a sai:
eρ − a = −ae
e do anterior: √
a2 − b 2
e= .
a
Já no caso 1 < e temos a equação
x2 2 y2
+ · x − =0
a2 a b2
para a cônica.
Portanto essa cônica intersecta a reta y = 0 em P0 = (0, 0) e em
P1 := (−2a, 0).
Considere o ponto médio do segmento P0 P1 :
C := (−a, 0).
Vamos transladar a origem do sistema de coordenadas para C. Para
isso usamos um novo sistema de coordenadas (x, y) onde:
x = x + a e y = y.
Então a equação da cônica vira:
(x − a)2 2 y2
+ · (x − a) − = 0,
a2 a b2
ou seja:
x2 y 2
− 2 = 1.
a2 b
2. A PARÁBOLA E SUA PROPRIEDADE REFLETIVA 220

O foco F tinha coordenada x dada por eρ e agora, no novo sistema,


terá coordenada x dada por:
eρ e2 ρ
eρ + a = eρ + = =
e−1 e−1
p p
e4 ρ2 e2 ρ2 + e2 ρ2 (e2 − 1)
= = =
e−1 e−1
s
e2 ρ2 e2 ρ2 (e2 − 1)
= + =
(e − 1)2 (e − 1)2

= a2 + b 2 .

2 2
A relação e = a a+b é imediata das definições de a e b.


2. A Parábola e sua propriedade refletiva


A parábola também aparecerá com destaque mais adiante, na Seção
6 do Capı́tulo 24, associada à balı́stica.
Um dos casos mais simples em que a reta tangente muda de acordo
com o ponto escolhido no gráfico é o caso das parábolas.
Mesmo assim já podemos obter algumas informações interessantes,
como o mostrarão as Seções seguintes, desde que soubermos calcular
essas tangentes.
Afirmação 2.1. Um ponto P satisfaz a equação
y = Cx2 , C∈R
1
se e somente se P equidista da reta horizontal y = − 4C e do ponto
1
F = (0, 4C ) (chamado de foco).
Demonstração.
Para provarmos isso, basta usarmos o caso e = 1 da Afirmação 1.1,
1
trocando x por y e fazendo C = 4ρ .
Mas também podemos fazer uma conta explı́cita, como segue.
Temos para P = (x, Cx2 ):
r
1 2
P F = (x − 0)2 + (Cx2 − ) =
4C
r
x2 1
= x2 + C 2 x4 − + 2 2 =
2 4C
r
x2 1
= C 2 x4 + + 2 2 =
2 4C
r
1 2
= (Cx2 + )
4C
CAPÍTULO 18. AS CÔNICAS E SUAS PROPRIEDADES
REFLETIVAS 221
1
e a distância de P até a reta y = − 4C é dada pelo tamanho
r
1 2
(Cx2 + ).
4C
Reciprocamente, se P = (x, y) satisfaz
r r
1 1 2
x2 + (y − )2 = (y + )
4C 4C
então
1 2 1 2
x2 + (y − ) = (y + )
4C 4C
de onde
y 1 y 1
x2 + y 2 − + 2 2 = y2 + + 2 2,
2C 4 C 2C 4 C
de onde:
y
x2 = e y = Cx2 .
C


1
Considere então a parábola y = Cx2 , com foco F := (0, 4C ) e reta
1
diretriz horizontal y = − 4C .
Dado um ponto P = (x, Cx2 ) qualquer de seu gráfico, denote p sua
a projeção vertical na reta diretriz:
1
p := (x, − ).
4C
Afirmação 2.2.
1 1
A reta rx que liga os pontos p = (x, − 4C ) e F = (0, 4C ) é ortogonal
à reta tangente Tx ao gráfico de y = Cx2 em P = (x, Cx2 ).
Ademais, rx e Tx se intersectam em Mx := ( x2 , 0), que é o ponto
médio do segmento de p e F .
Em suma, Tx é a reta mediatriz do segmento ligando p e F .
As Figuras a seguir ilustram a Afirmação:

0
-4 -2 0 2 4
x

-2

-4
2. A PARÁBOLA E SUA PROPRIEDADE REFLETIVA 222
2
Fig: y = x4 , tangente y = x − 1 em P = (2, 1),
onde F = (0, 1), M = (1, 0) e p = (2, −1).

2
x
-4 -2 0 2 4
0

-2

-4

-6

-8

Fig: A Figura de antes e ademais a tangente y = 23 x − 9


4
em P = (3, 1), M = ( 23 , 0) e p = (3, −1).

Demonstração.
Já sabemos que a reta tangente Tx tem equação:
y = (2Cx) · x − Cx2 .
E a reta rx ligando p e F tem coeficiente angular:
1 −1
4C
− 4C −1
= ,
0−x 2Cx
logo rx e Tx são ortogonais.
1
Por passar por F = (0, 4C ) a equação de rx é:
−1 1
rx : y = ·x+ .
2Cx 4C
Avaliando ambas as equações de retas em Mx = ( x2 , 0) vemos que
Tx e rx contêm Mx = ( x2 , 0).
Ademais as coordenadas de Mx são média aritmética das coorde-
1 1
nadas de (x, − 4C ) e (0, 4C ), logo Mx é ponto médio do segmento que
os une.


Agora vamos extrair consequências da Afirmação 2.2.

Note que os triângulos retângulos ∆F P Mx e ∆p P Mx são congru-


entes: de fato, P F = P p já que P está na parábola, F Mx = Mx p por
Mx ser ponto médio e P Mx ser lado comum a ambos.
Logo os ângulos ∠F P Mx e ∠Mx P p são congruentes.
CAPÍTULO 18. AS CÔNICAS E SUAS PROPRIEDADES
REFLETIVAS 223

Considere em torno de P os ângulos ∠Mx P p e seu ângulo oposto


pelo vértice. Como são congruentes, temos que o ângulo que a reta
vertical pP faz com a tangente Tx é congruente com o ângulo ∠F P Mx .

F
P

Em Ótica se postula que a luz se reflete numa curva da seguinte


forma:

o ângulo de incidência que se forma entre o raio de luz e a tangente


da curva é igual ao ângulo (não orientado) formado pelo raio refletido
e a tangente da curva.

Pelo que vimos, isso quer dizer que raios de luz que chegam ver-
ticalmente devem refletir na parábola y = Cx2 e passar todos pelo
1
ponto F = (0, 4C ) que por isso merece o nome de foco, por concentrar
a luz. Esse fato é usado em antenas, microfones, espelhos de formato
parabólico, para concentrar ondas, som, calor, luz em um ponto, que é
o Foco.
Como não posso plotar retas verticais, não pude fazer o Exemplo a
seguir na posição vertical. Tive que colocar na horizontal. E só pude
usar metade da parábola, para ter um gráfico. Então a Figura a seguir
ilustra a concentração de 5 raios horizontais refletidos no Foco:

2,5

1,5

0,5

0
0 0,20,40,60,8 1
x

y2
Figura: Braço da parábola x = 4
refletindo 5 raios horizontais no Foco F = (1, 0).
3. A ELIPSE E SUA PROPRIEDADE REFLETIVA 224

3. A Elipse e sua propriedade refletiva


Afirmação 3.1. Um ponto P = (x, y) satisfaz a equação
x2 y 2
+ 2 =1
a2 b
se e somente se
P F1 + P F2 = 2a,
onde F1 = (−c, 0) e F2 = (c, 0) são os dois focos e
a2 = b 2 + c 2
.

Observe que esta Afirmação 3.1 dá um método prático para traçar
uma elipse: fixe dois pontos F1 e F2 , com dois pregos, e ligue-os por
um cordão maior que a distância F1 F2 . Com um lápis estique o cordão
e agora mova o lápis, sempre mantendo o barbante esticado, traçando
pontos P . Você traçará uma elipse, pois F1 P + P F2 é constante.
Demonstração. (da Afirmação 3.1)
Como notamos após a Definição 1.3, uma elipse pode ser definida
com relação a dois pares Foco/diretriz: F, r ou F ′ r′ .
Para qualquer ponto P da elipse temos
PF = e · P r e P F ′ = e · P r′ ,
onde r, r′ são as retas diretrizes.

r r’

F F’
ρ a a ρ

Logo
P F + P F ′ = e · r r′ ,
onde r r′ é a distância entre essas duas retas (paralelas).
Ou seja, que P F + P F ′ ≡ C é constante para pontos na elipse.
Na descrição que demos, a excentricidade e da elipse verifica:

a=
1−e
CAPÍTULO 18. AS CÔNICAS E SUAS PROPRIEDADES
REFLETIVAS 225

ou seja, 2a − 2ae = 2eρ e portanto


2a = e · (2a + 2p).
Ora, como nos lembra a Figura acima:
2a + 2ρ = r r′
é a distância entre as duas retas diretrizes da elipse. Logo
P F + P F ′ ≡ 2a.
A Afirmação 1.2 e a simetria no eixo x dão que as coordenadas dos
focos são F1 = (−c, 0) e F2 = (c, 0), onde

c = a2 − b 2 .


A elipse tem a notável propriedade seguinte:

se P é um ponto da elipse e P F1 , P F2 duas semiretas que ligam P


aos focos, então os ângulos formados por P F1 e a tangente em P e o
formado por P F2 e a tangente em P são iguais.
Em outras palavras, se um raio de luz sai de um foco e reflete na
elipse então ele passa no outro foco.
Para provar isso, notamos primeiro o seguinte:
Afirmação 3.2. Se uma reta só intersecta uma elipse num único
ponto P , então essa reta é a reta tangente à elipse em P .
Demonstração.
2 2
Considerarei apenas pontos da elipse xa2 + yb2 = 1 com coordenada
y > 0, ou seja, onde posso representar a elipse pelo gráfico de
r
x2
y = b · 1 − 2,
a
pois para os outros é análogo, usando outros gráficos do tipo y = y(x)
ou x = x(y). q
2
Uma reta y = A · x + B que passa por (x, b · 1 − xa2 ) tem equação:
r
x2
y = A x + (b · 1 − 2 − Ax).
a
2
x2
Se a intersecto com a elipse + yb2 = 1 obtemos:
a
q
2
x 2 (A x + b 1 − xa2 − Ax)2
+ − 1 = 0,
a2 b2
3. A ELIPSE E SUA PROPRIEDADE REFLETIVA 226

que é uma equação quadrática em x:


q
x2
A 2
1 −2A x2 2 1− a2
A a2 x 2 x 2
( 2 + 2 ) · x2 + ( + )·x+ − 2 =0
a a b2 b b2 a
2
(note que de fato é quadrática em x, pois Aa2 + a12 > 0).
O dicriminante desta função quadrática em x é:
q
2
4(−a4 A2 + a2 A2 x2 − 2a2 b 1 − xa2 Ax − b2 x2 )
,
b 2 a4
e procuramos valores de A tais que, ∀x, anulem esse discriminante (pois
isso dirá que para esses valores de A há apenas 1 intersecção da reta
com a elipse).
Ou seja, buscamos A que anulem o numerador
r
x2
4 2 2 2 2
−a A + a A x − 2a b 1 − 2 Ax − b2 x2 .
2
a
Uma conta tediosa prova que:
r
x2
−a4 A2 + a2 A2 x2 − 2a2 b 1 − 2 Ax − b2 x2 =
a
bx
= (−a4 + a2 x2 ) · ( A + q )2
x2
a2 1 − a2
e portanto
−b x
A= q
x2
a2 1 − a2

é o valor de A que anula o discriminante acima, ∀x.


Por outro lado reconhecemos que
−bx
q = f ′ (x),
x2
a2 1 − a2

onde r
x2
f (x) = b · 1− .
a2
Logo a reta que só corta a elipse em P é de fato a sua reta tangente.


A seguinte afirmação explica o fato de que um raio e luz saindo de


um foco da elipse e refletindo na elipse passará necessariamente pelo
outro foco:
CAPÍTULO 18. AS CÔNICAS E SUAS PROPRIEDADES
REFLETIVAS 227

Afirmação 3.3. As semiretas que ligam um ponto P da elipse aos


dois focos F1 , F2 formam os mesmos ângulos (não-orientados) com a
tangente à elipse passando por P .
Demonstração.
Considere P na elipse e o triângulo ∆F1 P F2 .
Tome um ângulo externo α desse triângulo (veja a Figura).
F2 ’

F1 F2

Considere a bissectriz desse ângulo (ou seja, uma semireta que o


divide em dois ângulos iguais, de valores α2 ).
Marque um ponto F2′ no ângulo externo, cuja distância até P seja a
mesma de F2 (denote essas distâncias por P F2 = P F2′ ). Veja a Figura:
r
F2 ’

α/2
β
α/2
Q

F1 F2

Tome qualquer ponto Q da reta r que contém essa bissectriz, Q 6= P .


Já que o Q não está alinhado com F1 e F2′ , temos:
F1 Q + QF2′ > F1 P + P F2′ =
= F1 P + P F2 .
Já que a elipse é o lugar dos pontos P com
F1 P + P F2 ≡ 2a
vemos que Q não está na elipse.
Ou seja que o único ponto da reta r que está na elipse é P .
A Afirmação 3.2 anterior garante então que r é a tangente por P .
Mas o ângulo β é oposto pelo vértice ao ângulo que mede α2 .
Ou seja que as semiretas ligando P aos focos determinam ângulos
com reta tangente que medem ambos α2 .

4. A HIPÉRBOLE E O ANÁLOGO DA PROPRIEDADE
REFLETIVA 228

4. A Hipérbole e o análogo da propriedade refletiva


Afirmação 4.1. Um ponto P = (x, y) satisfaz a equação

x2 y 2
− 2 =1
a2 b
se e somente se
| P F1 − P F2 | = 2a,
onde F1 = (−c, 0) e F2 = (c, 0) são os dois focos e b2 = c2 − a2 .
Demonstração.
Por exemplo suponhamos que P F1 − P F2 ≥ 0.
Por definição
P F1 − P F2 = e · P r1 − e · P r2 .

= e · r1 r2
logo P F1 − P F2 ≡ C é constante.
Pela Afirmação 1.2,
ep
a= ,
e−1
ou seja 2ae − 2a = 2ep e
2a = e · (2a − 2p).
Mas 2a − 2p = r1 r2 .
Também a Afirmação 1.2 e a simetria da hipérbole no eixo x dão
que os focos têm essas coordenadas.


A hipérbole tem uma propriedade do mesmo tipo da elipse, a saber:

Os segmentos de reta que ligam um ponto de uma hipérbole aos


seus dois focos ficam bissectados pela reta tangente naquele ponto.

Para provarmos isso, como fizemos no caso da elipse, primeiro


provaremos o seguinte:
Afirmação 4.2. Se uma reta só intersecta uma hiperbole de equação
2
x2
a2
− yb2 = 1 ( a, b > 0 ) num único ponto P , então
• i) essa reta é reta tangente à hiperbole em P ou
• ii) é uma reta paralela à reta y = ab ou
• iii) é uma reta paralela à reta y = − ab .
CAPÍTULO 18. AS CÔNICAS E SUAS PROPRIEDADES
REFLETIVAS 229

3
2
1
y 0
-6 -4 -2 0 2 4 6
-1
x
-2
-3

2
Figura: a hipérbole x22 − y 2 = 1 e retas paralelas
às retas y = 12 x e y = − 12 x.
Demonstração. (Afirmação 4.2)
2 2
Considero pontos da hipérbole xa2 − yb2 = 1 com coordenada y > 0,
ou seja, onde posso representar a hipérbole pelo gráfico de
r
x2
y=b − 1.
a2
q
2
Quero intersectar uma reta qualquer por (x, b xa2 − 1):
r
x2
y = Ax + b − 1 − Ax,
a2
com a hipérbole. Obtendo então de
q
x2 2
x2 (A x + b 1 − a2 − Ax)
− − 1 = 0,
a2 b2
a equação em x:
q q
x2 x2
1 A 2
2 2A x 2 a2 − 1 A
2
x A x 2 a2 − 1 Ax
2 2 2
( 2 − 2 ) x +( 2 − ) x− 2 − 2 + = 0.
a b b b a b b2
Essa equação em x deixa de ser quadrática quando
1 A2
− = 0.
a2 b2
Ou seja, as retas passando por P com coeficientes angulares
b
A=±
a
só cortam a hipérbole em P .
4. A HIPÉRBOLE E O ANÁLOGO DA PROPRIEDADE
REFLETIVA 230

1 A2
Quando a2
− 6= 0, o dicriminante da função quadrática em x é:
b2
q
2
4(−a A + a A x − 2a b xa2 − 1 Ax + b2 x2 )
4 2 2 2 2 2
,
b 2 a4
e procuramos por coeficientes angulares A tais que, ∀x, anulem esse
discriminante (pois isso dará apenas uma única interseccçao da reta
com a elipse).
Ou seja, buscamos A que anulem o numerador
r
x2
−a4 A2 + a2 A2 x2 − 2a2 b − 1 Ax + b2 x2 .
a2
Mas uma conta tediosa mostra que:
r
4 2 2 2 2 2 x2
−a A + a A x − 2a b 2
− 1 Ax + b2 x2 =
a
bx
= (−a4 + a2 x2 ) · ( A − q )2
x 2
a2 a 2 − 1
e portanto
bx
A= q
x2
a2 a2
−1
é o valor de A que anula o discriminante acima, ∀x.
Por outro lado reconhecemos que
bx
q = f ′ (x),
2 x2
a a2
−1
onde r
x2
f (x) = b − 1.
a2
Logo a reta que só corta a hipérbole em P é de fato a reta tangente.


2 2
Afirmação 4.3. Quando |x| → ∞ os pontos da hiperbole xa2 − xy 2 =
1 se aproximam das reta y = ab x ou da reta y = − ab x (chamadas de
assı́ntotas).

Com esta Afirmação e a Afirmação 4.2 podemos dizer:

fora as tangentes, as únicas retas que só cortam a hipérbole em 1


ponto são as retas paralelas às assı́ntotas da hipérbole dada.

Demonstração. (Afirmação 4.3)


CAPÍTULO 18. AS CÔNICAS E SUAS PROPRIEDADES
REFLETIVAS 231
2 2
Cada ponto da hipérbole xa2 − xy 2 = 1 pode ser descrito ou como
ponto do gráfico de
r
x2 b√ 2
f1 (x) = b − 1 = x − a2 ,
a2 a
ou como ponto do gráfico de
r
x2 b√ 2
f2 (x) = −b 2
−1=− x − a2 .
a a
Se vamos fazer |x| → ∞, obviamente podemos supô-lo |x| =
6 0 e
escrever: r r
b a 2 b a2
f1 (x) = x2 (1 − 2 ) = |x| 1 − 2 ,
a x a x
r r
b a2 b a2
f2 (x) = − x2 (1 − 2 ) = − |x| 1 − 2 ,
a x a x
e claramente: r
a2
lim 1 − 2 = 1.
|x|→+∞ x
Ou seja que quando |x| → ∞ o gráfico de f1 tende ao gráficos de
y = ab |x| enquanto que o de f2 tende ao de y = − ab |x| .

Podemos ser mais detalhados:


Se x → +∞, temos o gráfico de f1 (x) se aproximando do de y = ab x.
Mas se x → −∞ temos f1 (x) se aproximando de
b b
y = (−x) = − x.
a a
Se x → +∞, temos o gráfico de f2 (x) se aproximando do de y =
− ab x. Mas se x → −∞ temos f2 (x) se aproximando do de
b b
y = − (−x) = x.
a a


Afirmação 4.4. As semiretas que ligam um ponto P da hipérbole


aos dois focos F1 , F2 formam os mesmos ângulos (não-orientados) com
a tangente à hipérbole em P .
Demonstração.
Considere P um ponto da hipérbole. Como | P F1 − P F2 | ≡ C > 0
posso supor que tomei P no ramo da hipérbole onde P F1 − P F2 ≡ C >
0 (seria análogo o outro caso, trocando os papéis de F1 e F2 ).
4. A HIPÉRBOLE E O ANÁLOGO DA PROPRIEDADE
REFLETIVA 232

F2 ’
α/2 α/2

F1 F2

Marque no segmento de reta [F1 P ] o ponto F2′ que tem P F2 = P F2′ .


Considere a bissectriz r do ângulo α em P que faz parte do triângulo
∆F1 P F2 .
Tome um ponto Q ∈ r, Q 6= P .

Caso 1: Suponhamos QF1 ≥ QF2′ :

Então como Q não está alinhado com F1 , F2′ , P , temos:


QF2′ + F2′ F1 > F1 Q,
e portanto:
F2′ F1 > F1 Q − QF2′ ≥ 0.
Note que a nossa reta r funciona também como mediatriz do segmento
[F2′ F2 ] (por ser a bissectriz do triângulo isósceles ∆F2′ P F2′ ). Logo
QF2′ = QF2
e portanto:
F2′ F1 > F1 Q − QF2 .
Por outro lado, já que o ponto F2′ está no segmento [F1 P ], temos:
F2′ F1 = P F1 − P F2′ =
= P F1 − P F2 .
Como este último valor é positivo, pela escolha de P ,
| P F1 − P F2 | = P F1 − P F2 ≡ C > 0
e
| P F1 − P F2 | > F1 Q − QF2 ≥ 0
nos faz concluir que Q não pertence à elipse.
Ou seja, que da reta r somente o ponto P está na elipse.
CAPÍTULO 18. AS CÔNICAS E SUAS PROPRIEDADES
REFLETIVAS 233

Vemos em seguida que r não é paralela a nenhuma das assı́ntotas da


hipérbole. Portanto, pela Afirmação 4.2, conclı́mos que r é a tangent
à hipérbole no ponto P .

Caso 2: Suponhamos QF2′ ≥ QF1 :

Então como Q não está alinhado com F1 , F2′ , P , temos:


QF1 + F1 F2′ > QF2′ ,
e portanto:
F2′ F1 > QF2′ − QF1 ≥ 0.
O Resto da prova neste Caso 2 é exatamente igual ao do Caso 1.


5. Exercı́cios
Exercı́cio 5.1. 2
2
Chamamos uma hipérbole xa2 − yb2 = 1 de retangular se suas assı́ntotas
são ortogonais entre si.
Qual a relação entre a e b que é necessária e suficiente para termos
uma hipérbole retangular ?
CAPı́TULO 19

Integração e o Primeiro Teorema Fundamental

1. Área sob um gráfico positivo

Dado um gráfico de uma função contı́nua y = f (x) ≥ 0 quero


entender qual a Área compreendida sob esse gráfico e acima do eixo x,
da vertical x = a até a vertical x = b.
Se y = f (x) = ax + b é uma reta tudo ok, já sabemos o que são
áreas de triângulos, retângulo, trapézios, etc. Mas e se y = f (x) não for
uma reta ? Se f (x) não é a equação de uma reta, vemos que realmente
precisamos definir de maneira matematicamente correta a intuição que
temos de que há uma figura sob esse gráfico e que ela tem uma certa
área.
A idéia de Bernard Riemann é de ir subdividindo o domı́nio da
f e colocando lado a lado retângulos sob o gráfico (vou chamá-los
de retângulos justapostos sob o gráfico). A soma das áreas desses
retângulos é menor que a área buscada, mas a medida que se refina
a subdivisão do domı́nio a soma de áreas dos retângulos justapostos
sob o gráfico se aproxima de um certo valor.

Figura: Cinco retângulos sob o gráfico, de mesma largura (1/5 do intervalo).

235
2. QUAL FUNÇÃO DESCREVE AS ÁREAS SOB GRÁFICOS?236
1
Figura: 12 retângulos sob o gráfico, de mesma largura ( 12 do intervalo).

1
Figura: 24 retângulos sob o gráfico, de mesma largura ( 24 do intervalo).
Nem precisam ter os retângulos a mesma largura, como nas Figuras;
basta que o máximo das larguras dos retângulos tenda a zero à medida
que refinamos as escolhas dos retângulos.
Isso parece ainda um pouco vago, mas na Seção seguinte faremos
alguns Exemplos explı́citos, onde fazemos a partição da base ficar cada
vez mais fina e obtemos via um limite um valor bem determinando,
que será a área. É possı́vel provar um teorema geral do seguinte tipo:
Afirmação 1.1. (B. Riemann)1 Seja f : [a, b] → R, f (x) ≥
0 contı́nua.2 Então a soma das áreas de retângulos justapostos sob
o gráfico tende a um certo número bem definido, quando a largura
máxima dos retângulos tende a zero.
Esse número é por definição a Área sob o gráfico de f , de a até b,
denotada por Af,a (b).

2. Qual função descreve as Áreas sob gráficos?

Dado uma função y = f (x) não-negativa, fixado um ponto inicial a


de seu domı́nio definimos acima a área sob seu gráfico até b.
Vamos agora fixar a e mudar o nome de b, passando a chamar-se
agora x para significar que vamos variar o b.
Então a área sob o gráfico vira uma nova função Af,a (x), que para
cada valor de x dá um resultado de Área.
Qual é essa função A(x)? E que propriedades ela tem?
Certamente é uma função crescente, será que Af,a (x) é contı́nua?
Será que ela é derivável ?
Com o que sabemos do colégio, só consigo ver dois tipos de exemplos
simples de f , onde responderı́amos facilmente sobre Af,a (x):

1Observo desde já que se pode dar versões bem mais fortes desse teorema de
Riemann.
2Note que se f não fosse contı́nua, quem sabe f não tivesse máximo global em
[a, b] e a figura sob seu gráfico fosse infinitamente alta. Nesse caso a área poderia ser
infinita, não ser um número determinando. Mas sendo f contı́nua e [a, b] intervalo
fechado limitado então ela tem um máximo global
CAPÍTULO 19. INTEGRAÇÃO E O PRIMEIRO TEOREMA
FUNDAMENTAL 237

• Exemplo 1 : Se y = C ≥ 0 é constante e a = 0, então AC,0 (x)


é a área de um retângulo de largura x e altura C; portanto
essa área é dada pela função AC,0 (x) = C · x.
• Exemplo 2 : Se y = Cx e a = 0 então ACx,a (x) é a área de um
triângulo de largura x e altura Cx, portanto essa área é dada
2
pela função ACx,a (x) = x·C·x 2
= Cx2 .
Com o que sabemos do colégio não conseguimos prever que acontece
com a área de 0 até x sob o gráfico, se o gráfico fosse o de y = C · x2 .
Mas com o que já aprendemos neste Curso sim podemos !
• Exemplo 3: Seja y = C · x2 , C ≥ 0, a = 0 escolha um x, 0 < x.
Considere a soma de áreas dos seguinte retângulos sob o
gráfico de y = C · x2 : particione o intervalo [0, x] em n inter-
valos de mesmo tamanho:
x x 2x (n − 1)x nx
[0, x] = [0, ] ∪ [ , ] ∪ . . . ∪ [ , ].
n n n n n
Tome como primeiro retângulo sob o gráfico o retângulo de
base [ nx , 2x
n
] e altura C( nx )2 , o segundo retângulo de base [ 2x
n n
, 3x ]
x 2
e altura C(2 n ) e assim até o (n−1)-ésimo retângulo, cuja base
é [ (n−1)x
n
, nx
n
] e altura C((n − 1) nx )2 .
Como esses retângulos estão sob o gráfico, a soma de suas
áreas é menor que a área real sob o gráfico.
Mas se fazemos n cada vez maior, a soma de área de
retângulos vai tender à área real, que queremos conhecer.
De fato, dado n ∈ N, a soma das áreas dauqeles (n − 1)
retângulos é:
x x2 x 22 x2 x (n − 1)2 x2
· C · 2 + · C · 2 + ... + · C · =
n n n n n n2
x x2
= C · · 2 · [12 + 22 + . . . (n − 1)2 ].
n n
No item iii) da Afirmação 1.1 vimos a fórmula:
n(n + 1)(2n + 1)
12 + 22 + . . . + n2 = , ∀n ∈ N,
6
que dá quando aplicada ao nosso n − 1:
(n − 1)(n − 1 + 1)(2(n − 1) + 1)
12 + 22 + . . . + (n − 1)2 = =
6
(n − 1)n(2n − 1)
= =
6
2n3 − 3n2 + n
= , ∀n ∈ N.
6
Ora, então a soma de áreas dos (n − 1) retângulos é de fato:
x x2 2n3 − 3n2 + n 2n3 − 3n2 + n
C· · 2· = Cx3 .
n n 6 6n3
2. QUAL FUNÇÃO DESCREVE AS ÁREAS SOB GRÁFICOS?238

Mas pelo que já vimos na Parte 1 (já que C e x não mudam
com n):
2n3 − 3n2 + n 31 Cx3
lim Cx3 = Cx = .
n→+∞ 6n3 3 3
Cx3
Então é ACx2 ,0 (x) = 3
.

• Exemplo 4: Seja y = C · x3 , C ≥ 0. Considere a soma de áreas


dos seguinte retângulos sob o gráfico de y = C · x3 : particione
o intervalo [0, x] em n intervalos de mesmo tamanho:
x x 2x (n − 1)x nx
[0, x] = [0, ] ∪ [ , ] ∪ . . . ∪ [ , ].
n n n n n
Tome como primeiro retângulo sob o gráfico o retângulo de
base [ nx , 2x
n
] e altura C( nx )3 , o segundo retângulo de base [ 2x
n n
, 3x ]
e altura C(2 nx )3 e assim até o (n−1)-ésimo retângulo, cuja base
é [ (n−1)x
n
, nx
n
] e altura C((n − 1) nx )3 .
Dado n ∈ N, a soma das áreas desses (n − 1) retângulos é:
x x3 x 23 x3 x (n − 1)3 x3
· C · 3 + · C · 3 + ... + · C · =
n n n n n n3
x x3
= C · · 3 · [13 + 23 + . . . (n − 1)3 ].
n n
Os itens i) e ii) da Afirmação 1.1 dão juntos a fórmula:
n(n + 1) 2
13 + 23 + . . . + n3 = ( ) , ) ∀n ∈ N,
2
que dá quando aplicada ao nosso n − 1:
(n − 1)2 (n)2 n4 − 2n3 + n2
13 + 23 + . . . + (n − 1)3 = = , ∀n ∈ N.
4 4
Ora, então a soma de áreas dos (n − 1) retângulos é de fato:
x x3 n4 − 2n3 + n2 n4 − 2n3 + n2
C· · 3· = Cx3 · .
n n 4 4n4
Mas pelo que já vimos na Parte 1 (já que C e x não mudam
com n):
n4 − 2n3 + n2 Cx4
lim Cx3 · = .
n→+∞ 4n4 4
4
Então ACx3 ,0 (x) = Cx4 .
• Exemplo 5) Também podemos combinar dois Exemplos desses,
por exemplo perguntar pela área sob o gráfico de y = C1 x2 +
C2 x3 , C1 , C2 ≥ 0 de 0 até x.
A soma de área de retângulos será:
x x2 x3 x (n − 1)2 x2 (n − 1)3 x3
· (C1 2 + C2 3 ) + . . . + · (C1 + C2 )=
n n n n n2 n3
CAPÍTULO 19. INTEGRAÇÃO E O PRIMEIRO TEOREMA
FUNDAMENTAL 239

x3 x4
= C1 3 · (1 + 2 + . . . + (n − 1) ) + C2 4 · (13 + 23 + . . . + (n − 1)3 ),
2 2 2
n n
e pelo que vimos nos dois exemplos anteriores 3),4) (e pelo
limite de somas):

x3 2 2 2 x4 3 3 3
lim C1 · (1 + 2 + . . . + (n − 1) ) + C2 4 · (1 + 2 + . . . + (n − 1) ) =
n→+∞ n3 n

x3 x4
= C1 + C2 .
3 4
Nos 5 Exemplos acima há, digamos assim, uma coincidência notável:

A Área como função de x é uma função derivável e ademais a


derivada da Área é a função de partida

Cx2
A(x) = Cx ⇒ A′ (x) = C, A(x) = ⇒ A′ (x) = Cx,
2

Cx3 Cx4
A(x) = ⇒ A′ (x) = Cx2 , A(x) = ⇒ A′ (x) = Cx3 .
3 4

C1 x3 C2 x4
A(x) = + ⇒ A′ (x) = C1 x2 + C2 x3 .
3 4
Como veremos isso não é uma coincidência ! O fato geral por trás
disso, de que derivando a função Área sob o gráfico voltamos na função
que dá o gráfico, será o Primeiro Teorema Fundamental do Cálculo.
E de fato é a chave para se calcular áreas sob gráficos incrivelmente
complicados (no Segundo Teorema fundamental do Cálculo).

3. Primeira Versão do Primeiro Teorema fundamental do


Cálculo
A princı́pio não sabemos muito sobre o gráfico de Af,a (x), porém o
próximo teorema vai nos dizer muito.
Para demonstrarmos o Teorema, começo com uma Afirmação, ilustrada
na figura que segue:

Afirmação 3.1. Suponha f : [a, b] → R é contı́nua e f (x) ≥ 0.


Tome x ∈ [a, b) e h > 0 suficientemente pequeno para que x + h ∈
[a, b]. Então:
Af,x (x + h) = f (ξ) · h,
para algum ponto ξ ∈ [x, x + h].
3. PRIMEIRA VERSÃO DO PRIMEIRO TEOREMA
FUNDAMENTAL DO CÁLCULO 240

M_f

f (ξ)

m_f

Figura: A área sob o gráfico é igual à do retângulo de altura f (ξ), mf < f (ξ) < Mf
Demonstração.
Começo observando que, dado o h > 0, o valor Af,x (h) tem que
estar entre:
mf · h ≤ Af,x (x + h) ≤ Mf · h
onde mf · h é a Área de uma retângulo com base h e altura mf (o
mı́nimo de f em [x, x + h]) e Mf · h é a Área de uma retângulo com
base h e altura Mf (o máximo de f em [x, x + h]).
Divido por h > 0:
Af,x (x + h)
mf ≤ ≤ Mf ,
h
A (x+h)
e portanto f,x h é um valor intermediário da f : [a, b] → R, um
valor entre seu mı́nimo e seu máximo.
Logo pelo T.V.I. existe ξ ∈ [x, x + h] tal que
Af,x (x + h)
= f (ξ),
h
logo Af,x (x + h) = f (ξ) · h.


O Teorema a seguir diz que sempre a derivada da função que mede


áreas sob um gráfico é a função original que dá o gráfico.
Também pode ser lido assim: a operação de derivar cancela o efeito
da operação de tomar área sob o gráfico:
Teorema 3.1. (Primeira versão)
Seja f : [a, b] → R contı́nua, f ≥ 0 e x ∈ [a, b). Então
A′f,a (x) = f (x).
Demonstração. Uso primeiro a definição de derivada em x ∈
[a, b]:
Af,a (x + h) − Af,a (x)
A′f,a (x) = lim .
h→0 h
CAPÍTULO 19. INTEGRAÇÃO E O PRIMEIRO TEOREMA
FUNDAMENTAL 241

Em seguida, a aditividade da Área:


Af,a (x + h) = Af,a (x) + Af,x (x + h)
portanto

Af,a (x) + Af,x (x + h) − Af,a (x)


A′f,a (x) = lim =
h→0 h
Af,x (x + h)
= lim .
h→0 h
Agora uso a Afirmação 3.1 acima, de que
Af,x (x + h) = f (ξ) · h,
onde ξ ∈ [x, h]. Então juntando tudo:
Af,x (h)
A′f,a (x) = lim =
h→0 x+h
f (ξ) · h
lim =
h→0 h
= lim f (ξ).
h→0
Para terminar basta ver que
lim f (ξ) = f (x).
h→0

Mas quando h tende a zero, ξ ∈ [x, x + h] tende a x.


Logo f (ξ) tende a f (x), porque f é contı́nua.


4. A Integral e o Primeiro Teorema fundamental


Até aqui só falamos de funções contı́nuas que são f ≥ 0, pois que-
riamos falar de áreas sob seu gráfico e acima do eixo dos x.
Mas é claro que se f < 0 na região [a, b] faz sentido definir a área da
região compreendida entre o eixo dos x e seu gráfico, que denotaremos
ainda por Af,a (b).
Sem entrar em detalhes técnicos, quero apresentar uma operação
chamada integral definida de f de a até b, de uma função f contı́nua
definida em [a, b] denotada:
Z b
f (x)dx.
a
Dada y = f (x) contı́nua em [a, b] escolha uma lista de pontos,
começando em a e terminando em b:
a = x0 < x1 < . . . < xn = b,
que chamamos de partição de [a, b].
4. A INTEGRAL E O PRIMEIRO TEOREMA FUNDAMENTAL
242

Chamamos de norma dessa partição o máximo dos tamanhos |xi −


xi−1 |. dizer que a norma fica pequena é dizer que aumenta o número
de pontos xi e também que eles ficam bem distribuı́dos em [a, b].
Dada uma partição, escolha uma lista de pontos ξi ∈ [xi , xi + 1].
Tome os valores da f nesses ξi e faça a soma:
(x1 − x0 ) · f (ξ0 ) + (x2 − x1 ) · f (ξ1 ) + . . . + (xn − xn−1 ) · f (ξn−1 )
que chamaremos de somas de Riemann.
Note que agora pode haver parcelas negativas nessa soma, se f < 0.

Fig.: Retângulos na parte y > 0 contribuem sua área na soma de Riemann,


enquanto os na parte y < 0 contribuem com o negativo da área

Se acontecer de f ≥ 0 então essa soma se parece muito com as


somas de áreas de retângulos sob o gráfico, que fizemos na Seção 2.
É possı́vel refinarmos as partições [a, b], colocando mais pontos xi e
escolhendo mais pontos ξi . Isso produz novas somas de Riemann, como
acima.
E podemos passar ao limite, fazendo a norma das partições tender
a zero (ou seja, o número n de pontos é feito n → +∞).
Teorema 4.1. Seja f (x) contı́nua em [a, b]. Então
• i) passando ao limite, com as normas das partições tendendo
a zero, as somas de Riemann
(x1 − x0 ) · f (ξ0 ) + (x2 − x1 ) · f (ξ1 ) + . . . + (xn − xn−1 ) · f (ξn−1 )
Rb
convergem para um número denotado a f (x) dx.

• ii) esse limite não depende do tipo particular de soma de Rie-


mann, apenas de que as normas das partiões de [a, b] tendam
a zero.
Rb
• iii) se f ≥ 0 então a f (x)dx = Af,a (b).
Rb
• iv) se f < 0 então a f (x)dx = −Af,a (b), onde esta área
Af,a (b) é compreendida entre o eixo dos x e o gráfico.
CAPÍTULO 19. INTEGRAÇÃO E O PRIMEIRO TEOREMA
FUNDAMENTAL 243

Rc
• v) c
f (x)dx = 0 para qualquer c ∈ [a, b].

• vi) se escolhemos c com a < c < b então vale


Z c Z b Z b
f (x)dx + f (x)dx = f (x)dx.
a c a
Ra Rb
• vii) b
f (x)dx = − a
f (x)dx.
Rb Rb
• viii) | a
f (x) dx | ≤ a
| f (x) | dx.

• ix) Se f, g são contı́nuas em [a, b] e c1 , c2 ∈ R, então


Z b Z b Z b
(c1 · f (x) ± c2 · g(x)) dx = c1 · f (x) dx ± c2 · g(x) dx.
a a a

Demonstração.
Me contentarei com dar algumas idéias sobre cada item. Os detalhes
se vêem em cursos de Análise Matemática.
i), ii) e iii) são técnicas, e nos dão a liberdade na escolha das
partições.
iv): óbvia se sabemos iii).
v): óbvia, pois posso pensar em no domı́nio [a′ , b′ ] := {c}.
vi): decorre da liberdade que temos nas partições de [a, b] = [a, c] ∪
[c, b].
vii): pode ser tomado como uma definição.
viii): Decorre da desigualdade triangular que:
| (x1 − x0 ) · f (ξ0 ) + (x2 − x1 ) · f (ξ1 ) + . . . + (xn − xn−1 ) · f (ξn−1 ) | ≤
≤ | (x1 − x0 ) · f (ξ0 ) | + | (x2 − x1 ) · f (ξ1 ) | + . . . + | (xn − xn−1 ) · f (ξn−1 ) | =
= (x1 − x0 ) · |f (ξ0 ) | + (x2 − x1 ) · | f (ξ1 ) | + . . . + (xn − xn−1 ) · | f (ξn−1 ) |,
e reconhecemos que esta última expressão é uma soma de Riemann
da função | f (x) |.
Logo ao passar ao limite obtemos a desigualdade entre as integrais.
ix) Decorre de
(x1 −x0 )·( c1 f (ξ0 )±c2 g(x0 ) )+. . .+(xn −xn−1 )·( c1 f (ξn−1 )±c2 g(xn−1 )) =
= c1 · [(x1 − x0 ) · f (ξ0 ) + . . . + (xn − xn−1 ) · f (ξn−1 )]±
± c2 · [(x1 − x0 ) · g(ξ0 ) + . . . + (xn − xn−1 ) · g(ξn−1 )].


Rb
Exemplo: Chamo a atenção que quando tivermos a f (x)dx = 0
isto não dirá em geral que f ≡ 0. Por exemplo se tomo [a, b] = [0, 2π]
5. FUNÇÕES COM DERIVADA, MAS SEM SEGUNDA
DERIVADA 244

e f (x) = sin(x), então o fato que veremos a seguir:


Z 2π
sin(x)dx = 0
0
significa que a área sob o gráfico do seno, de [0, π], é a mesma área da
região sobre o gráfico, de [π, 2π].

Por último, se f e g são contı́nuas e definidas em [a, b] em geral:


Z b Z b Z b
f (x) · g(x)dx 6= f (x)dx · g(x)dx,
a a a
x3
o que se vê comparando áreas Ax2 ,0 (x) = 3 com o produto de áreas
2 2
Ax,0 (x) · Ax,0 (x) = x2 · x2 .
Veremos mais tarde uma técnica para fazer as
Z b
f (x) · g(x)dx
a
chamada integração por partes.
O Teorema 3.1 que vimos acima, Rtem uma versão mais geral que
x
usa, ao invés de Af,a (x), a noção de a f (t)dt3 (chamada de integral
indefinida, por depender x).
A demonstração dessa versão não é mais difı́cil do que a que já
demos, por isso apenas enuncio:
Teorema 4.2. (Primeiro Teorema fundamental do Cálculo)
Seja f : [a, b] → R contı́nua e x ∈ [a, b]. Então
Z x
( f (t)dt )′ (x) = f (x).
a

Notação: Rx
Como esse Teorema diz que a f (t)dt é uma primitiva de f e como
duas primitivas definidas num mesmo intervalo só diferem por con-
stante C, podemos usar como sı́mbolo para a(s) primitiva(s) de f :
Z
f dx,

onde deixamos de fixar os limites de integração, o que acarreta que


deixamos de determinar o valor da constante C.

5. Funções com derivada, mas sem segunda derivada


Acostumados com os polinômios, que têm derivadas de todas as
ordens (mesmo que ≡ 0 a partir de um a certa ordem), poderı́amos
pensar que sempre que uma função tem alguma derivada tenha também
as de ordem seguinte.
3Note que usei t em f (t) dt para deixar x indicando o ponto escolhido
CAPÍTULO 19. INTEGRAÇÃO E O PRIMEIRO TEOREMA
FUNDAMENTAL 245

Isso é falso. Por exemplo, considere a função


Z x
F1 : [−1, 1] → R, F1 (x) := | t | dt.
−1
Pelo Primeiro Teorema Fundamental, F1′ (x) = | x |.
Logo F1 não terá F ′′ (0) (já que sabemos que | x | não tem derivada
em x = 0).
Agora façamos,
Z x
F2 : [−1, 1] → R, F2 (x) := F1 (t) dt.
−1
Pelo Primeiro Teorema fundamental, F2′ (x)
= F1 (x) e F2′′ (x) = | x |.
Logo F2 tem primeira e segunda derivadas em todos os pontos de seu
domı́nio, mas não terá F2′′′ (0).
E assim sucessivamente, podemos definir Fn , que vai bem até as
derivadas de ordem n, mas que não terá F (n+1) (0).
6. Exercı́cios
Exercı́cio 6.1. (resolvido)
O computador da as seguintes aproximações para:
π π
x1 := · (sin( ) + sin(π) ) = 1.570796327,
2 2
π π 2π
x2 := · (sin( ) + sin( ) + sin(π) ) = 1.813799365,
3 3 3
π π 2π 3π
x3 := · (sin( ) + sin( ) + sin( ) + sin(π) ) = 1.896118898,
4 4 4 4
π π 2π
x4 := · (sin( ) + sin( ) + . . . + sin(π) ) = 1.933765598.
5 5 5
i) qual uma possibilidade de termo geral da sequência xn da qual
exibimos os quatro primeiros termos ?
ii) Por quê os itens i) e ii) do Teorema 4.1 implicam que existe
limn→∞ xn ?
Exercı́cio 6.2. (resolvido)
i) Descreva a função F : [−1, 1] → R dada por
Z x
F (x) = | t |dt,
−1
onde | t | é o módulo.
Como é o gráfico de F (x) ?
Exercı́cio 6.3. (resolvido) Verifique que
x√ 1
F (x) = 1 − x2 + arcsin(x)
√ 2 2
é primitiva de y = 1 − x2 , para x ∈ [0, 1].
CAPı́TULO 20

Logaritmo natural e sua inversa, a exponencial

Definição 0.1. Considere a função


1
f : R>0 → R>0 , f (x) = .
x
A função de R>0 → R dada por
Z x
1
ln(x) := dx
1 x
é o logaritmo natural de x.
Observe que:
• ln(1) = 0
• se 1 < x então ln(x) = A 1 ,1 (x) > 0.
x
• se x < 1 então
Z x Z 1
1 1
dx = − dx
1 x x x
R1
e x x1 dx = A 1 ,x (1) > 0 é uma área. Logo ln(x) < 0 se
x
0 < x < 1.
• como ln′ (x) = x1 > 0, o ln(x) é uma função estritamente cres-
cente.
Pelo Primeiro Teorema Fundamental(Teorema 4.2, Capı́tulo 19)
ln(x) tem a propriedade de que ln′ (x) = x1 .
A importância dessa função na teoria do Cálculo já foi discutida,
pois sua inversa é imune às derivações.
A importância prática dos logaritmos é enorme, como veremos na
próxima Seção.
Denoto sua inversa de R → R>0 por exp(y):
exp(ln(x))) = x, ∀x ∈ R>0 .
Em particular o número exp(1) será denotado por e, ou seja ln(e) =
ln(exp(1)) = 1.
m m
1. A propriedade ln(x n ) = n
· ln(x)
A propriedade a seguir faz o logaritmo muito útil para lidar com
quantidades que crescem muito, por exemplo a massa de um animal,
que cresce na ordem de 107 de um rato até uma baleia.
Vamos provar que
247
2. A PROPRIEDADE LN(X · Y ) = LN(X) + LN(Y ) 248

Afirmação 1.1. ∀m, n ∈ N e ∀x ∈ R>0 ,


m m
ln(x n ) = · ln(x).
n
Demonstração. Considero a função diferença:
m m
φ(x) = ln(x n ) − · ln(x).
n
Gostarı́amos de provar que φ(x) ≡ 0.
Sejamos modestos, provando primeiro que φ(x) ≡ C.
Para isso derivamos φ(x), para ver se φ′ (x) ≡ 0.
Pelas regras de derivação do logarimo, da composta e da diferença
de funções:
1 m m m 1
φ′ (x) = m · ( · x n −1 ) − · =
xn n n x
≡ 0.
Ou seja, que φ(x) ≡ C. Qual o valor de C ? Se avalio φ(x) no único
ponto fácil de avaliar x = 1 obtenho:
m m
φ(1) = ln(1 n ) − ln(1) = 0,
n
logo φ ≡ 0 o que querı́amos. 

2. A propriedade ln(x · y) = ln(x) + ln(y)


Afirmação 2.1. ∀x, y > 0 vale ln(x · y) = ln(x) + ln(y).
Demonstração.
Para recairmos em uma variável fixe y e olhe a função diferença:
φ(x) := ln(x · y) − ln(x) − ln(y),
como função de x apenas.
Temos pela regra da composta e pelo Primeiro Teorema Fundamen-
tal:
1 1
φ′ (x) = ·y−
xy x
onde derivei xy como função apenas de x, para cada y fixado, obtendo
(xy)′ = x. Ora então φ′ (x) ≡ 0, portanto φ(x) ≡ C.
Qual C ? Avalio φ(1) = ln(1y) − 0 − ln(y) = 0, logo C = e φ(x) ≡ 0
como querı́amos.

CAPÍTULO 20. LOGARITMO NATURAL E SUA INVERSA, A
EXPONENCIAL 249

3. A composta ln | x | e sua derivada


Note que a composição ln( |x| ) faz sentido para todo x 6= 0. Para
melhorar a notação escrevo ln( |x| ) = ln | x |.
Esta função será muito útil na resolução de equações diferenciais,
devido a:
Afirmação 3.1.
1
( ln |x| )′ = , ∀x 6= 0.
x
Demonstração.
De fato, se x > 0 já sabemos que ln′ (x) = x1 pelo Primeiro Teorema
Fundametal do Cálculo.
Se x < 0, então |x| := −x e temos pela regra da composta
1 1
(ln(−x))′ = · (−1) = , onde − 1 = (−x)′ ,
(−x) x
como querı́amos.


0,5
x
0,5 1 1,5 2 2,5 3
0

-0,5

-1

-1,5

-2

Figura: O gráfico de y = ln(x).

0,5
x
-3 -2,5 -2 -1,5 -1 -0,5
0

-0,5

-1

-1,5

-2

Figura: O gráfico de y = ln(−x).


4. CRESCIMENTO MUITO LENTO DO LOGARITMO E MUITO
RÁPIDO DA EXPONENCIAL 250

x
-4 -2 0 2 4
0

-2

-4

-6

Figura: O gráfico de y = ln | x |.

4. Crescimento muito lento do logaritmo e muito rápido da


exponencial
A Afirmação a seguir diz que o logaritmo natural cresce, mas cresce
mais lentamente até que y = x. E que, por outro lado, a exponencial
cresce mais rápido que qualquer xn , n ∈ N:
Afirmação 4.1.
i) lim ln(x) = +∞,
x→∞
mas
ln(x)
ii) lim = 0.
x→∞ x
E para qualquer n ∈ N:
xn
iii) lim = 0.
x→∞ exp(x)
Demonstração.
De i): Por definição ln(x) para x > 1 é a área sob o gráfico de x1 ,
de x = 1 até x. Precisamos mostrar que à medida que x cresce a área
cresce ano quanto quisermos.
Dito de outro modo, precisamos mostrar que a área sob o gráfico
de x1 à direita de x = 1 é tão grande quanto quisermos, desde que
avancemos para a direita o suficiente.
Note que posso tomar os retângulos justpostos
1 1 1
[1, 2] × [0, ] ∪ [2, 3] × [0, ] ∪ . . . ∪ [n − 1, n] × [0,
2 3 n
cuja soma de áreas é
1 1 1
+ + ... + .
2 3 n
CAPÍTULO 20. LOGARITMO NATURAL E SUA INVERSA, A
EXPONENCIAL 251

Agora vamos ver que essa soma se faz tão grande quanto quisermos,
quando n cresce, o que implica que a área sob o gráfico à direita de 1
fica tão grande quanto quisermos.
De fato, denote:
1 1 1
sn := + + . . . +
2 3 n
e portanto com essa notação:
1 1 1 1 1 1 1
s2n := + ( + ) + ( + + + ) + . . . +
2 | 3 {z 4 } | 5 6 {z 7 8 }
21 parcelas 22 parcelas

1 1 1
+ ( n−1 + n−1 + ... n).
|2 + 1 2 {z + 2 2 }
2n−1 parcelas
Olhano o menor termo em cada grupo destacado acima,vemos que
1 1 1 2n−1 1
s2n ≥ + 2 · 2 + 4 · 3 + ... + n = n · .
2 2 2 2 2
n
Ora como limn→+∞ 2 = +∞ obtemos que limn→+∞ s2n = +∞ e por-
tanto limn→+∞ sn = +∞. Isso diz que 12 + 13 + . . . + n1 fica tão grande
quanto eu quiser, se n crescer o suficiente.

De ii):
Só com a definição de ln(x) é imediato que ∀x > 1: ln(x) < x − 1,
pois x − 1 é a área do retângulo de altura 1 e base [1, x].
E como x − 1 < x concluo:
0 < ln(x) < x, ∀x ≥ 1.
Por outro lado é claro que
1
x > 1 ⇔ x2 > 1
(passe da esquerda para a direita tirando a raı́z quadrada, e da dirita
para a esquerda elevando ao quadrado).
Ou seja:
1 1
0 < ln(x 2 ) < x 2 , se x > 1,
e pela propriedade do logaritmo:
1 1
0 < ln(x) < x 2 , se x > 1.
2
Agora eleve tudo ao quadrado obtendo:
(ln(x))2
0< < x, se x > 1
4
e daı́
ln(x) 4
0< < , se x > 1.
x ln(x)
4. CRESCIMENTO MUITO LENTO DO LOGARITMO E MUITO
RÁPIDO DA EXPONENCIAL 252

Como sabemos que


4
lim =0
ln(x)
x→+∞

fazendo x → +∞ na desigualdade obtemos:


ln(x)
0 = lim .
x→∞ x
De iii):
Agora vamos ver que do ponto de vista de sua inversa temos o efeito
contrário, ou seja, que a exponencial cresce mais rápido que qualquer
polinômio.
Como observamos acima, ln(x) < x − 1, se x > 1. Um tal x > 1 se
escreve como x = 1 + x com x > 0. Ou seja, obtenho:
ln(1 + x) < (1 + x) − 1 = x, se x > 0.
Agora que já sei isso volto à notação anterior, escrevendo:
ln(1 + x) < x, se x > 0.
x
Já que isso vale ∀x > 0 uso para n+1 > 0 obtendo:
x x
ln(1 + )< , se x > 0.
n+1 n+1
Agora tomo exponencial, obtendo:
x x
1+ < exp( )
n+1 n+1
e portanto:
x x
< exp( ).
n+1 n+1
Elevo tudo à n + 1:
x n+1 x n+1
( ) < (exp( )
n+1 n+1
x x
e usando a propriedade da exponencial expm ( m ) = exp(m m ) = exp(x)
obtemos
xn+1
< exp(x), ∀x > 0
(n + 1)n+1
e portanto
x
xn · < exp(x), ∀x > 0
(n + 1)n+1
e finalmente:
xn (n + 1)n+1
< , ∀x > 0.
exp(x) x
Mas n é fixado e x cresce, logo:
xn
lim = 0,
x→+∞ exp(x)

como querı́amos.
CAPÍTULO 20. LOGARITMO NATURAL E SUA INVERSA, A
EXPONENCIAL 253

5. Uma função extremamente achatada


As funções y = f (x) = xn com n ∈ N se anulam em x = 0 e tem
até a derivada de ordem n − 1 nula em x = 0: f (0) = f ′ (0) = . . . =
f ( (n − 1))(0) = 0. Quando n ∈ N cresce cada vez mais o gráfico dessas
funções se achata cada vez mais em torno ao x = 0:

0,8

0,6

0,4

0,2

0
-1 -0,5 0 0,5 1
x

Figura: Os gráficos de y = x2 (vermelho), y = x4 (verde)


e y = x6 (amarelo) para x ∈ [−1, 1].

Seria possı́vel uma função (diferente da função nula, obviamente)


que tenha derivadas de todas as ordens nulas em x = 0 ? Será que
se todas as (infinitas !) derivadas são nulas em x = 0 mesmo assim a
função consegue decolar ?
Vamos ver que sim, usando o que aprendemos na Seção anterior.
A função que consideraremos é:
f (x) = exp(−x−2 ), se x 6= 0, e f (0) = 0.
Vou me contentar em mostrar que sua primeira e segunda derivada são
zero na origem, mas o leitor verá que o que uso para isso servirá em
todas as derivadas.
Para calcularmos sua derivada fora da origem podemos usar a regra
da derivada da composta. Mas para calcular sua derivada em x = 0
vamos precisar usar a definiçãod e derivada:
exp(−h−2 ) − 0
f ′ (0) = lim .
h→0 h
Ora isso é o mesmo que:
1
f ′ (0) = lim h
h→0 exp( h12 )
5. UMA FUNÇÃO EXTREMAMENTE ACHATADA 254
1
e mudando de notação com z = h
é o mesmo que
z
f ′ (0) = lim
z→∞ exp(z 2 )

(deverı́amos considerar separadamente o caso h ց 0 e z → +∞ e a


outra possibilidade h ր 0 e z → −∞, mas veremos que o resultado
final não se altera). Mas vimos acima que
z
lim =0
z→∞ exp(z)

e portanto, como exp(z 2 ) > exp(z) se |z| > 1, com mais razão:
z
lim =0
z→∞ exp(z 2 )
logo f ′ (0) = 0.
Agora para a segunda derivada, lembro a definição:
f ′ (h) − f ′ (0)
f ′′ (0) = lim .
h→0 h
Se h 6= 0, o valor de f ′ (h) é dado pela regra da composta:

f ′ (h) = 2 exp(−h−2 )h−3 .

Logo:
2 exp(−h−2 )h−3
f ′′ (0) = lim =
h→0 h
1
h4
=2 .
exp( h12 )
1
Agora com a notação z = h2
temos

z2
f ′′ (0) = lim ,
z→+∞ exp(z)
e já vimos que
z2
lim =0
z→+∞ exp(z)
logo
f ′′ (0) = 0.
Deixo como exercı́cio para o leitor mostrar, do mesmo jeito, que
f ′′′ (0) = 0. Etc.
O Maple dá ao seu gráfico o seguinte formato:
CAPÍTULO 20. LOGARITMO NATURAL E SUA INVERSA, A
EXPONENCIAL 255

0,35

0,3

0,25

0,2

0,15

0,1

0,05

0
-1 -0,5 0 0,5 1
x

Fig.: Como o Maple representa a função extremamente achatada, x ∈ [−1, 1].

Mas note que parece que ela é zero em todo esse intervalo. Se
diminuo o intervalo ainda assim o gráfico dado pelo programa é en-
ganador : parece que se anula ainda em todo esse intervalo.

0,016

0,012

0,008

0,004

0
-0,4 -0,2 0 0,2 0,4
x

Figura: Assim o Maple representa a função extremamente achatada...

Por isso é sempre importante a teoria junto com o uso do computa-


dor pois sabemos que a função
f (x) = exp(−x−2 ), se x 6= 0, e f (0) = 0
só se anula em x = 0 !
Para terminar, um comentário.
Em geral, dada uma função f com todas as derivadas, onde f (x) =
f (x) é derivada de ordem 0 e f (i) (x) é a de ordem i, a série:
(0)

+∞
X f (i) (0) i
x,
i=0
i!
é a chamada série de Taylor de f em x = 0 (pois os cálculos de seus
(i)
coeficientes f i!(0) foram feitos na origem x = 0).
No nosso caso como f (0) = f (i) (0) = 0, ∀i ∈ N, então a sua série de
Taylor de f em x = 0 é identicamente nula. Como cada série de Taylor
6. EXERCÍCIOS 256

converge em um intervalo (pode se degenerar a um ponto) teremos que


dizer que a série de Taylor de nossa f achatada converge em toda a
reta.
Mas no entanto essa série só coincide com o valor da f em x = 0 !

6. Exercı́cios
Exercı́cio 6.1. Derive:

i) exp(x ln(x)), ii) x2 ln(x2 ) + x, iii) ln( x2 + 1),
iv) ln(x2 + 1), v) x2 ln(x), se x > 0, vi) exp(x2 ln(x)), vii) ln(x4 ),
1
viii) ln( ), 0 < x ≤ 1, ix) ln(x6 + 4x2 ).
x
Exercı́cio 6.2. Sejam f > 0 e g > 0.
ii) Usando a propriedade
f (x)
ln( ) = ln(f (x)) − ln(g(x))
g(x)
4 2
derive ln( 3xx4 +4x
+x +1
2 +1 ).
4 +x2 +1
iii) Como seria derivar ln( 3xx4 +4x 2 +1 ) sem usar i) ?

Exercı́cio 6.3. (resolvido)


ln(1+x)
O programa Maple plota y = x
para x ∈ [−0.9, 2]:
2,5

1,5

-0,5 0 0,5 1 1,5 2


x

sem se questionar sobre o que fazer em x = 0. Explique o que está


acontecendo, com os conceitos do Cálculo. Dica: Existe:
ln(1 + x)
lim ?
x→0 x
Quanto vale? Por quê ?
Exercı́cio 6.4. (resolvido)
Vimos dois fatos importantes do Cálculo:
ln(x)
lim ln(x) = +∞ mas lim = 0.
x→+∞ x→+∞ x
Ou seja que o logaritmo natural cresce, mas cresce mais lentamente
que a própria função y = x. A Figura mostra o gráfico de y = ln(x)
x
,
para x ∈ [1, 10], onde se ve que há um ponto de máximo, depois dele a
função y = ln(x)
x
vai caindo para cada vez mais próximo do zero.
CAPÍTULO 20. LOGARITMO NATURAL E SUA INVERSA, A
EXPONENCIAL 257
lnx
Determine o ponto de máximo de y = x
.

0,35

0,3

0,25

0,2

0,15

0,1

0,05

0
2 4 6 8 10
x

Exercı́cio 6.5. Vimos que que:


xn
lim exp(x) = +∞ e ainda lim = 0, ∀n ∈ N.
x→+∞ x→+∞ exp(x)

Ou seja, que a exponencial cresce e cresce mais rapidamente que qual-


quer polinômio xn .
xn
A Figura mostra o gráfico de y = exp(x) , para n = 2, 3 e para
x ∈ [0, 4], onde se vê que que cada um deles tem um ponto de máximo,
depois dele a função vai caindo ficando cada vez mais próxima de zero.
xn
Determine o ponto de máximo de cada y = exp(x) , onde n ∈ N .

1,2

0,8

0,6

0,4

0,2

0
0 1 2 3 4
x

Exercı́cio 6.6. (resolvido)


Neste exercı́cio se trata de encontrar primitivas sem ajuda de técnica
nenhuma.
Tenha em mente que a primitiva de um produto não é o produto de
primitivas. Quando aparecer um produto f ·g, lembre que a derivada da
composta faz aparecer produtos ! Por exemplo (sin(x2 ))′ = cos(x2 )·2x.

sin(x) cos(x)
i) , ii) x sin(x2 ) cos(x2 ),
6
2x + cos(x)
iii) 2 , se x2 + sin(x) ≥ 1,
x + sin(x)
1+x m
iv) , se x > 0, v) x n , m, n ∈ N, vi)2x cos(x2 ),
x
x
vii) cos(x2 ), viii) x exp(x2 ), ix) exp(x) cos(exp(x)),
2
x)f (x) = a0 xn + a1 xn−1 + . . . + an , ai ∈ R,
4x3 + 4x x19 exp(x20 )
xi) , xii) ,
x4 + 2x2 + 1 20
6. EXERCÍCIOS 258

exp( x1 )
xiii) , xiv) sin(x) sin(cos(x)),
x2
6x5 + 4x x19 exp(x20 )
xv) expn (x), n ∈ N xvi) , xvii)
x6 + 2x2 + 1 20
7
xviii) , xix) cos(x) cos(sin(x)).
x7
Exercı́cio 6.7. Apenas usando a definição dada em aula de lnar-
itmo natural, mostre que 0.5 < ln(2) < 2
Apenas com essa definição de e mostre que e < 4 (dica: compare
com áreas de retângulos adequados)
Exercı́cio 6.8. Prove que ∀k número Inteiro: ln(xk ) = k ln(x),
∀x ≥ 1. Em aula já fizemos para k nos Naturais (veremos depois que
vale ∀x > 0).
Exercı́cio 6.9. Prove que ∀x, y ≥ 1 vale ln(x · y) = ln(x) + ln(y).
(depois veremos que o mesmo vale ∀x, y > 0).
Para isso: i) fixe y e deixe livre apenas a variável x. Prove usando
o Cálculo que ln(x · y) − (ln(x) + ln(y)) ≡ C, ii) depois verifique que
C = 0.
Exercı́cio 6.10. Com os itens 7) e 8) prove que:
x
ln( ) = ln(x) − ln(y),
y
x
onde x ≥ 1, y ≥ 1, y ≥ 1 (veremos depois que vale para x, y > 0.
Exercı́cio 6.11. 11) defina o lnaritmo em base a por lna (x) :=
ln(x)
ln(a)
. Verifique que
i) lna (x) está bem definido se a, b, x ≥ 1 (mais tarde vermos que
basta ser a, b, x > 0) e que lne (x) = ln(x), ii) lna (a) = 1, iii) lnb (x) =
lna (x) · ln(a)
ln(b)
. iv) calcule a derivada de f (x) = lna (x), v) usando essa
derivada e as idéias do Cálculo, mostre que
lna (x · y) = lna (x) + lna (y), x, y ≥ 1
e que lna (xn ) = n lna (x).
Exercı́cio 6.12. Como definir uma função inversa de lna (x) ? Tem
que ser alguma g(y) tal que g(lna (x)) = x, ou seja g( ln(x)
ln(a)
) = x e
1
portanto g tem que eliminar o ln(a) e depois o efeito de exp. que tal
g(y) = exp(ln(a)y).
Teste-a e chame-a de ay = g(y).
Exercı́cio 6.13. Seja exp(y) a inversa de y = ln(x).
i) O objetivo é provar que ∀y1 , y2 na imagem de ln vale
exp(y1 + y2 ) = exp(y1 ) · exp(y2 ).
CAPÍTULO 20. LOGARITMO NATURAL E SUA INVERSA, A
EXPONENCIAL 259

(depois veremos que vale ∀y1 , y2 ) Para isso note que:


y1 + y2
pode se escrever como
ln(exp(y1 )) + ln(exp(y2 ))
ou como
ln(exp(y1 + y2 )).
Agora use o item 8) para concluir que
ln(exp(y1 )) + ln(exp(y2 )) = ln(exp(y1 ) · exp(y2 )).
Ou seja, juntando tudo:
ln(exp(y1 + y2 )) = ln(exp(y1 ) · exp(y2 )).
Afirmo que a demonstração de que
exp(y1 + y2 ) = exp(y1 ) · exp(y2 )
já acabou. Por quê ?
ii) aplique i) ao caso particular exp(n y), n ∈ N.
1
iii) prove que exp(−x) = exp(x) e que limx→+∞ exp(−x) = 0.
Exercı́cio 6.14. Derive:
i) exp(x2 ),
ii) exp(cos(x)),
iii) exp(cos6 (x)),
exp( 1 )
iv) x x , se x > 0,
v) exp(tan(x)),
vi) exp(exp(exp(x))).
Exercı́cio 6.15. Vamos definir as seguintes funções
exp(x) − exp(−x) exp(x) + exp(−x)
f1 (x) := e f2 :=
2 2
Prove que vale:
f1 (x)2 − f2 (x)2 ≡ 1, ∀x
de dois modos:
i) só fazendo contas que usam potências e produtos de exponenciais.
ii) usando a filosofia do Cálculo, ou seja, de derivar uma função, ver
que a derivada é zero, lno constante.
Exercı́cio 6.16. Seja um k > 0. Prove a equivalência:
lim exp( kx ) = +∞ ⇔ lim exp( −kx ) = 0.
x→+∞ x→+∞

2) Os gráficos a seguir são de funções f (x) = f (0) exp(−x), para


diferentes valores de f (0).
i) Confira que esses gráficos nunca se intersectam, mesmo quando
x fica muito grande.
6. EXERCÍCIOS 260

ii) mostre que em todos esses gráficos as inclinações tendem a zero


quando x cresce.
iii) Calcule em cada x qual é quociente das inclinações de dois desses
gráficos.

2,5

1,5

0,5

0
0 1 2 3 4
x

Exercı́cio 6.17. (resolvido)


A função y = f (x) = exp(−x2 ) (vermelho), sua derivada f ′ (x)
(verde) e sua segunda derivada f ′′ (x) (amarelo) são dadas na Figura a
seguir, para x ∈ [−2, 2]:

0,5
x
-2 -1 0 1 2
0

-0,5

-1

-1,5

-2

i) Calcule f ′ (x), f ′ (0), f ′′ (x) e f ′′ (0).


Note que o gráfico de f ′ (x) tem um máximo local e um mı́nimo
local (que são pontos de inflexão da f , portanto).
ii) Determine os pontos de mı́nimo/máximo locais de f ′ (x) resol-
vendo f ′′ (x) = 0.

Exercı́cio 6.18. (resolvido)


Prove que a tangente ao gráfico de y = ln(x) no ponto (e, 1) é uma
reta que passa pela origem. Dica: equação de uma reta dado um ponto
e o coeficiente angular.
Então conclua, de preferência sem fazer contas, que a tangente ao
gráfico de y = exp(x) no ponto (1, e) tamém é uma reta que passa pela
origem.
CAPÍTULO 20. LOGARITMO NATURAL E SUA INVERSA, A
EXPONENCIAL 261

1
x
0,5 1 1,5 2 2,5 3 3,5 4
0

-1

-2

-3

-4
CAPı́TULO 21

Segundo Teorema Fundamental e Áreas

1. A descoberta de Gregory e Sarasa sobre área


A propriedade ln(xy) = ln(x) + ln(y), que vimos na Seção 2 do
Capı́tulo anterior, tem uma contrapartida interessante.
Suponha x ≥ 1 e y ≥ 1. Como xy ≥ x e as áreas as áreas sob o
gráfico de x1 são aditivas, podemos escrever:
A 1 ,1 (xy) = A 1 ,1 (x) + A 1 ,x (xy).
x x x

Mas ln(xy) := A 1 ,1 (xy), ln(x) := A 1 ,1 (x) e ln(y) := A 1 ,1 (y) obtemos:


x x x

A 1 ,1 (x) + A 1 ,1 (y) = A 1 ,1 (x) + A 1 ,x (xy)


x x x x

e portanto:
A 1 ,1 (y) = A 1 ,x (xy).
x x

Por exemplo, com x = 2 e y = 2, A 1 ,1 (2) = A 1 ,2 (4) (quem consegue


x x
consegue intuir isso na Figura abaixo?)

1
0,9
0,8
0,7
0,6
0,5
0,4
0,3
1 1,5 2 2,5 3 3,5 4
x

1
Figura: As áreas sob x
entre 1 e 2 ou entre 2 e 4 são iguais !.

Como se aprende no livro C.H. Edwards, The historical development


of the Calculus, Springer, 1979 esta propriedade
A 1 ,1 (y) = A 1 ,x (xy),
x x

foi observada por Gregory St. Vincent e A.A. Sarasa, antes do Cálculo.
263
2. SEGUNDO TEOREMA FUNDAMENTAL DO CÁLCULO 264

Será que conseguimos verificar que


A 1 ,1 (y) = A 1 ,x (xy)
x x

diretamente, apenas com a definição de Área da Seção 1 do Capı́tulo


19 ?
Para definir A 1 ,1 (y) a primeira etapa é partimos o intervalo [1, y] em
x
n subintervalos de tamanho y−1 n
, e levantarmos retângulos com altura
1
f (x) = x , somando as suas Áreas. Depois a segunda etapa é passar ao
limite n → +∞.
Façamos a primeira etapa:
y−1 y − 1 −1 2(y − 1) −1 n(y − 1) −1
· [(1 + ) + (1 + ) + . . . + (1 + ) ].
n n n n
Por outro lado, a primeira etapa da definição de A 1 ,x (xy) é levantarmos
x
retângulos de base xy−x
n
e somarmos suas áreas, ou seja:
xy − x xy − x −1 2(xy − x) −1 x + n(xy − x) −1
·[(x+ ) +(x+ ) +. . .+( ) ]=
n n n n
y − 1 −1 (y − 1) −1 −1 2(y − 1) −1 n(y − 1) −1
=x ·[x (1+ ) +x (1+ ) +. . .+x−1 (1+ ) ],
n n n n
que, após cancelar x, dá o mesmo de antes ! Por isso ao passar ao
limite n → +∞ dará o mesmo e:
A 1 ,1 (y) = A 1 ,x (xy).
x x

2. Segundo Teorema Fundamental do Cálculo


Teorema 2.1. Seja f : [a, b] → R contı́nua. Então
Z b
f (x)dx = F (b) − F (a),
a

onde F (x) é qualquer função com


F ′ (x) = f (x), ∀x ∈ [a, b].
Ou seja,dito de outro modo
Z b
F ′ (x)dx = F (b) − F (a).
a

Essa função F com F ′ (x) = f (x) ∀x é chamada de primitiva da f .

Demonstração.
Tome uma F (x) com F ′ (x) = f (x) ∀x ∈ [a, b] (não importa como
se achou).
CAPÍTULO 21. SEGUNDO TEOREMA FUNDAMENTAL E
ÁREAS 265

Agora lembreR que o Primeiro Teorema Fundamental 4.2 diz que a


x
função G(x) := a f (x)dx tem
G′ (x) = f (x), ∀x ∈ [a, b].
Então
F ′ (x) = G′ (x), ∀x ∈ [a, b],
o que diz que
F (x) = G(x) + C, ∀x ∈ [a, b],
pelo Teorema Fundamental das Equações diferenciais (ver Capı́tulo 7
da Parte 1 deste Curso). em particular:
F (b) = G(b) + C.
Ra
Mas que constante C é essa ? Temos que G(a) = a
f (x)dx = 0, logo
F (a) = 0 + C,
ou seja C = −F (a) e
F (b) = G(b) − F (a)
e portanto:
Z b
G(b) := f (x)dx = F (b) − F (a),
a
como querı́amos.


Exemplo: Agora podemos justificar que


Z 2π
sin(x) dx = 0,
0
pois pelo Teroema 2.1:
Z 2π
sin(x)dx = − cos(2π) − (− cos(0)) = −1 + 1 = 0.
0

3. Regiões entre dois gráficos


Afirmação 3.1. Suponha f, g duas funções contı́nuas tais que no
intervalo [a, b] tenham:
f (x) ≥ g(x), ∀x ∈ [a, b].
Então a área da região, de x = a até x = b, abaixo do gráfico de
f (x) mas acima do gráfico de g(x) é dada por:
Z b
f (x) − g(x) dx.
a
3. REGIÕES ENTRE DOIS GRÁFICOS 266

Demonstração.
Suponhamos primeiramente o caso em que
g(x) ≥ 0, ∀x ∈ [a, b].
Então f (x) ≥ 0, ∀x ∈ [a, b], já que f (x) ≥ g(x).
Rb
Por um lado, a f (x) dx é a Área da região de x = a até x = b
abaixo do gráfico de f (x) e acima do eixo dos x, já que f (x) ≥ 0.
Rb
Enquanto que a g(x) dx é a Área da região de x = a até x = b
abaixo do gráfico de g(x) e acima do eixo dos x, já que g(x) ≥ 0.
Por uma propriedade da Integral:
Z b Z b Z b
f (x) − g(x) dx = f (x) dx − g(x) dx
a a a
Rb
e, como f (x) ≥ g(x), a f (x) − g(x) dx dá área da região de x = a até
x = b, abaixo do gráfico de f (x) mas acima do gráfico de g(x).
Agora, no caso geral, pode acontecer que g(x) < 0 para algum
ponto no intervalo [a, b].
Como g(x) é contı́nua, ela tem um valor mı́nimo global em [a, b].
Chame-o de −C < 0. Então as novas funções
f (x) := f (x) + C e g(x) := g(x) + C
têm
g(x) ≥ 0, ∀x ∈ [a, b],
(se não fosse assim para algum x ∈ [a, b] então g(x) + C < 0 e g(x) <
−C, contradizendo a escolha de −C como mı́nimo da g) e
f (x) ≥ g(x), ∀x ∈ [a, b].

0
-1 -0,5 0 0,5 1
x
-1

-2

Figura: f vermelho, g verde, f amarelo, g azul, [a, b] = [−1, 1].

Pelo que já vimos no primeiro caso da demonstração, agora aplicado


a f , g, o valor de
Z b
f (x) − g(x) dx
a
CAPÍTULO 21. SEGUNDO TEOREMA FUNDAMENTAL E
ÁREAS 267

dá a área da região de x = a até x = b, abaixo do gráfico de f (x) mas


acima do gráfico de g(x).
Como os gráficos de f (x) = f (x) + C e g(x) = g(x) + C diferem
dos de f (x) e g(x) apenas por uma translação vertical, então
Z b
f (x) − g(x) dx
a
dá a área da região de x = a até x = b, abaixo do gráfico de f (x) mas
acima do gráfico de g(x).
Finalmente: Z b
f (x) − g(x) dx =
a
Z b
(f (x) + C) − (g(x) + C) dx =
a
Z b
= f (x) − g(x) dx, ,
a
o que conclui a demonstração.


4. Exercı́cios
Exercı́cio 4.1. Usando o Segundo Teorema Fundamental do Cáculo
1
determine a área compreendida entre os gráficos de y = x3 e de y = x 3 .

1,5

0,5

0
0 0,2 0,4 0,6 0,8 1 1,2
x

Obs. Nesse tipo de questão é preciso verificar onde os gráficos se


intersectam e qual gráfico está por cima do outro.
Exercı́cio 4.2. (resolvido) √
É um fato que para b = −2+3 22 ∼ 0, 9 vale:
Z b
x − x2 − x3 dx = 0.
0
Interprete isso geometricamente, como sendo equivalente a uma igual-
dade entre duas Áreas de duas regiões comprendidas entre gráficos de
certas funções. √
Dica: podes ser útil saber que 5 ∼ 2.2.
Exercı́cio 4.3. Através do Teorema Fundamental, determine a
área da região compreendida entre os gráficos de y = x2 e y = −x2 + 8.
4. EXERCÍCIOS 268

Exercı́cio 4.4. Encontre a reta y = a · x adequada para que a


área compreendida entre seu gráfico e o de y = x2 seja exatamente 1.
Dica: vá té o fim sem determinar o a, ao final, peça que a área seja 1
e obtenha assim o a.

0
0 0,5 1 1,5 2
x

Exercı́cio 4.5. (resolvido)


Determine o valor adequado de a para que a área da região com-
prendida entre os gráficos de y = x4 e y = a seja exatamente A = 1.

1,5

0,5

0
-1 -0,5 0 0,5 1
x

Exercı́cio 4.6. A figura a seguir mostra os gráficos de y = xn ,


para n = 1, 2, 3, 4, 5, 6, na região x ∈ [0, 1].
i) na região x ∈ [0, 1] o gráfico de y = xn está por cima ou por baixo
do de y = xn+1 ?
ii) Determine para qual n a região compreendida entre os gráficos
1
de y = xn e y = xn+1 tem área exatamente igual a 12 .
1

0,8

0,6

0,4

0,2

0
0 0,2 0,4 0,6 0,8 1
x

Exercı́cio 4.7. A figura a seguir mostra os gráficos de y = xn −


xn+1 , para n = 1, 2, 3, 4, x ∈ [0, 1]. Determine para qual n a região sob
1
o gráfico de y = xn − xn+1 tem área 20 .
0,25

0,2

0,15

0,1

0,05

0
0 0,2 0,4 0,6 0,8 1
x
CAPÍTULO 21. SEGUNDO TEOREMA FUNDAMENTAL E
ÁREAS 269

Exercı́cio 4.8. A figura a seguir mostra os gráficos de y = fn (x) :=


xn − x2n , para n = 1, 2, 3, 4, no domı́nio x ∈ [0, 1] (que se parecem com
chicotes):
0,25

0,2

0,15

0,1

0,05

0
0 0,2 0,4 0,6 0,8 1
x

i) Calcule fn′ (x), ∀n ∈ N .


ii) Determine a equação y = ax + b da reta tangente ao gráfico de
fn (x) no ponto (1, 0).
iii) Explique o que acontece com os coeficientes angularres das retas
de ii), quando n cresce.
iv) Se vê que cada y = fn (x) tem um ponto de máximo em seu
domı́nio [0, 1]. Determine-o (claro dependendo de n).
v) todas as fn valem o mesmo nos seus pontos de máximo, quanto
?
vi) Determine a área An da região sob o gráfico de y = fn (x) =
xn − x2n , de x = 0 até x = 1.
vii) A quanto tendem essas áreas quando n aumenta? Ou seja, qual
o
lim An ?
n→+∞

Exercı́cio 4.9. A figura a seguir mostra os gráficos de y = fn (x) :=


x − x2n+1 , para n = 3, 6, 10, 50, x ∈ [0, 1]:

0,8

0,6

0,4

0,2

0
0 0,2 0,4 0,6 0,8 1
x

i) Calcule fn′ (x), ∀n ∈ N .


ii) Determine as equações y = ax + b das retas tangentes ao gráfico
de fn (x) no ponto (0, 0), ∀n.
iii) Determine as equações y = ax + b das retas tangentes ao gráfico
de fn (x) no ponto (1, 0), ∀n.
iv) O que acontece com as retas dos itens ii) e iii), quando n → +∞
?
v) Se vê que cada y = fn (x) tem um ponto de máximo em [0, 1].
Determine-o (dependendo de n).
4. EXERCÍCIOS 270

vi) Determine a área An da região sob o gráfico de y = fn (x) =


x − x2n+1 , de x = 0 até x = 1.
vii) O que acontece com An quando n → +∞, ou seja, existe o
limn→+∞ An ? Se existe quanto é ?
CAPı́TULO 22

Técnicas de Integração

1. Integração por partes


Vamos explicar agora uma técnica útil para encontrrar primitivas
de funções e expressá-las concretamente como funções.
Lembro primeiro que criamos uma função completamente nova ao
fazermos Z x
1
ln(x) := dx.
1 x
UmaRpergunta natural é: R x será criamos algo radicalmente novo se faze-
x
mos a ln(x)dx ou essa a ln(x)dx se pode expressar através de funções
conhecidas ?
Veremos que sim, se pode expressar através de funções conhecidas,
de fato:
Z x
ln(x) dx = x ln(x) − x + C.
a
Verificamos facilmente que (x ln(x) − x + C)′ = ln(x).
Mas como chegamos numa primitiva dessas? Há alguma técnica ?
O Teorema a seguir dá uma técnica útil, embora à primeira vista não
pareça, para encontrar primitivas:
Teorema 1.1. Sejam f e g definidas num intervalo, com f ′ e g ′
funções contı́nuas.
Rx Rx Rx
Então a f ′ (x) · g(x)dx = a f (x) · g(x)dx − a f (x) · g ′ (x)dx.
Demonstraç
R ão.x
Note que ( a (f (x) · g(x))′ dx)′ (x) = (f (x) · g(x))′ (x) pelo Primeeiro
Teorema RFundamental do Cálculo.
x
Logo a (f (x) · g(x))′ dx = f (x) · g(x) + C pelo Teorema Fundamnal
da Equações Diferenciais.
Mas pela derivado do produto:
(f (x) · g(x))′ = f ′ (x) · g(x) + f (x) · g ′ (x).
Logo pelas propriedades aditivas da integral:
Z x Z x

(f (x) · g(x)) dx = (f ′ (x) · g(x) + f (x) · g ′ (x))dx =
a a
Z x Z x

= f (x) · g(x)dx + f (x) · g ′ (x)dx
a a
271
1. INTEGRAÇÃO POR PARTES 272

e portanto:
Z x Z x

f (x) · g(x)dx = f (x) · g(x) − f (x) · g ′ (x)dx + C
a a
como querı́amos 

Vamos aplicá-lo nos exemplos a seguir, onde se vê que


• cuidado ao escolher quem fará o papel de f ′ e quem será g
• pode ser preciso usá-lo mais de uma vez
Rb
Exemplo 1.1. i) a ln(x) dx:
Z Z
1
1 ln(x) dx = x ln(x) − x dx =
| {z } | {z } x
|{z}
f ′g fg
f g′
= x ln(x) − x + C.
Rb
ii) a
x ln(x) dx:
Z Z 2
x2 x 1
x ln(x) dx = ln(x) − dx =
| {z } |2 {z } 2 x
|{z}
f g

fg f g′
2 2
x x
= ln(x) − + C.
Rb 2 4
ln(x)
iii) a x
dx:
Z Z
1 1
ln(x) dx = ln(x) ln(x) − ln(x) dx.
|x {z } | {z }
fg
| {z x}
f ′g f g′
Logo: Z b
ln(x)
2· dx = ln2 (x) + C
a x
ou seja Z b
ln(x) ln2 (x)
· dx = + C.
a x 2
Rb ln(x)
iv) a x2
dx:
Z Z
1 −1 −1 1
2
ln(x) dx = ln(x) − dx =
|x {z } |x {z } |x{zx}
f ′g fg f g′
Z
− ln(x) 1
= + dx =
x x2
− ln(x) 1
= − + C.
Rb x x
v) a
cos2 (x) dx:
CAPÍTULO 22. TÉCNICAS DE INTEGRAÇÃO 273

Z Z
cos(x) cos(x) dx = sin(x) cos(x) − sin(x)(− sin(x)) dx =
| {z } | {z } | {z }
f ′g fg f g′
Z
sin(x) cos(x) + sin2 (x)dx =
Z
= sin(x) cos(x) + (1 − cos2 (x))dx =
Z
= sin(x) cos(x) + x + C − cos2 (x)dx.
Logo Z
2· cos2 (x)dx = sin(x) cos(x) + x + C
e portanto: Z
sin(x) cos(x) + x
cos2 (x)dx = + C.
2
Rb
vi) a
cos3 (x) dx:
Z Z
2 2
cos(x) cos (x) dx = sin(x) cos (x) − sin(x)(−2 cos(x) sin(x)) dx =
| {z } | {z } | {z }
f ′g fg f g′
Z
= sin(x) cos2 (x) + 2 sin2 (x) cos(x)dx =
Z
= sin(x) cos (x) + 2 (1 − cos2 (x) cos(x)dx =
2

Z Z
= sin(x) cos (x) + 2 cos(x)dx − 2 cos3 (x)dx.
2

Logo
Z Z
3· cos (x)dx = sin(x) cos (x)+2 cos(x)dx = sin(x) cos2 (x)+2 sin(x)+C,
3 2

e portanto:
Z
sin(x) cos2 (x) + 2 sin(x)
· cos3 (x)dx = + C.
3
Rb
vii) a
x2 cos(bx) dx:
Z Z
2 sin(bx) 2 sin(bx)
cos(bx)x dx = x − 2x dx =
| {z } | b{z } | b{z }
f ′g
fg f g′
Z
sin(bx) 2 2
= x − sin(bx)x =
b b
Z
sin(bx) 2 2
x − sin(bx)x F ′ Gdx =
b b | {z }
2. INTEGRAÇÃO POR SUBSTITUIÇÃO 274
Z
sin(bx) 2 2 cos(bx)
= x − [− x− sin(bx)1 dx] =
b b | {z
b } | {z }

sin(bx) 2 2 2
= x + 2 cos(bx)x − 2 cos(bx) + C.
b b b
Rb
viii) a exp(ax) cos(bx) dx:
Z Z
sin(bx) sin(bx)
cos(bx) exp(ax) dx = exp(ax) − a exp(ax) dx =
| {z } | b {z } | b {z }
f ′g
fg f g′
Z
sin(bx) a
= exp(ax) − sin(bx) exp(ax) dx =
b b | {z }
F ′G
Z
sin(bx) a − cos(bx) − cos(bx)
= exp(ax) − [ exp(ax) − a exp(ax) .
b b | b {z } | b {z }
FG F G′
Logo
Z
sin(bx) exp(ax) a
2 · cos(bx) exp(ax)dx = + 2 cos(bx) exp(ax) + C
b b
e
Z
1 sin(bx) exp(ax) a
cos(bx) exp(ax)dx = ( + 2 cos(bx) exp(ax)) + C.
2 b b

2. Integração por substituição


Suponha uma f : J → R contı́nua e uma g : I → J contı́nua
também. A variável do domı́nio de f será u, f = f (u), e no domı́nio
de g será x, g = g(x).
Como g(I) ⊂ J, então u = g(x) e faz sentido a composição de
funções f (g(x)).
Note que em geral:
Z b Z g(b)
f (g(x)) dx 6= f (u) du.
a g(a)

Por exemplo, se f (u) = u e u = g(x) = x2 então:


Z b Z b2
b 3 − a3 2 b 4 − a4
= x dx 6= u du =
3 a a2 2
O que precisamos para corrigir esse erro é dado pelo seguinte Teo-
rema:
Teorema 2.1. Seja f : J → R contı́nua e g : I → J derivável,
u = g(x) com g ′ (x) contı́nua. Então:
• faz sentido a composição f (g(x)),
CAPÍTULO 22. TÉCNICAS DE INTEGRAÇÃO 275

• f (g(x))g ′ (x) é integrável e de fato


Z b Z g(b)

f (g(x)) g (x) dx = f (u) du.
a g(a)

Supondo por um momento esse resultado, corrigimos o erro anterior:


Z b Z b2
b 4 − a4 2 b 4 − a4
2( )= x 2x dx = u du = .
4 a a2 2
O Teorema 2.1
Z b Z g(b)

f (g(x)) g (x) dx = f (u) |{z}
du .
a | {z } g(a)

sugere uma notação:


du = g ′ (x) dx,
que sugere por sua vez, para u = g(x), a notação:
du
= g ′ (x).
dx
O lado esquerdo dudx
é o modo como Leibniz se referia à derivada
de u = g(x), que na notação do Newton é g ′ (x). Ou seja, a última
expressão que escrevemos corresponde a dois modos de se escrever a
mesma coisa.

Demonstração. (do Teorema 2.1)


Note que pelo Segundo Teorema do Cálculo:
Z g(b)
f (u)du = F (g(b)) − F (g(a)),
g(a)

onde F (u) é uma primitiva de f (u). Mas por outro lado, pela regra da
composta:
(F (g(x)))′ = F ′ (g(x))g ′ (x) = f (g(x))g ′ (x)
ou seja que F (g(x)) é primitiva da função:
f (g(x))g ′ (x).
Portanto se aplico o Segundo Teorema para calcular
Z b
f (g(x))g ′ (x)dx
a
tenho Z b
f (g(x))g ′ (x)du = F (g(b)) − F (g(a)).
a
Logo
Z g(b) Z b
f (u)du = f (g(x))g ′ (x)dx.
g(a) a
2. INTEGRAÇÃO POR SUBSTITUIÇÃO 276

2 ln(x)
Exemplo 2.1. Vamos provar aqui que a área sob o gráfico de x
,
de x = 1 até x = e := exp(1) vale exatamente 1.
Ou seja, que Z e
2 ln(x)
dx = 1.
1 x
Faço u = ln(x), du = x1 dx e acerto os liitesd e integração:
Z e Z 1
2 ln(x) u2 u2
dx = 2 u du = 2 [ (1) − (0)] = 1.
1 x 0 2 2

Vamos ver como a linguagem da Integração por Substituição se


aplicaria pra encontrar algumas primitivas.
Exemplo 2.2. Por exemplo, para começar, primitivas de
sin(x) cos(x).
Deixando de lado os limites de integração estamos deixando livre a
escolha da constante C. Portanto com:
u = sin(x), du = cos(x)dx
temos pelo Teorema 2.1:
Z Z
sin(x) cos(x) dx = u du =

u2
= +C =
2
sin2 (x)
= + C.
2
Se quisermos destacar os limites de integração então faremos:
Z b Z sin(b)
sin(x) cos(x) dx = u du =
a sin(a)

sin2 (b) sin2 (a)


= − .
2 2
Exemplo 2.3. Agora primitivas de
sinn (x) cos(x), n ∈ N.
Sem nos fixarmos em limites de integração. com:
u = sin(x), du = cos(x)dx
temos pelo Teorema 2.1:
Z Z
sin (x) cos(x) dx = un du =
n
CAPÍTULO 22. TÉCNICAS DE INTEGRAÇÃO 277

un+1
= +C =
n+1
sinn+1 (x)
= + C.
n+1
Se atentamos aos limites de integração:
Z b Z sin(b)
n
sin (x) cos(x) dx = un du =
a sin(a)

sinn+1 (b) sinn+1 (a)


= − .
n+1 n+1
Exemplo 2.4. Agora quero as primitivas de
4x3 + 4x
.
x4 + 2x2 + 1
Para isso faço
u = x4 + 2x2 + 1, du = (4x3 + 4x) dx
e portanto pelo Teorema 2.1:
Z Z
4x3 + 4x 1
4 2
dx = du =
x + 2x + 1 u
= ln(u) + C =
= ln(x4 + 2x2 + 1) + C.
Exemplo 2.5.
Z

x3 · x − 5 dx, x − 5 > 0.

Faço
u = x − 5, du = dx
3 3
e escrevo x = (u + 5) . Daı́:
Z Z
√ 1
3
x · x − 5 dx = (u + 5)3 u 2 du =
Z
1
= (u3 + 15u2 + 75u + 125)u 2 du =
7 5 3 1
= u 2 + 15u 2 + 75u 2 + 125u 2 du =
2 9 30 7 5 250 3
= u 2 + u 2 + 30u 2 + u2 + C =
9 7 3
2 9 30 7 5 250 3
= (x − 5) 2 + (x − 5) 2 + 30(x − 5) 2 + (x − 5) 2 + C.
9 7 3
3. SUBSTITUIÇÕES TRIGONOMÉTRICAS, ÁREAS DO
CÍRCULO E ELIPSE 278

Exemplo 2.6.
Z
1
√ √ dx, x > 0.
x exp( x)
Faço
√ 1
u= x, du = √ ,
2 x
logo Z Z
1
√ √ dx = exp(−u) 2 du =
x exp( x)
1
= 2 (− exp(−u)) + C = −2 √ + C.
exp( x)

3. Substituições trigonométricas, áreas do Cı́rculo e Elipse


Até aqui usamos as substituições u = g(x) e du = g ′ (x) dx para
simplificar a expressão que estamos integrando. A seguir usamos o
Teorema 2.1 de um jeito diferente, que parece complicar o integrando:
mas no final tudo acaba bem !
Por ter sido demonstrado há tanto tempo por Arquimedes que a
área do cı́rculo de raio r é πr2 , acabamos por trivializar esse fato
notável.
Vejamos o que dá se tento calcular a área do Cı́rculo usando inte-
grais/primitivas.
Vamos fazer o seguinte, vamos calcular primeiro a área de um quarto
de Cı́rculo de raio r, aquele que fica no primero quadrante e multiplicar
depois o resultado por 4.
A área do √Cı́rculo no primeiro quadrante é a área sob o gráfico de
y = f (x) = + r2 − x2 , para x ∈ [0, r]. Quero calcular portanto:
Z r√
r2 − x2 dx.
0

Faço a substituição:
x = r sin(θ).
Pelo Teorema 2.1 acima tenho que calcular:
Z π q Z r=r sin( π ) √
2 2
2
r2 − r2 sin (θ) · r cos(θ) dθ = r2 − x2 dx.
0 0=r sin(0)
π
Ora como na região 0 ≤ θ ≤ temos cos(θ) ≥ 0 posso dizer que:
2
q
cos(θ) = 1 − sin2 (θ)
então escrevo:
Z π q Z π q
2 2
2 2
2 2
r − r sin (θ) · r cos(θ) dθ = r 1 − sin2 (θ) · cos(θ) dθ =
0 0
CAPÍTULO 22. TÉCNICAS DE INTEGRAÇÃO 279
Z π
2
2
=r cos2 (θ) dθ.
0
Já fizemos na Seção 1 a integral:
Z
cos2 (θ) dθ

e obtivemos como primitiva1 de cos2 (θ):


sin(θ) cos(θ) + θ
.
2
Logo o Segundo Teorema do Cálculo dá:
Z π
2 sin(θ) cos(θ) + θ π sin(θ) cos(θ) + θ
cos2 (θ) dθ = ( )( ) − ( )(0) =
0 2 2 2
π
= .
4
Logo a área do setor no primeiro quadrante é π4 r2 e a área do cı́rculo é
πr2 .

Note que
x
x = r sin(θ) ou seja θ = arcsin( )
r
dão que
sin(θ) cos(θ) + θ 1 x x x
= · [ · cos(arcsin( )) + arcsin( )] =
2 2 r r r
r
1 x x2 x
= · [ · 1 − 2 + arcsin( )] =
2 r r r
1 x √ x
= · [ 2 · r2 − x2 + arcsin( )].
2 r r
Ou seja,
Z √
r2 x √ 2 x
r2 − x2 dx = · [ 2 · r − x2 + arcsin( )] + C
2 r r
ou finalmente
Z √ √
1 x
r2 − x2 dx = · [x · r2 − x2 + r2 arcsin( )] + C
2 r
(ufa, que primitivas !).
1Outra opção para continuar seria usar a fórmula trigonométrica: cos2 (θ) =
1+cos(2θ)
2 e depois uma primitiva de 1+cos(2θ)
2 , que é naturalmente
θ sin(2θ) sin(θ) cos(θ) + θ
+ = .
2 4 2
3. SUBSTITUIÇÕES TRIGONOMÉTRICAS, ÁREAS DO
CÍRCULO E ELIPSE 280

Exemplo 3.1. É claro que podemos inverter a questão e, supondo


que sabemos a área de cı́rculos, usar isso para calcular integrais.
Por exemplo, para r > 0 e r2 − x4 > 0, vamos provar que
Z √r √
8
π = 2· r2 − x4 · x dx.
r 0

De fato fazendo u = x2 , du = 2x dx e acertando os limites de integração


temos:
Z √r √ Z r√
2 4
du
r − x · x dx = r 2 − u2 =
0 0 2
1 1
· · πr2 ,
=
2 4
Rr√
pois 0 r2 − u2 du é área de 41 de Cı́rculo de raio r.
Agora mostro que uma pequena adaptação do que fizemos para
calcular a área do cı́rculo nos dá a área de elipses.
2 2
Considere a elipse xa2 + yb2 = 1.
Vamos primeiro considerar 41 de sua área, que é a área sob o gráfico
q
2
de y = b2 (1 − xa2 ), com x ∈ [0, a].
Então quero calcular:
Z ar
x2
b2 (1 − 2 ) dx
0 a
e o farei com a substituição:
x = a sin(u), dx = a cos(u) du,
que nos dá:
Z ar Z πq
x2 2
2
b (1 − 2 ) dx = b2 (1 − sin2 (u))a cos(u) du =
0 a 0
Z π
2
= ab cos2 (u) du.
0
Mas pelo que já vimos acima:
Z π
2 π
cos2 (u) du =
0 4
e portanto
Z r
a
x2 π
b2 (1 − 2
) dx = ab .
0 a 4
2 2
Logo a área toda da elipse xa2 + yb2 = 1 é πab.
Quando b = a temos um cı́rculo x2 + y 2 = a2 , cuja área é πa2 .
CAPÍTULO 22. TÉCNICAS DE INTEGRAÇÃO 281

4. Outras substituições trigonométricas


As integrais do tipo Z
1
√ dx
1 + x2
podem ser feitas com a substiuição:
x = tan(u), dx = sec2 (u) du
√ p
pois 1 + x2 = sec2 (u) = sec(u) e
Z Z
1 1
√ dx = sec2 (u) du =
1+x 2 sec(u)
Z
= sec(u) du.
Só que agora somos obrigados a saber fazer esta última integral.
Para isso vamos fazer uns pequenos malabarismos2:
Z Z
1
sec(u) du := du =
cos(u)
Z
1 + sin(u)
= du =
cos(u) (1 + sin(u))
Z
sin2 (u) + cos2 (u) + sin(u)
= du =
cos(u)(1 + sin(u))
Z
cos(u) sin(u)
= + du =
1 + sin(u) cos(u)
Z Z
cos(u) − sin(u)
= du − du ==
1 + sin(u) cos(u)
= ln | 1 + sin(u) | − ln | cos(u) | + C =
1 + sin(u)
= ln | |+C =
cos(u)
=: ln | sec(u) + tan(u) | + C.
Finalmente então podemos completar a integração anterior:
Z √
1
√ dx = ln | sec(u) + tan(u) | + C = ln( x2 + 1 + x) + C.
1 + x2

Já as integrais do tipo


Z
1
√ dx
x2 −1
2Adaptando esses passos se prova também que
Z
csc(u) du = − ln | csc(u) + cot(u)| + C
R
6. (AX 2 + BX + C)−1 DX 282

podem ser feitas com a substiuição:


x = sec(u), dx = sec(u) tan(u) du
√ p
pois x2 − 1 = tan2 (u) = tan(u) e
Z Z
1 1
√ dx = sec(u) tan(u) du =
2
x −1 tan(u)
Z
= sec(u) du =

= ln | sec(u) + tan(u)| + C

= ln |x + tan( x2 − 1)| + C.

5. Integração de algumas funções racionais


A questão em geral é como integrar
Z
p(x)
dx
q(x)
onde p(x), q(x) são polinômios.
A técnica geral para integrar funções racionais (quocientes de polinômios)
é conhecida como integração por frações simples.
Procederemos por etapas, começando com casos simples, sem pre-
tensão de esgotar o assunto.
R
6. (ax2 + bx + c)−1 dx
Começo explicando o que fazer, quando queremos calcular, com
0 6= a, b, c ∈ R: Z
1
2
dx.
ax + bx + c
Há três casos a considerar, dependendo do discriminante b2 −4ac =
0:
• i) b2 − 4ac = 0, ou seja, ax2 + bx + c = (x − x)2 tem uma raı́z
real dupla,

• ii) b2 − 4ac > 0, ou seja, ax2 + bx + c = (x − x1 ) · (x − x2 ) tem


duas raı́zes reais diferentes ou

• iii) b2 −4ac < 0, ou seja, ax2 +bx+c tem duas raı́zes complexas
conjugadas (não tem raı́zes Reais).

No caso i):
Faço u = x − x, du = dx e
Z Z
1 1
2
dx = dx =
ax + bx + c (x − x)2
CAPÍTULO 22. TÉCNICAS DE INTEGRAÇÃO 283
Z
1 −1 1
= 2
du = +C = + C.
u u x−x
No caso ii):
Gostaria de escrever, para A e B números bem escolhidos:
1 1 A B
dx = = + ,
ax2 + bx + c (x − x1 ) · (x − x2 ) x − x1 x − x2
pois então terı́amos:
Z Z Z
1 A B
dx = dx + dx =
(x − x1 ) · (x − x2 ) x − x1 x − x2
Z Z
1 1
=A· du + B · dv,
u v
onde u = x − x1 e v = x − x2 e daqui chegaamos em:
1
= A · ln |x − x1 | + B · ln |x − x2 | + C.
(x − x1 ) · (x − x2 )
Como encontrar A e B como queremos ? Queremos que valha:
1 A B
= + ,
(x − x1 ) · (x − x2 ) x − x1 x − x2
ou seja, somando as frações à direita:
1 (A + B)x − Ax2 − Bx1
= .
(x − x1 ) · (x − x2 ) (x − x1 ) · (x − x2 )

Para que (A + B)x − Ax2 − Bx1 = 1 precisamos ter B = −A e −Ax2 +


Ax1 = 1, ou seja as escolhas de A e B são:
1 −1
A= e B= .
x1 − x2 x 1 − x2

Em suma, no caso ii) (x1 , x2 raı́zes Reais distintas):


Z
1 1 1
2
dx = · ln |x − x1 | − · ln |x − x2 | + C.
ax + bx + c x1 − x2 x 1 − x2

No caso iii):
Primeiro faço, já que a 6= 0:

Z Z Z
1 1 1 1
dx = b c
dx = · dx.
2
ax + bx + c a(x2 + a x + a ) a x2 + ab x + c
a
R
6. (AX 2 + BX + C)−1 DX 284

Agora escrevo3:
b c b c b2
x2 + x + = (x + )2 + − 2 =
a a 2a a 4a
b 4ac − b2
= (x + )2 + .
2a 4a2
Como estamos no caso iii) sabemos que
4ac − b2 > 0
e por isso posso escrever:
b
b 2 4ac − b2 4ac − b2 x + 2a
(x + ) + 2
= 2
· [ ( √ )2 + 1. ]
2a 4a 4a ( 4ac−b 2
) 2a
Até aqui temos, no caso iii), apenas que:
Z Z
1 1 4a2 1
dx = · · dx.
ax2 + bx + c a 4ac − b2 x+ b
[( √ 2a
4ac−b2
)2 + 1 ]
( 2a
)

Chegou a hora de fazer uma substituição:


b
x+ 2a 1
u= √
2
, du = √ dx
4ac−b2
( 4ac−b
2a
) ( 2a
)
e obter
Z Z
1 2a 1
dx = √ · du.
x+ b 4ac − b2 u2 +1
[( √ 2a
4ac−b2
)2 + 1 ]
( 2a
)

Isso diz que terminamos pois sabemos calcular:


Z
1
2
du = arctan(u) + C.
u +1
Logo o resultado final é, no caso iii) (4ac − b2 > 0):
Z b
1 8a2 x + 2a
dx = 3 · arctan(
√ ) + C.
ax2 + bx + c (4ac − b2 ) 2
2
( 4ac−b ) 2a

3Se continuamos um pouquinho obteremos a fórmula de Báskara: já que a 6= 0,


b c b 4ac − b2
x2 + x + = (x + )2 + .
a a 2a 4a2
De onde,
b 2 b2 − 4ac
(x + ) = 2
,
2a √ 4a
b ± b2 − 4ac
x+ =± ,
2a 2a
e finalmente: √
−b ± b2 − 4ac
x= .
2a
CAPÍTULO 22. TÉCNICAS DE INTEGRAÇÃO 285
R
7. (Ax3 + Bx2 + Cx + D)−1 dx
Agora quero tratar do quê fazer para calcularmos:
Z
1
dx, A 6= 0.
Ax3 + Bx2 + Cx + D
Vimos como aplicação do T.V.I. que sempre um polinômio de grau
ı́mpar com coeficientes Reais tem ao menos uma raı́z Real x = x1 .
Há 4 caso possı́veis:
• i) Ax3 + Bx2 + Cx + D tem uma raı́z Real tripla,
• ii) Ax3 + Bx2 + Cx + D tem uma raı́z dupla e uma simples
(Reais),
• iii) Ax3 +Bx2 +Cx+D tem três raı́zes Reais distintas, x1 , x2 , x3 ∈
R.
• iv) Ax3 + Bx2 + Cx + D tem apenas uma raı́z simples Real e
duas raı́zes complexas (conjugadas).
São representados na figura a seguir:

1
x
-1 -0,5 0 0,5 1
0

-1

-2

-3

-4

Figura: Casos i) em vermelho, ii) em verde, iii) em amarelo e iv) em azul.

Qual o análogo do discriminante b2 − 4ac de ax2 + bx + c no caso


de Ax3 + Bx2 + Cx + D ? Isso veremos em detalhe no Capı́tulo 30.

Suponhamos que conhecemos as raı́zes Reais do Ax3 +Bx2 +Cx+D


(se não forem fáceis de achar em exemplos concretos, pode-se apelar
para a fórmula de Cardano - ver Capı́tulo 30, onde começei a dedução
da fórmula).
Então no caso i), já sabemos o que fazer:

Z Z
1 1 −1
dx = dx = +C
Ax3 + Bx2 + Cx + D (x − x1 )3 (x − x1 )2
R
7. (AX 3 + BX 2 + CX + D)−1 DX 286

No caso ii):
Z
1 1
dx = dx
Ax3 + Bx2 + Cx + D (x − x1 )2· (x − x2 )
vamos ser otimistas e tentar escrever, para ci constantes bem escolhidas:
1 c1 c2 c3
2
= + 2
+
(x − x1 ) · (x − x2 ) (x − x1 ) (x − x1 ) (x − x2 )
pois então obterı́amos:
Z
1 −1
2
dx = c1 ·ln |x−x1 |+c2 · +c3 ·ln |x−x2 |+C.
(x − x1 ) (x − x2 ) x − x1
Para encontrarmos ci adequadas, façamos primeiro a soma de frações
‘a direita:
c1 c2 c3
+ + =
(x − x1 ) (x − x1 )2 (x − x2 )
c1 (x − x1 )(x − x2 ) + c2 (x − x2 ) + c3 (x − x1 )2
= =
(x − x1 )2 (x − x2 )
(c1 + c3 )x2 + (c2 − c1 (x1 + x2 ) − 2c3 x1 )x + (c1 x1 x2 − c2 x2 + c3 x21 )
= .
(x − x1 )2 (x − x2 )
Como o numerador dessa última expressão tem que igual ao numerador
de (x−x )12 (x−x ) otemos um sistema de três equações:
1 2

c1 + c3 = 0, c2 − c1 (x1 + x2 ) − 2c3 x1 = 0
c1 x1 x2 − c2 x2 + c3 x21 = 1.
As duas primeiras equações dão:
c3 = −c1 , c2 = c1 (x2 − x1 ),
que quando substituidas na terceira equação dão:
1 −1
c1 = 2 2
= ,
2x1 x2 − x1 − x2 x1 − x2 )2
ou seja encontramos assim c1 , c2 , c3 , se conhecemos as raı́zes Reais x1 6=
x2 .
No caso iii) gostarı́amos de escrever :
1 c1 c2 c3
= + +
(x − x1 )(x − x2 )(x − x3 ) x − x1 x − x1 x − x 3
pois então integrarı́amos usando ln | |.
Somamos
c1 c2 c3
+ + =
x − x1 x − x 1 x − x3
(c1 + c2 + c3 ) x2 − (c1 (x2 + x3 ) + c2 (x1 + x3 ) + c3 (x1 + x2 )) x
= +
(x − x1 )(x − x2 )(x − x3 )
c 1 x x + c 2 x1 x 3 + c 3 x1 x2
+ 2 3
(x − x1 )(x − x2 )(x − x3 )
CAPÍTULO 22. TÉCNICAS DE INTEGRAÇÃO 287

e igualo seu numerador a 1, obtendo um sistema de três equações:


c1 + c2 + c3 = 0, c1 (x2 + x3 ) + c2 (x1 + x3 ) + c3 (x1 + x2 ) = 0,
c1 x2 x3 + c2 x1 x3 + c3 x1 x2 = 1.
Da primeira posso pôr c3 em função dos outros, da segunda posso por
c2 em função de c1
c1 (x3 − x1 )
c3 = −(c1 + c2 ), c2 = − ,
(x3 − x2 )
e substituindo na terceira determinamos o c1 .

Caso iv):
Aqui temos Ax3 + Bx2 + Cx + D = (x − x1 ) · (ax2 + bx + c) onde
2
ax + bx + c tem raı́zes complexas conjugadas. Se conhecemos x1 ,
também conhecemos a, b, c por divisão de polinômios. Portanto con-
sidero conhecidos esses coeficentes a, b, c.
Seremos otimistas tentando escrever, para c1 , c2 , c3 adequados:
1 c1 c2 x + c3
2
= + 2 .
(x − x1 ) · (ax + bx + c) x − x1 ax + bx + c
Note que:
1 c1 c2
2
6= + 2 , ∀c1 , c2
(x − x1 ) · (ax + bx + c) x − x1 ax + bx + c
pois se por absurdo fazemos:
1 c1 c2
= + =
(x − x1 )(ax2 + bx + c) x − x1 ax2 + bx + c
ac1 x2 + (bc1 + c2 )x + (c1 c − c2 x1 )
=
(x − x1 )(ax2 + bx + c)
poduzimos equações:
ac1 = 0, quadbc1 + c2 .
Como a 6= 0 neste caso, então c1 = 0 e daı́ obtemos c2 = 0, absurdo.
Mas por outro lado sim é possı́vel:
1 c1 c2 x + c3
2
= + 2 ,
(x − x1 ) · (ax + bx + c) x − x1 ax + bx + c
pois
c1 c2 x + c3 (ac1 + c2 )x2 + (bc1 − c2 x1 + c3 )x + (c1 c − c3 x1 )
+ 2 = ,
x − x1 ax + bx + c (x − x1 )(ax2 + bx + c)
produz as equações:
ac1 + c2 = 0, bc1 − c2 x1 + c3 = 0,
c1 c − c3 x1 = 1.
8. EXERCÍCIOS 288

A primeira me permite escrever c2 = −ac1 e a segunda dá


c3 = −bc1 + x1 c2 = −bc1 − x1 ac1 .
Ou seja c3 é função de c1 . Substituido c3 na terceira equação
c1 c − c3 x1 = 1,
esta vira uma equação de grau um em c1 e descobrimos
R c2 x+c3 o valor de c1 .
Achados os c1 , c2 , c3 precisamos integrar ax2 +bx+c dx para termos
então
Z finalmente: Z
1 c2 x + c3
3 2
dx = c1 · ln |x − x1 | + dx.
Ax + Bx + Cx + D ax2 + bx + c
Já que c2 6= 0 e a 6= 0, escrevo:
c2 x + c3 c2 2ax + b c3 − c2a
2b

= · ( ) + .
ax2 + bx + c 2a ax2 + bx + c ax2 + bx + c
Ora Z
2ax + b
dx
ax2 + bx + c
se faz com a substituição:
u = ax2 + bx + c, du = (2ax + b) dx
virando Z Z
2ax + b 1
dx = du = ln |u| + C.
ax2 + bx + c u
Por outro lado, falta fazer
Z
1
2
dx
ax + bx + c
e isso faz como no item iii) da Seção 6.
8. Exercı́cios
Exercı́cio 8.1.
i) verifique que se x ∈ [0, π2 ] então
x ≥ x sin(x) ≥ 0.
ii) Usando integração por partes e o segundo teorema fundamental,
calcule a área da região compreendida entre os gráficos de y = x e de
y = x sin(x) de x = 0 até x = π2 , mostrada na figura a seguir:

1,6

1,2

0,8

0,4

0
0 0,2 0,4 0,6 0,8 1 1,2 1,4
x
CAPÍTULO 22. TÉCNICAS DE INTEGRAÇÃO 289

Exercı́cio 8.2. Faça por substituição as integrais seguintes:


Z Z
1
i) tan(x) dx = − · (− sin(x)) dx,
cos(x)
Z Z
1
ii) cot(x) dx = · cos(x) dx,
sin(x)
Z Z Z
1 sin(x) −1
iii) sec(x) tan(x) dx := dx = ·(− sin(x)) dx
cos(x) cos(x) cos2 (x)
Z Z
1 1 1
iv) , dx = · dx.
ln(x) x ln(x) x
Dica: O lado direito das igualdades dá uma pista das substituições
u = g(x) e du = g ′ (x)dx adequadas para simplificar a expressão a ser
integrada)
CAPı́TULO 23

A curvatura dos gráficos

1. O comprimento de um gráfico
Considere o gráfico de uma função f : [a, b] → R. Gostarı́amos
nesta Seção de definir e calcular o comprimento desse gráfico.
Considere uma partição
a = t0 < t1 < . . . < tn = b
do domı́nio [a, b] e considere o comprimento da poligonal inscrita no
gráfico de f formada de n segmentos:
p p
pn := (t1 − t0 )2 + (f (t1 ) − f (t0 ))2 +. . .+ (tn − tn−1 )2 + (f (tn ) − f (tn−1 ))2 .
Ou seja,
s s
f (t1 ) − f (t0 ) 2 f (tn ) − f (tn−1 ) 2
pn = 1+( ) ·(t1 −t0 )+. . .+ 1 + ( ) ·(tn −tn−1 ).
t1 − t0 tn − tn−1
Se usamos em cada sub-intervalo [ti−1 , ti ] da partição o Teorema do
Valor Médio de Lagrange, então:
f (ti ) − f (ti−1 )
= f ′ (ξi ), ξi ∈ (ti−1 , ti ).
ti − ti−1
Então
p p
pn = 1 + (f ′ (ξ1 ))2 · (t1 − t0 ) + . . . + 1 + (f ′ (ξn ))2 · (tn − tn−1 ).
Refinando a partição esperamos estar inscrevendo uma poligonal
cujo tamanho cada vez mais aproxima o tamanho do gráfico de f .
A passagem ao limite n → +∞, com a norma da partição de [a, b]
tendendo a zero, sugere que definamos
Definição 1.1. Suponha um gráfico de f : [a, b] → R, com f
derivável e f ′ (x) uma função contı́nua.
O comprimento do gráfico de (a, f (a)) até (b, f (b)) será definido
pela integral Z bp
1 + f ′ (x)2 dx.
a

A primeira coisa que vemos nessa Definição 1.1 é que provavelmente


em muitos casos não será fácil calcular esse comprimento, pois dará uma
integral complicada. Mas como f ′ (x) é contı́nua se vê que de qualquer
forma existe essa integral.
291
1. O COMPRIMENTO DE UM GRÁFICO 292

Exemplos:
• no caso y = f (x) = A · x + B uma reta, nossa definição é
apenas o conteúdo do teorema de Pitágoras:
Z bp √
1 + f ′ (x)2 dx = 1 + A2 · (b − a) =
a
p p
= (b − a)2 + (A(b − a))2 = (b − a)2 + (Ab + B − Aa − B))2 .
• no caso y = x2 já não é tão evidente quanto mede seu gráfico:
Z bp Z b√
′ 2
1 + f (x) dx = 1 + 4x2 dx.
a a
Faço:
u = 2x, e du = 2dx
e Z Z 2b √
b √ 1
1 + 4x2 dx = · 1 + u2 du.
a 2 2a

Uma primitiva de 1 + u2 é
u√ 1 √
1 + u2 + ln(u + 1 + u2 ).
2 2
Logo:
Z b√
1 2b √ 1 √
1 + 4x2 dx = · [ · 1 + 4b2 + ln(2b + 1 + 4b2 )−
a 2 2 2
2a √ 1 √
− · 1 + 4a2 − ln(2a + 1 + 4a2 )].
2 2
Para a = 0, b = 1 isso dá:
1 √ 1 √
· [ 5 + ln(2 + 5)] ∼ 1.478942857
2 2

• como o segmento de reta de (0, 0) a (1, 1) mede 2 ∼ 1.414213562,
e
3
x2 < x 2 < x, se x ∈ [0, 1],
3
é natural que o comprimento do gráfico de y = x 2 de x = 0
até x = 1 seja um valor entre 1.414213562 e 1.478942857.
De fato,
Z bp Z 1r
3 1
1 + f ′ (x)2 dx = 1 + ( x 2 )2 dx =
a 0 2
Z 1 r
9
= 1 + x dx =
0 4
Z 13 3
4 4 √ 4 2 13 2
= · u du = · · [( ) − 1] ∼
9 1 9 3 4
∼ 1.439709873
CAPÍTULO 23. A CURVATURA DOS GRÁFICOS 293

Será muito útil mais adiante trabalharmos também com curvas


parametrizadas, ou seja, com aplicações
Γ : R → R2 , (x(t), y(t)), t ∈ [a, b]
que supomos sempre deriváveis (tanto x(t) quanto y(y)).
O traço de uma curva parametrizada Γ é o conjunto imagem Γ([a, b]).
Observo que nem sempre Γ([a, b]) é gráfico de alguma função; por
exemplo, Γ([0, 2π]) é um cı́rculo inteiro, quando tomamos
Γ : R → R2 , (cos(t), sin(t)), t ∈ [0, 2π]

2. Velocidade de um gráfico ou de uma curva


Como pelo Primeiro Teorema do Cálculo:
p Z tp
′ 2
1 + (f (x)) = ( 1 + f ′ (x)2 dt )′ (x)
a

é natural denotarmos
ds p
= 1 + (f ′ (x))2 .
dx
Essa grandeza será chamada velocidade do gráfico no instante x.
Note que sempre
ds
>0
dx
o que diz o comprimento do gráfico sempre é uma função estritamente
crescente. E ademais, isso diz que existe uma função inversa: x = x(s).
Logo dado um comprimento desde f (a) = A determino univocamente
x e daı́ um único ponto no gráfico. Portanto existe uma função bem
definida P = P (s) que descreve os pontos do gráfico.
Para curvas parametrizadas
Γ : R → R2 , (x(t), y(t)), t ∈ [a, b]
podemos definir seu comprimento por:
Z bp
s := (x′ (x)2 + (y ′ (x))2 dx.
a

Se chamamos o vetor (x′ (t), y ′ (t)) de vetor tangente a Γ e o deno-


tamos por Γ′ (t) então
Z b
s= || Γ′ (t) || dt.
a

Também é natural considerar:


p
f racd sd t = || Γ′ (t) || = (x′ (x)2 + (y ′ (x))2 .
3. DEFINIÇÃO DE CURVATURA E SUA FÓRMULA 294

3. Definição de curvatura e sua fórmula


A noção intuitiva de curvatura é a de uma medida de quanto mudam
as direções das retas tangentes (em relação a algum eixo fixado como
referência).
Mas, para que a curvatura de um gráfico G seja um conceito geométrico,
vamos defini-la como uma medida de quanto mudam as direções das
tangentes num trecho de um gráfico em relação a quanto vale o com-
primento da porção do gráfico.
Como critério de adequação de um possı́vel definição exigiremos
que um cı́rculo Cr de raio r tenha curvatura constante e de fato κ = 1r
(para que os cı́rculo muito grandes se curvem muito pouco).
Essa exigência é natural, pois quando percorremos todo o cı́rculo,
percorremos s = 2πr e o ângulo θ formado pelas retas tangentes variou
2π. Logo
∆θ 1
κ(Cr ) := = .
∆s r
Para motivarmos a Definição e Fórmula 3 abaixo, considero θ =
θ(s) uma função que mede como varia o ângulo formado pelas direções
tangentes em relação ao comprimento do gráfico percorrido.
Então a regra da derivada da composta diz1:
d tan(θ(s)) d tan(θ(s)) d θ(s)
= · =
ds dθ ds
d θ(s)
= sec2 (θ(s)) · .
ds
Por outro lado,
dy
(x(s)) = tan(θ(s))
dx
e a regra da composta dá:
d tan(θ(s)) d d y (x(s)) d x
= dx · (s) =
ds dx ds
d2 y dx
= 2
(x(s)) · (s).
dx ds
A taxa de variação que queremos para definir curvatura é
d θ(s)
.
ds
Até agora temos:
d2 y dx
d θ(s) 2 (x(s)) · d s (s)
= dx .
ds sec2 (θ(s))
Mas definimos na Seção 1 anterior:
Z xr
dy
s(x) := 1 + ( )2 dt,
a dx
1A notação de Leibniz deixa mas claro em relação a que variável derivamos
CAPÍTULO 23. A CURVATURA DOS GRÁFICOS 295

ou seja, pelo Primeiro Teorema do Cálculo:


s
ds dy 2
(x) = 1 + ( ) .
dx dx
Pela derivada da função inversa teremos:
dx 1
(s) = q .
ds dy 2
1 + (dx)
E também podemos escrever:
r
dy 2
sec(θ(s)) = 1+( ).
dx
Logo obtivemos:
d y 2
d θ(s) 2 (x(s))
= dx d y 3 .
ds (1 + ( d x )2 ) 2
Essa é a justificação da seguinte definição:
Definição 3.1. A curvatura2 do gráfico de y = f (x) é:
2
| ddx2y |
κ(x) := 3 .
(1 + ( dd xy )2 ) 2
A Figura a seguir dá um exemplo de como varia a curvatura:

0
-2 -1 0 1 2
x

Figura: Em vermelho y = x2 e em verde sua função curvatura.

Observação 3.1. Note que acima obtivemos:


dx
= cos(θ(s)).
ds
Como
dy
(x(s)) = tan(θ(s))
dx
2por enquanto não nos interessa ter sinais, por isso tomamos o módulo
4. QUAL A CURVATURA DE UMA QUINA ? 296

então a regra da composta dá:


dy dy dx
= ·
ds dx ds
ou seja:
dy
= sin(θ(s)).
ds
Novamente, no caso de uma curva parametrizada, podemos estender
a Definição 3.1 para:
Definição 3.2. Se
Γ : R → R2 , (x(t), y(t)), t ∈ [a, b]
é uma curva parametrizada então sua curvatura é dada por:
| x′ (t)y ′′ (t) − x′′ (t)y ′ (t) |
κ(t) := 3 .
(x′ (t)2 + y ′ (t)2 ) 2

Note que esta Definição 3.2 é realmente é uma estensão da Definição


3.1, pois quando t = x, temos x′ (x) ≡ 1 e x′′ (x) ≡ 0.

4. Qual a curvatura de uma quina ?


A curvatura de uma reta certamente é zero, já que a segunda
derivada é zero. Mas numa linha quebrada, formada de pedaços de
retas, que curvatura faria sentido associar à um ponto que é uma quina
??
Após a Afirmação seguinte daremos uma resposta:
Afirmação 4.1. Considere um braço de hipérbole:
ǫ
y = fǫ (x) = , ∀x > 0,
x
onde ǫ > 0 é fixado. Então:
3
i) sua função curvatura é κ(x) = 42ǫ·x2 3 .
(x +ǫ ) 2
ii) limx→+∞ κ(x) = 0 e limxց0 κ(x) = 0. √
iii) o ponto de máximo de κ(x) é em x = ǫ. Nele a curvatura é:

2
√ .
2 ǫ

iv) limǫց0 κ( ǫ) = +∞.
Demonstração.
A função curvatura é para x > 0:

x3 2ǫ · x3
κ(x) = 2 3
= 3 .
(1 + xǫ 4 ) 2 (x4 + ǫ2 ) 2
CAPÍTULO 23. A CURVATURA DOS GRÁFICOS 297

Portanto:
2ǫ · x3 x3
lim 3 = lim =0
x→+∞ (x4 + ǫ2 ) 2 x→+∞ x6
1
e, já que limxց0 3 = ǫ13 > 0, então claramente
(x4 +ǫ2 ) 2

2ǫ · x3
lim 3 = 0,
xց0 (x4 + ǫ2 ) 2
Para buscarmos mı́nimo de κ(x) a derivamos:
−6 ǫ · x2 · (x4 − ǫ2 )
κ′ (x) = ,
(x4 + ǫ2 )5/2
e vemos que:

κ′ (x) > 0 se 0 < x < ǫ,

κ′ (x) = 0 se x = ǫ,

κ′ (x) < 0 se ǫ<x

o que diz nitidamente que x = ǫ é o ponto de máximo de k(x). Que
nele vale: √
√ 2
κ( ǫ) = √ .
2 ǫ


A Figura a seguir dá o gráfico da curvatura para ǫ = 1:

2,5

1,5

0,5

0
0,5 1 1,5 2 2,5 3 3,5 4
x

1 √1
Figura: O gráfico de y = x
(vermelho), sua κ(x) (verde) e o valor y = 2
em azul

Quando ǫ → 0 o ponto x = ǫ tende a x = 0, assim como todo o
gráfico de y = fǫ (x) = xǫ tende à união de retas x · y = 0, pois:
y·x=ǫ
ao longo do gráfico de y = fǫ (x).
E pelo item iv) da Afirmação 4.1:
4. QUAL A CURVATURA DE UMA QUINA ? 298


lim κ( ǫ) = +∞
ǫց0
Assim se fôssemos atribuir um valor de curvatura a (0, 0) como
ponto da união de retas
y·x=0
deverı́amos pôr: κ = +∞.
CAPı́TULO 24

Rudimentos de Equações diferenciais e Aplicações

1. A exponencial e as equações diferenciais


A função y = f (x) = exp(x) já nasceu com a propriedade de satis-
fazer a equação:
f ′ (x) = f (x), ∀x ∈ R.
Vamos ver agora algumas pequenas modificações da exponenciale e que
tipo de equações satisfazem:
Afirmação 1.1. Seja y = f (x) derivável e suponha que para k ∈ R
tenhamos
f ′ (x) = k · f (x), ∀x ∈ R.
• Dado o valor f (0), então:
f (x) = f (0) · exp(k x), ∀x ∈ R.
• Mais em geral, dado f (x) para algum x, então:
f (x) = f (x) · exp(k (x − x)), ∀x ∈ R.
A Figura a seguir ilustra as soluções de f ′ (x) = −2 f (x) para quatro
diferentes valores iniciais f (0): 0.5, 1, 2, 3.

2,5

1,5

0,5

0
0 0,5 1 1,5 2 2,5 3
x

Demonstração.
Vamos provar diretamente o caso geral, onde nos damos o valor
f (x).
299
2. A DEFINIÇÃO ORIGINAL DE NAPIER PARA O
LOGARITMO 300

Se k = 0 então a hipótese vira f ′ (x) ≡ 0. Já sabemos que nesse


caso f (x) ≡ C e portanto f (x) = f (x). Ou seja,
f (x) = f (x) · 1 = f (x) · exp(0),
como querı́amos.
Logo podemos supôr que k 6= 0.
Considero a função g(x) := exp(k (x − x)).
Note que g(x) = exp(k (x − x)) > 0 para todo x ∈ R.
Verifico pela regra da derivada da composta que:
g ′ (x) = k · exp(k (x − x)) = k g(x), ∀x ∈ R.
Se tomo qualquer outra função f satisfazendo f ′ (x) = k · f (x), faço
o quociente
f
g
e derivo pela regra da derivada do quociente:
f f ′g − f g′
( )′ (x) = =
g g2
(kf )g − f (kg)
= ≡ 0,
g2
o que nos faz concluir que fg ≡ C. Ou seja, f (x) = C · g(x).
Para descobrir C avalio tudo em x:
f (x) = C · g(x) =
= C · exp(k 0) = C.
Portanto f (x) = f (x) · exp(k (x − x)) como querı́amos.


2. A definição original de Napier para o logaritmo


A obra do escocês John Napier (1550-1617) é o começo da longa
história do conceito de logaritmo.
Seguindo a exposição de C.H. Edwards (op.cit), podemos entender
a definição original de logaritmo de Napier do ponto de vista do Cálculo,
e qual a relação com o ln(x).
Esse anacronismo serve para entender o que fez Napier, mas lembre
que, historicamente, Napier trabalhou só com sua definição e conseguiu
fazer tabelas imensas de logaritmos !
A definição de Napier envolve dois pontos se movendo:
• N um segmento [P0 , O] de comprimento P0 O = 107 , determi-
namos a posição x(t) de um ponto P (t) que se move de P0 até
O através da distância P (t) O:
x(t) = P (t) O.
CAPÍTULO 24. RUDIMENTOS DE EQUAÇÕES DIFERENCIAIS
E APLICAÇÕES 301

• supomos que que a velocidade x′ (t) de P (t) satisfaz ∀t


x′ (t) = −x(t).
• ou seja, a velocidade inicial de P (t) é x′ (0) = 107 = x(0), mas
a velocidade vai caindo e quando P (t) está chegando no ponto
O ele está parando, pois x′ (t) = −x(t) ≈ 0.
• Com esse mesmo parâmetro de tempo t, num segundo seg-
mento de origem Q0 , se move um um ponto Q(t), se afastando
de Q0 e a posição de Q(t) é Q(t) = 107 t (ou seja, Q(t) tem
velocidade constante 107 ).
• Napier define o tamanho Q0 Q(t) como sendo o logaritmo de
x(t) := P (t) O.
• Chamemos o logaritmo definido assim por Napier de Nog(x).
Vamos traduzir isso na linguagem do Cálculo e obter:
Afirmação 2.1.
7
i) Nog(x) = 107 ln( 10x ).
ii) Nog(x1 x2 ) = Nog(x1 ) + Nog(x2 ) − 107 ln(107 ).
Demonstração.
De i):
A solução de x′ (t) = −x(t) é x = x(0) exp(−t) pela Afirmação 1.1,
ou seja,
x = 107 exp(−t).
Tomando logaritmo natural:
ln(x) = ln(107 ) + ln(exp(−t))
logo
ln(x) − ln(107 ) = −t
e
107
t = ln( )
x
logo
107
Nog(x) := 107 t = 107 · ln( ).
x
De ii)

107
Nog(x1 x2 ) = 107 · ln( )=
x1 x2
= 107 (ln(107 ) − ln(x1 x2 )) =
= 107 ln(107 ) − 107 ln(x1 ) − 107 ln(x2 ) =
1 1
= 107 ln(107 ) + 107 ln( ) + 107 ln( ) =
x1 x2
3. DECAIMENTO RADIOATIVO E A DATAÇÃO DE FÓSSEIS,
ROCHAS, OSSOS 302

1 1
= 107 ln(107 ) −2 · 107 ln(107 ) + 2 · 107 ln(107 ) +107 ln( )+107 ln( ) =
| {z } x1 x2
0
1 1
= −107 ln(107 ) + 107 ln(107 ) + 107 ln( ) + 107 ln(107 ) + 107 ln( ) =
x1 x2
107 107
= −107 ln(107 ) + 107 ln( ) + 107 ln( )=
x1 x2
= −107 ln(107 ) + Nog(x1 ) + Nog(x2 ).


3. Decaimento radioativo e a datação de fósseis, rochas,


ossos
Algumas substâncias quı́micas tem estrutura nucleares diferentes
mas compostam-se do ponto de vista quı́mico do mesmo jeito. São os
chamados isótopos diferentes da mesma substância.
Uma das mais importantes, por estar na base das moléculas orgânicas,
é o Carbono. O isótopo chamado Carbono 14 é radioativo enquanto o
isótopo mais comum, o Carbono 12 não é radioativo.
A radioatividade surge coma desintegração do núcleo e portanto
as substâncias radioativas são instáveis, se degradam com o passar do
tempo. Por isso se fala em decaimento da substância, a quantidade
tende a zero com o tempo.
Por exemplo, quando um organismo morre, deixa de assimilar Car-
bono à sua estrutura (madeira, ossos, etc) e a proporção entre o Car-
bono 14 e o Carbono 12 (de um para um trilhão quando vivo) começa
a mudar, já que o Carbono radioativo se decompõe.
Se considero a função y = f (x) para descrever a quantidade de
uma substância radioativa no tempo x, começando num tempo que
fixo como x = 0, então
• f é uma função decrescente,
• f ′ (x) é sempre negativa
• f (x) tende a zero
Mais precisamente, a quantidade y = f (x) de cada substância
quı́mica radioativa satisfaz uma equação:
f ′ (x) = −kf (x), k > 0,
onde x ∈ R é o tempo e o valor de k > 0 depende especialmente de
cada substância.

Já sabemos pela Afirmação 1.1 que

f (x) = f (0) exp(−k x), ∀R


CAPÍTULO 24. RUDIMENTOS DE EQUAÇÕES DIFERENCIAIS
E APLICAÇÕES 303

e também pelo que sabemos sobre a exponencial:


lim exp(−kx) = 0, k > 0.
x→+∞

Para o Carbono 14, k ≈ 3.8394 × 10−12 m/s (unidades de massa


por segundo).
Ora, isso dá um decaimento em unidade de massa por ano próximo
de:
−12
|3.8394{z· 10 } ·60 ·60 ·24 ·365 ≈ 0.0001210793184.
m/segundo
| {z }
m/minuto
| {z }
m/hora
| {z }
m/dia
| {z }
m/ano
Define-se meia-vida como o tempo τ no qual a quantidade inicial
f (0) de uma substancia radioativa se reduz à metade, ou seja:
f (0)
f (τ ) := .
2
Mas também temos:
f (0)
= f (0) · exp(−kτ ),
2
e daı́:
1
= exp(−kτ ).
2
E tomando logaritmo:
1
ln( ) = −kτ.
2
1
Como − ln( 2 ) = ln(2), obtemos:
ln(2)
τ= .
k
No caso do Carbono 14 temos:
ln(2)
τ= ≈ 5724.736394
0.0001210793184
(e textos de fı́sica certamente o leitor encontrará aproximações mais
corretas dessa meia-vida)
Uma meia-vida relativamente curta (na escala geológica !) como a
do Carbono 14 serve para datar madeira ou a historia da humanidade
(na arqueologia).
Para rochas/fósseis é preciso substâncias com meia-vida maiores.
Por exemplo, a lava das erupções se esfria, crsitalizando-se, formando
rochas cujo surgimento pode ser datado. Isso porque ocorre o decai-
mento do potássio 40 (radioativo) em argônio 40 (estável), que é uma
gás mas que fica retido na lava transformada em cristal. A meia vida
4. OBJETOS EM QUEDA-LIVRE 304

do potássio 40 é 1, 3 bilhão de anos e portanto rochas muito antigas


podem ser datadas1
Por coincidência, vendo um documentário sobre a Evolução aprendi
o seguinte: foram encontrados restos de um hominı́dio que fora um dos
primeiros a andar em duas patas, e que se conjecturava ter em torno
de 4 milhões de anos, quase um milhão a mais que a famosa Lucy. Mas
sua idade certamente não seria datável via Carbono 14.
Vieram então geólogos e determinaram que os restos de ossos es-
tavam localizados entre duas camadas distintas de sedimentos de erupçoes
vulcânicas.
Pelo método potássio/argônio as duas camadas de sedimentos vulcânicos
forma datadas em torno de 4 milhões de anos. Logo esses ossos tinham
essa idade !

4. Objetos em queda-livre
Vamos aplicar alguns conceitos que aprendemos para entender o
que acontece quando um corpo de massa m cai (desde um altura ra-
zoavelmente baixa).
Sejam y = f (x) a posição do corpo no instante x, que supomos
aumenta2 à medida que o corpo se aproxima da superfı́cie da Terra e
f ′ (x) sua velocidade.
Segundo Newton a aceleração f ′′ (x) de um corpo é dada por
F
f ′′ (x) = ,
m
onde F é a força resultante sobre o corpo que cai e m sua massa (em
geral F é uma grandeza vetorial, mas nesta situação particular podemos
pensá-la como escalar).
Agora vamos postular que a Força resultante F tem duas origens:
uma dependendo apenas da atração gravitacional e outra dependendo
da resistência que surge quando o objeto que se desloca atinge uma
velocidade alta.
• Ao nı́vel do mar, para quedas de não muito alto, a aceleração g
impressa pela gravidade é da ordem de 9.8 m/s
s
. Galileu já tinha
estomativas dessa aceleração e foi o primeiro a notar que essa
aceleração não depende da massa do corpo (desprezando-se o
atrito).
• Já o atrito e a resistência do ar contam no segundo tipo de
força, do tipo
−γ · f ′ (x),
1Aprendi isso no livro de Richard Dawkins, A grande história da evolução- Na
trilha de nossos ancestrais, Companhia das Letras, 2009.
2Também poderı́amos medir a posição desde o solo, e então adaptarı́amos a
grandeza g que aparecerá a seguir por −g, para indicar que a gravidade traz para
o solo
CAPÍTULO 24. RUDIMENTOS DE EQUAÇÕES DIFERENCIAIS
E APLICAÇÕES 305

onde γ > 0 depende da forma do objeto, do peso, do material,


etc e onde o sinal negativo tem a ver com o fato que aqui nos
opomos ao efeito da gravidade.
Então obtemos a aceleração:
−γ ′
f ′′ (x) = f (x) + g
m
Queremos descobrir quem é f ′ (x) e depois f (x).
Como tratamos de uma queda-livre, ou seja, o objeto não deve ser
empurrado, vamos supor
f ′ (0) = 0
e também f (0) = 0 para começarmos a medir a distância percorrida a
partir do instante x = 0.
A Afirmação a seguir resolve uma equação que já sabemos resolver
quando A = 0 ou B = 0:
Afirmação 4.1. Uma equação do tipo:
g ′ (x) = Ag(x) + B, ∀x, A, B ∈ R
tem como solução:
g(x) = Bx + g(0), se A = 0,
g(x) = g(0) exp(Ax), se B = 0,
B B
g(x) = (g(0) + ) exp(Ax) − , se A · B 6= 0.
A A
Demonstração. (Afirmação 4.1)
Os casos em que A = 0 ou B = 0 já nos são conhecidos. Por isso
suponhamos AB 6= 0.
Então escrevo:
B
g ′ (x) = A · (g(x) + ),
A
e agora, com a suposição extra de que ∀x: g(x) + B
A
6= 0 obtenho:
g ′ (x)
= A.
g(x) + B A
Agora tomo primitivas. O lado esquerdo reconheço ter como primitivas:
B
ln |g(x) + | + C1
A
onde C1 é qualquer constante e o lado direito tem como primitivas:
Ax + C2
onde C1 é qualquer constante. Ou seja, agrupando as constantes como
C3 := C2 − C1 , obtenho tomando primitivas:
B
ln |g(x) + | = Ax + C3 .
A
4. OBJETOS EM QUEDA-LIVRE 306

Tomando exponencial:
B
exp( ln |g(x) + | ) = exp(Ax + C3 ),
A
de onde
B
|g(x) + | = exp(Ax) · exp(C3 ).
A
Como g(x) + BA
é uma função contı́nua, ela não pode mudar de sinal
sem se anular (Teorema Valor Intermediário) e como supusemos que
A
g(x) + B nunca se anula, temos que ∀x:
• ou bem g(x) + B A
= exp(Ax) · exp(C3 ) > 0
B
• ou bem g(x) + A = − exp(Ax) · exp(C3 ) < 0.
Por isso agora adoto uma nova constante C, que pode ser positiva
se C = exp(C3 ) ou neqativa se C = − exp(C3 ) e escrevo:
B
g(x) = C exp(Ax) − .
A
Para determinar C avalio tudo em x = 0:
B
g(0) = C − ,
A
e portanto:
B
C = g(0) + ,
A
o que dá
B B
g(x) = (g(0) + ) · exp(Ax) − .
A A
B
Agora volto à hipótese de que g(x) + A 6= 0. Observe que se pomos
C = 0 em
B
g(x) = C exp(Ax) −
A
temos
B
g(x) ≡ .
A
B
E note que g(x) ≡ A também satisfaz
B
0 ≡ g ′ (x) = Ag(x) + B = A(− ) + B ≡ 0.
A
Se pode provar usando fatos gerais sobre equações diferenciais que essas
são todas a soluções da equação
g ′ (x) = Ag(x) + B.


Na figura a seguir resolvi plotar a solução especial g(x) = − B A


ao
lado das soluções positivas e negativas g(x) = (g(0) + A ) · exp(Ax) − B
B
A
.
CAPÍTULO 24. RUDIMENTOS DE EQUAÇÕES DIFERENCIAIS
E APLICAÇÕES 307

7,4

7,2

6,8

6,6

0 1 2 3 4
x

Fig.: Gráfico de y = 7 (vermelho) e gráficos de y = C exp(−x) + 7,


com C = − 41 , − 12 , 12 , 41 .

Como consequência da Afirmação 4.1, se fazemos:


−γ
g(x) = f ′ (x), A= , B=g
m
e
f ′ (0) = 0
temos
f ′ (x) = gx, se γ = 0,
ou
−gm −γ gm
f ′ (x) = exp( x) + , se γ 6= 0.
γ m γ
Agora vamos impor que f (0) = 0 pois queremos medir a distância
percorrida no tempo x > 0.
Se γ = 0 obtemos
g · x2
f (x) = .
2
Ma se γ 6= 0:
Z
−gm −γ gm
f (x) = [ exp( t) + ] dt =
γ m γ
−m −gm −γ gm
= ( ) exp( x) + x+C
γ γ m γ
e a imposição f (0) = 0 dá:
−m gm
C= ( )
γ γ
e portanto:
gm −γ
f (x) = · (1 − exp( x)).
γ m
4. OBJETOS EM QUEDA-LIVRE 308

Os valores de γ se determinam experimentalmente. Por exemplo,


para m = 10 kg pode-se3 atribuir o valor γ = 2 kg
s
. A Figura a seguir
compara a queda sem resistencia com a queda com resistência, onde
γ = 2 kg
s
.

300

250

200

150

100

50

0
1 2 3 4 5 6 7 8
x

2
Fig.: Gráficos de y = g·x2 (vermelho) e y = gm
γ
· (1 − exp( −γ
m
x)) (verde),
com g = 9.8, m = 10, γ = 2, para x ∈ [1, 8]

Seria muito interessante para um pára-quedista por exemplo ter em


f (x) uma função linear ou que cresça menos que isso ainda.
Por curiosidade plotei ao lado dos gráficos anteriores o da reta y =
gx:

300

250

200

150

100

50

0
1 2 3 4 5 6 7 8
x

2
Fig.: Gráficos de y = g·x2 (vermelho) e y = gm
γ
· (1 − exp( −γ
m
x)) (verde),
e y = gx (amarelo) com g = 9.8, m = 10, γ = 2, para x ∈ [1, 8]
a seguinte afirmação trata da conservação de energia4 na queda-
livre:
Afirmação 4.2. Considere um objeto de massa m que cai em
queda-livre, verticalmente, sem efeito de atrito. Se f (x) dá a distância
vertical percorrida desde que o objeto é largado em queda livre, então
a grandeza chamada Energia Total:
(f ′ (x))2
m· − mg · f (x)
2
3Boyce e DiPrima, Equações diferencias elementares e problemas de valores de
contorno, LTC.
4Se medı́ssemos a posição desde o solo, terı́amos que ter usado g < 0 ao invés
de g > 0 e a energia total seria uma soma, não uma subtração
CAPÍTULO 24. RUDIMENTOS DE EQUAÇÕES DIFERENCIAIS
E APLICAÇÕES 309

é constante ∀x.
Demonstração.
De fato, como vimos acima quando γ = 0, então f ′ (x) = g · x e
2
f (x) = g · x2 .


No que segue vamos supor a seguinte versão da:

(Lei de Newton) se dd xs é a velocidade de um ponto de massa m ao


longo de um gráfico, então a aceleração é:
d2 s F
2
= ,
dx m
onde F é a força resultante que atua sobre o corpo.
Afirmação 4.3. Considere dois pontos A, B num plano posi-
cionado verticalmente. Suponha que B = (0, 0) é a origem de um
sistema de coordenadas cartesiano e que
A = (a1 , a2 ), a1 6= 0, e a2 > 0.
Suponha que o gráfico Γ de y = f (x) (derivável) com f (a) = A a
f (b) = B descreve a trajetória de um corpo de massa m que cai ao
longo de Γ, apenas sob o efeito da gravidade, sem atrito, partindo de
A no tempo x = a com velocidade inicial 0 e chegando em B no tempo
x = b.
Então é constante, ∀x ∈ [a, b], a grandeza
( dd xs )2
m· + g · m · f (x),
2
onde g = 9.8 m/s2 .
Demonstração.
Derivando
( dd xs )2

2
obtemos:
d s d ( dd xs ) d s d2 s
m· · =m· · .
dx d x d x d x2
Como vimos na Seção 2, podemos determinar a posição de um ponto
P do gráfico em função de quanto vale o comprimento do gráfico desde
f (a) = A até f (x) = P . Ou seja, há uma função P = P (s).
A força resultante F em cada ponto P (s) do gráfico Γ depende do
efeito da gravidade na direção da tangente do gráfico, ou seja, é da
ordem de
F = −gm · sin(θ(s)),
4. OBJETOS EM QUEDA-LIVRE 310

onde θ(s) é o ângulo formado pela tangente de Γ em P (s) com a hori-


zontal e o sinal − se deve a que a força é no sentido oposto ao cresci-
mento de y (se θ = π2 temos toda a força gravitacional gm agindo
verticalmente).
Lembrando a Observação 3.1, temos então:
F dy
= −g · sin(θ(s)) = −g ·
m ds
e com a Lei de Newton obtemos:
d2 s dy
2
= −g · .
dx ds
Logo a derivada de
ds
m( )2
dx
é:
ds dy dy ds
m· · (−g · ) = −mg · =
dx ds ds dx
dy
= −mg · ,
dx
se usamos na última igualdade a regra da derivada da composta.
Portanto, como y = f (x), a derivada de
ds 2
m( ) + gm · f (x)
dx
é zero, o que diz que essa grandeza é constante.


A Afirmação 4.3 tem uma estensão na Afirmação 4.4 a seguir. A


prova desta é essencialmente a mesma da Afirmação 4.3, a diferença
está apenas no uso de noções vetoriais. Por isso a omitimos:
Afirmação 4.4. Considere dois pontos A, B num plano posi-
cionado verticalmente. Suponha que B = (0, 0) é a origem de um
sistema de coordenadas cartesiano e que
A = (a1 , a2 ), a1 6= 0, e a2 > 0.
Suponha que a curva parametrizada
Γ : (x(t), y(t)), t ∈ [a, b]
com A = (x(a), y(a)) a B = (x(b), y(b)) descreve a trajetória de um
corpo de massa m que cai ao longo de Γ, apenas sob o efeito da gravi-
dade, sem atrito, partindo de A no tempo t = a com velocidade inicial
0 e chegando em B no tempo t = b.
Então é constante, ∀t ∈ [a, b], a grandeza
( dd st )2
m· + gm · y(t),
2
CAPÍTULO 24. RUDIMENTOS DE EQUAÇÕES DIFERENCIAIS
E APLICAÇÕES 311

onde g = 9.8 m/s2 e


ds p ′ 2
= (x (x) + (y ′ (x))2 .
dt
5. A curva que minimiza o tempo
Considere um ponto A = (0, 0) e um ponto B = (b1 , b2 ), com b1 6= 0
e b2 < 0. Imagine um objeto de massa m = 1 que desliza pela reta
AB, de A até B, apenas sob o efeito da gravidade g, sem atrito.
O Exercı́cio 7.6 (resolvido) propõe descrever a reta AB como curva
parametrizada Γ tal que Γ(0) = A e Γ(t) dá a posição do objeto que
desliza. Também pede para calcular o tempo t0 em que Γ(t0 ) = B
(como função da distância de A até B e da inclinação da reta AB)
No caso particular em que B = (π, −2) (e no qual uso para acel-
eração g o valor π 2 ≈ 9.869604404) obtemos, segundo o Exercı́cio:

π2 + 4
t0 = ≈ 1.185447061.
π
O objetivo desta Seção é dar uma outra curva parametrizada β
ligando A = (0, 0) até B = (π, −2), com β(0) = A, β(t) dando a
posição do objeto que desliza apenas segundo a gravidade, sem atrito,
mas para a qual:
β(1) = B.
É claro que o comprimento de β, de A até B, é maior que a distância
p
b21 + b22 , porém afirmo que deslizando por β o objeto chega antes a B
do que se deslizasse pela reta AB !
A curva que considero é a ciclóide:
β(t) := ( πt − sin(πt) , cos(πt) − 1 ), t ∈ [0, 1]
que claramente sai de β(0) = A e chega em t0 = 1 em
β(1) = (π, −2) = B.
O que precisamos verificar é se a β(t) pode descrever a posição do
objeto que desliza. Para isso uso a Afirmação 4.4.
Temos para esta curva:
ds
( )2 = (x′ (x)2 + (y ′ (x))2 = 2π 2 · (1 − cos(πt)).
dt
Usando para g o valor π 2 ≈ 9.869604404, após derivar e simplificar
obtemos:
( d s )2
d ( d2t + π 2 · y(t) )
≡ 0,
dt
onde y(t) = cos(π · t) − 1.
A sequência de Figuras a seguir mostra a corrida entre a reta (em
verde) e a ciclóide (em vermelho), para ir de (0, 0) até (π, −2). Cuide
que as escalas dos eixos x, y vão mudando de figura para figura.
5. A CURVA QUE MINIMIZA O TEMPO 312

Os tempos transcorridos são

t = 0.05, 0.1, 0.3, 0.5, 0.7, 1.0, 1.18,

e em t = 1 a ciclóide já chegou no ponto (π, −2).

0 0,001
0,002
0,003
0,004
0,005
0

-0,002

-0,004

-0,006

-0,008

-0,01

-0,012

0 0,0050,010,0150,02
0

-0,01

-0,02

-0,03

-0,04

0 0,05 0,1 0,15 0,2


0

-0,1

-0,2

-0,3

-0,4
CAPÍTULO 24. RUDIMENTOS DE EQUAÇÕES DIFERENCIAIS
E APLICAÇÕES 313

0 0,1 0,2 0,3 0,4 0,5


0

-0,2

-0,4

-0,6

-0,8

-1

0 0,5 1 1,5 2 2,5


0

-0,5

-1

-1,5

0 0,5 1 1,5 2 2,5


0

-0,5

-1

-1,5

0 0,5 1 1,5 2 2,5 3


0

-0,5

-1

-1,5

-2
6. BALÍSTICA E O SUPER MÁRIO 314

0 1 2 3 4
0

-0,5

-1

-1,5

-2

Johann Bernouilli colocou, em 1696, o seguinte5 problema:

Problema da braquistócrona6:
Sejam dados dois pontos A, B num plano vertical. Se A e B não
estão numa reta vertical, encontrar qual a curva descrita por um corpo
M que sai de A e chega em B no menor tempo possı́vel, sob efeito
apenas da gravidade.

É possı́vel provar, com recursos mais avançados dos que dispomos


no momento, que a curva que minimiza o tempo é uma ciclóide.

6. Balı́stica e o Super Mário


Vários cientistas do Renascimento foram defrontados com proble-
mas fı́sico-matemáticos ligados à balı́stica, por exemplo Galileu, Torri-
celli e outros. Naquela época os mecenas eram os Reis e os Reis sempre
foram belicosos...
Por isso vou explicar o problema mais básico de balı́stica, mas o
leitor pacifista pode adaptá-lo ao jogo Super Mário, mais de acordo
com o espı́rito de nossa época. Nesse jogo o personagem salta para
nı́veis mais altos. O que pode ser interpretado como o ponto mais alto
da trajetória na Afirmação 6.1 a seguir.
O problema mais básico para açguém que atira com um canhão
é: dado um alvo encontrar o ângulo θ que se deve levantar um canhão
para atingir o alvo.
Mais precisamente, imagine o alvo no eixo x > 0 e com coordenada
(x, 0) enquanto o canhão está na origem (0, 0). Em geral a velocidade
escalar da bala do canhão não pode ser alterada, o que se pode é alterar
o ângulo 0 < θ < π2 que o canhão forma com o eixo x > 0.
Também se supõe que a bala sofre apenas o efeito da gravidade
(e que estamos a nı́vel do mar), sem sofrer resistências extra ao seu
deslocamento.
5y −2xπ
= xπ+2π−2x
6
braquistocrona vem do grego e significa menor tempo
CAPÍTULO 24. RUDIMENTOS DE EQUAÇÕES DIFERENCIAIS
E APLICAÇÕES 315

Se meditamos um momento vemos que, se x for grande demais em


relação a v0 pode acontecer da bala nunca alcançar o alvo. Aı́ é preciso
aproximar o canhão do alvo.
A Figura a seguir mostra 4 tentativas frustradas de se atingir o
alvo, onde v0 = 5 e x ≥ 3.

1
0,8
0,6
0,4
0,2
0
0 0,5 1 1,5 2 2,5

Figura: A tentativa em verde é a de θ = π4 .


Afirmação 6.1. Seja v0 > 0 a velocidade escalar com que a bala
sai do canhão e o alvo em (x, 0), com x > 0.
• o ângulo θ a ser escolhido para o tiro atingir o alvo (x, 0)
verifica
g·x
sin(2 · θ) = 2 ,
v0
2
onde g = 9.8 (m/s ).
• em geral, dado um 0 < θ < π2 , a trajetória da bala é descrita
pela parábola
g
y=− 2
· x2 + tan(θ) · x.
2 · v0 · cos2 (θ)
Em particular, a partir da parábola vemos que:
• o ponto mais alto atingido pela bala tem coordenadas:
v02 · sin(θ) cos(θ) v02 · sin2 (θ)
( , ).
g 2g
• o ponto onde a bala atinge o chão tem coordenada
sin(2θ) · v02
x= .
g
Em particular o ponto mais longe que pode ser atingido tem
coordenada
v2
x= 0
g
e corresponde à escolha θ = π4 .
6. BALÍSTICA E O SUPER MÁRIO 316

• o ponto mais alto da trajetória se dá no tempo


v0 · sin(θ)
tM = .
g
O tempo que transcorre entre a saı́da da bala e sua chegada ao
chão é 2 · tM .
A Figura a seguir ilustra um tiro certeiro:

1,6
1,2
0,8
0,4
0
0 2 4 6 8
x

Figura: θ = π5 , v0 = 10, x ∼ 9.7, altura máxima ∼ 1.7.

Demonstração.
A velocidade v0 tem uma componente horizontal e uma vertical.
A horizontal é x′ (0) = v0 · cos(θ) e a vertical y ′ (0) = v0 · sin(θ).
Não há componente horizontal da força de gravidade. Portanto,7 se
x(t) é a coordenada horizontal da posição da bala:
x′′ (t) ≡ 0
o que dá:
x′ (t) ≡ C = x′ (0)
e portanto:
x(t) − x(0) = x′ (0) · t.
Como (x(0), y(0)) = (0, 0) temos:
x(t) = x′ (0) · t = v0 · cos(θ) · t, ∀t ≥ 0.
Mas a gravidade g afeta a componente vertical. De fato:
y ′′ (t) = −g,
(onde o sinal vem da oposição entre o sentidos).
Logo
y ′ (t) − y ′ (0) = −g · t,
7E se supõe que a bala não sofre resistência
CAPÍTULO 24. RUDIMENTOS DE EQUAÇÕES DIFERENCIAIS
E APLICAÇÕES 317

ou seja,
y ′ (t) = y ′ (0) − g · t,
e daı́ obtemos:
g · t2
y(t) − y(0) = y ′ (0) · t − .
2
Ou seja
g · t2
y(t) = v0 sin(θ) · t − .
2
Substituindo
x(t) x
t= =
x′ (0) x′ (0)
em
g · t2
y(t) = v0 sin(θ) · t −
2
obtemos a parábola
g
y=− · x2 + tan(θ) · x,
2· v02 · cos2 (θ)
que é a descrição da trajetória da bala.
Sabemos encontrar o ponto de máximo de uma parábola y = ax2 +
bx + c, onde a < 0. Esse ponto é x = −b 2a
. No caso da parábola acima
obtemos:
v 2 · sin(θ) cos(θ)
x= 0
g
e daı́ obtemos a altura máxima.
O tempo tM em que se atinge essa altura máxima é obtido de igualar
a componente vertical da velocidade a zero:
0 = y ′ (tM ) = y ′ (0) − g · tM ,
portanto:
y ′ (0)
tM = .
g
E o tempo tF > 0 no qual a bala atinge o alvo é obtido de igualar
y(tF ) = 0 e resolver:
g · t2
0 = v0 sin(θ) · t −
2
cujas raı́zes são t = 0 e
2 · y ′ (0)
tF = = 2 · tM .
g
A coordenada x do alvo atingido pode ser obtida ou avaliando x(t)
em tF ou vendo-se a intersecção da parábola acima com o eixo x. De
ambos os modos obtêm-se:
v 2 · sin(2 · θ)
x= 0 .
g
7. EXERCÍCIOS 318

Deixo para o Exercı́cio 7.7 a prova de uma propriedade de balı́stica


conhecida por Galileu, exemplificada na Figura a seguir:

0
0 2 4 6 8 10

7. Exercı́cios
Exercı́cio 7.1. Quanto tempo tem que ter passado para que uma
mostra de osso tenha menos que 10−3 vezes a quantidade original de
C14 ?
Exercı́cio 7.2. Em quanto tempo duplica uma dı́vida que cresce
segundo a equação f ′ (x) = 2 · f (x) ?
Exercı́cio 7.3. (resolvido)
A 12 -vida é o tempo τ transcorrido para que uma substância radioa-
tiva tenha massa f (τ ) igual à metade da massa inicial f (0).
i) Suponha que defino a 41 -vida como o tempo τ̂ transcorrido para
que uma substância radioativa tenha massa f (τ̂ ) igual a um quarto da
massa inicial f (0). Qual a relação entre τ̂ e τ ?
ii) Suponha agora que defino a √12 -vida como o tempo τ̌ transcorrido
para que uma substância radiotiva tenha massa f (τ̌ ) igual f√(0)
2
. Qual
a relação entre τ̌ e τ ?
iii) Mais geralmente, chamo agora de 11 -vida o tempo τn transcor-
2n
f (0)
rido para que uma substância radiotiva tenha massa f (τn ) igual 1 .
2n
Qual a relação entre τn e τ ?
Exercı́cio 7.4. Em 10 anos a quantidade inicial f (0) de uma
substância radioativa caiu para f (0)
3
.
i) qual o valor de k na equação f ′ (x) = −kf (x) do decaimento ?
ii) qual a meia-vida dessa substância (em função do k do item i) ?
CAPÍTULO 24. RUDIMENTOS DE EQUAÇÕES DIFERENCIAIS
E APLICAÇÕES 319

Exercı́cio 7.5. (resolvido)


Considere a equação f ′ (x) = −kf (x), com −k < −1 e f (0) = 1.
Note que então f ′ (0) = −k < −1.
Para qual tempo x temos que o coeficiente angular da tangente ao
gráfico da solução y = f (x) é exatamente −1 ?
Exercı́cio 7.6. (resolvido)
Suponha g = 9.8 m/s2 ao nı́vel do mar.
Considere dois pontos A, B, com B mais baixo que A, mas não
vertical com A. Imagine que um objeto é largado em A, com velocidade
inicial zero, e desliza até B, sem atrito, apenas sob o efeito da gravidade,
ao longo do segmento de reta não-vertical que liga esses dois pontos.
i) Descreva a reta AB como curva parametrizada Γ, com parâmetro
t escolhido de modo que (x(0), y(0)) = A e que (x(t), y(t)) dê a posição
do objeto no tempo t.
ii) quanto tempo leva o objeto para ir de A a B ? (a resposta certa-
mente depende da distância entre esses pontos e também da inclinação
da reta AB)
Exercı́cio 7.7. A Figura a seguir ilustra em vermelho a trajetória
de uma bala de canhão que forma ângulo de π4 com o eixo x, atingindo
o alcance máximo.
E em amarelo e verde dois lançamentos com ângulos π4 +0.4 e π4 −0.4,
respectivamente.

0
0 2 4 6 8 10

Por quê atingiram o mesmo ponto ?


Galileu já conhecia essa propriedade !
CAPı́TULO 25

Newton e a gravitação

(...) Halley colocou a questão diretamente para Newton em agosto


de 1684: supondo-se uma lei do inverso do quadrado da distância para
a atração do Sol, que tipo de curva faria o planeta ? Newton lhe disse,
uma elipse. Disse-lhe que havia calculado isso havia muito tempo. (..)
que não conseguia achar os cálculos, mas prometeu refazê-los e enviá-
los mais tarde (...)
(trecho da biografia de Newton, de J. Gleick)

Este Capı́tulo explicará alguns dos cálculos que Newton queria


mostrar a Halley.
1. Atração segundo o inverso do quadrado da distância
Se lembramos como é enorme raio do globo terrestre, podemos pen-
sar que a distância entre os objetos caindo (em queda-livre ou arremes-
sados, nas Seções anteriores) e o centro da Terra é muito próxima do
valor do Raio da Terra1:
R ∼ 6.378 · (10)6 m.
Estabeleçamos a lei de atração universal, de Newton, que é formu-
lada para dois pontos com massa:

dois pontos de massa m0 e m se atraem recı́procamente com uma


força da ordem de G·mr20 ·m , onde G é uma constante universal e r é a
distância entre eles.
Agora imaginemos a massa da Terra M ∼ 5.98 · 1024 concentrada
no seu centro (centro de gravidade). O que acontece quando queremos
usar a lei de atração para explicar a atração mútua exercida pelo centro
de gravidade da Terra e um ponto de massa m = 1?
Obteremos:
g G·M ·m
=g= ∼
m R2
G · 5.98 · 1024
∼ ,
(6.378)2 · (10)12
e portanto
G ∼ 6.67 · (10)−11 ,
1Os dados sobre a Terra obtive em R. Resnick e D. Halliday, Fı́sica, LTC.
321
2. TEMPO DE COLISÃO E VELOCIDADE DE ESCAPE 322

em unidades m3 /(s2 kg).


F
Ademais como a massa da Terra é enorme, sua aceleração M
pode
ser considerada nula.

2. Tempo de colisão e velocidade de escape


Agora que já colocamos os fenômenos de queda-livre e balı́stica no
quadro da lei geral da atração gravitacional, consideremos um ponto
de massa M na origem (0, 0) com massa muito grande relativamente à
massa m = 1 de um corpo cuja posição inicial é (x(0), 0).
Afirmação 2.1. Suponha um ponto na origem de massa M e outro
ponto P de massa m = 1 na posição (x(0), 0). Suponha M tão grande
que possamos considerar o ponto na origem como parado.
Suponha que o ponto em (x(0), 0) que se movimenta apenas segundo
o eixo dos x.
Então
• É constante ∀t a grandeza:2
(x′ (t))2 GM
− .
2 x(t)
• Se x′ (0) = 0 (velocidade inicial zero) então o tempo de colisão
entre o ponto P e a origem é de:
r
π x(0) 3
· .
2 GM
• Para escapar da atração do ponto na origem e se afastar tanto
quanto quisermos da origem, a velocidade inicial x′ (0) tem que
ser necessariamente:
s
2 · GM
x′ (0) ≥ .
x(0)
• em particular, para um foguete lançado da superfı́cie da Terra
escapar da atração da Terra e se afastar da Terra, necessari-
amente:
s
2 · GM
x′ (0) ≥ ∼ 11.184 m/s.
x(0)
Demonstração.
Como m = 1, a lei de atração diz:
x′′ (t) Gm0
= x′′ (t) ≡ − ,
m x(t)2
2chamada (x′ (t))2
de Energia total, onde 2 é chamada de energia cinética e − GM
x(t)
de energia potencial.
CAPÍTULO 25. NEWTON E A GRAVITAÇÃO 323

onde o sinal − deve-se a que a atração é oposta ao sentido positivo dos


x.
Logo
x′ (t)
x′′ (t) · x′ (t) ≡ −Gm0 ,
x(t)2
e portanto
(x′ (t))2 ′ 1 ′
[ ] ≡ Gm0 · [ ],
2 x(t)
ou seja
(x′ (t))2 Gm0 ′
[ − ] ≡0
2 x(t)
e
(x′ (t))2 Gm0
− ≡ C.
2 x(t)
Se o corpo foi largado com velocidade inicial
x′ (0) = 0,
então obtenho
Gm0
C=− ,
x(0)
e portanto
s
Gm0 Gm0
x′ (t) = − 2 · ( + )
x(0) x(t)
(onde tomo a raı́z negativa poque o ponto P se aproximará da origem).
Como x′ (t) < 0, para t > 0, a função x(t) é estritamente decres-
cente.
Logo posso considerar a função inversa t = t(x). A fórmula da
derivada da função inversa dá:
1
t′ (x) = − q .
2 · ( Gm 0
x(0)
+ Gm0
t(x)
)

Para calcular o tempo t de colisão entre P e a origem podemos fazer


a integral
Z 0
t−0= t′ (x) dx,
x(0)

pois assim estaremos calculando o tempo que trancorre para sairmos


de x(0) > 0 e chegarmos em x = 0 (a origem).
Ou seja,
Z x(0) Z x(0)
′ 1
t=− t (x) dx = q dx.
Gm0 Gm0
0 0 2 · ( x(0) + t(x) )
2. TEMPO DE COLISÃO E VELOCIDADE DE ESCAPE 324

Se somamos frações, simplificamos, e usamos que as constantes saem


da integral, obtemos:
Z x(0) r Z x(0) √
1 x(0) x
q dx = · p dx,
0 2 · ( Gm 0
+ Gm0
) 2GM 0 x(0) − x
x(0) t(x)

onde se nota que x(0) − x > 0.


Agora faço a substituição para u > 0:
x = u2 e dx = 2u du,
obtendo:
r Z x(0) √ r Z x(0)
x(0) x x(0) u2
· p dx = 2 · p du.
2GM 0 x(0) − x 2GM 0 x(0) − u2
u2
Não é difı́cil conferir que uma primitiva de √ é:
x(0)−u2

up x(0) u
− x(0) − u2 + · arcsin( p ).
2 2 x(0)
Portanto: r Z x(0)
x(0) u2
t=2 · p du =
2GM 0 x(0) − u2
r q p
x(0) x(0) p x(0) x(0)
=2 · [− x(0) − ( x(0))2 + · arcsin( p )] =
2GM 2 2 x(0)
r
x(0) x(0) π
=2 · · =
2GM 2 2
r
π x(0)3
= ,
2 2GM
como querı́amos demonstrar.
Agora consideremos a situação em que x′ (0) > 0.
Determinemos a condição necessária sobre x′ (0) > 0 para que o
ponto P escape da atração do ponto na origem e se afaste tanto quanto
quisermos da origem.
Já vimos que:
(x′ (t))2 GM
− ≡ C,
2 x(t)
ou seja
(x′ (t))2 GM
0≤ ≡C+ .
2 x(t)
GM
Mas, se há um escape onde x(t) → +∞, então x(t)
→ 0 e daı́:
0 ≤ C.
CAPÍTULO 25. NEWTON E A GRAVITAÇÃO 325

Portanto:
(x′ (0))2 GM
− ≡ C ≥ 0,
2 x(0)
de onde
s
2GM
x′ (0) ≥ .
x(0)


3. Órbitas planetárias
Na Seção anterior estudamos como se dá a colisão entre um corpo e
outro de massa muito maior, que o atrai de acordo com a lei de Newton.
Mas a situação mais interessante é quando o objeto de pequena
massa (planeta, satélite, cometa, etc) gravita em torno do de grande
massa (estrela) sem colidir.
A princı́pio esta Seção usa dados do plano e de funções duas variáveis,
portanto seria mais natural num curso de Cálculo em duas variáveis,
enquanto o nosso tem sido em uma variável.
Mas ela é tão profundamente ligada à origem e ao objetivo do cri-
ador do Cálculo, que se torna inevitável apresentá-la.
Vamos nos situar num plano onde suporemos que viaja o planeta
em sua órbita, para simplificar o problema.
De fato, a primeira etapa do problema geral é mostrar que, apesar
de estar num espaço 3-dimensional, a órbita do planeta é de fato plana.
Ou seja, que cada planeta não sai de uma fatia plana do espaço.
Para obter os resultados de Newton, começo lembrando que agora
há duas coordenadas

P (t) = ( x(t) , y(t) ).

do planeta, que mudam com o tempo t.


Ademais a velocidade instantânea P ′ (t) será3

P ′ (t) := ( x′ (t) , y ′ (t) ).

Enquanto que a aceleração instantânea será:

P ′′ (t) := ( x′′ (t) , y ′′ (t) ).

3Tomo isto como uma definição, mas se pode mostrar que de fato o vetor
( x (t) , y ′ (t) ) é o vetor que gera a reta cuja posição é limite de retas secantes à

curva descrita por P (t) à medida que t ∈ R muda.


4. VELOCIDADE E ACELERAÇÃO EXPRESSAS EM
COORDENADAS POLARES 326

4. Velocidade e aceleração expressas em coordenadas polares


Por um motivo que vai ficar claro um pouco mais adiante, vamos
criar um novo modo de descrever a posição P (t) = (x(t), y(t)), a ve-
locidade P ′ (t) e a aceleração P ′′ (t).
Estamos acostumados a encontrar um ponto especı́fico do plano
através de um par de informações sobre ele, a coordenda x e a coorde-
nada y. Mas o sistema cartesiano ortogonal é apenas um instrumento
para determinar pontos. Podemos usar outro par de informações, por
exemplo a distância r do ponto até um ponto - chamado Pólo - e o
ângulo θ que o vetor posição forma com uma semireta - chamada eixo
polar.
A partir de agora determinaremos um ponto do plano dizendo qual
a distância r(t) que o ponto tem da origem e qual o ângulo θ(t) que o
vetor (x(t), y(t)) forma com o eixo x > 0. Ou seja,
p x(t) y(t)
r(t) = x(t)2 + y(t)2 , cos(θ(t)) = e sin(θ(t)) = .
r(t) r(t)
Note que r(t), cos(θ(t)) e sin(θ(t)) são deriváveis se r(t) 6= 0. E
também θ(t) = arcsin( y(t)
r(t)
) é derivável se r(t) 6= 0.
Temos também:
x(t) = r(t) · cos(θ(t)) e y(t) = r(t) · sin(θ(t))
e, pelas regras de derivação de produto e composta:
P ′ (t) := ( x′ (t) , y ′ (t) ) =
= ( r′ (t)·cos(θ(t))−r(t)·sin(θ(t))·θ′ (t) , r′ (t)·sin(θ(t))+r(t)·cos(θ(t))·θ′ (t) ).
Note que4
||P ′ (t)||2 = x′ (t)2 + y ′ (t)2 = r′ (t)2 + r(t)2 · (θ′ (t))2 .
A expressão de
P ′′ (t) := ( x′′ (t) , y ′′ (t) )
é bem maior, como o leitor pode verificar.
Agora vem uma etapa engenhosa: vamos querer obter as projeções
dos vetores P ′ (t) e P ′′ (t) em duas direções: numa direção paralela a
P (t) e numa direção ortogonal a P (t).
A direção paralela a P (t) é dada pelo vetor de módulo 1:
1
( cos(θ(t)) , sin(θ(t)) ) = · P (t).
r(t)
Já a direção ortogonal a P (t) será dada pelo vetor de módulo 1:
( − sin(θ(t)) , cos(θ(t)) ).
4O

módulo de um vetor v = (a, b) do plano é ||v|| = a2 + b2
CAPÍTULO 25. NEWTON E A GRAVITAÇÃO 327

Afirmação 4.1. Dado um vetor V = (A, B), sua projeção num


eixo gerado por um vetor v = (a, b) de módulo 1 é dada pelo vetor
(A · a + B · b) · v,
Esse número (A · a + B · b) é chamado de produto escalar de V por v.
Suponhamos essa Afirmação 4.1, para usá-la como método para
obter projeções. Então que a projeção de V = P ′ (t) na direção
v = ( cos(θ(t)) , sin(θ(t)) )

= r′ (t) · ( cos(θ(t)) , sin(θ(t)) )
pois (sem t para simplificara notãção):
r′ = (r′ cos(θ) − r sin(θ)θ′ ) cos(θ) + (r′ sin(θ) + r cos(θ)θ′ ) · sin(θ).
E do mesmo modos se obtêm que a projeção de V = P ′ (t) na direção
v = (− sin(θ(t)) , cos(θ(t)))
é:
r(t) · θ′ (t) · (− sin(θ(t)) , cos(θ(t))).
Essa projeção diz que, para uma mesma mudança de ângulo θ′ (t),
quanto maior for r mais rápido vamos na direção ortogonal a P (t).
Pense em você parado fazendo um cavalo fazer um cı́rculo em um tempo
determinado: quanto maior a corda mais rápido terá que correr o cavalo
para fechar o cı́rculo.
Uma conta um pouco maior dará que a projeção de P ′′ (t) na direção
v = ( cos(θ(t)) , sin(θ(t)) )
é:
[r′′ (t) − r(t) · (θ′ (t))2 ] · ( cos(θ(t)) , sin(θ(t)) ).
Note que se o movimento é perfeitamente circular, r(t) = r e o
módulo dessa projeção vira r · (θ′ (t)))2 : esse termo está ligado à força
centrı́peta, que aumenta com o aumento de (θ′ (t)))2 .
E uma conta também mais longa dá que a projeção de P ′′ (t) na
direção de
v = (− sin(θ(t)) , cos(θ(t)))
é:
[r(t) · θ′′ (t) + 2 · r′ (t) · θ′ (t)] · (− sin(θ(t)) , cos(θ(t))).
Note agora que essa projeção da aceleração muda quando r(t) au-
menta ou diminui: isso é o que faz um patinador girando ao abrir ou
fechar os braços, para diminuir ou aumentar a velocidade do giro.
4. VELOCIDADE E ACELERAÇÃO EXPRESSAS EM
COORDENADAS POLARES 328

Afirmação 4.2. Suponha um ponto sendo atraı́do por força grav-


itacional radialmente dirigida para a origem.
Suponha M tão grande relativo a m que possamos supôr o ponto na
origem tem aceleração nula.
Suponha que ∀t, r(t) 6= 0 (não haja colisão) e que θ′ (0) 6= 05.
O fato da força ser radial para a origem implica que ∀t é constante
a grandeza
C := r(t)2 · θ′ (t),
chamada de momento angular.
GM m
O fato adicional do módulo da força ser r(t)2
implica que ∀t é
constante a grandeza
m · ||P ′ (t)||2 GM m
E := − ,
2 r(t)
chamada de Energia total, soma da energia cinética
||P ′ (t)||2
Ec := m ·
2
e da energia potencial
GM m
Ep := − .
r(t)
Demonstração.
Lidaremos com velocidade e aceleração em coordenadas polares,
como explicamos na Seção anterior.
A hipótese sobre a direção radial da força de atração se expressa
como:
r(t) · θ′′ (t) + 2 · r′ (t) · θ′ (t) ≡ 0.
Já a hipótese sobre o módulo da atração se expressa como
GM m
m · (r′′ (t) − r(t) · (θ′ (t))2 ) = −
r(t)2
(onde o sinal menos está ligado ao sentido da atração para a origem,
oposto ao do vetor posição P (t)).
A condição
r(t) · θ′′ (t) + 2 · r′ (t) · θ′ (t) ≡ 0
produz:
( r(t)2 · θ′ (t) )′ (t) = 2 · r(t) · r′ (t) · θ′ (t) + r(t)2 · θ′′ (t) =
= r(t) · (2r′ (t) · θ′ (t) + r(t) · θ′′ (t)) ≡ 0,
e portanto
r(t)2 · θ′ (t) ≡ C,
5essashipóteses garantem que o momento angular r(0)2 · θ′ (0) não é nulo, o
que diz que o objeto não vai em trajetória radial - caso já estudado na Seção 2
CAPÍTULO 25. NEWTON E A GRAVITAÇÃO 329

desde que r(t) 6= 0 ∀t (não haja colisão). Isso pode ser garantido,
pois supusemos que estamos no caso em que P (t) tem uma distância
mı́nima positiva em relação à origem.
Ademais, como
r(0)2 · θ′ (0) = C,
as hipóteses r(0) 6= 0 e θ′ (0) 6= 0 fazem que C 6= 0.
Temos também r(t)4 · (θ′ (t))2 ≡ C 2 e daı́
C2
r(t) · (θ′ (t))2 = .
r(t)3
Colocando isso na relação anterior:
GM
r′′ (t) − r(t) · (θ′ (t))2 =
r(t)2
obtenho
C2 GM
r′′ (t) − 3
=−
r(t) r(t)2
ou seja,
C2 GM
r′′ (t) = 3
− .
r(t) r(t)2
Se r′ (t) ≡ 0 então r(t) ≡ r constante. E como r2 ·θ′ (t) = C, concluimos
que θ′ (t) = rC2 é constante. Então
C2 C2
||P ′ (t)||2 = r′ (t)2 + r(t)2 · (θ′ (t))2 = r2 · = .
r4 r2
Portanto
||P ′ (t)||2 GM m C2 GM m
m· − =m· 2 −
2 r(t) 2r r
é constante, como afirmamos.
Portanto posso considerar no que segue que r′ (t) 6≡ 0. Daı́, multi-
plicando por r′ (t), e tomando primitivas temos:
Z t
r′ (t)2
= r′′ (s) · r′ (s) · ds =
2 t0
Z t 2
C GM
= ( 3
− 2
) · r′ (s) ds.
t0 r(s) r(s)
Reconhecemos aı́ uma fórmula de integração por substituição:
Z r(t) 2
r′ (t)2 C GM
= ( 3 − 2 ) dr =
2 r(t0 ) r r
C2 GM
=− + + C2 ,
2 · r(t)2 r(t)
onde C2 é uma constante. Ou seja,
C2 2GM
r′ (t)2 + 2
− ≡ C3 .
r(t) r(t)
4. VELOCIDADE E ACELERAÇÃO EXPRESSAS EM
COORDENADAS POLARES 330

onde C3 = 2 · C2 . Já observamos que:


x′ (t)2 + y ′ (t)2 = r′ (t)2 + r(t)2 · (θ′ (t))2
e também que
C2
r(t)2 · (θ′ (t))2 = .
r(t)2
Portanto
C2
x′ (t)2 + y ′ (t)2 = r′ (t)2 + ,
r(t)2
que quando substituı́do na anterior dá:
2GM
x′ (t)2 + y ′ (t)2 − ≡ C3 .
r(t)
Se consideramos a velocidade inicial P ′ (0) concluı́mos que
2GM 2GM
x′ (t)2 + y ′ (t)2 − = C3 = x′ (0)2 + y ′ (0)2 − .
r(t) r(0)
m
Multiplicando por 2
, concluı́mos que é constante a grandeza:
m · ||P ′ (t)||2 GM m
− .
2 r(t)


Afirmação 4.3.
Nas mesmas hipóteses da Afirmação 4.2 (anterior), a trajetória
de P (t) = (r(t), θ(t)) pode ser descrita em coordenadas polares (r, θ)
através de uma função r = r(θ).
De fato, precisamente:
C2
GM
r(θ) = √
m2 G2 M 2 +2mEC 2
1+ GM m
· cos(θ)
onde mC = mr2 (t) · θ′ (t) é o momento angular e E = Ec + Ep é a
energia total da trajetória.

Na próxima Seção (Seção 5) explicaremos a geometria da trajetória


r(θ) dada na Afirmação 4.3.

Demonstração. (da Afirmação 4.3)


Já vimos que
r(t)2 · θ′ (t) ≡ C = r(0)2 · θ′ (0) 6= 0,
portanto6 θ′ (t) > 0 ∀t ou θ′ (t) < 0 ∀t.
6θ ′ (t) como função de t é contı́nua, pois de fato existe θ′′ (t).
CAPÍTULO 25. NEWTON E A GRAVITAÇÃO 331

Posso pensar em determinar a coordenada r de P (t) como função de


θ, ao longo da trajetória, pois o próprio θ(t) é ou bem uma função estri-
tamente crescente ou estritamente decrescente de t. Assim t determina
θ e θ determina r.
1
Considero uma nova variável u(t) = r(t) .
Então
1
r′ (t) = [r(θ(t))]′ (t) = [ ]′ (t) =
u(θ(t))
1
=− · u′ (θ(t)) · θ′ (t) =
u(θ(t))2
= −r(t)2 · θ′ (t) · u′ (θ) = −C · u′ (θ).
Coloquemos
r′ (t) = −C · u′ (θ)
e
C
r(t) · θ′ (t) = = C · u(t)
r(t)
na fórmula da energia cinética:
||P ′ (t)||2 (r′ (t)2 + r(t)2 θ′ (t)2 )
Ec := m · =m· =
2 2
′ 2 2
2 (u (θ)) + u(θ)
= mC · ,
2
ou seja,
2Ec
(u′ (θ))2 + u(t)2 = .
mC 2
Ora,
GM m
Ec = E − Ep = E + =
r(t)
= E + GM m · u(t).
Logo
2
(u′ (θ))2 + u(θ)2 = (E + GM m · u(t)).
mC 2
Como u = u(r) e r pode ser pensado como função de θ, que à sua vez
é função (estritamente crescente ou decrescente) de t.
Por isso pensamos em u = u(θ) e escrevo:
2
(u′ (θ))2 + u(θ)2 = (E + GM m · u(θ)).
mC 2
Lembro que a energia total E é constante ao longo da trajetória, por-
tanto a derivada de E como função de θ é zero. Logo, derivando em θ
a expressão anterior, temos:
2GM ′
2 · u′ (θ) · u′′ (θ) + 2u(θ)u′ (θ) = u (θ).
C2
Ou seja,
GM
2u′ (θ) · [u′′ (θ) + u(θ) − 2 ] = 0.
C
4. VELOCIDADE E ACELERAÇÃO EXPRESSAS EM
COORDENADAS POLARES 332

Conforme provaremos na Afirmação 6.1 da Seção 6, todas as soluções


da equação diferencial
GM
u′′ (θ) + u(θ) − 2 = 0
C
são do tipo:
GM
u(θ) = 2 + A · cos(θ − q)
C
onde A e q são constantes arbitrárias.
Suponhamos por um momento isso.
Então u′ (θ) = −A sin(θ − q) e portanto
(u′ (θ))2 = A2 sin2 (θ − q)
e
GM
(u′ (θ))2 + u(θ)2 = A2 sin2 (θ − q) + ( + A · cos(θ − q))2 =
C2
G2 M 2 GM
= A2 + 4
+ 2A · 2 · cos(θ − q)
C C
e por outro lado já tinhamos
2
(u′ (θ))2 + u(θ)2 = (E + GM m · u(θ)) =
mC 2
2 GM
= 2
(E + GM m · ( 2 + A · cos(θ − q))) =
mC C
2 2
2E 2G M GM
= 2
+ 4
+ 2A · 2 · cos(θ − q).
mC C C
Reunindo isso obtenho:
G2 M 2 2E m2 G2 M 2 + 2mEC 2
A2 = + =
C4 mC 2 m2 C 4
o que dá: √
m2 G2 M 2 + 2mEC 2
A=± .
mC 2
Logo

1 GM m2 G2 M 2 + 2mEC 2
= u(θ) = 2 ± · cos(θ − q).
r(θ) C mC 2
Como cos(θ − q + π) = − cos(θ − q) não precisamos manter o ± e
módulo translação em θ, podemos escrever:

1 GM m2 G2 M 2 + 2mEC 2
= 2 + · cos(θ),
r(θ) C mC 2
C2
e multiplicando tudo por GM
:

C2 1 m2 G2 M 2 + 2mEC 2
· =1+ · cos(θ),
GM r(θ) GM m
CAPÍTULO 25. NEWTON E A GRAVITAÇÃO 333

de onde finalmente:
C2
GM
r(θ) = √ .
m2 G2 M 2 +2mEC 2
1+ GM m
· cos(θ)


5. Cônicas em coordenadas polares


Se o eixo polar é identificado com o dos x > 0 e o Pólo com (x, y) =
(0, 0) então:
p y
r = x2 + y 2 e tan(θ) = .
x
No Capı́tulo 18 definimos a excentricidade e o semi-latus rectum de
uma cônica qualquer.
Afirmação 5.1. Seja uma cônica com foco F , semi-latus rectum
l e excentricidade e > 0.
Tome coordenadas polares cujo Pólo é F . Use o eixo da cônica
como eixo dos x e ponha como eixo polar o eixo x > 0.
Então nessa coordenada polar a cônica é dada por:
l
r(θ) = ,
1 + e · cos(θ)
onde θ é o ângulo medido com o eixo polar.
Em particular:
x2 y2
• as elipses a2
+ b2
= 1 viram
b2
a
r(θ) = √ .
a2 −b2
1+ a
· cos(θ)
Essa descrição se estende ao cı́rculo x2 +y 2 = a2 , pondo e = 0,
o que dá a equação r(θ) = l = a.
2 2
• As hipérboles xa2 − yb2 = 1 viram
b2
a
r(θ) = √ .
a2 +b2
1+ a
· cos(θ)

• as parábolas y 2 = 4ρ · x viram r(θ) = 1+cos(θ)
.
Demonstração.
Como o Pólo é F , temos para um ponto P da cônica
r(P ) = e · P r
onde r é diretriz da cônica.
Considere x = −(ρ+eρ) a equação da diretriz, P0 = (−eρ, 0) vértice
da cônica e o foco F = (0, 0). Ou seja, que a distância entre a diretriz
e o foco F é ρ + eρ.
5. CÔNICAS EM COORDENADAS POLARES 334

Denote x(P ) a coordenada x de P (que pode assumir valores posi-


tivos ou negativos). Então
P r = (ρ + eρ) + x(P )
e portanto
r(P ) = e · (ρ + eρ + x(P ))
Um ponto P̂ da cônica com P̂ r = (ρ+eρ) está situado verticalmente
sobre o foco. Pela Definição 1.1 de cônica do Capı́tulo 18,
P̂ F = e · (ρ + eρ).
Mas o semi-latus rectum l foi definido como a distância P̂ F , ou seja,
l = e · (ρ + eρ).
Ou seja, temos
r(P ) = l + e · x(P ).
Podemos tomar o ângulo θ̂ que o vetor posição faz com a semi-
reta que sai de F = (0, 0) e chega no vértice P0 = (−eρ, 0). Assim
x(P0 ) = r(P0 ) cos(0). Assim em geral,
x(P ) = r(P ) cos(θ̂) = −r(P ) cos(π − θ̂) = −r(P ) cos(θ)
onde θ é o ângulo formado com o eixo x > 0. Daı́
r(P ) = l − e · r(P ) cos(θ)
e portanto
l
r(P ) = r(θ) = .
1 + e · cos(θ)


Afirmação 5.2. A trajetória determinada na Afirmação 4.3 como


C2
GM
r(θ) = √
m2 G2 M 2 +2mEC 2
1+ GM m
· cos(θ)
C 2
é uma cônica com semi-latus rectum GM e excentricidade

m2 G2 M 2 + 2mEC 2
e= .
GM m
Ademais, é uma elipse (cı́rculo), parábola ou hipérbole se respecti-
2 2
vamente E < 0 (E = − mG2CM 2 ), E = 0 ou E > 0.
Demonstração.
A Afirmação 5.1 já demonstrada nos diz que se trata de uma cônica
com essa excentricidade e esse semi-latus rectum.
Agora noto que:
e<1 ⇔ m2 G2 M 2 + 2mEC 2 < G2 M 2 m2 ⇔
2
⇔ 2mEC < 0 ⇔ E < 0.
CAPÍTULO 25. NEWTON E A GRAVITAÇÃO 335

E do mesmo modo
mG2 M 2
e=0 ⇔ E=− ,
2C 2
e=1 ⇔ E=0
e > 1 ⇔ E > 0.


Exemplo:
As órbitas dos planetas dos sistema Solar tem excentricidade muito
pequena.
Mercúrio é o planeta do sistema solar cuja órbita tem a maior
excentricidade, da ordem de e = 0.205630. Seu semi-latus rectus é
5.54430 × 1010 m.

4E10

2E10

-6E10 -4E10 -2E10 0E0 2E10 4E10


0E0

-2E10

-4E10

l
Figura: Elipse r(θ) = 1+e cos(θ)
, e = 0.205630 e l = 5.54430 × 1010 (notação 5.5 E 10).

6. Oscilador harmônico
A Afirmação a seguir prova um fato que já usamos na prova da
Afirmação 4.3, além de reforçar o conteúdo da Afirmação 2.1 do Capı́tulo
12:
Afirmação 6.1.
i) Todas as soluções do problema
f ′′ (x) = −k 2 · f (x) + H, ∀x ∈ R
onde k, H ∈ R, são da forma
H
f (x) = a · cos(k · x) + b · sin(k · x) +
k2
onde a, b são constantes arbitrárias. Essas constantes ficam determi-
nadas por a = f (0) e b = f ′ (0).
6. OSCILADOR HARMÔNICO 336

ii) Ademais,
a · cos(k · x) + b · cos(k · x) ≡ A · cos(k · x − q)
onde √ a
A= a2 + b 2 e cos(q) = .
a2 + b2
Demonstração.
Se k = 0 tudo é muito fácil. Por isso suponho k 6= 0.
De i): Derivando duas vezes as funções a cos(k · x) + b · cos(k · x) + kH2
se verifica facilmente que elas satisfazem:
f ′′ (x) = −k 2 · f (x) + H, H ∈ R.
O que precisamos provar é que não há outros tipos de função satis-
fazendo essa equação.
Considere uma misteriosa função f que satisfaça
f ′′ (x) = −k 2 · f (x) + H, H∈R
bem como a função muito simples g(x) ≡ kH2 , que certamente também
verifica essa equação.
Então a nova função φ := f − g = f (x) − kH2 satisfaz o problema:
φ′′ (x) = −k 2 · φ(x).
Se conseguirmos provar que as únicas soluções de φ′′ (x) = −k 2 ·φ(x)
são da forma a · cos(k · x) + b · sin(k · x), com a, b constantes arbitrárias,
então nossa outrora misteriosa função vira:
H
f (x) =: φ(x) + g(x) = a · cos(k · x) + b · sin(k · x) + 2 ,
k
que é o que queremos provar.
Portanto recaı́mos num problema levemente mais fácil:
φ′′ (x) = −k 2 · φ(x).
Nessa direção, vamos provar primeiro o seguinte:
Caso 1: se φ(x) satisfaz φ′′ (x) = −k 2 · φ(x) e ademais φ(0) =

φ (0) = 0 então φ(x) ≡ 0.
De fato, terı́amos:
φ′′ (x) + k 2 · φ(x) ≡ 0
e portanto
2φ′ (x) · [φ′′ (x) + k 2 · φ(x)] ≡ 0
ou seja,
[(φ′ (x))2 + (k 2 φ(x))2 ]′ ≡ 0
e portanto
(φ′ (x))2 + (k 2 φ(x))2 ≡ C.
Mas φ(0) = φ′ (0) = 0 dão que (φ′ (x))2 + (k · φ(x))2 ≡ 0 e isso implica
que φ′ (x) ≡ φ(x) ≡ 0, como querı́amos.
Agora atacaremos o caso geral:
CAPÍTULO 25. NEWTON E A GRAVITAÇÃO 337

Caso 2: φ(x) satisfaz φ′′ (x) = −k 2 · φ(x) mas a := φ(0) e b := φ′ (0)


são arbitrários.
Derivando duas vezes se vê que ψ(x) := a · cos(k · x) + b · sin(kx)
satisfaz ψ ′′ (x) = −k 2 · ψ(x). Então
(φ − ψ)(x) := φ(x) − ψ(x)
satifaz
(φ − ψ)′′ (x) = −k 2 · (φ − ψ)(x).
Mas agora (φ − ψ)(0) = 0 e (φ − ψ)′ (0) = 0 e pelo Caso 1 aplicado à
função (φ − ψ)(x) concluo que φ − ψ ≡ 0, ou seja φ = a · cos(k · x) +
b · sin(kx) como querı́amos. 
CAPı́TULO 26

Cinética quı́mica e crescimento bacteriano

Quando saı́mos do campo das equações diferenciais lineares, em


geral topamos com equações difı́ceis de serem resolvidas explicitamente
(ou mesmo impossı́veis ...).
Mas algumas equações diferenciais não-lineares bem especiais são
ainda fáceis de serem resolvidas e muito úteis.

1. Crescimento bacteriano
Por exemplo, quando uma quantidade de bactérias é posta num
meio de cultivo adequado, inicialmente a população cresce muito rápido;
mas, ao longo do tempo, quando começam a aparecer detritos e começa
a haver competição por nutrientes há uma desaceleração do crescimento
e a populaçção atinge um platô: ou seja, ainda nascem e morrem in-
divı́duos mas a população fica mais ou menos estável.
Obtemos a mesma descrição no caso das populações humanas em
paı́ses desenvolvidos, que inicialmente cresceram muito mas atualmente
atingiram platôs.
O tipo de equações diferenciais simples que modela o crescimento
bacteriano é a seguinte:
f ′ (x) = r · f (x) − s · f 2 (x),
onde f (x) é a população em cada instante, r > 0, s > 0.
É do tipo não-linear pois aparece esse termo quadrático f 2 .
Note que para f (x) < 1 temos f (x) < f 2 (x) e a contribuição de
−sf 2 (x) pode ser pouco relevante, mas à medida que f aumenta, essa
parte quadrática da equação se manifesta.
É claro que f (x) ≡ rs é solução de
r r
0 ≡ f ′ (x) = r( ) − s( )2 ≡ 0.
s s
Por isso afirmamos:
Teorema 1.1. Seja f (x) > 0 derivável com f (0) > 0 satisfazendo
f ′ (x) = rf (x) − sf 2 (x), r, s > 0, ∀x ∈ R.
Suponha que ∀x : f (x) 6= rs . Então
f (0) · exp(r · x)
f (x) = s .
1 − · f (0)(1 − exp(r · x))
r

339
1. CRESCIMENTO BACTERIANO 340
r
A seguir ploto a solução especial f (x) = s
ao lado de soluções não
constantes:

10

0
0 0,2 0,4 0,6 0,8 1
x

Figura: O gráfico de y = 10 (vermelho) e os gráficos de


y = 1− 1 f·f(0)·exp(10·x)
(0)(1−exp(10·x))
, com f (0) = 0.05, 0.5, 1, x ∈ [0, 1]
10

Pode ser interessante para o leitor considerar um gráfico de cresci-


mento bacteriano, ao lado do de suas derivadas:

2
x
0 0,5 1 1,5 2 2,5 3
0

-2

-4

-6

0.05·exp(4·x)
Figura: O gráfico de y = f (x) = 1− 1 ·0.05(1−exp(4·x)) (vermelho), de
4
′ ′′
y = f (x) (verde) e y = f (x) (amarelo), x ∈ [0, 3]

Demonstração.
Divido
f ′ (x) = rf (x) − sf 2 (x), r, s > 0, ∀x ∈ R,
por
s
rf (x) − sf 2 (x) = rf (x)(1 − f (x)) 6= 0,
r
CAPÍTULO 26. CINÉTICA QUÍMICA E CRESCIMENTO
BACTERIANO 341

que nãos e anula pelas nossas hipóteses. Obtendo então


f ′ (x)
=1
rf (x) − sf 2 (x)
e portanto:
f ′ (x)
= r.
f (x) · (1 − rs f (x))
Desenvolvo então, como soma de frações:
s
f ′ (x) f ′ (x) r
· f ′ (x)
= + = r
f (x) · (1 − rs f (x)) f (x) 1 − rs f (x)
e tomo primitivas obtendo:
s
ln |f (x)| − ln |1 − f (x)| = r · x + C, C ∈ R.
r
Por propriedade do lnaritmo do quociente:
f (x)
ln | | = rx + C.
1 − rs f (x)
Para eliminar o lnaritmo aplicamos a exponencial, obtendo:
f (x)
| | = exp(rx) · C ′ , C ′ = exp(C).
1 − rs f (x)
Determinamos C ′ avaliando em x = 0:
f (0)
| | = exp(0)C ′ = C ′ ,
1 − rs f (0)
portanto
f (x) f (0)
| s | = exp(r · x) · | |.
1 − r f (x) 1 − rs f (0)
Por hı́pótese temos que
s
1 − f (x) 6= 0, ∀x,
r
mas como se trata de uma função contı́nua, o T.V.I. garante que ou
s
1 − f (x) > 0, ∀x ∈ R,
r
ou
s
1 − f (x) < 0, ∀x ∈ R.
r
lno ao tomar seu módulo, ∀x temos ou |1 − rs f (x)| = 1 − rs f (x) ou ∀x
temos |1 − rs f (x)| = −(1 − rs f (x)).
Como f (x) > 0 ∀x e exp > 0 podemos retirar o módulo da igualdade
que obtivemos acima:
f (x) f (0)
| s | = exp(r · x) · | |,
1 − r f (x) 1 − rs f (0)
2. CINÉTICA QUÍMICA 342

o que nos dá:


f (x) f (0)
s = exp(rx) · .
1 − r f (x) 1 − rs f (0)
Daqui obtenho:
s s
f (x)(1 − f (0)) = f (0) · exp(r · x) − f (0) · exp(r · x)f (x),
r r
e daı́:
s s
f (x) · (1 − f (0) + f (0) · exp(r · x)) = f (0) · exp(r · x),
r r
de onde sai o resultado que querı́amos:
f (0) · exp(r · x)
f (x) = .
1 − r f (0) + rs f (0) · exp(r · x))
s

2. Cinética quı́mica

(em elaboração)
CAPı́TULO 27

Área de regiões ilimitadas e Séries convergentes

1
Vimos na Afirmação 4.1 que a área sob o gráfico de y = x
à direita
de x = 1 é infinita, ou em outras palavras:
lim ln(x) = +∞.
n→+∞

Mas uma conseguência do Teorema 2.1 escandalizou o filósofo Hobbes,


no sec. XVII: existem regiões ilimitadas cuja Área é finita !
Vejamos exemplos a seguir:
Afirmação 0.1. A Área da região ilimitada à direita de um a > 0
sob o gráfico de
1
y = k , k ≥ 2, k ∈ N,
x
é igual a
1
.
(k − 1) · ak−1

Demonstração.
A área sob o gráfico de a até um certo x na linguagem do Cálculo
se escreve como: Z x
A(x) = x−k dx
a
e Pelo Segundo Teorema Fundamental do Cálculo Teroema 2.1 :
Z x
1 1
x−k dx = ( x−k+1 )(x) − ( x−k+1 )(a), k ≥ 2
a −k + 1 −k + 1
pois se k ≥ 2:
1 −k + 1 −k
( x−k+1 )′ (x) = x = x−k .
−k + 1 −k + 1
Mas a área de toda a região à direita de a é:
1 1
lim [ ( x−k+1 )(x) − ( x−k+1 )(a)) ] =
x→+∞ −k + 1 −k + 1
1 1 1 k−1
= lim [ k−1
+ a ]=
x→+∞ (−k + 1) x k−1
1 k−1
= a .
k−1


343
1. SÉRIES K-HARMÔNICAS, K > 1 344

1. Séries k-harmônicas, k > 1


Note que a área da região à direita de a = 1 sob o gráfico de y = x1k
é maior que a soma de áreas dos retângulos justapostos
1 1 1
[1, 2] × [0, k ] ∪ [2, 3] × [0, k ] ∪ . . . ∪ [n, n + 1] × ∪[0, ]...
2 3 (n + 1)k
onde os três pontos significam que podemos ir colocando sempre retângulos
à direita.
Mas a área desses retangulos todos é (ainda num sentido vago) uma
soma infinita:
1 1 1
k
+ k + ... + k ...
2 3 n
Pela Afirmação 0.1 -i), com a = 1 temos:
1 1 1 1
∀n ∈ N, k
+ k + ... + k < .
2 3 n k−1
O quer dizer essa soma infinita:
1 1 1
+ + . . . + ... ?
2k 3k nk
Simplesmente quer dizer que existe o limite da sequência xn dada por
1 1 1
xn := k + k + . . . + k , k ≥ 2.
2 3 n
Aqui é importante que k ≥ 2, pois pelo que vimos na prova da
Afirmação 4.1 a soma infinita
1 1 1
+ + ... + ...
2 3 n
tem um comportamento diferente, ela fica tão grande quanto quisermos.
Definição 1.1. As séries 21k + 31k + . . . + n1k . . . são chamadas k-
harmônicas. A série 1-harmônica 12 + 31 + . . . + n1 . . . é chamada apenas
de harmônica.
Como a Afirmação 0.1 diz que
1
∀n ∈ N, xn <
k−1
1
dizemos que a sequência (xn )n é limitada superiormente por k−1 (a
definição de limitada infeiormente é análoga). E nitidamente é cres-
cente, ou seja:
xn ≤ xn+1
1
pois xn+1 = xn + (n+1)k
(a definição de decrescente é análoga).
CAPÍTULO 27. ÁREA DE REGIÕES ILIMITADAS E SÉRIES
CONVERGENTES 345

Então a nossa (xn )n é um exemplo de sequência limitada superior-


mente e crescente, se
1 1 1
xn := k + k + . . . + k , k ≥ 2.
2 3 n

Teorema 1.1. (Teorema fundamental sobre sequências)


i) toda sequência (xn )n limitada superiormente e crescente tem
lim xn .
n→+∞

ii) toda sequência (xn )n limitada inferiormente e decrescente tem


lim xn .
n→+∞

A prova deste teorema se discute em cursos de Análise matemática.


A prova não dá nenhuma pista em geral de quanto vale esse limite,
apenas que existe.
Prefiro dar uma aplicação importante na Seção que segue.

2. A série geométrica
Afirmação 2.1. Seja r um número Real, com 0 ≤ |r| < 1. Defina
a sequência cujo xn := 1 + r + r2 + . . . + rn . Então
n+1
• i) ∀n ∈ N, xn = 1−r1−r
.

• ii) limn→+∞ |r|n = 0 e limn→+∞ rn = 0.


1
• iii) limn→+∞ xn = 1−r n
.
Demonstração.
Claro que se |r| = 0 então r = 0 e tudo que afirmamos é obviamente
válido. Logo no que segue 0 < |r| < 1.

Prova de i), por indução:


1−r 2
Se n = 1, então de fato vale 1 + r = 1−r
. Supondo a fórmula até
n − 1:
1 − rn
1 + r + r2 + . . . + rn−1 =
1−r
e
1 − rn rn · (1 − r)
1 + r + r2 + . . . + rn−1 + rn = + =
1−r 1−r
1 − rn+1
= .
1 − rn
Para provar ii), note que 0 < |r| < 1 implica (multiplicando por r
positivo):
0 < |r|2 < |r| < 1,
2. A SÉRIE GEOMÉTRICA 346

e assim obtemos por indução:


0 < |r|n < |r|n−1 < 1, ∀n ∈ N
Mas então a sequencia (|r|n )n é decrescente e obviamente limitada in-
feriormente pelo 0. Pelo Teorema 1.1) existe
lim |r|n = L.
n→+∞

Mas afirmo que L = 0 (a principio seria apenas 0 ≤ L ≤ |r| < 1).


Meu argumento agora usará uma analogia1: se uma fila completa
de pessoas tende a um lugar, as pessoas nas posições pares também
tendem a esse lugar.
Ou seja, quero dizer que:
lim |r|n = L ⇒ lim |r|2n = L.
n→+∞ n→+∞

Por outro lado


lim |r|2n = lim (|r|n )2
n→+∞ n→+∞
e pelo limite de produtos de sequências:
lim (|r|n )2 = lim |r|n · lim |r|n = L2 .
n→+∞ n→+∞ n→+∞
2
Então L = L . Logo L(L − 1) = 0 e L = 0 ou L = 1. Mas
|r|n < |r| < 1.
impede que seja L = 1, ou seja, temos L = 0.
Bom agora só resta obervar que também limn→+∞ rn = 0. Mas o
que significa limn→+∞ rn = 0 ? Significa que se n é suficientemente
grande temos para qualquer ǫ dado:
|rn − 0| < ǫ,
ou seja, pelas propriedades do módulo:
|rn | = |r|n < ǫ.
Mas temos já provado que
lim |r|n = 0
n→+∞

e isso diz que se n é suficientemente grande temos para qualquer ǫ dado:


| |r|n − 0 | < |r|n < ǫ,
como querı́amos. ou seja:

Prova de iii):
Do item i) já temos que
1 − rn+1
xn = , ∀n ∈ N
1−r
1Rigorosamente trata-se de argumentar com uma subsequência da sequência
toda
CAPÍTULO 27. ÁREA DE REGIÕES ILIMITADAS E SÉRIES
CONVERGENTES 347

e do item ii) temos limn→+∞ rn = 0. Com as propriedades de limites


de somas/produtos obtemos:
1 − limn→+∞ rn 1
lim xn = = .
n→+∞ 1−r 1−r


3. Um argumento geométrico para a série geométrica


Arquimedes provava com um argumento geométrico que
1 1 1 1
+ ( )2 + ( )3 + . . . =
4 4 4 3
o que dá em seguida
1 1 1 1
1 + + ( )2 + ( )3 + . . . = 1 + =
4 4 4 3
4 1
= = ,
3 1 − 41
em perfeita concordância com nossa Afirmação 2.1.
Seu argumento é o seguinte. Tome um quadrado de lado 1 e inscreva
nele um quadrado de lado 12 (e área 41 portanto). Na figura a seguir
é o maior quadrado em vermelho. Note que à direita e acima desse
quadrado vermelho há quadrados verde e amarelos de mesma área 41 .

Figura: Três etapas do processo de Arquimedes

Agora justaponha ao quadrado vermelho um segundo quadrado ver-


melho, de lado 14 e área 412 = 16
1
, como mostra a figuraa seguir (note
1
que aparecem então dois quadrados de área 16 à direita e acima dele).
Assim sucessivamente, quadrados vermelhos de lado 21n e área 41n
são justapostos, ∀n ∈ N.
Arquimedes argumenta que esse processo continuado preenche todo
o quadrado de lado 1 com infinitos quadrados vermelhos, verdes e
amarelos. A soma das áreas dos vermelhos é a mesma soma das áreas
dos verdes e da dos amarelos. Mas então
1 1 1
3 · ( + 2 + 3 + . . .) = 1,
4 4 4
3. UM ARGUMENTO GEOMÉTRICO PARA A SÉRIE
GEOMÉTRICA 348

e portanto
1 1 1 1
+ 2 + 3 + ... = .
4 4 4 3
CAPı́TULO 28

Aproximação de Números e Funções importantes

Neste Capı́tulo mostro que o cálculo permite, através da iteração


das operações elementares +, −, /, x, obter aproximações com a pre-
cisão que se quiser de:
• funções fundamentais como arctan(x), ln(x), etc

• números como p (p primo), π, e = exp(1).
Ou seja, o Cálculo transforma a gente num McGiver , aquele per-
sonagem que quase sem nenhum instrumento fabricava aparelhos incrı́veis
em suas missões. Nós só com as quatro operações faremos tudo (e aı́
a gente entende um pouco do que acontece quando se usa uma calcu-
ladora cientı́fica ...).

1. Aproximações de raı́zes quadradas por números racionais

Pensando bem, é curiosa a nomenclatura números Reais, pois esses


números não estão próximos da nossa realidade nem são dados de forma
natural. Quem aparece no dia-a-dia são os Naturais, os Inteiros e os
Racionais, esses sim presentes nas operações matemáticas mais simples
do dia a dia.
Quando falamos números Reais estamos nos referindo a um con-
junto de números muito maior que o conjunto dos números Racionais
(isso s eprova nos cursos de Análise Matemática).
√ Apesar de que só
saibamos citar um ou outro exemplo decor : 2, π, etc.
De fato quando Arquimedes se refere a π no seu trabalho A medida
do cı́rculo, ele o define como quociente entre o perı́metro e o diâmetro
de um cı́rculo. Ele não prova que π ∈ / Q, mas por outro lado dá um
método para aproximá-lo tanto quanto se quiser por números racionais.
E seu método, que √ é geométrico, usa em certos momentos aproximações
de números como 3 por números Racionais.
Essa é uma visão muito interessante (como todas as do gênio Ar-
quimedes) de que números Reais são limites de sequências de números
Racionais. Um ponto de vista bastante útil e prático para as aplicações
da matemática e ao mesmo tempo um ponto de vista que, convenien-
temente adaptado produz um construção lógica dos Reais (um pouco
mais adiante volto nisto).
349
3. COMO TIRAR RAÍZ QUADRADA SÓ COM +, −, ×, / 350

2. Raı́zes quadradas que são irracionais

Que tal primeiro nos convercermos


√ de que existem números Irra-
cionais, por exemplo, que 2 ∈ /Q √?
Suponha por absurdo que sim 2 = pq , onde p, q ∈ N com mdc(p, q) =
1√ (máximo divisor comum é um). Ou seja, uso por ex. por absurdo
2 = 1/3 ao invés de 2/6.
2
Mas então obtenho: 2 = pq2 e portanto: 2 · q 2 = p2 . O número
Natural p se escreve como um produto de números primos, e nesse
produto o fator 2 aparece um c k ≥ 0 de vezes. Por ex. no 12 = 22 · 3
o fator 2 aparece k = 2 vezes. Mas em p2 há 2k fatores 2 e 2k é sempre
um número Par. Por outro lado p2 = 2 · q 2 e na decomposiçao do
número 2 · q 2 em primos, o fator 2 aparece
√ um número Ímpar de vezes.
Essa contradição surgiu de supor que 2 é racional.
√ Se olharmos bem o argumento que demos para convencernos que
2∈/ Q,√notamos que serviria para provar que qualquer número primo
P tem P ∈ / Q.

3. Como tirar raı́z quadrada só com +, −, ×, /


Vamos aplicar alguns itens do Teorema 3.1 do Capı́tulo 4 para fazer
uma mágica.
Tome um número positivo A. Tome um número positivo arbitrário,
qualquer x > 0 e defina
x0 := x
e
1 A
· (x + ).
x1 :=
2 x
1
Daı́ em diante, recursivamente, defina
1 A
xn := · (xn−1 + )
2 xn−1
2
Afirmação 3.1.
Se a sequência
1 A
· (xn−1 +
xn := )
2 xn−1
tem limn→+∞ xn = L > 0 então de fato

L= A
(a raı́z positiva de A).
1O formato peculiar da sequência será explicado mais tarde
2Uma afirmação mais forte - e verdadeira - é de que de fato a sequência definida
recursivamente tem um limite L e esse limite é um número positivo.
CAPÍTULO 28. APROXIMAÇÃO DE NÚMEROS E FUNÇÕES
IMPORTANTES 351


√ Em particular, se A for um número Irracional como por exemplo
2 e se x for Racional, então estamos dando um método para aproxi-
mar o número irracional pelos números Racionais
1 A
xn := · (xn−1 + ).
2 xn−1
Demonstração.
Para começarmos a prova da Afirmação 3.1, argumentaremos através
de uma analogia.3
Imagine uma fila de pessoas e que a fila se move para algum lugar.
Então vemos elemento n-ésimo caminhando em direção a esse lugar
e o elemento (n − 1)-ésimo que o segue para lá. Isso quer dizer em
linguagem do dia a dia que:

se limn→+∞ xn = L (como supomos) então limn→+∞ xn−1 = L


também.
Para provar a Afirmação toda, note que o Teorema 3.1 do Capı́tulo
4 vai dando, já que limn→+∞ xn−1 = L :
1 1
lim = ,
n→+∞ xn−1 L
A 1 A
lim =A· = ,
n→+∞ xn−1 L L
A 1
lim (xn−1 + )=L+
n→+∞ xn−1 L
1 A 1 1
lim · (xn−1 + ) = · (L + ).
n→+∞ 2 xn−1 2 L
Mas temos
1 A
xn =· (xn−1 + )
2 xn−1
e limn→+∞ xn = L; logo juntando temos:
1 A
L = · (L + ),
2 L
de onde obtemos
L2 + A
2L =
L √
2
e portanto L = A; como L > 0 temos que L = A.


Fiz um exemplo na Calculadora, onde a cada etapa a calculadora


faz truncamentos.
3Rigorosamente trata-se de argumentar com uma subsequência da sequência
toda
4. TANGENTES A PARÁBOLAS E O FORMATO DA
SEQUÊNCIA DE NEWTON 352
1 2
Pondo A = 2 e ∀n ≥ 1, xn := 2
· (xn−1 + xn−1
):
x0 := 390, x1 := 195.0025641 x2 := 97.50641019,
x3 := 48.76346084, x4 := 24.40223758, x5 := 12.24209864,
x6 := 6.202734661, x7 := 3.262586543, x8 := 1.937798551,
x9 := 1.484948789, x10 := 1.415898291, x11 := 1.414214565,
x12 := 1.414213562
e aqui a calculadora
√ não sai mais desse número Racional, que para ela
é a própria 2.

4. Tangentes a parábolas e o formato da sequência de


Newton

Também as equações das retas tangentes às parábolas estão por


trás do formato da sequência de Newton. A idéia para oter o formato
da sequência é√ a seguinte.
Encontrar A é resolver a equação x2 − A = 0, com x > 0. Se
pensamos no gráfico de y = x2 − A, é uma parábola deslocada vertical-
mente. E a solução positiva da equação é onde o gráfico corta o eixo
positivo dos x.
A receita é a seguinte (acompanhe na Figura a seguir):

• tome um x0 = x (suponha por um momento x > A) levante
verticalmente até chegar no gráfico de y = f (x) := x2 − A e
daı́,
• do ponto (x, x2 −A) desça pela tangente do gráfico em (x, x2 −
A) até cortar o eixo positivo dos x.
É esse ponto de corte que será o x1 :

y= x^2 − A

x_1 x_0

Figura: Como se produz a sequência de Newton


Afirmação 4.1. O x1 obtido geometricamente como indicado, ver-
ifica
1 A
x1 = · (x + ).
2 x
CAPÍTULO 28. APROXIMAÇÃO DE NÚMEROS E FUNÇÕES
IMPORTANTES 353

Demonstração.
Começo observando que a Derivada de f (x) = x2 − A em x é 2x:
Logo o coeficiente angular da reta da tangente ao gráfico de y =
f (x) := x2 − A em (x, x2 − A) é 2x e sua equação se obtém como na
Afirmação 1.3:
y − (x2 − A)
= f ′ (x) = 2 · x,
x−x
ou seja:
y = 2 · x · (x − x) + x2 − A.
Descer por essa reta até atingir o eixo positivo dos x é fazer y = 0:
0 = 2 · x · (x − x) + x2 − A.
Se isolamos x aqui obtemos:
x2 + A 1 A
x= = · (x + ),
2·x 2 x
ou seja chegamos no ponto (x, 0) = (x1 , 0).


Depois se repete o processo com o x1 obtido. O próximo ponto


(x2 , 0) terá claramente o mesmo formato, por ter sido obtido pelo
mesmo processo.

5. O Reais através de sequências de números Racionais

Como sabemos não se pode ver um buraco negro, pelo simples mo-
tivo que ele atrai até mesmo os raios de luz. Então como os astrônomos
podem estar tão seguros de que existem esses misteriosos objetos?
O que eles vêem são estrelas sendo sugadas para um certa região,
onde se acumulam milhares de estrelas, apertando-se cada vez mais
numa pequena região do espaço. Daı́ deduzem que ali há um buraco
negro.
Voltando ao nosso tema, se um sequência de números xn tende a
um número L, então os seus termos vão se aproximando entre si :
Afirmação 5.1. Suponha limn→+∞ xn = L. Então dado ǫ > 0
existe um nǫ tal que
∀n1 ≥ nǫ e ∀n2 ≥ nǫ , |xn1 − xn2 | < ǫ.
Demonstração.
Pela definiçao de limn→+∞ xn = L, dado ǫ > 0, existe nǫ tal que
∀n ≥ nǫ temos |xn − L| < 2ǫ .
Então ∀n1 , n2 ≥ nǫ temos (pela desigualdade triangular):
|xn1 − xn2 | = |xn1 − L + L − xn2 | ≤
6. APROXIMAÇÕES DE E = EXP(1) POR NÚMEROS
RACIONAIS 354
ǫ ǫ
≤ |xn1 − L| + |xn2 − L| < + = ǫ.
2 2


Podemos também inverter as coisas !


Que tal lidarmos inicialmente apenas com números Racionais e
fazermos o seguinte: cada vez que vemos uma sequência de números
Racionais cujos termos se aproximam entre si tanto quanto quisermos
(como ocorre na conclusão da Afirmação 5.1), que tal imaginarmos,
postularmos, que ali há um número Real que os atrai ?

Chamaremos as sequências de números Racionais cujos termos se


aproximam entre si de sequências fundamentais.
Claro que pode acontecer que duas ou mais sequências fundamentais
se acumulem na mesma região, e as imaginamos estarem sendo atraı́das
pelo mesmo número Real.
Diremos que duas sequências fundamentais xn e x′n são equivalentes
se
lim (xn − x′n ) = 0.
n→+∞

Isso sugere então pensar que:

cada número Real é uma classe de equivalência de sequências fun-


damentais.

6. Aproximações de e = exp(1) por números Racionais


A prova da irracionalidade de e = exp(1) é dada com detalhes no
livro do M. Spivak, Calculus. Aqui o que discuto é como aproximá-lo
por números Racionais.
Primeiro veremos uma sequência que o aproxima, mas o faz de
modo bastante lento, depois indicaremos outro modo de aproximá-lo,
este sim rápido.

Sabemos pelo Teorema Fundamental e pela definição de logaritmo


natural que:
1
ln′ (x) = , ∀x > 0
x
e portanto:
1
ln′ (1) = = 1.
1
Se olhamos isso pela definição de derivada o que temos é que
ln(1 + h) − ln(1) ln(1 + h)
1 = lim = lim .
h→0 h h→0 h
CAPÍTULO 28. APROXIMAÇÃO DE NÚMEROS E FUNÇÕES
IMPORTANTES 355

Mas se isso vale para quaisquer números h tendendo a zero, podemos


tomá-los da forma:
1
h= com n → +∞.
n
Ou seja que limh→0 ln(1+h)
h
= 1 vira
ln(1 + n1 ) 1
1 = lim 1 = lim n · ln(1 + ).
n→+∞
n
n→+∞ n
Pela propriedade de que
ln(xn ) = n · ln(x), ∀x > 0, ∀n ∈ N
obtenho:
1 n
1 = lim ln( (1 + ) ).
n→+∞ n
Suponha por um momento que a sequência xn := (1 + n1 )n tem
um limite L.
Então como o ln(x) é uma função contı́nua tenho
1 1
lim ln( (1 + )n ) = ln( lim (1 + )n ) = ln(L).
n→+∞ n n→+∞ n
Aplicando exponencial:
exp(1) = exp(ln(L)) = L,
ou seja concluı́mos que xn := (1 + n1 )n é uma sequência de Racionais
tendendo ao e.
Vamos dar agora uma prova de que a sequência xn := (1 + n1 )n
converge para um número entre 2 e 3:
Afirmação 6.1. A sequência xn := (1 + n1 )n tem
1
lim (1 + )n = L, com 2 < L < 3.
n→+∞ n
Demonstração.
Basta verificar que que essa sequência é limitada superiormente-
mente por um número menor que 3. Pois como é nitidamente crescente
e x1 = 2, o Teorema 1.1 garantirá que ela converge.
Começo escrevendo pela fórmula do binômio:
n  
1 n X n 1 j
(1 + ) = ( ) =
n j=0
j n
1 n(n − 1) 1 1
=1+n· + 2
+ ... + n.
n 2! n n
Agora vamos escrever essa soma de um jeito adequado ao que segue:
1
(1 + )n =
n
1 n(n − 1) 1 n(n − 1)(n − 2) . . . 2 1
=1+n· + + . . . + =
n 2! n2 n! nn
6. APROXIMAÇÕES DE E = EXP(1) POR NÚMEROS
RACIONAIS 356

1 1 1 1 2 n−2
=1+1+ (1 − ) + . . . + (1 − )(1 − ) . . . (1 − ).
2! n n! n n n
Agora vamos dar quotas superiores para cada parcela desta soma, ob-
tendo:
1 1 1 1 2 n−2
1 + 1 + (1 − ) + . . . + (1 − )(1 − ) . . . (1 − )<
2! n n! n n n
1 1
< 1 + 1 + + ... + .
2! n!
Para darmos novas cotas superiores a essa soma lembro um Exercı́cio
de Indução:
n! ≥ 2n−1 ∀n ∈ N.
Então
1 1 1 1
1 + 1 + + ... + ≤ 1 + 1 + . . . + n−1 .
2! n! 2 2
ou seja, que (1 + n1 )n é sempre estritamente menor que
1 1
1+1+ . . . + n−1 .
2 2
É nı́tido que esta última soma é o resultado de adicionar 1 a um pedaço
da série geométrica infinita:
1 1
1 + . . . + n−1 + . . . ,
2 2
que já vimos vale:
1 1 1
1 + . . . + n−1 + . . . = = 2.
2 2 1 − 12
Logo ∀n ∈ N:
1 n 1 1
(1 + ) < 1 + (1 + . . . + n−1 + . . .) = 3,
n 2 2
como querı́amos.


Fiz algumas contas no computador, obtendo os primeiros 10 valores


(truncados na 10 casa após a virgula) para xn := (1 + n1 )n :

x1 = 2, x2 = 2.250000000, x3 = 2.370370370, x4 = 2.441406250,


x5 = 2.488320000, x6 = 2.521626372, x7 = 2.546499697,
x8 = 2.565784514, x9 = 2.581174792, x10 = 2.593742460,
e assim por diante, se vê que a sequência vai crescendo lentamente.
Tive que ir até n = 120 para obter

x120 = 2.707041491.
CAPÍTULO 28. APROXIMAÇÃO DE NÚMEROS E FUNÇÕES
IMPORTANTES 357

Se pode provar que a sequência x′n := 1 + 1/1! + 1/2! + . . . + 1/n!


também tende para e = exp(1).
Fiz as contas de n = 1 até n = 12 e já aqui o computador diz
que cheguei no limite, ou seja o erro entre e = exp(1) e x′12 está na
décima-primeira casa decimal:

x′1 = 2, x′2 = 2.500000000, x′3 = 2.666666667,


x′4 = 2.708333333, x′5 = 2.716666667, x′6 = 2.718055556,
x′7 = 2.718253968, x′8 = 2.71827877, x′9 = 2.718281526
x′10 = 2.718281801, x′11 = 2.718281826, x′12 = 2.718281828.
Veja por comparação como a sequência anterior xn = (1 + 1/n)n é
lenta em sua covergência para e, pois x112 = 2.707041491 ainda está
bem longe de x′12 = 2.718281828.

7. Arcotangente e cartografia
Nos mapas as curvas de nı́vel dão a informação de quanto variou
a coordenada vertical ∆y entre dois pontos e a escala do mapa te dá
informação da variação da coordenada horizontal ∆x.
∆y
Logo se obtém um valor tan(α) = ∆x e torna-se relevante calcular
arctan(α).
Logo é importante sabermos calcular o arcotangente com a precisão
que quisermos.
Mas o que a calculadora cientı́fica de fato faz, quando calcula essa
função ?
E se eu tiver apenas uma calculadora que faz as 4 operações, será
que consigo calcular arctan(α) com a precisão que quiser ?
Vou explicar o que fazer, para dar o arctan(x) pelo menos para x ∈
(−1, 1), com a ordem de precisão que se quiser, ou seja, com quantas
casas quisermos depois da vı́rgula, apenas fazendo repetidamente as 4
operações +, −, /, x.
Primeiro começo lembrando da fórmula (Seção 4 do Capı́tulo 16 ):
1
arctan′ (x) = , ∀x ∈ R.
1 + x2
Escrevendo:
1 1
2
= ,
1+x 1 − (−x2 )
podemos usar a Afirmação 2.1 na região x ∈ (−1, 1):
1
= 1 − x2 + x4 − x6 + . . . se |x| < 1.
1 + x2
Sabemos pelo Primeiro Teorema Fundamental que:
Z x
1
2
dt = arctan(x) − arctan(0) = arctan(x).
0 1+t
7. ARCOTANGENTE E CARTOGRAFIA 358

Agora vamos ser otimistas 4: vamos imaginar que podemos usar a pro-
priedade
Z x Z x Z x
(f + g) dt = f dt + g dt
a a a

não apenas para a soma de duas funções f + g mas para a soma de


uma infinidade de funções.
Ou seja, com otimismo, asssumo que a integral de uma soma infinita
de funções é a soma infinita de integrais. Esse otimismo nos permitiria
escrever:
Z x
x3 x5 x7
(1 − t2 + t4 − t6 + . . .) dt = x − + − + . . . , se |x| < 1.
0 3 5 7
O fascinante é que sim, podemos fazer isso ! pelo menos nessa situação
especı́fica... Ou seja, igualando o lado esquerdo com o direito:

x3 x5 x7
arctan(x) = x − + − + ..., se |x| < 1.
3 5 7
E é isso que a calculadora faz: ela trunca a soma

x3 x5 x7
x− + − + ..., se |x| < 1
3 5 7
num grau suficientemente alto para termos a precisão desejada do
arctan(x). E fazer somas e produtos como os que aparecem em

x3 x5 x7
x− + − + ..., se |x| < 1
3 5 7
é fácil para uma calculadora !
As Figuras a seguir comparam o gráfico real de arctan : (−1, 1) → R
3
com os gráficos dos truncamentos y = x : (−1, 1) → R, y = x − x3 :
3 5
(−1, 1) → R e x − x3 + x5 : (−1, 1) → R.

0,5

0
-0,8 -0,4 0 0,4 0,8
x

-0,5

-1

4Justificado na Afirmação 2.1 do Capı́tulo 29


CAPÍTULO 28. APROXIMAÇÃO DE NÚMEROS E FUNÇÕES
IMPORTANTES 359

Figura: O gráfico de y = arctan(x) (vermelho) e y = x (verde) para x ∈ [−0.99, 0.99].

0,8

0,4

0
-0,8 -0,4 0 0,4 0,8
x

-0,4

-0,8

x3
Figura: O gráfico de y = arctan(x) (vermelho) e y = x − 3
(verde) para x ∈ [−0.99, 0.99].

0,8

0,4

0
-0,8 -0,4 0 0,4 0,8
x

-0,4

-0,8

x3 x5
Figura: O gráfico de y = arctan(x) (vermelho) e y = x − 3
+ 5
(verde)
para x ∈ [−0.99, 0.99].

8. A aproximação de π dada por Leibniz


Uma prova de que π é Irracional é dada no excelente livro Calculus,
de M. Spivak, usando com astúcia o Cálculo.
O que quero dar aqui é uma aproximação de π por Racionais, que
remonta a Leibniz.
Mostraremos aqui que a série
x3 x5 x7
arctan(x) = x − + − + ...
3 5 7
funciona para x = 1 ! E como arctan(1) = π4 , teremos:
π 1 1 1
= arctan(1) = 1 − + − + . . . ,
4 3 5 7
de onde:
1 1 1
π = 4(1 − + − + . . .).
3 5 7
.
Essa aproximação de π, apesar de bonita, é lenta e é feita por falta
e excesso, de modo oscilante: de fato as somas parciais de ordem ı́mpar
da soma são maiores que π e decrescem:
1 1
s1 := 4 · 1 = 4, s3 := 4(1 − + ) = 3.466666667,
3 5
8. A APROXIMAÇÃO DE π DADA POR LEIBNIZ 360

1 1 1 1
s5 = 4(1 −+ − + ) = 3.339682540, . . .
3 5 7 9
enquantos as somas parciais de ordem par são menores que π e crescem:
1 1 1 1
s2 := 4(1 − ) = 2.666666667, s4 := 4(1 − + − ) = 2.895238095,
3 3 5 7
1 1 1 1 1
s6 := 4(1 − + − + − ) = 2.976046176, . . .
3 5 7 9 11
Queremos provar que uma fila sn vai toda para algum lugar deter-
minando quando n cresce. Se mostro que as posições pares s2n a fila
vão para o lugar L e se mostro que as posições ı́mpares s2n+1 também
vão para esse lugar L, então a fila toda vai.
É isso que queremos verificar, pois queremos mostrar que para
1 1 1
sn := 4(1 − + + . . . + (−1)n )
3 5 2n − 1
existe
lim sn = L.
n→+∞

Reparando no formato das somas sn , vemos que para n ≥ 2:


• s2n+1 < s2(n−1)+1 pois
1 1
s2n+1 = s2(n−1)+1 − 4( − )
2(2n + 1) − 3 2(2n + 1) − 1
e portanto as somas parciais ı́mpares s2n+1 formam elas mes-
mas uma sequência decrescente,
• s2n > s2(n−1) pois
1 1
s2n = s2(n−1) + 4( − )
2n − 3 2(2n) − 1
e portanto as somas parciais pares s2n+1 formam elas mesmas
uma sequência crescente.
• s2n ≤ s1 = 4 e s2 = 4(1 − 31 ) < s2n+1
Logo o Teorema 1.1 aplicado separadamente às sequências (s2n )n e
(sn+1 )n , diz que ambas convergem:
lim s2n = L1 e lim s2n+1 = L2 .
n→+∞ n→+∞

Mas para terminar note que L1 = L2 pois


4
| s2n+1 − s2n | =
2(2n + 1) − 1
e
4
lim = 0.
n→+∞ 2(2n + 1) − 1
CAPÍTULO 28. APROXIMAÇÃO DE NÚMEROS E FUNÇÕES
IMPORTANTES 361

9. Aproximações de logaritmos
Se |x| < 1 então 1 + x > 0 e posso tomar ln(1 + x). Pela regra da
composta:
1
ln(1 + x) ′ = .
1+x
Agora escrevo:
1 1
=
1+x 1 − (−x)
e uso a Afirmação 2.1 para x ∈ (−1, 1):
1
= 1 − x + x2 − x3 + . . . , se |x| < 1.
1 − (−x)
O Teorema Fundamental do Cálculo dá:
Z x
1
dt = ln(1 + x) − ln(1 + 0) = ln(1 + x)
0 1+t

Vamos ser novamente otimistas novamente e supor que a integral de


uma soma infinita é uma soma infinita de integrais5, obtendo então:
Z x
x2 x3 x4
ln(1 + x) = (1 − t + t2 − t3 + . . .) dt = x − + − . . . , |x| < 1.
0 2 3 4

As Figuras a seguir comparam o gráfico real de ln(1+x) : (−1, 1) →


2
R com os gráficos dos truncamentos y = x : (−1, 1) → R, y = x − x2 :
2 3
(−1, 1) → R e x − x2 + x3 : (−1, 1) → R.
Para que os gráficos ficassem mais destacados não usei a mesma
escala nos eixos x e y:

1
x
-0,8 -0,4 0 0,4 0,8
0

-1

-2

-3

-4

Figura: O gráfico de y = ln(1 + x) (vermelho) e y = x (verde)


para x ∈ [−0.99, 0.99].

5Justificado na Afirmação 2.1 do Capı́tulo 29


10. APROXIMAÇÃO DE LOGARITMOS DE NÚMEROS
QUAISQUER 362

x
-0,8 -0,4 0 0,4 0,8
0

-1

-2

-3

-4

x2
Figura: O gráfico de y = ln(1 + x) (vermelho) e y = x − 2
(verde)
para x ∈ [−0.99, 0.99].

x
-0,8 -0,4 0 0,4 0,8
0

-1

-2

-3

-4

x2 x3
Figura: O gráfico de y = ln(1 + x) (vermelho) e y = x − 2
+ 3
(verde)

10. Aproximação de logaritmos de números quaisquer


Agora vamos ver o que fazer para aproximar ln(z) de um número
z > 0 qualquer.
Se |x| < 1 então 1 − x > 0 e posso tomar ln(1 − x). Pela regra da
derivada da composta:
1 −1
ln(1 − x) ′ = (−1) =
1−x 1−x
Se |x| < 1 escrevo pela Afirmação 2.1:
1
= 1 + x + x2 + x3 + . . . , se |x| < 1
1−x
e se pode também escrever (ver Afirmação 2.1 da Seção 29):
−1
= −1 − x − x2 − x3 − . . . , se |x| < 1.
1−x
Pelo Teorema Fundamental:
Z x
−1
ln(1 − x) − ln(1 − 0) = ln(1 − x) = dt,
0 1−t
CAPÍTULO 28. APROXIMAÇÃO DE NÚMEROS E FUNÇÕES
IMPORTANTES 363

e se formos otimistas trocaremos a integral de uma soma infinita pela


soma de infinitas integrais (ver Afirmação 2.1 do Capı́tulo 29):
Z x
2 3 x2 x3
ln(1 − x) = (−1 − t − t − t − . . .) dt = −x − − . . . |x| < 1.
0 2 3

Agora vamos precisar de um truque:

Afirmação 10.1. Todo número z > 0 se escreve de modo único


como:
1+x
z= , com |x| < 1.
1−x
Demonstração.
Dado z > 0 quero resolver em x a equação:
1+x
= z.
1−x
Para isso faço z · (1 − x) = 1 + x, logo −zx − x = 1 − z, ou seja,
−x(1 + z) = 1 − z e daı́:
z−1
x= .
z+1
Note que x < 1 pois z − 1 < z < z + 1.
Também note −1 < x pois −(z + 1) = −z − 1 < z − 1, já que 0 < z.
Ou seja, |x| < 1. 

Usando dessa Afirmação e da propriedade do logaritmo do quo-


ciente, escrevo:
1+x
ln(z) = ln( ) = ln(1 + x) − ln(1 − x) z > 0, |x| < 1
1−x
e portanto, pelo que já vimos:

x2 x3 x4 x2 x3
ln(z) = (x − + − . . .) − (−x − − . . .), |x| < 1.
2 3 4 2 3
Se as somas acima fossem finitas, poderı́amos subtrair termo a termo.
Sejamos otimistas e imaginemos que podemos subtrair termo a termo
nas somas infinitas (ver Afirmação 1.1 do Capı́tulo 29), obtendo (já
que os termos de grau par se cancelam):

x3 x5 z−1
ln(z) = 2(x + + + . . .), onde z > 0, x= , |x| < 1
3 5 z+1
12. EXERCÍCIOS 364

0
10 20 30 40 50
z

Figura: O gráfico de y = ln(z) (vermelho), z ∈ [0.5, 50], y = 2x (verde)


3 3 5
y = 2(x + x3 ) (amarelo) e y = 2(x + x3 + x5 ) (azul), onde x = z−1
z+1
.

11. Aproximação de ln(2)


Lembro que só usando a definição já sabı́amos que
1
< ln(2) < 1.
2
z−1 1
Com os resultados anteriores, para z = 2 e portanto x = z+1 = 3
,
obtemos ln(2) com a precisão que quisermos:
1 11 11 11
ln(2) = 2( + 3
+ 5
+ . . .).
3 33 53 7 37
Meu computador aproxima ln(2) ≈ 0.6931471806.
Enquanto isso, obtenho:
1 1 11
s1 := 2( ) = 0.6666666667, s2 := 2( + ) = 0.6913580247
3 3 3 33
1 11 11
s3 := 2( + 3
+ ) = 0.6930041152
3 33 5 35
1 11 11 11
s4 := 2( + 3
+ 5
+ ) = 0.6931347573.
3 33 53 7 37
1 11 11 11 11
s5 := 2( + 3
+ 5
+ 7
+ ) = 0.6931460474
3 33 53 73 9 39
1 11 11 11 11 1 1
s6 := 2( + + + + + ) = 0.6931470738.
3 3 33 5 35 7 37 9 39 11 311
12. Exercı́cios
Exercı́cio 12.1. Obtenha uma sequência definida recursivamente
que tende para a raı́z cúbica de A. Para isso:
i) levante (x0 , 0) verticalmente no gráfico de y = x3 − A
ii) encontre a tangente ao gráfico de y = x3 − A no ponto obtido
em i),
iii) desça pela tangente até encontrar o eixo x, determinando x1 e
assim sucessivamente.
CAPÍTULO 28. APROXIMAÇÃO DE NÚMEROS E FUNÇÕES
IMPORTANTES 365

iv) teste a sequência obtida, numericamente, numa calculadora.


CAPı́TULO 29

Séries numéricas e de funções

1. Séries de números
Um série infinita é uma soma infinita:
x1 + x2 + x3 + . . .
O sentido preciso dos três pontinhos é o seguinte: considere uma soma
parcial de orde n:
sn := x1 + x2 + . . . + xn .
Quando cresce o n os números sn forma eles mesmos uma sequência
infinta (sn )n . Então
x1 + x2 + x3 + . . . := lim sn ,
n→+∞

que pode existir ou não.

Quando existe esse limite dizemos que a soma infinita x1 + x2 +


x3 + . . . converge e quando não existe dizemos que x1 + x2 + x3 + . . .
diverge.

O sı́mbolo x1 + x2 + x3 + . . . não é muito conciso, por isso uso:


n
X +∞
X
sn := xi , e x1 + x2 + x3 + . . . = xi .
i=1 i=1

A Afirmação a seguir justifica alguns dos truques usados nas Seções


anteriores:

P+∞ão 1.1.
Afirmaç P
i) Se i=1 xi converge e C ∈ R então +∞
i=1 C · xi também converge
e
+∞
X +∞
X
C · xi = C · xi .
i=1 i=1
P P+∞
ii) Se +∞ x i e y são duas séries convergentes então também
i=1 Pi=1 i P+∞
convergem as séries +∞i=1 (xi + yi ) e i=1 (xi − yi ) e ademais:
+∞
X +∞
X +∞
X
(xi + yi ) = xi + yi ,
i=1 i=1 i=1
367
1. SÉRIES DE NÚMEROS 368

+∞
X +∞
X +∞
X
(xi − yi ) = xi − yi .
i=1 i=1 i=1
P
iii) Sejam xi > 0 e yi > 0. Se xi ≤ yi ∀i ∈ N e se +∞ i=1 yi converge
P+∞
então também coverge i=1 xi converge
P P+∞
iv) Se +∞i=1 |xi | converge então i=1 xi . A recı́proca não é ver-
dadeira.
Demonstração.
P +∞
De i): Como i=1 xi converge, então existe
n
X
lim sn = L, onde sn := xi .
n→+∞
i=1

Mas pelas propriedades de limites de sequências:


+∞
X
lim C · sn = C · lim sn := C · xi
n→+∞ n→+∞
i=1

Pela distributividade do produto e soma (finita)


n
X n
X
C · sn := C · xi = C · xi ,
i=1 i=1

e portanto
+∞
X
lim C · sn = C · xi ,
n→+∞
i=1
como querı́amos.
De ii): P P
Denoto por sxn := ni=1 xi e syn := ni=1 yi . Temos por hipótese que
existem
lim sxn = L1 e lim syn = L2 .
n→+∞ n→+∞

Então pelas propriedades de soma/diferença de sequências, aplicadas


às sequências (sxn )n e (syn )n , temos:
lim (sxn ± syn ) = lim sxn ± lim syn ,
n→+∞ n→+∞ n→+∞

que é o que queremos provar.


De iii): Sem entrar m muitos P detalhes,a idéia é que se consegui
somar as infinitas parcelas de +∞ i=1 yi com mais razão poderei somas
P+∞
as infinitas parcelas de i=1 xi , já que xi ≤ yi .
De iv): Sem entrar em detalhes que se vêem em textos de Análise
Matemática, o que posso dizer é que se conseguimos somar todos os
módulos |xi | > 0 é razoável que consigamos também somar as parcelas
xi , já que nessas há mudanças de sinais de > 0 para < 0, que produzem
subtrações e cancelamentos.
CAPÍTULO 29. SÉRIES NUMÉRICAS E DE FUNÇÕES 369

Sobre a recı́proca : a série 1− 12 + 31 − 14 +. . . converge (e o argumento


é análogo ao que usamos na aproximação de π). Mas como vimos na
prova da Afirmação 4.1, 1 + 12 + 13 + 41 + . . . fica tão grande quanto
quisermos.


2. Séries de funções
Agora precisamos justificar que, sob certas condições, a integral
de uma soma infinita é a soma infinita de integrais. Por exemplo, o
otimismo:
Z x
x2 x3
(−1 − t − t2 − t3 − . . .) dt = −x − − . . . |x| < 1,
0 2 3
que podemos reescrever, se preferirmos, numa nova notação:
Z xX+∞ +∞ Z x
X
i
−t dt = −ti dt =
0 i=0 i=0 0

+∞
X −xi+1
= , |x| < 1.
i=0
i+1
Esta última expressão é uma série infinita, mas que depende de
cada x com |x| < 1 para dar um valor determinado.
Por isso se chama série infinita de funções, e pode ser pensada como
uma fábrica de séries de números, pois:
+∞
X −xi+1
x 7−→ ∈ R,
i=0
i+1
desde que |x| < 1.
Esse é só um exemplo, em geral uma série infinita de funções é algo
do tipo:
+∞
X
fi (x)
i=0
e o principal problema é saber para quais x as séries numéricas
+∞
X
x 7−→ fi (x)
i=0
convergem.
No que segue nos limitaremos apenas a funções
fi (x) = ai xi
onde ai são números (chamadas séries de potências).
2. SÉRIES DE FUNÇÕES 370
P+∞
Afirmação 2.1. Suponha uma série de funções i=1 ai ti tal que
para um certo t = x > 0 convirja a série numérica:
+∞
X
|ai ||xi |.
i=1

Então:
• convergem também as séries
+∞
X +∞
X
|ai ti | e ai ti , ∀t ∈ [−x, x].
i=1 i=1

• A função
+∞
X
f : [−x, x] → R, f (t) := ai ti
i=1

é integrável e
Z x X+∞ +∞ Z x +∞
i
X
i
X ai i+1
ai t dt = ai t dt = x .
0 i=1 i=1 0 i=1
i+1
Demonstração.
Temos para |t| ≤ x:
+∞
X +∞
X +∞
X
i i
|ai t | = |ai ||t | ≤ |ai |xi |
i=1 i=1 i=1

e está última série converge por hipótese. P+∞


Então também convergem as séries numéricas i=1 |ai ti |, obtidas
escolhendo t com |t| ≤ x (para cada t, aplique a Afirmação 1.1 item
iii)). P
Então para cada t escolhido com |t| ≤ x convergem +∞ i
i=1 ai t (para
cada t, aplique a Afirmação 1.1 item iv)).
Logo a função
+∞
X
f : [−x, x] → R, f (t) := ai ti
i=1

está bem definida.


A integrabilidade dessa f se explica nos textos de Análise Matemática.
Me concentrarei apenas em mostrar que
Z x +∞ Z x
X
f (t) dt = ai ti dt,
0 i=1 0

ou seja que
Z x n Z
X x
f (t) dt = lim ai ti dt,
0 n→+∞ 0
i=1
CAPÍTULO 29. SÉRIES NUMÉRICAS E DE FUNÇÕES 371

ou ainda (já que integral de soma finita é a soma finita de integrais)


que
Z x Z x X n
f (t) dt = lim ( ai ti ) dt.
0 n→+∞ 0 i=1

Para isso tenho que mostrar que:

dado ǫ > 0 qualquer, se n for suficientemente grande, então


Z x Z x Xn
| f (t) dt − ( ai ti ) dt | < ǫ.
0 0 i=1

Ora, do item ix) do Teorema 4.1, Capı́tulo 19:


Z x Z x Xn Z x n
X
i
f (t) dt − ( ai t ) dt = (f (t) − ai ti ) dt.
0 0 i=1 0 i=1

Pelo item viii) do Teorema 4.1, Capı́tulo 19:


Z x n
X Z x n
X
i
| (f (t) − ai t ) dt | ≤ | f (t) − ai ti | dt.
0 i=1 0 i=1
P+∞
Agora, por definição f (t) := i=1 ai ti , logo
n
X +∞
X
f (t) − ai ti = ai ti
i=1 i=n+1

e portanto
n
X +∞
X
i
| f (t) − ai t | = | ai ti | ≤
i=1 i=n+1

+∞
X +∞
X
i
≤ |ai ||t | ≤ |ai ||xi |, se |t| ≤ x
n+1 n+1
P
O que vem a ser esse termo +∞ i
n+1 |ai ||x | ?
P+∞
Se denoto n+1 |ai ||xi | = L, então
+∞
X n
X
i
|ai ||x | = L − |ai ||xi |.
i=n+1 i=1
Pn
Mas as somas parciais sn := i=1 |ai ||xi | convergem para o limite L,
logo
+∞
X
|ai ||xi | = L − sn
i=n+1
2. SÉRIES DE FUNÇÕES 372

se faz tão pequeno quanto quisermos, se n cresce o suficiente. Posso


tomar n tal que
+∞
X ǫ
|ai ||xi | < , onde x > 0.
i=n+1
x
Em conclusão:
Z x Z x n
X
| f (t) dt − ( ai ti ) dt | ≤
0 0 i=1
Z +∞
x X
≤ |ai ||xi | dt ≤
0 i=n+1
Z x
ǫ ǫ
≤ dt = · x = ǫ,
0 x x
se n cresce o suficiente. Era o que querı́amos demonstrar.

CAPı́TULO 30

O discriminante de polinômios de grau 3

Neste Capı́tulo nos perguntamos sobre raı́zes múltiplas de polinômios.


Ou seja pontos x ∈ R onde não somente o polinômio y = f (x) se anula
mas onde há tangência do gráfico com o eixo dos x. Ou seja, pontos
onde também valha f ′ (x) = 0.
No caso de um polinômio de grau 2, f (x) = ax2 + bx + c, o sistema

f (x) = f ′ (x) = 0

significa:
ax2 + bx + c = 0 e 2ax + b = 0.
−b
Da segunda equação temos x = 2a
e substituindo na primeira obtemos:

ab2 b2 b2 − 4ac
0= 2 − +c=
4a 2a 4a2
ou seja, obtemos que onde há raı́z dupla x é onde há a anulação do
discriminante:
b2 − 4ac = 0.
A conhecida fórmula de Báskara dá a localização da raı́z dupla: x = −b
2a
O objetivo deste Capı́tulo é explicar que há um discriminante de
polinômios de grau 3 e que sua anulação determina a existência de uma
raı́z Real dupla dos polinômiso de grau 3.

1. Quase a fórmula de Cardano


Consideremos um polinômio de grau exatamente 3, que após divisão
pelo seu coeficiente de grau 3 pode ser escrito como:

f (x) = x3 + a1 x2 + a2 x + a3 , ai ∈ R.

É muito útil a mudança de coordenada


a1
x=x− .
3
Em termos geométricos x = x − a31 desloca o gráfico horizontal-
mente, como mostra a figura a seguir:
373
1. QUASE A FÓRMULA DE CARDANO 374

20

10

x
-3 -2 -1 0 1 2
0

-10

-20

Figura: Os gráficos de y = x3 + 3x2 e de y = (x − 1)3 + 3(x − 1)2 .

Mas em termos algébricos a mudança x = x− a31 produz o polinômio


a seguir, livre de monômio de grau 2:
a21 a1 a2 2a3
f (x) = x3 + (a2 − )x − + a3 + 1 .
3 3 27
Essa notação está pesada, por isso volto a usar como variável x e ponho
a21 a1 a2 2a31
b = a2 − a=− + a3 + .
3 3 27
Ou seja que podemos nos restringir a considerar:
f (x) = x3 + bx + a.
Afirmação 1.1. Seja um polinômio de grau 3 da forma
f (x) = x3 + bx + a
(sem termo quadrático).
Então

i) f (x) tem uma raı́z múltipla (dupla ou tripla) se e somente se


4b3 + 27a2 = 0.

ii) Se vale i) então a raı́z simples é


r
−a
x1 = 2 3
2
e a raı́z dupla é
r
−a
x2 = − 3 .
2
Se vale i), as raı́zes dupla e simples coincidem exatamente quando
a = b = 0.
CAPÍTULO 30. O DISCRIMINANTE DE POLINÔMIOS DE
GRAU 3 375

Demonstração.
Primeiro provemos que 4b3 + 27a2 = 0 é condição necessária, se
existe raı́z múltipla.
Analisar as raı́zes Reais múltiplas de f (x) = x3 + bx + a é analisar
x onde
f (x) = f ′ (x) = 0,
o que significa resolver o sistema:
x3 + bx + a = 0 3x2 + b = 0.
A segunda
b = −3x2
e substituindo na primeira obtemos:
−2x3 + a = 0
ou seja
a = 2x3 .
Então
b3 = −27x6 e a2 = 4x6
ou seja, que temos a anulação do seguinte discriminante:
4b3 + 27a2 = 0.
Agora vamos ver que 4b3 + 27a2 = 0 nos permite encontrar as raı́zes
de f (x) = x3 + bx + a e determianr qual é a múltipla.
Começo com a fórmula do binômio:

(v + u)3 = v 3 + 3v 2 u + 3vu2 + u3 =
= v 3 + u3 + 3uv(u + v).
Portanto posso escrever a identidade:
(v + u)3 − 3uv(v + u) − (u3 + v 3 ) ≡ 0.
Pensemos por um momento em x = v+u e busquemos v, u satisfazendo:
−3uv = b, e − (u3 + v 3 ) = a.
Se conseguimos estas duas últimas condições então
(v + u)3 − 3uv(v + u) − (u3 + v 3 ) ≡ 0
diria que x = v + u seria raı́z de
x3 + bx + a = 0.
Ora, a primeira condição:
−3uv = b,
dá (supondo u 6= 0)
−b
v=
3u
1. QUASE A FÓRMULA DE CARDANO 376

e, substituindo isso na segunda, u3 + v 3 = −a, obtemos:


−b3
u3 + = −a.
27u3
Se multiplicamos isso tudo por u3 , obtemos uma equação:
b3
u6 + au3 − = 0.
27
Note que esta equação é do tipo:
3 2 3 b3
(u ) + a(u ) − = 0,
27
ou seja , uma equação quadrática na nova variável u3 .
Portanto as raı́zes u3 podem ser descobertas pela fórmula de Báskara:
q
3
−a ± a2 − 4 −b 27
u3 = =
2
q
4a2 3
−a 4
+ 4b27
= ± =
2 r 2
−a a2 b3
= ± + .
2 4 27
1
Logo s r
3 −a a2 b3
u= ± +
2 4 27
2 3
Estamos supondo 27a + 4b = 0, o que dá no mesmo que
a2 b3
+ = 0.
4 27
Logo obtenho r
−a
u= 3
2
e a condição v 3 + u3 = −a dá
r
−a
v= 3 .
2
Logo
x=v+u=
r
−a
=2· 3 .
2
q
Esse ponto x1 = 2· 3 −a 2
é raı́z de f (x) = x3 +bx+a, mas é raı́z simples
se a 6= 0.
1Se continuamos poderemos chegar na fórmula de Cardano para as raı́zes, mas
para entender essa fórmula em detalhe é preciso lidar com números complexos,
fugindo ao objetivo do Curso, que é Cálculo em variável Real
CAPÍTULO 30. O DISCRIMINANTE DE POLINÔMIOS DE
GRAU 3 377

Observe agora que se denoto por x1 , x2 , x3 as raı́zes Reais ou com-


plexas de f (x) = x3 + bx + a, podendo ser repetidas no caso múltiplo
(xi = xj ) temos:
x1 + x2 + x3 = 0.
Isso é fácil de se ver, pois se escrevo:
x3 + bx + a = (x − x1 )(x − x2 )(x − x3 ) =

= x3 + (−x1 − x3 − x2 ) · x2 + (x1 x3 + x1 x2 + x2 x3 ) · x − x1 x2 x3 ,
temos que concluir que x1 + x2 + x3 = 0.
Ou seja, no caso de raı́z dupla x2 temos que x1 + x2 + x2 = 0, ou
seja,
−x1
x2 = .
2
Verifiquemos então que o ponto
r
−x1 −a
x2 = =−3
2 2
é de fato raı́z dupla de f (x) = x3 + bx + a, calculando primeiro f (x)
nesse ponto:
r r
3 −a 3 −a
(− ) + b(− 3 )+a=
2 2
r r
a 3 27 a4 3 −a
= − − +a=
2 4 2
r
a 3
3 27 a a 3a
= − +a= − + a = 0.
2 8 2 2
E a seguir calculando f ′ (x) nesse ponto:
r r
2
3 −a 2 3 a
3( − ) +b=3 +b=
2 4
r
3
3 −b
3 + b = −b + b = 0
27
a4 b3
Calor que se a = 0 e 4
+= 0 então b = 0 e f (x) = x3 tem raı́z
27 q
tripla em x = 0. E também é claro que se a raı́z dupla − 3 −a
2
coincide
q
com a raı́z simples 2 3 −a
2
então a = 0.

2. O DISCRIMINANTE COMO CURVA 378

2. O discriminante como curva

Vamos interpretar geometricamente a Afirmação 1.1.


Pensemos num plano cujas coordenadas são (a, b) e o lugar de an-
ulação 4b3 + 27a2 = 0. Isso define uma curva Γ no plano (a, b).
O traço da curva Γ : 4b3 + 27a2 = 0 é dado na Figura a seguir:

-0,2 -0,1 0 0,1 0,2


0

-0,1

-0,2

-0,3

-0,4

-0,5

-0,6

-0,7

Note que a imagem de

γ : R → R2 = (a, b), γ(t) := (2t3 , −3t2 )

satifaz

4( −3t2 )3 + 27( 2t3 )2 ≡ 0.

Por isso γ(t) é chamada de parametrização de Γ : 4b3 + 27a2 = 0.


Ou seja:

todas as cúbicas do tipo y = ft (x) = x3 −3t2 x+2t3 têm raı́z múltipla.

Pela Afirmação 1.1 a localização da raı́z dupla é

r
3 −2t3
x2 = − = t,
2

enquanto a raı́z simples é

r
3 −2t3
x1 = 2 = −2t.
2

Fiz quatro Exemplos na Figura a seguir:


CAPÍTULO 30. O DISCRIMINANTE DE POLINÔMIOS DE
GRAU 3 379

40

20

0
-4 -2 0 2 4
x
-20

-40

Figura: Gráficos de de y = ft (x) = x3 − 3t2 x + 2t3 , com t = −2, −1, 1, 2

Quando t → 0 a raı́z dupla de y = ft (x) = x3 − 3t2 x + 2t3 colide


com a terceira raı́z simples, formando a raı́z tripla de y = f0 (x) = x3 .
Veja a Figura a seguir:

60

40

20
x
-4 -2 0 2 4
0

-20

-40

-60

Figura: Gráficos de de y = ft (x) = x3 − 3t2 x + 2t3 , com t = −1, −1


2
, −1
4

A curva discriminante Γ separa o plano (a, b) em duas regiões, uma


onde 4b3 + 27a2 < 0, e que está acima da curva na Figura. Na figura
a seguir escolhi 4 pontos (a, b) nessa região e plotei as cúbicas y =
x3 + bx + a resultantes:
3. A CURVA DISCRIMINANTE ENTRE AS CÚBICAS
SINGULARES 380

100

50

0
-4 -2 0 2 4
x
-50

-100

A outra região do plano, determinada pela Γ, é onde 4b3 +27a2 > 0,


e que fica abaixo da curva na Figura. Na figura a seguir escolhi 4 pontos
(a, b) nessa região e plotei as cúbicas y = x3 + bx + a resultantes:

800

400

0
-10 -5 0 5 10
x

-400

-800

3. A curva discriminante entre as cúbicas singulares


Os pares ordenados de parâmetros (a, b) formam um plano, que será
para nós agora um plano (x, y). Vamos escolher novas coordenadas
(x, y) nesse plano, para que a curva discriminante
4y 3 + 27x2 = 0
seja dada por:
y 2 − x3 = 0,
√ √
Basta fazer uma mudança do tipo y := 27 · x e x := − 3 4 · y.
CAPÍTULO 30. O DISCRIMINANTE DE POLINÔMIOS DE
GRAU 3 381

Definição 3.1. Um ponto P = (x, y) é uma singularidade de uma


curva F (x, y) = 0 se nesse ponto
∂F (x, y) ∂F (x, y)
F (x, y) = = = 0.
∂x ∂y
Em geral as cúbicas apresentadas na Seção 3 do Capı́tulo 15 são da
forma:
y 2 = x3 + b x + a,
ou
F (x, y) = y 2 − x3 − b x − a = 0.
Logo para termos singularidades dessas cúbicas temos que ter:
y 2 − x3 − b x − a = 0, y=0 e − 3x2 − b = 0,
ou seja (já que o sinal não vai importar):
x3 + b x + a = 0 e 3x2 + b = 0.
Se denoto f (x) = x3 + b x + a, as singularidades terão coordenada x
verficando:
f (x) = f ′ (x) = 0,
quer dizer, raı́z multipla de f (x) = 0.
Mas então estamos recaindo no que aprendemos na Afirmação 1.1:

a condição para termos singularidades nas cúbicas y 2 = x3 + b x + a


é dada por 4b3 + 27 a2 = 0.

A Figura a seguir é o que o Maple consegue plotar da cúbica


y 2 − x3 + 3 x − 2 = 0,
que tem singularidade, pois 4 · (−3)3 + 27 · 22 = 0.
De fato o formato correto é o de um laço e a singularidade é o ponto
(1, 0).

y 0
-2 -1 0 1 2 3
x
-2

-4

-6
3. A CURVA DISCRIMINANTE ENTRE AS CÚBICAS
SINGULARES 382

Figura: A curva y 2 − x3 + 3 x − 2 = 0.

A Figura a seguir é como o Maple plota a curva

y 2 − x3 + 3 x + 2 = 0,

que tem singularidade pois 4 · (−3)3 + 27 · (−2)2 = 0.

y 0
2 2,4 2,8 3,2 3,6
x
-2

-4

-6

Figura: Atenção: esta curva y 2 − x3 + 3 x + 2 = 0


tem um ponto isolado em (−1, 0), que é a singularidade !

De fato, (−1, 0) está na curva, y 2 − x3 + 3 x + 2 = 0, pois esta é:

y 2 − (x + 1)2 · (x − 2) = 0.

Ademais ∂F
∂y
= 2y e ∂F
∂x
= −3x2 + 3 se anulam em (−1, 0).
Os dois últimos exemplos são casos da seguinte situação:

Afirmação 3.1. Suponha y 2 = f (x) = x3 + bx + a com (a, b) 6=


(0, 0) e 4 b3 + 27 a2 = 0.
2
• i) Se
q a < 0 então y = f (x) tem um ponto singular isolado em
(− 3 −a
2
, 0) e todos os outros pontos da curva tem coordenada
q
x ≥ 2 3 −a
2
.
2
• ii) Se a > 0 então yq = f (x) tem forma de laço com singular-
idade no ponto ( − 3 −a2
, 0 ).

Demonstração.
Se f (x) = x3 + bx + a tem

(a, b) 6= (0, 0) e 4b3 + 27 a2 = 0,


CAPÍTULO 30. O DISCRIMINANTE DE POLINÔMIOS DE
GRAU 3 383

então a Afirmação 1.1 diz que f (x) tem uma raı́z dupla e uma simples,
bem como que a raı́z simples é
r
−a
x1 = 2 3
2
enquanto que a raı́z dupla é
r
−a
x2 = − 3 .
2
Logo no caso i):
a > 0 ⇒ x1 < x2 ,
enquanto que, no caso ii):
a<0 ⇒ x2 < x1 .

Caso i): como a < 0,


∂F ∂F
= 2y e = 3x2 + b
∂y ∂x
q
se anulam em (− 3 −a2
, 0), pois
r r
3 −a 2 −a 2 b
3( − ) +b=0 ⇔ ( 3 ) =− ⇔
2 2 3
a2 b3
⇔ =− ⇔ 27 · a2 = −4 · b3 .
q 2 27
Logo (− 3 −a
2
, 0) é singularidade, cuja coordenada x negativa.
Note que
f (x) = x3 + bx + a = (x − x2 )2 · (x − x1 ).
Como y 2 = f (x), é necessário que
r
3 −a
x ≥ x1 = 2
2
para termos números Reais
p p
y = (x − x2 )2 · (x − x1 ) ou y = − (x − x2 )2 · (x − x1 ).
q
Ou seja, fora o ponto (− 3 −a2
, 0) todos os outros pontos dessa curva
q
tem coordenada x ≥ 2 3 −a2
.

Caso ii): No caso a > 0 a verificação de que (x2 , 0) é ponto singular


de y 2 = f (x) é idêntica. O ponto (x1 , 0) não é singular para a curva,
que tem tangente vertical neste ponto.
Agora, neste caso, como x1 < x2 e
f (x) = (x − x1 ) · (x − x2 )2 ,
3. A CURVA DISCRIMINANTE ENTRE AS CÚBICAS
SINGULARES 384

basta que x ≥ x1 para que estejam definidas nos Reais as raı́zes:


p p
y = (x − x2 )2 · (x − x1 ) ou y = − (x − x2 )2 · (x − x1 ).
As duas opções distintas de raı́zes se colapsam para o valor y = 0 em
x = x1 . São distintas raı́zes no intervalo (x1 , x2 ), pois nesse intervalo
(x − x2 )2 · (x − x1 ) > 0.
E voltam a se colapsar para o valor y = 0 em x = x2 . Para x > x2 há
novamente duas opções distintas de raı́zes para y. Por isso se forma o
laço em (x2 , 0).

CAPı́TULO 31

3
Apêndice: O expoente 4 comanda a vida !

Neste capı́tulo dou uma aplicação à Biologia do logaritmo, da série


geométrica e da teoria de mı́nimos do Cálculo. Não sou nenhum es-
pecialista em bio-matemática, minha intenção é apenas mostrar como
conceitos matematicamente simples podem ser úteis em outras ciências.
Ademais, aqui exponho apenas um argumento para demonstrá-la,
que usa hipóteses fortes e na etapa final um tipo de limite no número
de nı́veis de ramificação do sistema circulatório.
Mas a lei de Kleiber se aplica até a seres unicelulares. Portanto
deve haver um argumento bem mais geral para demonstrá-la !
Minhas referências foram:
• R. Dawkins, A grande história da Evolução, Companhia das
Letras, 2009.
• J. West, J. Brown, B. Enquist, A general model for the origin
of allometric scaling laws in biology , Science, 1997.
• M. Kleiber, Body size and metabolic rate, Physiological Re-
views, vol. 27, n.4 , 1947.
• R. Etienne, M. Apol, H. Olff, Demystifying West, Brown, En-
quist model of the allometry of metabolism , Functional Ecol-
ogy, 2006.
Essencialmente o objetivo do Apêndice é apresentar algumas idéias
do último artigo.

1. Metabolismo versus massa corporal

Questão 1: Quem produz mais calor ao longo de dia, estando em


repouso, um homem ou um rato ?

Questão 2: Quem tem a maior taxa de produção de calor por


unidade de peso, um homem ou um rato ?

Os biólogos se interessam por essas questões, ou seja, entender a


relação entre o crescimento da massa corporal e o crescimento do
metabolismo basal dos organismos vivos.
O metabolismo basal B é essencialmente o consumo de oxigênio por
unidade de tempo (medido em kcal/dia).
385
3. RETA DE AJUSTE - MÉTODO DE MÍNIMOS QUADRADOS
386

Em 1883 Rubner propôs um modelo geométrico para explicar essa


relação:
• É preciso haver uma superfı́cie de área A para as trocas de O2
entre o organismo e o ambiente. Ou seja
B = τ1 · A,
(τ1 constante que não depende da massa).
• Por outro lado, a massa corporal M verifica
M = τ2 · V.
• Mas A = τ3 · L2 enquanto V = τ4 · L3 , onde L é uma medida
de comprimento.
Ou seja
B = τ 5 · L2 e M = τ 6 · L3 .
Pelo modelo de Rubner já se prevê que não pode aparecer de uma
hora para outra uma aranha - Godzilla. Ela se sufocaria antes de
destruir qualquer coisa !

2. Escalas log/log para um experimento


A massa de um elefante é 1021 vezes a massa de uma ameba. Por
isso, quando se plota M versus B se usa log10 (M ) versus log10 (B). Pois
então se poder desfrutar da propriedade:
log10 (ak ) = k · log10 (a).
Escolha agora o grupo de seres vivos que mais lhe agrada (caninos,
felinos, primatas, mamı́feros, aves, peixes, crustáceos, plantas, etc). De
preferência com bastante variabilidade de massa corporal.
Plote os pares ( log10 (M ) , log10 (B) ) obtidos por observação no
grupo de seres vivos escolhidos.
Suponha que voce tem então sua lista
( log10 (M1 ), log10 (B1 ) ), . . . , ( log10 (Mk ), log10 (Bk ) )

Agora o problema é definir a Reta que mais se ajusta a esses


pontos, pois é dela que trata a Lei de Kleiber.

3. Reta de ajuste - método de mı́nimos quadrados


Se o leitor já conhece esse conceito, pode ir para a Seção seguinte.
Chamo de distância vertical de um ponto (x, y) a uma reta y =
ax + b o número
p
|(ax + b) − y| = (ax + b − y)2 .
Como há uma raı́z quadrada, torna-se complicado derivar. Por isso
vamos elevar ao quadrado a distância e tentar minimizar o quadrado
da soma de distâncias verticais até uma reta.
3
CAPÍTULO 31. APÊNDICE: O EXPOENTE 4
COMANDA A
VIDA ! 387

Problema 2: Determinar reta y = ax + b que minimiza a soma dos


quadrados das distâncias verticais até k pontos dados.

Vamos mostrar apenas como obter um candidato a reta que min-


imiza a soma dos quadrados das distâncias. a verificação completa
depende de noções de Cálculo em duas variáveis.
Imagine para as retas a notação:
y = ξx + β,
já que os coeficientes angulares ξ e lineares β são os que queremos
determinar. O que quero dizer é que devemos pensar na função:

z = f (ξ, β) = (ξx1 + β − y 1 )2 + (ξx2 + β) − y 2 )2 + . . . (ξxk + β − y k )2 .


como função de duas variáveis ξ, β.
O gráfico de z = f (ξ, β) forma uma superfı́cie no espaço com coor-
denadas (ξ, β, z).

Figura: O gráfico de z = f (ξ, β)

O ponto (ξ0 , β0 ) que buscamos será um ponto de mı́nimo do gráfico


de z = f (ξ, β), portanto esperamos que ao intersectar essa superfı́cie
com os planos ξ = ξ0 e com β = β0 produzam gráficos de funções
z = f (ξ, β0 e z = f (ξ0 , β) que tenham pontos de mı́nimo.
Ou seja, esperamos que as derivadas de z = f (ξ, β0 ) e de z =
f (ξ0 , β) sejam zero em (ξ0 , β0 ). Ou seja, devemos parar a variável ξ e
derivar em β e vice-versa, e buscar pelos zeros dessas derivadas.
∂g
Quando paramos ξ = ξ0 e derivamos em β usamos o sı́mbolo ∂β .
∂g
Quando paramos β = β0 e derivamos em ξ usamos o sı́mbolo ∂ξ . Então
∂g
= 2(ξx1 + β − y 1 )x1 + 2(ξx2 + β) − y 2 )x2 + . . . 2(ξxk + β − y k )xk =
∂ξ
Xk Xk k
X
2
= 2 · (ξ ( xi ) + β ( xi ) − xi y i )
i=1 i=1 i=1
4. A LEI EXPERIMENTAL DE KLEIBER 388

∂g
= 2(ξx1 + β − y 1 ) + 2(ξx2 + β) − y 2 ) + . . . 2(ξxk + β − y k ) =
∂β
Xk k
X
= 2(ξ ( xi ) + k · β − y i ).
i=1 i=1
Fazendo
∂g ∂g
= =0
∂ξ ∂β
estamos criando um sistema não-homogêneo de duas equações lineares,
com duas incógnitas ξ, β:
Xk Xk k
X
2
ξ( xi ) + β( xi ) = xi y i ,
i=1 i=1 i=1

Xk k
X
ξ( xi ) + k · β = yi.
i=1 i=1
Podemos usar a Regra de Cramer para resolvê-lo, pois o determinante
formado com os coeficientes do sistema é:
Xk Xk
2
k·( xi ) − ( xi )2 > 0,
i=1 i=1

pelo item ii) da Afirmação 6.1 do Capı́tulo 11.


Obteremos por Cramer:
P P P
k · ki=1 xi y i − ( ki=1 xi )( ki=1 y i )
ξ0 = P P
k · ki=1 x2i − ( ki=1 xi )2
e P P P P
( ki=1 x2i )( ki=1 y i ) − ( ki=1 xi )( ki=1 xi y i )
β0 = P P
k · ki=1 x2i − ( ki=1 xi )2

4. A Lei experimental de Kleiber


Se verifica experimentalmente (com as ressalvas como k suficiente-
mente grande, etc) que:

(Lei de Kleiber - 1947) O coeficiente angular da reta de ajuste


independe do grupo de seres vivos escolhidos e vale 34 .

Observo que 34 < 1 implica que há uma lentificação do metabolismo,


à medida que a massa corporal aumenta.
Evidências:
• M. Kleiber se baseia numa tabela de k = 26 pontos, com Massa
M dada em kg e B dado em kcal/dia.
3
CAPÍTULO 31. APÊNDICE: O EXPOENTE 4
COMANDA A
VIDA ! 389

• A tabela analisa mamı́feros. Começa com dados do camundongo,


com (M, B) = (0.021, 3.6), passa por exemplo pelo gato (M, B) =
(3, 162) e vai até dados da vaca (M, B) = (435, 8166).
• Usando sua tabela, se obtém (conferi !) a0 = 0.7497881511 ∼
3
4
.
No livro de Dawkins (2004) a lei de Kleiber é aplicada em três
grupos:
• organismos unicelulares,
• organismos de sangue frio e
• de sangue quente.
Aı́ se vê que os coeficientes lineares b0 das retas de ajuste mudam
bastante.

Além disso, Dawkins usa a lei de Kleiber para estudar outra cor-
relação: massa corporal versus massa cerebral.

Das retas de ajuste log10 (B) = 43 log10 (M ) + b, obtemos:


3 3
B = 10b · M 4 = τ · M 4
onde τ depende do tipo de organismo (sangue frio x sangue quente, por
ex.)
Vou introduzir a notação
3
B ∝ M4
para dizer só nos interessa o expoente de M e expressar a Lei de Kleiber.
Para termos uma comparação, a seguir plotei y = x (vermelho),
2 3
y = x 3 (verde) e y = x 4 (amarelo), para x ∈ [1, 10]

10

2 4 6 8 10
x

5. Justificação racional da Lei de Kleiber


Até 1997 não havia nenhuma justificação teórica da lei experimen-
tal de Kleiber. Então o fı́sico West e os biólogos Brown e Enquist
6. O ARGUMENTO 390

trataram de provar a lei de Kleiber, em artigo publicado na Revista


Science.
A idéia deles foi de que a eficiência de um sistema metabólico está
intimamente relacionada à eficiência do sistema respiratório/circulatório.
A ”demonstração”’ deles se baseou em:
• hipóteses sobre a geometria do sistema circulatório.
• hipóteses da fı́sica de fluidos, sobre a eficiência do processo de
distribuição (ou seja, minimização das perdas, resistência, etc)
O artigo WEB teve um grande impacto. Em 2004, R. Dawkins diz:

(...) A Lei de Kleiber, seja para plantas, animais ou até mesmo no


nı́vel do transporte dentro de uma única célula, encontrou finalmente
sua base racional. Ela pode ser derivada da fı́sica e da geometria das
redes de suprimento.(...)
No entanto, houve crı́ticas. Fora debates sobre as ”contas”que fiz-
eram, criticou-se
• que há hipóteses fortes sobre a geometria dos sistema circu-
latório (algumas retomaremos mais adiante)
• que o postulado de eficiência do sistema circulatório parece
sugerir que a Evolução já acabou, já estarı́amos otimamente
adaptados ...
O artigo de Etienne, Apol e Olff, de 2006, esclarece quais as su-
posições de WBE, destaca pontos obscuros de WBE e permite dar
uma versão light de WBE.
Seguirei EAO, mas visando apenas explicar algumas das muitas
idéias de WBE, aquelas que dispensam a fı́sica dos fluidos.

6. O argumento
6.1. Hipótese 1. Hip. 1: Os sistemas circulatórios são árvores,
onde:
• Cada ramo de ordem k pode ser considerado um cilindro, de
comprimento lk , cuja base é um disco de raio rk .

r _k

l _k

• Há 1 =: N1 ramo de ordem 1 (a aorta), que se subdivide em


ν1 ≥ 2 ramos de ordem 2,
3
CAPÍTULO 31. APÊNDICE: O EXPOENTE 4
COMANDA A
VIDA ! 391

• cada ramo de ordem k se subdivide em νk ≥ 2 ramos de ordem


k + 1. Há Nk ramos de ordem k.

• Observe que
Nk N2
Nk = · ... · = νk−1 · . . . · ν1
Nk−1 1

6.2. Capilares.
• o processo de ramificação da aorta em artérias e depois arterı́olas
continua até ramos finais, chamados de capilares.
• cuja ordem na ramificação será designada por C e cujo número
total será NC .
• Saiba que as paredes dos capilares são unicelulares ! 0 diâmetro
externo de um capilar é de 5 a 10 µ m (micrômetros, 10−6 m).
• Nos capilares se dão os processos fı́sicos como difusão, osmose,
etc. Através dos quais oxigênio / nutrientes passam para os
tecidos enquanto gás carbônico/ dejetos passam para o sangue.
• esses dados dos capilares são praticamente universais.
• Se sabe que no ser humano há ≈ 20 bilhões de capilares.
• As hemáceas humanas tem 8 µ m de diâmetro. Para trafe-
garem pelos capilares elas formam fila indiana !
• Para se ver o grau de ramificação do sistema circulatório, a
aorta de uma baleia pode chegar a 23 cm de diâmetro.

6.3. Relação com os Capilares. Como νk := NNk+1 k


, defino analoga-
mente:
lk+1 rk+1
λk := e ρk := .
lk rk
Note que vale
rk+1 rC
rk · ρk · ρk+1 . . . · ρC−1 = rk · · ... · = rC ,
rk rC−1
Ou seja:
rC
rk = QC−1
i=k ρi
6. O ARGUMENTO 392

e exatamente do mesmo jeito se obtém:


lC NC
lk = QC−1 e Nk = QC−1
i=k λi i=k νi
Imagine cada ramo cheio de sangue ou de seiva (já pensamos em
sistemas não-pulsáteis ...)
Considere πrk2 · lk o volume de cada ramo de ordem k.
A soma de todos os volumes de ramos de nı́vel k é portanto:
NC · r2 · lC
Vs,k := Nk · (πrk2 · lk ) = π QC−1 C 2 .
i=k νi ρi λi

Logo o volume total no sistema


C
X
Vs := Vs,k
k=1

é:
C
X 1
Vs = πNC · rC2 · lC · ( QC−1 2
).
k=1 i=k νi ρi λi

6.4. Definição de S1 e de S2 . Para facilitar, chamar


C
X 1
S1 := QC−1 .
k=1 i=k νi ρ2i λi
Com essa nova notação temos:
Vs = πNC · rC2 · lC · S1 .
Considere
• Ak o quociente das somas de áreas de seções transversas dos
ramos
• Ek o quociente de somas de volumes de esferas cujos diâmetros
são o comprimento dos ramos.

2
Nk+1 πrk+1
Ak := 2
= νk · ρ2k ,
Nk πrk

Nk+1 34 π( lk+1 )3
Ek := 2
= νk · λ3k .
Nk 34 π( l2k )3
Essa esferas de volume 43 π( l2k )3 serão supostos os volumes servidos
pelos ramos, ou seja partes do corpo que recebem nutrientes dos ramos
cilı́ndricos de ordem k, de comprimento lk .
3
CAPÍTULO 31. APÊNDICE: O EXPOENTE 4
COMANDA A
VIDA ! 393

l _k

E agora defino outra grandeza:


C
X 1
S2 := QC−1 1 ,
1/3
k=1 Nk i=k Ai · Ei3
PC 1
Afirmação: S1 := k=1
Q C−1
νi ρ2i λi
pode ser escrito como:
i=k

1
S1 = NC3 · S2
1
De fato, como νi · ρ2i = Ai e λi = ( Eνii ) 3 :

C
X 1
S1 = QC−1 1 =
k=1 i=k Ai · ( Eνii ) 3

QC−1 1
C
i=k νi
X 3

= QC−1 1 =
k=1 i=k A i · Ei
3

1
C
X ( NNCk ) 3
= QC−1 1 =
k=1 i=k Ai · Ei 3

C
1 X 1
= NC ·3
1 QC−1 1
k=1 Nk 3
i=k Ai · Ei3
o que prova a Afirmação. Portanto:
4
Vs = π NC · rC2 · lC · S1 = π NC3 · rC2 · lC · S2 .

Ou seja:
3
Vs 4
NC = ( 2 )
πrC · lC · S2
6. O ARGUMENTO 394

6.5. Hipótese 2. A hipótese a seguir faz mais sentido para sis-


temas circulatórios não-pulsáteis. Mas tomemo-a para simplificar a
exposição.

Hip. 2 O metabolismo basal B é proporcional ao fluxo total pela


aorta Q1 :

B = τ Q1 ,
onde a constante τ não depende da massa M .
Se pode mostrar que a incompressibilidade do fluido (sangue/seiva)
implica:
Q1 = Nk Qk , ∀k = 1, . . . C,
onde Qk é fluxo em cada ramo de ordem k.
Logo:
B = τ NC QC
onde QC é o fluxo por cada capilar.

6.6. Hipótese 3. Obtemos da expresão anterior de NC :


3
Vs 4
B = τ QC ( 2 ) .
πrC · lC · S2
Lembre que Vs é o volume total (sangue/seiva).
Em mamı́feros, o volume de sangue ocupa 6 − 7
Há evidências experimentais para:

Hip. 3 Vs = ηM , onde η não depende da massa M .


Ou seja, do anterior obtenho:
3
M4
B ∝ QC 3 .
(rC2 · lC · S2 ) 4

6.7. Hipótese 4. Aqui retomamos o que já dissemos antes sobre


o caráter universal dos capilares:

Hip. 4 As grandezas QC , rC , lC não dependem da massa M .


• Esta hipótese tem evidências experimentais, diz por exemplo
que os dados dos capilares de uma baleia e de um rato são
essencialente os mesmos !
• Isso deve estar ligado ao fato de que, a partir dos capilares, o
sistema de distribuição só se baseia em processos fı́sicos uni-
versais, como a difusão.
• Ou visto de outro modo, que os sistemas circulatórios todos
começaram modestamente como redes capilares ...
3
CAPÍTULO 31. APÊNDICE: O EXPOENTE 4
COMANDA A
VIDA ! 395

• Porém o número de nı́veis C e NC claramente depende de M :


maior o animal, maior o número de etapas de ramificação e
maior o número de capilares.
6.8. S2 invariante. Ou seja, do anterior obtenho agora:
3
M4
B∝ 3 .
(S2 ) 4
EAO dão argumentos no sentido de que a dependência entre S2 e
M é negligenciável, o que concluiria a dedução da Lei de Kleiber.
Mas eu gostaria de seguir a exposição na linha do argumento orig-
inal de WBE, onde há algumas hipóteses (fortes) a mais, com con-
sequências sobre S2 .
6.9. Hipótese 5. A resistência ao fluxo de sangue/seiva fica diminuida
pela suposição (natural para o sistema circulatório de plantas):
Hip. 5 A soma das áreas das seções transversais é preservada a
cada ramificação.
Ou seja :
Ak = 1, ∀k = 1, . . . , C.
6.10. Hipótese 6. A hipótese a seguir diz uma soma de volumes
ao redor dos vasos permanece constante em cada etapa da subdivisão:

Hip. 6 As quantidades Nk · 43 π( l2k )3 são preservadas nas rami-


ficações.
Ou seja:
Ek ≡ 1, ∀k = 1, . . . C.
Esta última hipótse deu origem a muita controvérsia.

Como mostra EAO, as Hipóteses 5 e 6 são fortes, poderiam ser


enfraquecidas pois em
C
X 1
S2 = Q 1 ,
1/3 C−1
k=1 Nk i=k Ai · Ei
3

os Ai e Ei podem se compensar, mesmo que mudem a cada etapa.


6.11. Hipótese 7. Com as Hipóteses 5 e 6, S2 se reduz a:
C
X
S2 = Nk −1/3 .
k=1
A hipótese a seguir diz que ou sempre há dicotomias, ou sempre
tricotomias , etc:

Hipótese 7: νk = ν , ∀k = 1, . . . , C (onde o Natural ν ≥ 2 não


6. O ARGUMENTO 396

depende de M ).

6.12. Número de ramificações. Portanto da Hipótese 7,

Nk = ν k−1 , k = 1 . . . C.

Por exemplo, em seres humanos, NC ≈ 2 × 1010 . De

NC = ν C−1

obtemos:
ν = 2 ⇒ C ≈ 35 e ν = 3 ⇒ C ≈ 22.
Ou seja, chegamos da aorta ao capilar em 35 dicotomias !
Ou chegamos da aorta ao capilar em 22 tricotomias !

Voltando ao S2 , note que ele se transforma numa soma geométrica


(finita):
C
X
S2 = Nk −1/3 =
k=1

C
X −(k−1)
= ν 3 =
k=1

−C
1−ν 3
= −1 .
1−ν 3

6.13. S2 como função de C.


O número de nı́veis C depende de M .
Portanto precisamos ver que a dependência entre S2 e C é negli-
genciável.
O argumento de EAO é o seguinte: vamos plotar S2 como função
de C, bem como sua assı́ntota horizontal:
−C
1−ν 3 1
lim −1 = −1 ,
C→+∞ 1−ν 3 1−ν 3

−1
(que existe pois ν 3 < 1). E vejamos se a função S2 = S2 (C) se
aproxima rapidamente de sua assı́ntota. Se isso acontecer, a conclusão
será que a partir de uma certo C, S2 pouco muda com C.
Para ν = 2 obtemos y = S2 (C):
3
CAPÍTULO 31. APÊNDICE: O EXPOENTE 4
COMANDA A
VIDA ! 397

1
5 10 15 20 25 30 35
x

Note que a escala no eixo y é menor que no eixo x.

Para ν = 3 obtemos y = S2 (C):

2,5

1,5

1
5 10 15 20
x

Note que a escala no eixo y é menor que no eixo x.

A velocidade com que os gráficos se aproximam do limite é o que


EAO consideram ”dependência negligenciável”entre S2 e C.
E obtemos de 3
M4
B∝ 3
(S2 ) 4
o resultado: 3
B ∝ M 4.
CAPı́TULO 32

Soluções detalhadas de alguns Exercı́cios

0.14. Capı́tulo 2: Exercı́cio 9.6:


v) O enunciado não diz, mas de fato y > 0, pois x ∈ (0, 1) dá
x
1 − x2 > 0 e portanto y = 1−x 2 > 0.

Agora
x
y= ⇔ y · x2 + x − y = 0,
1 − x2
e precisamos resolver essa equação quadrática em x, para termos x =
x(y).
Ora, por Báskara as soluções são:
p p
−1 + 1 − 4y (−y) −1 + 1 + 4y 2
x1 = = ,
2y 2y
p
−1 − 1 + 4y 2
x2 = .
2y
Precisamos ficar com a solução que seja positiva, pois por hipótese
x
x ∈ (0, 1). Como y = 1−x 2 > 0 e a solução positiva é:
p
−1 + 1 + 4y 2
x := x1 = .
2y
Ou seja, a candidata a função inversa é:
p
−1 + 1 + 4y 2
x= ,
2y
que faz sentido ∀y > 0 (mostraremos mais adiante que a imagem de
x >0
y = 1−x 2 é de fato todo R ).
Preciso conferir que x( y(x) ) ≡ x, o que não está nada óbvio neste
exemplo.
Vejamos: q
x 2
−1 + 1 + 4( 1−x 2)
x( y(x) ) = x =
2 ( 1−x 2)
q 2 )2 +4x2
−1 + (1−x (1−x2 )2
= x =
2 ( 1−x 2)
q 2 )2
−1 + (1+x (1−x2 )2
= x =
2 ( 1−x 2)

399
400

1+x 2
−1 + 1−x 2
x = x.
2 ( 1−x2 )
0.15. Capı́tulo 3:
Exercı́cio 3.1
Suponhamos por absurdo que exista o número 01 .
Então 0 · 10 = 1, pois o sentido de x1 é ser o inverso multiplicativo
de x.
Agora afirmo que
∀x ∈ R, 0 · x = 0.
Se provo isso teremos em aprticular que
1
0· =0
0
e portanto 0 = 1. Isso contradiz um princı́pio básico: 0 6= 1.
Ora,
0·x=0 ⇔ (1 − 1) · x = 0 ⇔ x = x,
e este último fato é verdade: x = x.

Exercı́cio 3.2:
i) Dados x, y, z, w ∈ R com
x≥y e z ≥ w,
podemos traduzir isso em:
(x − y) ≥ 0 e (z − w) ≥ 0.
Queremos provar que
x + z ≥ y + w,
que se traduz em
(x + z) − (y + w) ≥ 0,
ou, o que diz o mesmo:
(x − y) + (z − w) ≥ 0.
Isso é o que queremos. Para termos isso, podemos usar o Princı́pio 1,
pois então com esse princı́pio:
(x − y) ≥ 0 e (z − w) ≥ 0 ⇒ (x − y) + (z − w) ≥ 0.
ii) Temos que x > 0. Caso y = z então x · y = x · z. Por isso
supomos que y > z, ou seja, y − z > 0.
Queremos provar que x · y > x · z, ou seja, que
x · y − x · z > 0,
o que é o mesmo que dizer que
x · (y − z) > 0.
CAPÍTULO 32. SOLUÇÕES DETALHADAS DE ALGUNS
EXERCÍCIOS 401

Isso é o que queremos. Então podemos usar o Princı́pio 2, que dá:


x>0 e y−z >0 ⇒ x · (y − z) > 0.
iii) Temos agora −x > 0. Caso y = z então x · y = x · z.
Por isso supomos y > z, ou seja, y − z > 0. Então o Princı́pio 2 dá:
(−x) · (y − z) > 0,
ou seja
−x · y + x · z > 0,
ou seja,
x · y − x · z < 0,
que é o que buscávamos provar:
x · y < x · z.
1
iv) Temos x > 0 e suponhamos por absurdo que x
< 0.
Então − x1 > 0 e pelo Princı́pio 2:
1
x · (− ) > 0.
x
1
Mas x·(− x ) = −1. Logo obtemos −1 > 0 ou seja 1 < 0. Para obtermos
uma contradição usamos o Principio 3.
v) Seja x > 1. Suponhamos por absurdo que x1 ≥ 1.
Se x1 = 1 então chegamos na contradição: 1 = x.
Se x1 > 1 então multiplicando esta desigualdade por x > 1 > 0,
temos
1
x· >x·1
x
(pelo item ii) já provado).
Como x· x1 = 1 pela própria definição de x1 e como x·1 pela definição
do neutro 1, obtemos
1 > x,
que contradiz x > 1.

Exercı́cio 3.4:
ii) Primeiro noto que:
x2 − x > 0 ⇔ x · (x − 1) > 0 ⇔
x > 0 e x − 1 > 0 ou x < 0 e x − 1 < 0.
Ou seja, se x > 1 (mais forte que x > 0) ou se x < 0 (mais forte que
x < 1).
Em suma, se x ∈ (−∞, 0) ∪ (1, +∞).
iii) As raı́zes de 3x2 − 2x − 1 = 0 são: x1 = − 13 e x2 = 1. Logo
1
3x2 − 2x − 1 = (x + ) · (x − 1).
3
Portanto preciso determinar onde o produto (x + 13 ) · (x − 1) é positivo.
402

Ou ambos fatores nesse produto são positivos ou ambos são nega-


tivos, ou seja:
1 1
x>− e x > 1 ou x < − e x < 1.
3 3
Tomando apenas as informações mais fortes:
1
x > 1 ou x < − ,
3
ou seja, x ∈ (−∞, − 31 ) ∪ (1, +∞).

Exercı́cio 3.5
O que se quer provar é que:
 + △ ≤ |  | + |△|, caso 0 ≤  + △,
ou que
−( + △) ≤ | | + |△|, caso  + △ < 0.
Caso 0 ≤  + △: obviamente que valem
 ≤ | | e △ ≤ |△|,
e somando essas duas desigualdades obtemos o desejado:
 + △ ≤ | | + |△|.
Caso  + △ < 0: então pelo menos um deles é negativo, por exem-
plo, suponhamos que  < 0. Por absurdo, suponha que
|| + |△| < −( + △).
Como || = −, cancelamos esses termos na desigualdade anterior e
obtemos então que:
|△| < −△.
Se 0 < △ então chegamos no absurdo:
0 < △ =: |△| < −△ < 0.
Se △ ≤ 0 então −△ =: |△| < −△ é outro absurdo.
Logo
−( + △) ≤ || + |△|, caso ( + △) < 0.

0.16. Capı́tulo 4:

Exercı́cio 4.5:
Não temos informação nenhuma sobre a sequência, exceto que seus
termos são negativos. Por isso o melhor é raciocinar por absurdo.
Suponha por absurdo que limn→+∞ xn = L > 0. Considere
ǫ := L = |L − 0|,
CAPÍTULO 32. SOLUÇÕES DETALHADAS DE ALGUNS
EXERCÍCIOS 403

ou seja, a distância entre L e 0. Pela definição de limn→+∞ xn , dado


esse ǫ tem que haver um nǫ ∈ N tal que:
n > nǫ ⇒ |xn − L| < ǫ.
Mas coma escolha de ǫ := L isto quer dizer:
n > nǫ ⇒ |xn − L| < L,
ou seja, ou bem
xn − L < L, se 0 ≤ xn − L,
ou bem
−(xn − L) = L − xn < L, se xn − L < 0.
No primeiro caso, 0 < L ≤ xn e no segundo caso 0 = L − L < xn .
em ambos chegamos numa contradição com a hipótese xn < 0 ∀n.
Logo L ≤ 0.
Por exemplo, a sequência − n1 < 0 tem L = 0.

0.17. Capı́tulo 5:

0.18. Capı́tulo 6:
Exercı́cio 8.4:
Se x 6= 0 a função é resultado da composição de duas funções
contı́nuas, x1 e sin(x), e do produto com x: logo é contı́nua em x 6= 0.
Precisamos mostrar que em x = 0 temos:
1
lim x sin( ) = 0,
x→0 x
pois esse foi o valor associado a f (0) = 0.
Ou seja, precisamos ver que se xn é qualquer sequência com limn→+∞ xn =
0 então:
1
lim xn sin( ) = 0.
n→+∞ xn
Mas como | sin( x1n ) | ≤ 1, dado ǫ tomamos nǫ tal que:
| xn | < ǫ
e teremos:
1 1
| xn sin( ) | = | xn | · | sin( ) | <
xn xn
< ǫ · 1 = ǫ,
o que siginifica
1
lim xn sin( ) = 0.
n→+∞ xn
O Maple plota assim o gráfico de y = x sin( x1 ) perto da origem:
404

0,04

x
-0,1 -0,05 0 0,05 0,1
0

-0,04

-0,08

Exercı́cio 8.9
Esse limite pode ser feito de dois modos. Podemos calcular assim:
√ 5·x +x

2
5 · x2 + x x
lim = lim x+2 =
x→+∞ x+2 x→+∞
x
q q
5·x2 +x
x2
5 + x1 √
= lim 2 = lim 2 = 5,
x→+∞ 1+ x x→+∞ 1 +
x
onde se usou a continuidade da raı́z quadrada.
Mas poderı́amos primeiro calcular
√ 2
5 · x2 + x 5 · x2 + x
lim ( ) = lim 2 =
x→+∞ x+2 x→+∞ x + 2 · x + 4

x2 · (5 + x1 ) 5 + x1
= lim 2 = lim =5
x→+∞ x · (1 + 2 + 42 ) x→+∞ 1 + 2 + 4
x x x x2
e depois aplicar a raı́z quadrada:
s
√ √ 2
5· +x x2 5 · x2 + x
lim = lim ( ) =
x→+∞ x+2 x→+∞ x+2
s
√ 2
5 · x2 + x √
= lim ( ) = 5,
x→+∞ x+2
onde nesta última linha usamos a continuidade da raı́z quadrada.

Exercı́cio 8.10:
CAPÍTULO 32. SOLUÇÕES DETALHADAS DE ALGUNS
EXERCÍCIOS 405

Fazemos aparecer quocientes:



√ √ x2 + x + x
lim ( x2 + x − x ) = lim ( x2 + x − x ) · [ √ ]=
x→+∞ x→+∞ x2 + x + x
x2 + x − x2 x
= lim √ = lim √ =
x→+∞ x2 + x + x x→+∞ x2 + x + x
x
1 1
= lim √ 2 x = lim q = .
x→+∞ x +x+x x→+∞ x2
+ x +1 2
x x2 x2

0.19. Capı́tulo 7:
Exercı́cio 7.3:
Resolver o sistema
y − 5x − 2 = 0 e 2y − 10x − 1 = 0,
significa, geometricamente, intersectar as retas:
10x + 1 1
y = 5x + 2 e y = = 5x + .
2 2
Porém essas retas tem o mesmo coeficiente angular 5, logo são paralelas
e distintas (pois seus coeficientes lineares são distintos).
Por isso não consigo resolver o sistema.

Exercı́cio 7.6
i) Quero que o coeficiente angular a′ da reta contendo o segmento
P Q seja
1
a′ = −
a
paera que haja ortogonalidade com a reta y = ax + b.
Ora então quero:
(ax + b) − B 1
a′ := =− .
x−A a
Isso produz uma equação:
(a2 + 1) x + a(b − B) − A = 0.
A solução é
A − a(b − B)
x= .
a2 + 1
Portanto
A − a(b − B) A − a(b − B)
Q=( 2
, a·( ) + b ).
a +1 a2 + 1
ii) Se temos x = A então :
A − a(b − B)
A=
a2 + 1
isso dá
a2 A + a(b − B) = 0.
406

Supondo por um momento a 6= 0, divido por ele e obtenho:


a A + (b − B) = 0,
ou seja, aA + b = B. Mas isso significa que P = (A, B) ∈ r.
A conclusão é que, se x = A, então
ou P = Q = (A, B) ou a = 0.
No caso a = 0 temos uma reta r horizontal e Q é a projeção vertical
de P sobre essa reta.
0.20. Capı́tulo 8:
Exercı́cio 5.2:
A inversa de y = f (x) = x1 é x = f −1 (y) = y1 .
Se usamos o mesmo sistema cartesiano para a f (x) e sua inversa
diremos que
1
f (x) = = f −1 (x).
x
(Quando ocorre de uma função coincidir com sua inversa, dizemos
que a função é uma involução).
Como em geral o gráfico de f e de sua f −1 são refletidos um do
outro na diagonal, o que temos no caso f = f −1 é que o gráfico de
y = x1 é perfeitamente simétrico relativo à diagonal: o pedaço que fica
abaixo da diagonal é refletido no que fica acima e vice-versa.
0.21. Capı́tulo 9:
Exercı́cio 4.6:
Fixe x 6= 0. No que segue, se x < 0 tome x < 0 e se x > 0 tome
x > 0.
Traço retas secantes ao gráfico de y = x1 ligando (x, x1 ) a cada (x, x1 ),
cujo coeficente angular é:
1 1 x−x
x
− x xx
ax := = =
x−x x−x
x−x 1 −1
= · = < 0,
(x − x) x x xx
(pois x e x têm o mesmo sinal).
As secantes são portanto retas de coeficiente angular ax <. Pas-
sando ao limite quando x → x o que dá para prever é que a reta
tangente terá coefciente angular a ≤ 0.
Vejamos que de fato a < 0.
Pela definição de coeficiente angular da reta tangente, fixado x 6= 0:
f (x + h) − f (x)
a := f ′ (x) = lim =
h→0 h
1 1 x−(x+h)
x+h
− x (x+h) x
= lim = lim =
h→0 h h→0 h
CAPÍTULO 32. SOLUÇÕES DETALHADAS DE ALGUNS
EXERCÍCIOS 407

−h −1
= lim = lim =
h→0 (x + h) x h h→0 (x + h) x
−1
= <0
x2
−1
(na última etapa uso que a função de h dada por (x+h) x
é contı́nua !
Logo seu limite quando h → 0 é simplesmente seu valor em h = 0).

0.22. Capı́tulo 10:


Exercı́cio 5.4:
Primeiro testo se (−1, −1) e (2, 3) estão em todos os gráficos de:
y = fb (x) := (4/3 − b) · x2 + b · x + (2b − 7/3), b ∈ R.
De fato:
−3
(4/3 − b) · (−1)2 + b · (−1) + (2b − 7/3) = = −1,
3
e
9
(4/3 − b) · 22 + b · 2 + (2b − 7/3) =
= 3.
3
O coeficiente angular da secante a todos os gráficos y = fb (x) lig-
ando (−1, −1) a (2, 3) é:
3+1 4
a= = .
2+1 3
Pelo Teorema de Lagrange devem haver pontos xb (dependendo de
b, a princı́pio ...) tais que
4
xb ∈ (−1, 2) e fb′ (xb ) = .
3
Vejamos quem são os xb . Temos
fb′ (x) = 2 · (4/3 − b) · x + b,
4
e igualando a 3
criamos uma equção em x:
4
2 · (4/3 − b) · x + b = ,
3
de onde
1 34 − b 1
x= ·(4 )= ,
2 3 −b 2
ou seja ∀b: xb = 12 . Por isso quando fazemos um zoom numa faixa
vertical em torno de
1 1
( , fb ( ) )
2 2
vemos todos os gráficos parecidos com retas paralelas, de mesma in-
clinação 43 .
408

0.23. Capı́tulo 11:


Exercı́cio 10.5:

Nas Figuras a seguir não usei a mesma escala nos eixos x e y, por
isso as figuras são apenas qualitativamente corretas.

2
x
-1 -0,5 0 0,5 1
0

-2

-4

-6

-8

Figura: y = f1 (x) = x3 − x2 (verm.), f1′ (x) (verde), f1′′ (x) (amar.)

0
-1 -0,5 0 0,5 1 1,5

-2 x

-4

-6
CAPÍTULO 32. SOLUÇÕES DETALHADAS DE ALGUNS
EXERCÍCIOS 409

Figura: y = f2 (x) = x2 − x3 (verm.), f2′ (x) (verde), f2′′ (x) (amar.)

15

10

0
-1 0 1 2 3
x

-5

-10

Figura: y = f3 (x) = −2x2 + x3 (verm.), f3′ (x) (verde), f3′′ (x) (amar.)

20

15

10

0
-1 -0,5 0 0,5 1
x

-5

Figura: y = f4 (x) = x4 − 2x2 (verm.), f4′ (x) (verde), f4′′ (x) (amar.)
410

80

60

40

20

0
-1 -0,5 0 0,5 1 1,5 2
x
-20

Figura: y = f5 (x) = 3x4 − 4x3 (verm.), f5′ (x) (verde), f5′′ (x) (amar.)
Esta última Figura merece um zoom perto da origem:

20

15

10

0
-0,4 -0,2 0 0,2 0,4 0,6
x

-5

Exercı́cio 10.6:
Note que
x3 + C · x2 = −( (−x)3 − C(−x)2 ).
Ou seja que o gráfico de y = x3 + C · x2 pode ser obtido refletindo
o de y = x3 − C · x2 primeiramente no eixo x (passar de x a −x) e,
depois, refletindo no eixo y (passar de y para −y).
CAPÍTULO 32. SOLUÇÕES DETALHADAS DE ALGUNS
EXERCÍCIOS 411

A Figura a seguir mostra em vermelho y = x3 − C · x2 , em verde o


de y = (−x)3 − C(−x)2 e em amarelo o de y = x3 + C · x2 . para C = 3.

100

50

0
-3 -2 -1 0 1 2 3
x

-50

-100

0.24. Capı́tulo 12:


Exercı́cio 3.1:
Note que
sin(k · x) sin(k · x)
lim = lim [ · k].
x→0 x x→0 k·x
E que, se fazemos uma mudança de notação z := k · x, temos :
sin(k · x) sin(z)
lim = lim = 1.
x→0 k·x z→0 z
Então :
sin(k · x) sin(k · x)
lim = lim · k = k.
x→0 x x→0 k·x
0.25. Capı́tulo 13:
Exercı́cio 5.1:
Se n = 1 então claramente:
1! = 1 ≥ 20 = 1.
Supondo válida a desigualdade até n − 1 (n ≥ 2):
n! = n · (n − 1)! ≥ n · 2n−2 .
Ora,
2n−1
n · 2n−2 = n · =
2
n
= 2n−1 · ≥ 2n−1 ,
2
onde usei na última desigualdade que n ≥ 2.
412

0.26. Capı́tulo 14:


Suponha que sabemos:
sin(x + y) = sin(x) · cos(y) + cos(x) · sin(y),
Faço o seguinte: fixo y e olho a identidade acima apenas em x.
Derivo o lado esquerdo, pela regra da derivada da composta:
(sin(x + y))′ = cos(x + y) · 1,
e o lado direito:
(sin(x) · cos(y) + cos(x) · sin(y))′ = cos(x) · cos(y) + (− sin(x) · sin(y)) =
= cos(x) · cos(y) − sin(x) · sin(y).
Igualando o lado esquerdo e o direito:
cos(x + y) = cos(x) · cos(y) − sin(x) · sin(y).

0.27. Capı́tulo 15:


Exercı́cio 5.1:
Note que:
∂F (x, y) ∂F (x, y)
= −3 x2 e = 2 y,
∂x ∂y
logo calculados em (1, 1):
∂F (x, y) ∂F (x, y)
= −3 e = 2.
∂x ∂y
Então num pequeno entorno de (1, 1) a curva é dada pelo gráfico
de y = y(x).
Mas a curva não é globalmente um gráfico y = y(x), pois para cada
valor x > 0 temos dois valores de y.
Note que se um ponto da curva y 2 − x3 = 0 tem x = 0, então y 2 = 0
e portanto y = 0, ou seja é a origem.
E note que nenhum ponto da curva y 2 − x3 = 0 tem coordenada
x < 0.

0.28. Capı́tulo 16:


Exercı́cio 7.1:
iii): Usando a derivada a composta:
sin3 (x3 )′ = 3 sin2 (x) cos(x3 )(3x2 )
iv): Usando a regra da derivada do produto:
(sin(x) cos(x))′ = cos(x) cos(x) + cos(x)(− sin(x)) = cos2 (x) − sin2 (x).
v): Usando a regra da derivada do quociente:
x4 + x2 + 1 ′ (4x3 + 2x)(3x4 + 4x2 + 1) − (x4 + x2 + 1)(12x3 + 8x)
( ) = .
3x4 + 4x2 + 1 (3x4 + 4x2 + 1)2
CAPÍTULO 32. SOLUÇÕES DETALHADAS DE ALGUNS
EXERCÍCIOS 413

vi): Usando a regra da composta:


√ 1 1 −1 x
( 1 − x2 )′ = ((1 − x2 ) 2 )′ = (1 − x2 ) 2 (−2x) = − √
2 1 − x2
xv): pela composta:
((3x + 4)100 )′ = 300(3x + 4)99 .
0.29. Capı́tulo 17. Exercı́cio 3.1:
Defina a função:

f (x) := v2 · x2 + 25 + v1 · (8 − x),
que dá o tempo gasto pelo salva-vidas para chegar em B.
Ou melhor, considere:
√ v1
g(x) := x2 + 25 + · (8 − x) =
v2

= x2 + 25 + k · (8 − x),
cujo domı́nio é [0, 8]. Trata-se de minimizar f ou, equivalentemente,
minimizar g.
Para isso calcule separadamente

g(0) = 5 + 8k e g(8) = 89.
Mas: √
89 − 5
g(8) > g(0) ⇔ > k,

8
89−5
e como 0.55 ≈ 8
e supusemos k ≤ 0.5 então:
g(8) > g(0).
Agora basta buscar no intervalo aberto (0, 8) pelo ponto onde
g ′ (x) = 0.
Ora,
x √
g ′ (x) = √ −k =0 ⇔x=k· x2 + 25.
2
x + 25
Daı́ obtemos, elevando ao quadrado:
x2 = k 2 · (x2 + 25),
ou seja,
x2 (1 − k 2 ) = 25 · k 2
e r
25 · k 2 5k
x(k) = = √ ,
1 − k2 1 − k2
pois a solução negativa não nos interessa. Claramente:
5k 0
lim x(k) = lim √ = = 0.
k→0 k→0 1−k 2 1
414

E nesse ponto x(k) temos o valor:


r
1
g(x(k)) = 8k + 5(1 − k 2 ) · .
1 − k2
Agora r
2 1
g(0) − g(x(k)) = 5 + 5(k − 1) ·
1 − k2
e não está tão claro se g(0) − g(x(k)) ≥ 0, para todos os k no intervalo
0 ≤ k ≤ 0.5. Ora,
r
2 1
5 + 5(k − 1) · ≥0 ⇔
1 − k2
r
1
⇔ 5 ≥ 5(1 − k 2 ) ·
1 − k2
e elevando ao quadrado quero ter:
25 · (1 − k 2 )2
25 ≥
1 − k2
que equivale a :
1 − k 2 ≥ 1 − 2k 2 + k 4 ,
ou seja,
0 ≥ k 2 · (k 2 − 1)
e isso sempre temos se 0 ≤ k ≤ 0.5.

0.30. Capı́tulo 18:

0.31. Capı́tulo 19:


Exercı́cio 6.1:
Se escrevemos
π π π
x1 = sin( ) + sin(π),
2 2 2
π π π 2π π
x2 = sin( ) + sin( ) + sin(π),
3 3 3 3 3
π π π 2π π 3π π
x3 = sin( ) + sin( ) + sin( ) + sin(π),
4 4 4 4 4 4 4
π π π 2π π
x4 = sin( ) + sin( ) + . . . + sin(π),
5 5 5 5 5
fica mais fácil reconhecer que cada xi é uma soma de Riemann da
π
função sin : [0, π] → R, onde a partição tem norma i+1 .
Em geral:
π π π 2π π (i + 1)π
xi = sin( )+ sin( ) + ... + sin( ).
i+1 i+1 i+1 i+1 i+1 i+1
Quando i → ∞ a norma da partição tende a zero.
CAPÍTULO 32. SOLUÇÕES DETALHADAS DE ALGUNS
EXERCÍCIOS 415

Como sin(x) é uma função contı́nua, os itens i) e ii) garantem que

Z π
lim xi = sin(x) dx.
i→∞ 0

Mais adiante, pelo Segundo Teorema fundamental, veremos que:

Z π
sin(x) dx = 2.
0

Exercı́cio 6.2:
Se x < 0 então

Z x Z x
F (x) := | t | dt = −t dt =
−1 −1

−t2 −t2 −x2 1


=( )(x) − ( )(−1) = + .
2 2 2 2

Se x ≥ 0 podemos fazer:

Z x Z 0 Z x
F (x) = | t | dt = | t | dt + | t | dt =
−1 −1 0

Z x
1
= + t dt =
2 0

1 x2
= + .
2 2

Ou seja que a função F (x) obtida integrando o módulo tem uma


descrição diferente, dependendo se x < 0 ou x ≥ 0.
Note que pelo Primeiro Teorema Fundamental, F ′ (x) = | x |, logo
não existe F ′′ (0).
Ou seja, que F (x) é menos suave em em x = 0 que f (x) = x3 + 12 .
A figura a seguir apresenta F (x) (vermelho) e f (x) = x3 + 12 (verde):
416

1,5

0,5

0
-1 -0,5 0 0,5 1
x

-0,5

0.32. Capı́tulo 20:


Exercı́cio 6.4:
ln(x)
Primeiro busco o ponto de y = f (x) = x
onde f ′ (x) = 0. Pela
derivada do quociente:
1
x − ln(x) 1 1 − ln(x)
f ′ (x) = x
2
= ,
x x2
e f ′ (x) = 0 exatamente onde 1 − ln(x) = 0, ou seja, onde ln(x) = 1.
Sabemos então que a solução é x = exp(1).
Podemos calcular a segunda derivada f ′′ (x), para confirmarmos que
f ′′ (exp(1)) < 0. Caso isso valha, a Afirmação 2.1 do Capı́tulo 10 diz
que x = exp(1) é ponto de máximo local. E portanto concluiremos que
x = exp(1) é ponto de máximo global (já que não há outro candidato).
Ora,
(1 − ln(x))′ x2 − (1 − ln(x)) 2x
f ′′ (x) = =
x4
− x1 x2 − (1 − ln(x)) 2x −3x + 2x ln(x)
= 4
= ,
x x4
e portanto f ′′ (exp(1)) = − exp(1)
e4
< 0.

Exercı́cio 6.3:
Como arcsin′ (x) = √ 1
1−x2
então:
x√ 1
F ′ (x) = [1 − x2 ]′ + ( arcsin(x))′ =
2 2
1 √ x 1 1 1 1
=[ 1 − x2 + · √ · (−2x)] + √ =
2 2 2 1−x 2 2 1 − x2
CAPÍTULO 32. SOLUÇÕES DETALHADAS DE ALGUNS
EXERCÍCIOS 417

1√ 1 1 1 1
= 1 − x2 − x2 √ + √ =
2 2 1 − x2 2 1 − x2
1√ 2
1 1 − x2
1−x + √ =
2 2 1 − x2

= 1 − x2 .

Exercı́cio 6.3:
ln(1+x)
O programa Maple plota y = x
completando em x = 0 o valor
ln(1 + x)
lim =1
x→0 x
De fato posso escrever:
ln(1 + x) − 0 ln(1 + x) − ln(1)
lim = lim
x→0 x x→0 x
e esse último limite é nada mais nada menos que uma derivada:
ln(1 + x) − ln(1)
ln′ (1) := lim .
x→0 x
′ 1
Ora ln (1) = 1
= 1.

Exercı́cio 6.6:
As primitivas de produto/quociente NÃO são o produto/quociente
de primitivas. Quando aparecem produtos é natural imaginar qu sur-
giram de se derivar composições de funções.
vi): Por isso as primitivas de f (x) = 2x cos(x2 ) são
F (x) = sin(x2 ) + C.
x
vii): As primitivas de 2
cos(x2 ) são:
sin(x2 )
F (x) = + C.
4
viii): As primitivas de x exp(x2 ) são
exp(x2 )
2
e as de exp(x) cos(exp(x)) são
sin(exp(x)) + C.
As primitivas de soma/subtração são a soma/subtração de primiti-
vas.
x): Portanto as primitivas de f (x) = a0 xn + a1 xn−1 + . . . + an são
xn+1 xn
a0 + a1 + . . . + an x + C.
n+1 n

Exercı́cio 6.17:
418

A função y = f (x) = exp(−x2 ) tem, pela regra da composta e pelo


fato que exp′ (x) = exp(x), derivada
f ′ (x) = exp(−x2 ) · (−2x).
lno f ′ (x) se anula apenas em x = 0 (pois exp não se anula nunca). Já
a segunda derivada é (pela regra do produto e da composta):
f ′′ (x) = (exp(−x2 ) · (−2x))′ =

= (exp(−x2 ) · (−2x))(−2x) + exp(−x2 )(−2) =


= 2 exp(−x2 )(2x2 − 1).
q q
logo f (x) se anula em x = + 2 e x = − 12 .
′′ 1

Esses dois pontos são pontos de máximo/mı́nimo da f ′ (x) e pontos


de inflexão da f .

Exercı́cio 6.18:
Os pontos (x, y) da reta tangente ao gráfico de y = ln(x) no ponto
(e, 1) são os pontos que verificam:
y−1
= ln′ (e),
x−e
pois o valor da derivada ln′ (e) é por definição o coeficiente angular da
reta tangente.
Mas ln′ (e) = 1e , lno
y−1 1
=
x−e e
de onde
x
y−1= −1
e
x
e portanto y = e , que é uma reta pela origem.
Por reflexão na diagonal se obtem o gráfico da função inversa exp(x).
E a reflexão na diagonal da reta y = xe é x = ye , ou seja, a reta
y = ex. Essa é a tangente ao gráfico de y = exp(x) em (1, e), como
também se pode verificar a partir de:
y−e
= exp′ (1) = exp(1) =: e.
x−1

0.33. Capı́tulo 21:


Exercı́cio 4.2:
Note que
• o integrando é a diferença entre as funções x − x2 e a função
x3 .
• x − x2 > 0 para 0 < x < 1.
CAPÍTULO 32. SOLUÇÕES DETALHADAS DE ALGUNS
EXERCÍCIOS 419

• Ademais
x − x2 > x3 ,
para x pequenos, pois
x − (x2 + x3 ) > 0
para x pequenos.
• Porém certamente a partir de um certo x deve acontecer que
x − x2 < x3 ,
devido ao expoente 3.
Para qual x ≥ 0 temos x − x2 = x3 ? Ou seja, onde x3 + x2 − x = 0
? Nas soluções de:
x (x2 + x − 1) = 0,
ou seja, em x = 0 ou na solução positiva de (x2 + x − 1), que é

−1 + 5
a := ∼ 0.6.
2
A partir desse a ∼ 0.6 vale x − x2 < x3 .
Então escrevo:
Z b Z a Z b
2 3 2 3
x − x − x dx = x − x − x dx + x − x2 − x3 dx
0 0 a
e portanto: Z b
x − x2 − x3 dx = 0 ⇔
0
Z a Z b
2 3
⇔ x − x − x dx = − x − x2 − x3 dx.
0 a
Mas Z Z
b b
2 3
− x − x − x dx = −(x − x2 − x3 ) dx =
a a
Z b
= x3 − (x − x2 ) dx.
a
Em suma,
Z a Z b
2 3
x − x − x dx = x3 − (x − x2 ) dx.
0 a
Ora, Z a
(x − x2 ) − x3 dx
0
é uma Área, pois (x − x ) − x3 ≥ 0 na região x ∈ [0, a]. E também
2
Z b
x3 − (x − x2 ) dx
a

é uma Área, pois agora x − (x − x2 ) ≥ 0 se x ≥ a. 3


420

Na Figura a seguir os gráficos de y = x − x2 > 0 (vermelho) e de


y = x3 (verde) formam um peixe (x R a∈ 0, b]. 2
O peixe tem a área do corpo ( 0 (x − x ) − x3 dx) igual a área do
Rb
rabo a x3 − (x − x2 ) dx (b ∼ 0.9).

0,7

0,6

0,5

0,4

0,3

0,2

0,1

0
0 0,2 0,4 0,6 0,8
x

Exercı́cio 4.5:
Para saber de onde até onde considerar a Área precisamos saber
as abscissas dos pontos onde os gráficos de y = x4 e de y = a se
intersectam.
1 1
Ou seja, resolver x4 = a, o que dá x = −a 4 e x = a 4 .
1
Vamos subtrair da área do retângulo de base 2a 4 e altura a (que é
1 5
2a 4 a = 2a 4 ) a área sob o gráfico de x4 .
Esta última é dada pelo importante Teorema Fundamental do Cálculo.
Na notação do Curso:1
5
1 x5 1 x5 1 a4
Ax4 , −a 41 ( a ) = (a 4 ) − (−a 4 ) = 2
4
5 5 5
lno a área que buscamos é
5
5 a4 4 5
2a − 2
4 = 2( a 4 ).
5 5
Como exigimos que seja
5 4 5
= 2( a 4 )
2 5
concluimos que
5 25
a4 =
16
4
e portanto a = ( 25
16
)5 .

1Na
R a 14 x5 x5
notação usual de integrais −a 4
1 x4 dx = 1
5 |a 4 − 1
5 |−a 4
CAPÍTULO 32. SOLUÇÕES DETALHADAS DE ALGUNS
EXERCÍCIOS 421

0.34. Capı́tulo 22:


0.35. Capı́tulo 23:
0.36. Capı́tulo 24:
Exercı́cio 7.3:
A solução da equação f ′ (x) = −kf (x) é
f (x) = f (0) exp(−kx), ∀x.
f (0)
Portanto f (τ ) := 2
é também:
f (τ ) = f (0) exp(−kτ ).
Logo dividindo por f (0):
1
= exp(−kτ ).
2
Aplicando ln em ambos lados:
1
ln( ) = ln(exp(−kτ )) = −kτ,
2
e portanto:
ln( 21 ) − ln(2) ln(2)
τ= = = .
−k −k k
Por definição de τ̂ temos: f (τ̂ ) := f (0)
4
é também:
f (τ̂ ) = f (0) exp(−kτ̂ ).
lno dividindo por f (0):
1
= exp(−kτ̂ ).
4
Aplicando ln em ambos lados:
1
ln( ) = ln(exp(−kτ̂ )) = −kτ̂ ,
4
e portanto:
ln( 212 ) − ln(22 ) 2 ln(2)
τ̂ = = = .
−k −k k
Ou seja, τ̂ = 2τ .
Para a τ̌ temos por definição f (τ̌ ) := f√(0)
2
é também
f (τ̌ ) = f (0) exp(−kτ̌ ).
lno dividindo por f (0):
1
√ = exp(−kτ̌ ).
2
Aplicando ln em ambos lados:
1
ln( √ ) = ln(exp(−kτ̌ )) = −kτ̌ ,
2
422

e portanto
1
ln( 1 ) − ln(2 2 )
1
1 ln(2)
22
τ̌ = = = .
−k −k 2 k
1
Ou seja, τ̌ = 2
τ.

Exercı́cio 7.5:
Sabemos que a solução da equação, com f (0) = 1 é f (x) = exp(−kx).
Queremos x tal que f ′ (x) = −1, onde
f ′ (x) = −k exp(−kx).
Logo queremos encontrar x tal que:
−1 = −k exp(−kx),
1
ou seja, k
= exp(−kx), ou seja, ln( k1 ) = −kx, de onde
ln(k)
x= .
k
Resolvi fazer um exemplo, com k = 2 e portanto x = ln(2)2
.
Pedi para o Maple plotar os gráficos de y = f (x) = exp(−2x) e de
y = −x para
ln(2) ln(2)
x∈[ − 0.1, + 0.1]
2 2
e o resultado aparece a seguir:

0,6

0,4

0,2

0
0,28
0,32
0,36
0,4
0,44
x
-0,2

-0,4

Exercı́cio 7.6:
Item i):
A função que dá a posição a partir de A é parecida com àquela da
2
queda-livre vertical: g · t2 (já que f ′ (0) = 0 e f (0) = 0 e a aceleração é
constante ao longo da semireta AB).
Mas a diferença com aquele caso já estudado é que a gravidade
atua na semireta AB de acordo com a projeção de um vetor vertical
de módulo g nesta semireta; ou seja, com valor
g · sin(θ)
CAPÍTULO 32. SOLUÇÕES DETALHADAS DE ALGUNS
EXERCÍCIOS 423

onde θ é o ângulo entre a semireta AB e uma reta horizontal. Ou seja,


o efeito da gravidade vira zero se θ = 0 e volta a ser máxima se θ = π2 .
Por isso se tomamos um sistema cartesiano em que
A = (0, 0), B = (b1 , b2 ), com b1 6= 0, b2 < 0,
então o deslizamento do objeto ao longo da semireta AB
t2
g · sin(θ) · .
2
será descrito pela curva parametrizada:
b1 t2 b2 t2
(x(t), y(t)) = ( p · g sin(θ) · , p · g sin(θ) · ),
b21 + b22 2 b21 + b22 2
onde ( √ b21 , √ b21 ) é um vetor de módulo 1 que gera a semireta AB.
b1 +b22 b1 +b22
Já que
−b2
sin(θ) = p 2
b1 + b22
ficamos com:
−b1 · b2 t2 −b22 t2
(x(t), y(t)) = ( · g · , · g · ).
(b21 + b22 ) 2 (b21 + b22 ) 2
Item ii):
O tempo que leva para chegar em B se obtém igualando:
−b1 · b2 t2 −b22 t2
· g · = b 1 ou · g · = b2 ,
(b21 + b22 ) 2 (b21 + b22 ) 2
o que dá: s
2 · (b21 + b22 )
t= .
−g · b2
0.37. Capı́tulo 25:
0.38. Capı́tulo 26:
0.39. Capı́tulo 27:
0.40. Capı́tulo 28:
0.41. Capı́tulo 29:
0.42. Capı́tulo 30:

Você também pode gostar